CHAPTER 1 1)
The major groups of microorganisms studied by microbiologists include
.
A) bacteria B) plants C) helminths D) algae E) fungi F) viruses G) archaea H) protozoa
2)
Select all answers that are roles played by microorganisms in our environment.
A) B) C) D)
Carry out photosynthesis Biological decomposition Nutrient recycling Complex relationships with animals but not plants
3) Select statements that apply to the theory of evolution to test your understanding of evolution.
A) Has undergone years of testing B) Is a new untested hypothesis C) Has not been disproven D) Lacks supportive evidence E) Is a well-established natural phenomenon
4)
Which of the following are correctly matched?
Version 1
1
A) Bacteria—no nucleus B) Bacteria—true nucleus C) Archaea—true nucleus D) Archaea—no nucleus E) Eukaryotes—true nucleus F) Eukaryotes—no nucleus
5)
Select the main groups of macromolecules found in living things.
A) Nucleic acids B) Oxygen C) Carbon dioxide D) Lipids E) Proteins F) Carbohydrates
6)
Select the nitrogenous bases found in nucleotides that make up RNA.
A) Guanine B) Uracil C) Thymine D) Adenine E) Cytosine
7)
Select the nitrogenous bases found in nucleotides that make up DNA.
Version 1
2
A) Guanine B) Uracil C) Thymine D) Adenine E) Cytosine
8)
Select characteristics exhibited by all cells.
A) Cytoplasmic membrane B) Nucleus C) DNA D) Ribosomes E) Cell wall F) Organelles
9)
Select all areas that comprise the main aspects of the science of taxonomy. A) Classification B) Genetics C) Nomenclature D) Analysis E) Identification
10)
When humans manipulate the genes of microorganisms, the process is called
.
A) bioremediation B) genetic engineering C) epidemiology D) immunology E) taxonomy
Version 1
3
11)
Which of the following is not considered a microorganism?
A) Mosquito B) Protozoan C) Bacterium D) Virus E) Fungus
12)
All microorganisms are best defined as organisms that
.
A) cause human disease B) lack a cell nucleus C) are infectious particles D) are too small to be seen with the unaided eye E) can only be found growing in laboratories
13)
Which activity is an example of biotechnology?
A) Bacteria in the soil secreting an antibiotic to kill competitors B) A microbiologist using the microscope to study bacteria C) Egyptians using moldy bread on wounds D) Escherichia coli producing human insulin E) Public health officials monitoring diseases in a community
14)
Living things ordinarily too small to be seen with the unaided eye are termed
Version 1
.
4
A) bacteria B) viruses C) parasites D) microorganisms E) prokaryotes
15) The microorganisms that recycle nutrients by breaking down dead matter and wastes are called .
A) decomposers B) prokaryotes C) pathogens D) eukaryotes E) fermenters
16) Cells, like bacteria and archaea, that do not have a nucleus in their cells have traditionally been called . A) decomposers B) prokaryotes C) pathogens D) eukaryotes E) fermenters
17)
The first cells appeared about
Version 1
billion years ago.
5
A) 5.2 B) 4.6 C) 3.8 D) 2.9 E) 1.5
18)
Which of the following is not a human use of microorganisms?
A) Making bread B) Treating water and sewage C) Manufacturing copper wire D) Mass producing antibiotics E) Cleaning up oil spills
19)
Using microbes to detoxify a site contaminated with heavy metals is an example of .
A) biotechnology B) bioremediation C) decomposition D) immunology E) epidemiology
20)
Disease-causing microorganisms are called
Version 1
.
6
A) decomposers B) prokaryotes C) pathogens D) eukaryotes E) fermenters
21)
The most prevalent worldwide infectious diseases are
.
A) AIDS-related diseases B) diarrheal diseases C) malaria diseases D) measles E) respiratory diseases
22) Which of the following is a unique characteristic of viruses that distinguishes them from the other major groups of microorganisms?
A) Cause human disease B) Lack a nucleus C) Cannot be seen without a microscope D) Contain genetic material E) Lack cell structure
23)
Helminths are
Version 1
.
7
A) bacteria B) protozoa C) molds D) parasitic worms E) infectious particles
24) Which group of microorganisms is composed only of hereditary material wrapped in a protein covering?
A) Viruses B) Bacteria C) Parasites D) Fungi E) Helminths
25)
Which statement correctly compares the sizes of different microorganisms? A) B) C) D)
Bacteria are larger than viruses Bacteria are larger than eukaryotic microorganisms Eukaryotic microorganisms are smaller than viruses Archaea are larger than eukaryotic microorganisms but smaller than bacteria
26) The Dutch merchant who made and used quality magnifying lenses to see and record microorganisms was .
Version 1
8
A) Francesco Redi B) Antonie van Leeuwenhoek C) Louis Pasteur D) Joseph Lister E) Robert Koch
27)
Koch's postulates are criteria used to establish that
.
A) microbes are found on dust particles B) a specific microbe is the cause of a specific disease C) life forms can only arise from preexisting life forms D) a specific microbe should be classified in a specific kingdom E) microbes can be used to clean up toxic spills
28) was
The surgeon who advocated using disinfectants on hands and in the air prior to surgery .
A) Joseph Lister B) Ignaz Semmelweis C) Robert Koch D) Louis Pasteur E) Antonie van Leeuwenhoek
29)
Sterility refers to
Version 1
.
9
A) being pathogen free B) having an absence of spores C) having an absence of any life forms and viral particles D) being pasteurized E) being homogenized
30)
Which scientist showed that anthrax was caused by the bacterium, Bacillus anthracis?
A) Joseph Lister B) Ignaz Semmelweis C) Robert Koch D) Louis Pasteur E) Antonie van Leeuwenhoek
31) If you were a microbiologist in 1950, which of the following scientific principles would you already know? A) Aseptic techniques could reduce the number of wound infections in the surgical setting. B) Biofilms can form on implanted objects in the human body and be responsible for infection. C) Enzymes found in bacteria can be used to cut DNA. D) Very little DNA is transcribed into RNA that is then translated into proteins.
32)
Taxonomy does not involve
Version 1
.
10
A) nomenclature B) classification C) taxa D) identification E) common name
33) Which scientific field is involved in the identification, classification, and naming of organisms?
A) Nomenclature B) Taxonomy C) Phylogeny D) Woesean classification E) None of the choices are correct.
34)
The orderly arrangement of organisms into a hierarchy of taxa is called
.
A) classification B) identification C) nomenclature D) experimentation E) biotechnology
35)
Which of the following is a taxon that contains all the other taxa listed?
Version 1
11
A) Species B) Phylum C) Kingdom D) Genus E) Family
36)
The smallest and most significant taxon is
.
A) genus B) species C) kingdom D) family E) phylum
37)
Select the correct descending taxonomic hierarchy:
A) family, order,class B) family, genus,species C) genus, species,family D) class, phylum,order E) kingdom, domain,phylum
38)
Which of the following is a scientific name?
A) Gram-positive streptococcus B) Staphylococcus C) Streptococcuspyogenes D) Anthrax E) Streptobacilli
Version 1
12
39)
When assigning a scientific name to an organism,
.
A) the species name is capitalized B) the species name is placed first C) the species name can be abbreviated D) both genus and species names are capitalized E) both genus and species names are italicized or underlined
40)
The study of evolutionary relationships among organisms is called
,
A) biotechnology B) genetics C) recombinant DNA D) phylogeny E) taxonomy
41) Which area of biology states that living things undergo gradual, structural, and functional changes over long periods of time?
A) Morphology B) Phylogeny C) Evolution D) Genetics E) None of thechoices is correct.
42) A scientist studying the sequence of nucleotides in the rRNA of a bacterial species is working on .
Version 1
13
A) determining evolutionary relatedness B) bioremediation C) recombinant DNA D) nomenclature E) determining if that species is the cause of a new disease
43)
The scientist(s) who proposed organisms be assigned to one of three domains is(are) . A) Robert Koch and Louis Pasteur B) Antonie van Leeuwenhoek C) Carl Woese and George Fox D) Robert Whittaker E) Francesco Redi
44)
Which scientific name is written correctly?
A) Staphylococcusaureus B) staphylococcusaureus C) Staphylococcus Aureus D) Staphylococcus aureus E) STAPHYLOCOCCUS AUREUS
45) Organic chemicals always have a basic framework of the element atoms.
Version 1
bonded to other
14
A) carbon B) nitrogen C) oxygen D) hydrogen E) phosphorous
46)
Most biochemical macromolecules are polymers, which are chains of
.
A) hydrophobic molecules B) electrolytic molecules C) repeating monomers D) repeating carbohydrates E) hydrogen bonds
47)
All of the following are monosaccharides except
.
A) glucose B) glycogen C) fructose D) ribose E) deoxyribose
48)
All of the following are polysaccharides except
.
A) glycogen in liver and muscle B) agar used to make solid culture media C) a cell's glycocalyx D) cellulose in certain cell walls E) prostaglandins in inflammation
Version 1
15
49)
All of the following are lipids except
.
A) cholesterol B) starch C) phospholipid D) wax E) triglyceride
50)
What part of a phospholipid forms hydrophobic tails?
A) Fatty acids B) Glycerol C) Phosphate D) Alcohol E) All of thechoices are correct.
51) A fat is called if all carbons of the fatty acid chain are single bonded to two other carbons and two hydrogens. A) unsaturated B) polyunsaturated C) monounsaturated D) saturated E) None of the choices are correct.
52)
The lipid group that serves as energy storage molecules is
Version 1
.
16
A) prostaglandins B) waxes C) phospholipids D) steroids E) triglycerides
53)
The lipid group that is the major component of cell membranes is the
.
A) prostaglandins B) waxes C) phospholipids D) steroids E) triglycerides
54)
The building blocks of an enzyme are
.
A) nucleotides B) glycerol and fatty acids C) monosaccharides D) phosphate, glycerol, and fatty acids E) amino acids
55)
Which is not true about enzymes?
A) They are found in all cells. B) They are catalysts. C) Their shape determines their function. D) They can be denaturated by heat and other agents. E) They have high-energy bonds between phosphates.
Version 1
17
56) Which amino acid contains sulfur atoms that form covalent disulfide bonds in its tertiary structure?
A) Valine B) Cysteine C) Serine D) Alanine E) Tyrosine
57)
What type of bonds are formed between adjacent amino acids?
A) Glycosilic B) Ester C) Peptide D) Disulfide E) Phosphate
58)
The alpha helix is a type of
protein structure.
A) primary B) secondary C) tertiary D) quaternary E) None of thechoices is correct.
59)
One nucleotide contains
Version 1
.
18
A) one phosphate B) one pentose sugar C) one nitrogen base D) All of the choices are correct E) None of the choices are correct.
60)
Which pertains to DNA but not to RNA?
A) Contains ribose B) Contains adenine C) Contains thymine D) Contains uracil E) Contains nucleotides
61)
ATP is best described as
.
A) an enzyme B) a double helix C) an electron carrier D) the energy molecule of cells E) All of thechoices are correct.
62) You are trying to identify a chemical that consists of adenine, ribose, and three phosphates. What is this chemical? A) B) C) D)
Version 1
DNA RNA ATP Phospholipid
19
63) A student forgot to label a beaker containing a DNA solution and a beaker containing a glucose solution. If chemical analysis was performed to identify the contents of each beaker, which of the following would be found in the beaker of DNA but not in the beaker with glucose?
A) Amino acids B) Hydrogen and oxygen atoms C) Nitrogen and phosphorus D) Fatty acids E) Carbon atoms
64)
Purines and pyrimidines are components in the building block units of all
.
A) nucleic acids B) carbohydrates C) polysaccharides D) amino acids E) enzymes
65)
Which of the following is not a pyrimidine?
A) Uracil B) Adenine C) Thymine D) Cytosine E) All of these arepyrimidines.
66)
During protein synthesis,
Version 1
RNA is made as a copy of a gene from DNA.
20
A) B) C) D)
67)
Characteristics shared by all cells include
A) B) C) D)
68)
transfer messenger ribosomal All of the choices are correct.
a membrane serving as a cell boundary the possession of genetic information the presence of cellular fluid All of thesechoices are correct.
The purine
A) B) C) D)
.
always binds with the pyrimidine
in DNA and RNA.
guanine; cytosine cytosine; guanine adenine; guanine thymine; guanine
69) Which of the following statements is correct regarding the relationship between humans and microbes?
A) The majority of microorganisms that colonize humans are pathogenic B) Microorganisms are benefited from their colonization of humans, whereas humans are unaffected by the relationship C) Humans are colonized by bacteria and fungi, but not viruses D) Not only do the majority of colonizing bacteria cause no harm to humans, the relationship is beneficial for both microbe and human host
Version 1
21
70)
Which list correctly ranks the microorganisms from largest to smallest?
A) Zika virus, Bacillus anthracis, Aspergillis sp., Helminth B) Aspergillis sp., Zika virus, Bacillus anthracis, Helminth C) Bacillus anthracis, Helminth, Aspergillis sp., Zika virus D) Helminth, Aspergillis sp., Bacillus anthracis, Zika virus E) Helminth, Aspergillis sp., Zika virus, Bacillus anthracis
71) The Nobel Prize was awarded to Kary Mullis in 1993 for inventing what technique to amplify and subsequently analyze DNA?
A) Polymerase chain reaction B) The central dogma of biology C) Restriction enzyme analysis D) Human microbiomeproject E) Small RNA analysis
72) Which of the following statements correctly determines the process when following the scientific method?
A) B) C) D)
73)
Formulate question, conduct research, propose hypothesis, test hypothesis Propose hypothesis, test hypothesis, formulate question, conduct research Formulate question, propose hypothesis, test hypothesis, conduct research Conduct research, formulate question, propose hypothesis, test hypothesis
Which of the features listed below is not found in all cells?
Version 1
22
A) B) C) D)
Cytoplasmic membrane Ribosomes DNA Nucleus
74) Organisms were classified into kingdoms as they were defined. Which list reflects the order of discovery of the kingdoms as we know them today?
A) Monera, protista, fungi, plants and animals B) Plants and animals, protista, monera, fungi C) Fungi, monera, plants and animals, protista D) Protista, fungi, monera, plants and animals E) Monera, plants and animals, protista, fungi
75) Carl Woese and George Fox developed the three-domain system of taxonomy based on what molecular discovery?
A) Variations in the ribonucleic acid of the small ribosomal subunit of organisms B) Mutations in enzyme proteins C) Genetic analysis showing that bacteria and archaea are identical D) Molecular analysis of genes showing that eukaryotes evolved from bacteria, and bacteria evolved from archaea
76) The term used to describe the broad field of science that involves human manipulation of microbes for use in industrial processes is .
Version 1
23
A) B) C) D)
biotechnology bioremediation recombinant DNA technology biodegradation
77) The Dutch linen merchant ground glass lenses to detailedspecifications so that he was able to develop a microscope for observing and describing living microscopic animalcules. A) B) C) D)
78)
Leeuwenhoek Pasteur Lister Koch
Select the characteristic that isexhibited by viruses. A) Viruses are independent living cellular organisms. B) Viruses are much more complex than cells. C) Viruses are composed of both DNA and RNA. D) Viruses are parasitic particles that invade host cells. E) Viruses lack a protein coat.
79)
Alpha helices and beta pleated sheets are examples of A) B) C) D)
Version 1
protein structure.
primary secondary tertiary quaternary
24
80)
Disulfide bonds are involved in maintaining protein structure as well as the level of protein structure exhibited by complex proteins such as antibodies. A) tertiary; quaternary B) quaternary; tertiary C) secondary; quaternary D) secondary; tertiary E) primary; secondary
81) Choose the term that describes the formal system of identifying, arranging, and naming organisms. A) Nomenclature B) Identification C) Classification D) Taxonomy E) Hierarchy
82) Which of the following choices is a correct way to denotethe binomial name of a microorganism?
A) Staphylococcusaureus B) StaphylococcusAureus C) staphylococcusAureus D) staphylococcus aureus E) Staphylococcus aureus
83) Choose the statement or characteristicthat best describes the Woese-Fox taxonomic system to test your understanding of taxonomy.
Version 1
25
A) B) C) D)
84)
Three distinct cell lines called domains Five kingdoms Plants, animals, and microorganisms Prokaryotes versus eukaryotes
When classifying organisms, early taxonomists did NOTrely on which of the following?
A) B) C) D)
Analysis of theorganism's shape (morphology) Analysis ofstructural and organizational characteristics of the organism Analysis ofmetabolic (nutritional) characteristics of the organism Genetic analysisof the organism
85) NCLEX Prep - Test Bank Question: Please read the clinical scenario, and then answer the questions that follow to become familiar with the traditional NCLEX question format. Ms. Smith is a 29-year-old patient at the outpatient psychiatric clinic. While completing her assessment you notice her hands are red, raw, and show signs of recent bleeding. She explains that she washes all her clothes in bleach, and uses the chemical to clean her hands several times a day. She states “I need to sterilize myself and my environment of all germs so I do not get sick.” While developing her nursing plan of care, you educate her about the importance of bacteria to the health and well being of not only humans, but also our planet.
85.1) Microorganisms have inhabited the Earth for billions of years, and can be found inhabiting a variety of environments. In fact, microbes performing anoxygenic photosynthesis led to the oxygenation of early Earth’s atmosphere. These ancient organisms were .
Version 1
26
A) B) C) D)
bacteria eukaryotes viruses prions
85.2) The RN applies therapeutic communication techniques to assess of Ms. Smith’s understanding of the principle of sterility. Her statement of “I need to sterilize myself and my environment of all germs so I do not get sick” would be best followed by which of the following questions by the RN? A) B) C) D)
How does being in an unsterile environment make you feel? Can you tell me more about what sterility means to you? How does washing your hands with bleach make you sterile? Can you tell me more about why you are afraid of germs?
85.3) As Ms. Smith progresses with her plan of care, the RN provides education regarding beneficial applications of microbes. Scientists use microbes to produce drugs, hormones, and enzymes. This type of biotechnology involves the transfer of foreign genetic material into a microbe, a process called .
A) B) C) D)
Version 1
recombinant DNA technology gene therapy bioremediation polymerase chain reaction
27
86) NCLEX Prep - Test Bank Question: Please read the clinical scenario, and then answer the questions that follow to become familiar with the traditional NCLEX question format. Wanda is a medical assistant and the newest employee of your healthcare team. You notice that she does not wash her hands in between patient visits. From your microbiology background, you understand that microbes are not visible with the naked eye. As the only nurse in your small medical office, you provide education for Wanda on the importance of hand washing.
86.1) Many microbes that inhabit the skin have the potential to cause disease. One such pathogen is Staphylococcus aureus. The genus name of this organism is most properly represented as .
A) B) C) D)
aureus Staphylococcus staphylococcus Aureus
86.2) Wanda is receptive to the nurse’s teaching. In reinforcing the prevalence of microbes in our environment, the nurse describes the experiments of Louis Pasteur. Pasteur hypothesized that microbes were in the air and dust. Through experiments using swannecked flasks, he disproved the concept of . A) B) C) D)
spontaneous mutation spontaneous generation aseptic theory biogenesis
86.3) Hand washing in the healthcare environment is aimed at reducing the number of microbes in the medical setting to prevent the spread of infection and disease. Which of these terms best represents this technique?
Version 1
28
A) B) C) D)
Sterilization Asepsis Disinfection Antisepsis
86.4) Viruses may also be transmitted to patients, even though they differ from bacteria in that they are .
A) B) C) D)
parasitic invertebrate animals infectious proteins metabolically active eukaryotes noncellular particles
87) NCLEX Prep - Test Bank Question: Please read the clinical scenario, and then answer the questions that follow to become familiar with the traditional NCLEX question format. Breonna Jones is 16 years old, 5’4”, and weighs 93 lb. She was admitted to an inpatient medical unit 2 days ago after collapsing at the local high school. Her parents knew she was skinny and had lost weight in the past few months, but had no idea that her life was in danger. The medical team has instituted treatment for anorexia nervosa. As you develop Breonna’s nursing plan of care, you take into consideration the four major biological molecules that are building blocks of all cells.
87.1) The nurse implements an extensive nutrition education plan for Breonna, beginning at the molecular level. Carbohydrates, lipids, proteins, and nucleic acids are the four main families of biological molecules referred to as . A) B) C) D)
Version 1
macromolecules monosaccharides polysaccharides micromolecules
29
87.2) The structure of proteins is complex and unique, and only specific molecules can interact with their surface features. The natural shape of each protein is termed the native state. When proteins are exposed to heat, acid, or alcohol, their shape is disrupted and they become nonfunctional or .
A) B) C) D)
digested denatured distorted depolymerized
87.3) You inform Breonna that it is important for her to maintain a diet rich in carbohydrates, lipids, and proteins, so thateach of these macromolecules can be metabolized to form a high-energy compound called .
A) B) C) D)
88)
cGMP RNA ATP NAD
A scientist studying helminths is working with bacteria. ⊚ ⊚
true false
89) Current evidence indicates that bacteria and archaea existed on earth for approximately 2 billion years before eukaryotes appeared. ⊚ true ⊚ false
Version 1
30
90) A scientific theory, like the theory of evolution, is just our best guess at explaining a scientific phenomenon, but a theory cannot be considered fact. ⊚ true ⊚ false
91)
Many chronic conditions are found to be associated with microbial agents. ⊚ ⊚
true false
92)
All microorganisms are considered pathogens. ⊚ true ⊚ false
93)
The term sterile means free of all life forms. ⊚ true ⊚ false
94) Members of the same species share many more characteristics compared to those shared by members of the same kingdom. ⊚ ⊚
true false
95) Once an organism is assigned to a particular taxonomic hierarchy, it is permanent and cannot be revised.
Version 1
31
⊚ ⊚
true false
96)
Viruses are not classified in any of Whittaker's five kingdoms. ⊚ true ⊚ false
97)
The names of the three domains are: Bacteria, Protista, and Eukarya (Eukaryota). ⊚ true ⊚ false
98) One distinguishing characteristic of the archaebacteria is that they live in extreme environments. ⊚ ⊚
99)
Microbes have been found existing in salty, acidic lakes. ⊚ ⊚
100)
true false
true false
All proteins are enzymes. ⊚ ⊚
true false
101) The most important outcome of polypeptide intrachain bonding and folding is the unique shape of the protein.
Version 1
32
⊚ ⊚
102)
Nucleic acids have primary, secondary, tertiary, and quaternary levels of organization. ⊚ ⊚
103)
true false
true false
If a hypothesis is accepted, then the findings become a scientific law. ⊚ ⊚
true false
104) The acceptance or rejection of a hypothesis is based on a series of educated guesses and opinions. Once the opinion is widely accepted it becomes a theory. ⊚ ⊚
true false
105) Despite the lack of a membrane-bound nucleus, bacteria and archaea are cells with a complex organizational structure. ⊚ ⊚
true false
106) Most microorganisms that are found in and on humans do not cause harm and can sometimes benefit the host. ⊚ ⊚
Version 1
true false
33
Version 1
34
Answer Key Test name: Ch 1 4e 1) [A, C, D, E, F, G, H] 2) [A, B, C] 3) [A, C, E] 4) [A, D, E] 5) [A, D, E, F] 6) [A, B, D, E] 7) [A, C, D, E] 8) [A, C, D] 9) [A, C, E] 10) B 11) A 12) D 13) D 14) D 15) A 16) B 17) C 18) C 19) B 20) C 21) E 22) E 23) D 24) A 25) A 26) B Version 1
35
27) B 28) A 29) C 30) C 31) A 32) E 33) B 34) A 35) C 36) B 37) B 38) C 39) E 40) D 41) C 42) A 43) C 44) A 45) A 46) C 47) B 48) E 49) B 50) A 51) D 52) E 53) C 54) E 55) E 56) B Version 1
36
57) C 58) B 59) D 60) C 61) D 62) C 63) C 64) A 65) B 66) B 67) D 68) A 69) D 70) D 71) A 72) A 73) D 74) B 75) A 76) A 77) A 78) D 79) B 80) A 81) D 82) A 83) A 84) D 85) Section Break 85.1) A Version 1
37
85.2) B 85.3) A 86) Section Break 86.1) B 86.2) B 86.3) B 86.4) D 87) Section Break 87.1) A 87.2) B 87.3) C 88) FALSE 89) FALSE 90) FALSE 91) TRUE 92) FALSE 93) TRUE 94) TRUE 95) FALSE 96) TRUE 97) FALSE 98) TRUE 99) TRUE 100) FALSE 101) TRUE 102) FALSE 103) FALSE 104) FALSE 105) TRUE 106) TRUE Version 1
38
Version 1
39
CHAPTER 2 1)
Select the correct answers that identify the Five I's of culturing microorganisms. A) Identification B) Inspection C) Inhibition D) Inoculation E) Isolation F) Infection G) Incubation
2)
Select the methods below that enable the isolation of bacteria.
A) Quadrant streak plate B) Bright-field microscopy C) Loop dilution D) Use of selective media E) Spread plate F) Gram stain
3)
Choose the three basic ways that media are classified.
A) Chemical composition B) Purpose C) Temperature of incubation D) Physical state E) Ease of preparation
4)
Select the elements necessary for good microscopy.
Version 1
1
A) Adequate magnification B) Ability to observe cells in the living state C) Use of wavelengths other than the visible spectrum D) Contrast E) Resolution F) Specimen holder
5)
The Five I's of studying microorganisms include all of the following except
.
A) inoculation B) incubation C) infection D) isolation E) identification
6) The term that refers to the purposeful addition of microorganisms into a laboratory nutrient medium is .
A) isolation B) inoculation C) immunization D) infection E) contamination
7)
A pure culture contains
Version 1
.
2
A) only one species of microorganism B) only bacteria C) a variety of microbes from one source D) a variety of species from the same genus E) None of the choices are correct.
8) The correct microbiological term for the tiny sample of specimen that is put into a nutrient medium in order to produce a culture is the .
A) colony B) inoculum C) streak D) loop E) incubator
9)
Which of the following is essential for development of discrete, isolated colonies?
A) Broth medium B) Differential medium C) Selective medium D) Solid medium E) Assay medium
10) Which method often results in colonies developing down throughout the agar along with some colonies on the surface?
Version 1
3
A) Streak plate B) Spread plate C) Pour plate D) All of the choicesare correct. E) None of the choices are correct.
11)
What type of isolation technique is most effective for the majority of applications?
A) Pour plate B) Streak plate C) Spread plate D) Loop dilution E) Culture plate
12) Which of the following will result when 1% to 5% agar is added to nutrient broth, boiled, and cooled? A) A pure culture B) A mixed culture C) A solid medium D) A liquid medium E) A contaminated medium
13)
Agar is an important component of media because
Version 1
.
4
A) bacteria require agar to grow B) agar inhibits mold growth C) agar provides a solid surface for bacterial growth D) agar prevents contamination E) All of the choices are correct.
14)
The three physical forms of laboratory media are
.
A) solid, liquid, and gas B) solid, semisolid, and liquid C) streak plate, pour plate, and broth D) aerobic, anaerobic, and micro aerobic E) None of the choices are correct.
15)
Which of the following is not an inoculating tool?
A) Petri dish B) Loop C) Needle D) Pipette E) Swab
16)
Agar is a complex polysaccharide that comes from a(n)
.
A) green plant B) fungus C) mold D) algae E) euglena
Version 1
5
17)
Which of the following is not a benefit of agar as a solid medium?
A) Has flexibility B) Holds moisture C) Can be inoculated and poured at a temperature that is not harmful D) Is solid at room temperature E) Is digested by most microbes
18) A nutrient medium that has all of its chemical components identified, and their precise concentrations known and reproducible, would be termed . A) a complex media B) a reducing media C) an enriched media D) a chemically defined media E) None of the choices are correct.
19) A nutrient medium that contains at least one ingredient that is NOT chemically definable would be termed .
A) complex B) reducing C) enriched D) synthetic E) minimal
20)
All of the following are examples of different types of microbiological media except .
Version 1
6
A) broth B) enriched C) agar D) petri dish E) selective
21) Which type of media would be the best choice when shipping a sample of bacteria to a laboratory to be tested from a satellite office site?
A) Transport B) EMB C) Blood D) Thioglycollate E) General purpose
22) A microbiologist inoculates Staphylococcus epidermidis and Escherichia coli into a culture medium. Following incubation, only the E. coli grows in the culture. What is the most likely explanation?
A) Themicrobiologist used too much inoculum. B) The culture iscontaminated. C) The incubationtemperature was incorrect. D) The culturemedium must be selective. E) The culturemedium must be differential.
23)
A common medium used for growing fastidious bacteria is
Version 1
.
7
A) blood agar B) trypticase soy agar C) mannitol salt agar D) MacConkey medium E) a reducing medium
24)
A reducing medium contains
.
A) sugars that can be fermented B) extra oxygen C) hemoglobin, vitamins, or other growth factors D) substances that remove oxygen E) inhibiting agents
25) Which type of medium is able to distinguish different species or types of microorganisms based on an observable change in the colonies or in the medium?
A) Differential B) Selective C) Enumeration D) Enriched E) Reducing
26)
Differential media results in which of the following growth characteristics?
Version 1
8
A) Different color colonies B) Different media color post incubation C) Precipitates D) Gas bubbles E) All of the choices are correct.
27)
A reducing media is used to culture
.
A) fastidious organisms B) aerobic organisms C) anaerobic organisms D) any pathogenic organisms E) None of the choices are correct.
28)
For which bacterial genus does mannitol salt agar differentiate between species?
A) Salmonella B) Streptococcus C) Neisseria D) Staphylococcus E) Escherichia
29) A microbiologist must culture a patient's feces for intestinal pathogens. Which of the following would likely be present in selective media for analyzing this fecal specimen?
Version 1
9
A) NaCl B) Sheep red blood cells C) Bile salts D) Thioglycollic acid E) Peptone
30)
Bacteria that require special growth factors and complex nutrients are termed
.
A) aerobic B) anaerobic C) fastidious D) microaerophilic E) autotrophic
31) A microbiologist inoculates Staphylococcus aureus into a culture medium. Following incubation, both Staphylococcus aureus and Staphylococcus epidermidis are determined to be growing in this culture. What is the most likely explanation?
A) Themicrobiologist used too much inoculum. B) The culture iscontaminated. C) The incubationtemperature was incorrect. D) The culturemedium must be selective. E) The culturemedium must be differential.
32)
Newly inoculated cultures must be
Version 1
at a specific temperature to encourage growth.
10
A) streaked B) poured C) incubated D) contaminated E) All of thechoices are correct.
33) A rod-shaped bacterium is measured as 0.3 micrometers (μm) in length using an ocular micrometer. Your instructor wants you to report the length in millimeters (mm) to test your understanding of metric conversions. What is the length of the organism in millimeters? A) 0.0003 mm B) 300 mm C) 0.03 mm D) 3 mm
34) An enveloped virus measures 0.02 micrometers (μm) in diameter. What is the diameter of this virus in nanometers (nm)? A) 20 nm B) 0.00002 nm C) 2 nm D) 0.2 nm
35)
The
of the microscope holds and allows selection of the objective lenses.
A) stage B) condenser C) objective D) ocular E) nosepiece
Version 1
11
36)
Which of the following magnifies the specimen to produce its real image? A) Condenser B) Objective lens C) Ocular lens D) Body E) Nosepiece
37)
Which of the following magnifies the specimen to produce its virtual image? A) Objective lens B) Ocular lens C) Condenser D) Body E) Iris diaphragm
38)
Which of the following controls the amount of light entering the specimen?
A) Objective lens B) Ocular lens C) Condenser D) Body E) Iris diaphragm
39) If a microbiologist is studying a specimen at a total magnification of 950x, what is the magnifying power of the objective lens if the ocular lens is 10x?
Version 1
12
A) 100x B) 950x C) 85x D) 850x E) 95x
40) Magnification is achieved in a compound microscope through the initial magnification of the specimen by the lens. This image is then projected to the lens that will further magnify the specimen to form a virtual image received by the eye.
A) ocular; objective B) scanning; objective C) objective; ocular D) ocular; oil E) None of the choices are correct.
41) Which of the following characteristics refers to the microscope's ability to show two separate entities as separate and distinct?
A) Resolving power B) Magnification C) Refraction D) All of thechoices are correct. E) None of the choices are correct.
42) All of the following are diameters of cells that would be resolved in a microscope with a 0.2 µm limit of resolution except .
Version 1
13
A) 0.2 µm B) 0.2 mm C) 0.1 µm D) 0.3 µm E) 2.0 µm
43) The type of microscope in which you would see brightly illuminated specimens against a black background is .
A) bright field B) dark field C) phase contrast D) fluorescence E) electron
44) Which type of microscope shows cells against a bright background but also differentiates intracellular structures of unstained cells based on their varying densities? A) Bright field B) Dark field C) Phase contrast D) Differential interference E) Electron
45) Which type of microscope is the most widely used and shows cells against a bright background?
Version 1
14
A) Bright field B) Dark field C) Phase contrast D) Fluorescence E) Electron
46)
All of the following pertain to the fluorescence microscope except
.
A) it uses electrons to produce a specimen image B) it is a type of compound microscope C) it requires the use of dyes like acridine and fluorescein D) it is commonly used to diagnose certain infections E) it requires an ultraviolet radiation source
47)
A confocal scanning microscope
.
A) uses visible light to form a specimen image B) shows three-dimensional cell images from the cell surface to the middle of the cell C) produces specimen images on electron micrographs D) uses dyes that emit visible light when bombarded by electrons E) requires specimens to be stained
48)
Which type of microscope does not use light in forming the specimen image?
A) Bright field B) Dark field C) Phase contrast D) Fluorescence E) Electron
Version 1
15
49)
Which type of microscope achieves the greatest resolution and highest magnification?
A) Bright field B) Dark field C) Phase contrast D) Fluorescence E) Electron
50) Which type of microscope bombards a whole, metal-coated specimen with electrons moving back and forth over it?
A) Differential interference contrast B) Scanning electron C) Transmission electron D) Phase contrast E) Fluorescence
51)
The specimen preparation that is best for viewing cell motility is
.
A) hanging drop B) fixed stained smear C) Gram stain D) negative stain E) flagellar stain
52)
The purpose of staining cells on a microscope slide is to
Version 1
.
16
A) kill them B) secure them to the slide C) enlarge the cells D) add contrast in order to see them better E) see motility
53)
What do the Gram stain, acid-fast stain, and endospore stain have in common?
A) They are used on a wet mount of the specimen. B) They use heat to force the dye into cell structures. C) The outcome is based on cell differences. D) They use a negative stain technique. E) They are simple stains.
54)
Basic dyes are
.
A) attracted to the negatively charged acidic substances of bacterial cells B) anionic C) used in negative staining D) repelled by cells E) dyes such as India ink and nigrosin
55) A microbiologist makes a fixed smear of bacterial cells and stains them with Loeffler's methylene blue. All the cells appear blue under the oil lens. This is an example of .
Version 1
17
A) negative staining B) using an acidic dye C) simple staining D) using the acid-fast stain E) capsule staining
56)
The Gram staining procedure is best described as a(n)
staining technique.
A) acid-fast or Ziehl-Neelson B) differential C) capsule D) flagellar E) simple
57) The all-purpose media Tryptic Soy Agar (TSA) contains the following four ingredients; pancreatic digest of casein, digest of soybean meal, NaCl and agar. TSA is best described as .
A) a differential media B) a complex media C) a selective media D) a defined media E) a reducing media
58)
Comparing defined vs. complex media is analogous to comparing
Version 1
.
18
A) baby formula to breast milk B) Coca-cola ® to Pepsi ® C) the generic version of the drug Lipitor ® to the brand name produced by Pfizer Pharmaceuticals D) a strawberry-banana smoothie to a mixed berry smoothie
59) The diameter of field for a 4x lens is measured at 4.6 mm. How many bacterial cells, each measuring 4 μm, could be lined up along the diameter?
A) 1,150 cells B) 1.15 cells C) 18.4 cells D) 115 cells E) 1840 cells
60)
The relationship between a micrometer and a millimeter is
.
A) a millimeter is 10 times larger than a micrometer B) a micrometer is 1,000 times smaller than a millimeter C) a millimeter is 100 times smaller than a micrometer D) a micrometer is 1,000 times larger than a millimeter E) a millimeter is 10 times smaller than a micrometer
61)
Resolution differs from contrast in microscopy in that
Version 1
.
19
A) resolution refers to the ability to distinguish two cells from one another, whereas contrast refers to the ability to distinguish a cell from its surroundings B) contrast refers to the ability to distinguish two cells from one another, whereas resolution refers to the ability to distinguish a cell from its surroundings C) resolution is measured by the refractive index of light whereas contrast depends on the use of oil D) contrast is improved by adding oil to the specimen with the 100x objective lens and resolution is improved by adjusting the iris diaphragm control lever
62)
Choose the media categorization that includes enriched, selective, and differential media.
A) B) C) D)
Media categorized by its physical state Media categorized by its chemical composition Media categorized by its purpose Media categorized by its temperature of incubation
63) What type of medium contains blood, serum, or other special growth factors, and what type of bacterial growth is supported by this medium? A) B) C) D)
64)
Selective; gram-negative bacteria Differential; fastidious bacteria Enriched; fastidious bacteria Enriched; gram-negative bacteria
Choose the term that describes motility media.
Version 1
20
A) Solid, but can liquefy B) Solid C) Semisolid D) Liquid E) Synthetic
65) Choose the description that best fits a medium that has been designed to support the growth of MRSA (methicillin-resistant Staphylococcus aureus) while inhibiting all species and strains of other bacteria. A) Differential B) Selective C) Reducing D) Enriched E) Anaerobic
66) A bacterial cell that measures 2.5 μm (micrometers) would measure (nanometers).
nm
A) 2,500 nm B) 25 nm C) 0.0025 nm D) 0.25 nm
67) A virus that is 150 nm (nanometers) in diameter would be diameter.
Version 1
μm (micrometers) in
21
A) 0.150 μm B) 15 μm C) 150,000 μm D) 0.0150 μm
68) A human squamous epithelial cell measures 75 µm in diameter. The size of this cell represented in millimeters (mm) would be mm. A) 0.075 B) 0.75 C) 75,000 D) 750
69) Reducing the intensity of light using the iris diaphragm or staining a specimen can improve . A) B) C) D)
contrast magnification resolving power reflection
70) The resolving power of electron microscopes is much better than the resolving power of light microscopes because the wavelength of electron beams is than the wavelength of visible light. A) B) C) D)
Version 1
shorter longer faster slower
22
71) In a organism. A) B) C) D)
72)
stain, the dye stains the background, forming a silhouette around the unstained
negative differential positive fluorescent
Choose the differential stain that is most important in the diagnosis of tuberculosis.
A) Acid-faststain B) India inkstain C) Gram stain D) Endosporestain E) Flagellarstain
73) NCLEX Prep - Test Bank Question: Please read the clinical scenario, and then answer the questions that follow to become familiar with the traditional NCLEX question format. An RN is working at an urban low-income medical clinic when a young woman enters crying. She is 19 years old and 28 weeks pregnant with her second child. The woman reports that she woke this morning to find she was leaking milky-colored fluid vaginally. Her first child was born 6 weeks early due to premature rupture of membranes and she is worried this is happening again. You reassure the patient, explain that a vaginal speculum exam will be performed, and educate her about specimens that will be collected. Once the proper specimens are obtained and appropriately labeled, the wet mount and culturette are sent to the laboratory for processing.
Version 1
23
73.1) The RN understands that along with a pH test, a microscopic view is needed to perform the ferning test to detect an amniotic fluid leak. The patient sample is prepared on a glass slide and examined under 10x magnification. Which type of microscope will be used to make this observation of the patient sample? A) B) C) D)
Electron microscope Light microscope Confocal microscope Fluorescent microscope
73.2) When utilizing a light microscope, the specimen on the glass slide must be in proper position to ensure illumination of the specimen for visualization. The glass slide is placed in which of the following positions?
A) B) C) D)
Between the condenser lens and the objective lens Directly on top of the light source Between the ocular lens and the objective lens Between the light source and the condenser lens
73.3) The sterile vaginal fluid specimen is sent for culture. The RN educates the patient about the five basic techniques utilized by laboratory technicians to manipulate, grow, examine, and characterize any microorganisms present in the specimen. Which of the following is the correct order of steps for processing the specimen? A) B) C) D)
Isolation, incubation, inspection, identification, and inoculation Inspection, identification, isolation, incubation, and inoculation Identification, isolation, incubation, inspection, and inoculation Inoculation, incubation, isolation, inspection, and identification
73.4) The patient asks how microbes from her body can be grown in the lab. The RN explains that specimens are introduced to nutrient medium and that any growth of the microbe that appears after incubating the specimen is called the .
Version 1
24
A) B) C) D)
colony culture microorganism infectious agent
73.5) The patient is diagnosed with a bacterial infection after gram-positive cocci are detected in the fluid sample. The Gram stain involves .
A) forcing a dye into resistant bodies with heat to distinguish between spores and cells B) timed, sequential applications of crystal violet dye, iodine, an alcohol rinse, and a contrasting counterstain to the sample C) application of the dye, carbol fuchsin, followed by an acid alcohol rinse D) application of India ink to detect the presence of bacterial capsules
74) NCLEX Prep - Test Bank Question: Please read the clinical scenario, and then answer the questions that follow to become familiar with the traditional NCLEX question format. An RN is working at a public health clinic that sees many patients with infectious disease. Ms. Hungh, a Burmese immigrant, presents to the clinic with an interpreter, complaining of fatigue, weight loss, persistent cough, and rust-colored sputum. The interpreter explains that Ms. Hungh has had this cough for many months in her home country and, now that she is in America,is seeking treatment for her condition.
74.1) A sputum sample is ordered for microbial analysis in order to rule out the diagnosis of tuberculosis. Suspecting Mycobacterium tuberculosis may be the pathogen, the RN knows the laboratory technicians will perform which stain on the sample?
Version 1
25
A) B) C) D)
Endospore stain Negative stain Flagellar stain Acid-fast stain
74.2) Ms. Hungh’s acid-fast stain results resulted as inconclusive for the presence of acid-fast bacilli. Culturing of the sputum is performed in order to isolate microbial growth for further analysis. Lowenstein-Jensen medium is utilized to select for the growth of Mycobacterium species if present in the sample, while suppressing unwanted background organisms. What is the proper term for this type of medium?
A) B) C) D)
Differential medium Selective medium Differential medium and selective medium None of the choices are correct.
74.3) Culturing of the sputum resulted in the growth of distinct colonies and further isolation by subculturing is now needed. The RN understands that isolation is accomplished by taking a bit of growth from an individual colony and inoculating a separate medium, resulting in the production of a(n) .
A) B) C) D)
Version 1
simple culture pure culture isolated culture mixed culture
26
75) NCLEX Prep - Test Bank Question: Please read the clinical scenario, and then answer the questions that follow to become familiar with the traditional NCLEX question format. A 65-year-old homeless male presents to an urgent care clinic with a deep laceration on his left arm. He states that he cut his arm on an old piece of scrap metal two days prior. His wound is red, tender, hot to the touch, and has yellow drainage. The RN collects a sample of the drainage and sends it to the laboratory for microbial analysis, per provider orders.
75.1) The patient’s culture results positive for Staphylococcus. The RN understands that the culture most likely required growth on a complex medium, consisting of
A) B) C) D)
.
an exact chemical formula chemical growth inhibitors at least one ingredient that is not chemically defined None of the choices are correct.
75.2) Microscopic analysis revealed the presence of grapelike clusters of gram-positive cocci. The RN educates the patient that the species identification of the organism will require biochemical testing, which aids in microbial identification by providing information on cellular metabolism. Which of the following statements by the patient demonstrates understanding of the nurse’s teaching?
A) Biochemical tests can determine the organism’s nutrient requirements. B) Biochemical tests can determine the presence of enzymes in the sample. C) Biochemical tests can provide information about products given off during growth. D) Biochemical tests can provide information about the microbe’s mechanism for deriving energy. E) All of these statements are correct.
Version 1
27
75.3) Cultures and specimens pose a potential health hazard and require proper handling and disposal via specific medical waste policies. Some facilities are regulated to maintain living catalogs of specimens that may be subcultured into a fresh medium for research and educational purposes. Such collections are referred to as .
A) B) C) D)
76)
The procedures for culturing a microorganism require the use of a microscope. ⊚ ⊚
77)
live microbes stock cultures reserved specimens bacteriological reserve
true false
Some microbes are not capable of growing on artificial media. ⊚ ⊚
true false
78) A selective medium contains one or more substances that inhibit growth of certain microbes in order to facilitate the growth of other microbes. ⊚ ⊚
79)
true false
One colony typically develops from the growth of several parent bacterial cells. ⊚ ⊚
Version 1
true false
28
80)
Mixed cultures are also referred to as contaminated cultures. ⊚ ⊚
81)
true false
Bacterial cultures are easily identified from their microscopic appearance. ⊚ ⊚
true false
82)
Normal incubation temperatures range from 30°C to 60°C. ⊚ true ⊚ false
83)
The bending of light rays as they pass from one medium to another is called refraction. ⊚ true ⊚ false
84)
The real image is the reverse of the actual specimen. ⊚ ⊚
85)
true false
A confocal microscope uses visible light as its source of illumination. ⊚ ⊚
Version 1
true false
29
86) Fixed smears of specimens are required in order to perform the Gram stain and endospore stain on the specimens. ⊚ ⊚
87)
true false
At the end of the Gram stain, gram-positive bacteria will be seen as red/pink cells. ⊚ true ⊚ false
88) In the absence of immersion oil when using the 100x objective lens, some light would be lost to scatter resulting in a blurry image. ⊚ ⊚
Version 1
true false
30
Answer Key Test name: Ch 2 4e 1) [A, B, D, E, G] 2) [A, C, D, E] 3) [A, B, D] 4) [A, D, E] 5) C 6) B 7) A 8) B 9) D 10) C 11) B 12) C 13) C 14) B 15) A 16) D 17) E 18) D 19) A 20) D 21) A 22) D 23) A 24) D 25) A 26) E Version 1
31
27) C 28) D 29) C 30) C 31) B 32) C 33) A 34) A 35) E 36) B 37) B 38) E 39) E 40) C 41) A 42) C 43) B 44) C 45) A 46) A 47) B 48) E 49) E 50) B 51) A 52) D 53) C 54) A 55) C 56) B Version 1
32
57) B 58) A 59) A 60) B 61) A 62) C 63) C 64) C 65) B 66) A 67) A 68) A 69) A 70) A 71) A 72) A 73) Section Break 73.1) B 73.2) A 73.3) D 73.4) B 73.5) B 74) Section Break 74.1) D 74.2) B 74.3) B 75) Section Break 75.1) C 75.2) C 75.3) B Version 1
33
76) FALSE 77) TRUE 78) TRUE 79) FALSE 80) FALSE 81) FALSE 82) FALSE 83) TRUE 84) FALSE 85) FALSE 86) TRUE 87) FALSE 88) TRUE
Version 1
34
CHAPTER 3 1)
Select characteristics exhibited by all bacteria.
A) Chromosome B) Fimbriae C) Capsule D) Ribosomes E) Cell wall F) Flagella G) Endospores H) Cell membrane
2)
Select the structures that are possessed by some, but not all bacteria.
A) Flagella B) Cell membrane C) Endospores D) Ribosomes E) Pili F) Fimbriae G) Glycocalyx
3)
Select the structures that enable bacteria to be motile.
A) Capsule B) Flagella C) Axial filaments D) Fimbriae E) Glycocalyx F) Pseudopods
Version 1
1
4)
Select those structures that are found in a gram-positive cell envelope. A) Thick layer of peptidoglycan B) Outer membrane C) Cell membrane D) Teichoic acids E) Lipopolysaccharide F) Lipoteichoic acids G) Thin layer of peptidoglycan
5)
Select those structures that are found in a gram-negative cell envelope.
A) Thick layer of peptidoglycan B) Outer membrane C) Cell membrane D) Teichoic acids E) Lipopolysaccharide F) Lipoteichoic acids G) Thin layer of peptidoglycan H) Periplasmic space
6)
Select the items that would be found in bacterial cytoplasm.
Version 1
2
A) Water B) Nucleus C) Mitochondria D) Ribosomes E) Plasmids F) Bacterial chromosome G) Bacterial endospores H) Inclusion bodies I) Enzymes
7)
Select the terms which describe various members of the archaea.
A) Eukaryotes B) Hyperthermophiles C) Methanogens D) Acidophiles E) Extreme halophiles F) Extremophiles
8)
Which of the following is not a characteristic of bacteria? A) Its DNA is notencased in a membrane. B) It has a cellwall made of peptidoglycans or other distinct chemicals. C) It does not havemembrane-bound organelles. D) Its DNA iswrapped around histones. E) All of these are characteristics of bacteria.
9)
The two functions of bacterial appendages are
Version 1
.
3
A) attachment and protection B) attachment and motility C) motility and slime production D) energy reactions and synthesis E) protection and motility
10)
Bacterial cells could have any of the following appendages except
.
A) flagella B) cilia C) fimbriae D) periplasmic flagella (axial filaments) E) sex pili
11)
Spirochetes have a twisting and flexing locomotion due to appendages called
.
A) flagella B) cilia C) fimbriae D) periplasmic flagella (axial filaments) E) sex pili
12)
A flagellum is anchored into the bacterial cell envelope by its
.
A) hook B) outer membrane C) filament D) sheath E) basal body
Version 1
4
13)
The term that refers to the presence of flagella over the cell surface is
.
A) amphitrichous B) atrichous C) lophotrichous D) monotrichous E) peritrichous
14)
The term that refers to the presence of a tuft of flagella emerging from the same site is .
A) amphitrichous B) atrichous C) lophotrichous D) monotrichous E) peritrichous
15)
The term that refers to flagella at both poles is
.
A) amphitrichous B) atrichous C) lophotrichous D) monotrichous E) peritrichous
16)
Chemotaxis refers to the ability to
Version 1
.
5
A) move in response to light B) move in response to a chemical C) not move in response to a chemical D) transport desired molecules into cells E) None of the choices is correct.
17)
A bacterial cell exhibiting chemotaxis probably has
.
A) fimbriae B) a capsule C) thylakoids D) flagella E) metachromatic granules
18) What three components comprise the flagellum, extending from the cytoplasm to the exterior of the cell? A) Filament, hook, basal body B) Filament, basal body, hook C) Basal body, hook, filament D) Hook, basal body, filament E) Basal body, filament, hook
19)
Movement of a cell toward a chemical stimulus is termed
Version 1
.
6
A) positive phototaxis B) negative phototaxis C) positive chemotaxis D) negative chemotaxis E) a tumble
20) The short, numerous appendages used by some bacterial cells for adhering to surfaces are called .
A) flagella B) cilia C) fimbriae D) periplasmic flagella (axial filaments) E) sex pili
21) The transfer of genes during bacterial conjugation involves rigid, tubular appendages called .
A) flagella B) cilia C) fimbriae D) periplasmic flagella (axial filaments) E) sex pili
22)
Which structure protects bacteria from being phagocytized by white blood cells?
Version 1
7
A) Slime layer B) Fimbriae C) Cell membrane D) Capsule E) All of the choices are correct.
23)
The outcome of the Gram stain is based on differences in the cell's
.
A) ribosomes B) inclusions C) wall D) membrane E) flagella
24)
Which order below reflects the correct procedure for Gram staining?
A) Alcohol/acetone–crystal violet–safranin–iodine B) Crystal violet–alcohol/acetone–iodine–safranin C) Crystal violet–iodine–alcohol/acetone–safranin D) Iodine–safranin–crystal violet–alcohol/acetone E) Alcohol/acetone–safranin–crystal violet–iodine
25) The cell can be composed of three layers: the cytoplasmic membrane, the cell wall, and the outer membrane.
Version 1
8
A) glycocalyx B) envelope C) pathogenicpackage D) slime coat E) None of thechoices is correct.
26)
During the Gram stain,
cells decolorize when the alcohol is applied.
A) gram-positive B) gram-negative C) both gram-positive and -negative
27) If bacteria living in salty seawater were displaced to a freshwater environment, the cell structure that would prevent the cells from rupturing is the .
A) endospore B) cell wall C) cell membrane D) capsule E) slime layer
28)
Peptidoglycan is a unique macromolecule found in bacterial
.
A) cell walls B) cell membranes C) capsules D) slime layers E) inclusions
Version 1
9
29) A bacterial cell wall that has primarily peptidoglycan with small amounts of teichoic acid and lipoteichoic acid is . A) gram-negative B) gram-positive C) found in archaea D) a spheroplast E) acid fast
30)
A bacterial genus that has waxy mycolic acid in the cell walls is
.
A) Mycobacterium B) Mycoplasma C) Streptococcus D) Corynebacterium E) Salmonella
31) The difference in cell wall structure of Mycobacterium and Nocardia compared to the typical gram-positive bacterial cell wall structure is that they .
A) contain more peptidoglycan B) have a predominance of unique, waxy lipids C) are easily decolorized D) contain lipopolysaccharide E) All of the choices are correct.
32) The other bacteria.
Version 1
stain is used to stain and differentiate Mycobacterium and Nocardia from
10
A) acid-fast B) methyleneblue C) Gram D) negative E) basic
33) The enzyme , found in tears and saliva, can hydrolyze the bonds in the glycan chains of certain bacterial cell walls. A) penicillinase B) lysozyme C) peptidase D) All of thechoices are correct. E) None of the choices are correct.
34)
Lysozyme is most effective against
.
A) gram-negative organisms B) gram-positive organisms C) mycoplasmas D) cyanobacteria E) archaea
35) All of the following structures contribute to the ability of pathogenic bacteria to cause disease except the .
Version 1
11
A) inclusions B) fimbriae C) capsule D) slime layer E) LPS
36)
Which of the following does not pertain to endotoxin? A) It is a specific cell wall lipid B) It can stimulate fever in the human body C) It can cause septic shock in the human body D) It is involved in typhoid fever and some meningitis cases E) It is found in acid-fast bacterial cell walls
37)
The site(s) for most ATP synthesis in bacterial cells is(are) the
.
A) ribosomes B) mitochondria C) cell wall D) inclusions E) cell membrane
38)
Gram-negative bacteria
.
A) are more susceptible to antibiotics that target peptidoglycan than gram-positive organisms B) are less susceptible to antibiotics that target peptidoglycan than gram-positive organisms C) stain purple in the Gram stain D) encompass all pathogens E) None of thechoices is correct.
Version 1
12
39)
Which of the following is not true of the outer membrane?
A) The uppermost layer is made of lipopolysaccharides. B) The innermost layer is a phospholipid bilayer. C) The porinproteins create channels through the outer membrane. D) Gram-positivebacteria have an outer membrane. E) The lipid portionof the lipopolysaccharide layer is an endotoxin.
40)
All bacterial cells have
.
A) one or more chromosomes B) one or more fimbriae C) the ability to produce endospores D) capsules E) flagella
41)
The most immediate result of destruction of a cell's ribosomes would be
.
A) material would not be able to cross the cell membrane B) protein synthesis would stop C) destruction of the cell's DNA D) formation of glycogen inclusions E) loss of the capsule
42)
The bacterial chromosome
Version 1
.
13
A) is located in the cell membrane B) contains all the cell's plasmids C) is part of the nucleoid D) forms a single linear strand of DNA E) All of the choices are correct.
43)
Which of the following is mismatched?
A) Ribosomes - protein synthesis B) Inclusions - excess cell nutrients and materials C) Plasmids - genes essential for growth and metabolism D) Nucleoid - hereditary material E) Cytoplasm - dense, gelatinous solution
44)
Plasmids
.
A) are found in all bacteria B) are essential for growth and metabolism C) cannot be passed between organisms D) cannot be passed on to progeny E) are often the site of pathogenic genes
45)
The chemical components of ribosomes are proteins and
.
A) mRNA B) tRNA C) rRNA D) All of thechoices are correct. E) None of thechoices is correct.
Version 1
14
46)
The function of bacterial endospores is
.
A) to convert gaseous nitrogen to a usable form for plants B) reproduction and growth C) protection of genetic material during harsh conditions D) storage of excess cell materials E) to act as sites for photosynthesis
47) Chemical analysis of a bacterial cell structure detects calcium and dipicolinic acid. What is the identity of this structure? A) Cell wall B) Capsule C) Slime layer D) Nucleoid E) Endospore
48)
Endospores are
.
A) metabolically inactive B) resistant to heat and chemical destruction C) resistant to destruction by radiation D) living structures E) All of the choices are correct.
49)
Bacterial endospores are not produced by
Version 1
.
15
A) Staphylococcus B) Sporosarcina C) Bacillus D) Clostridium E) All of thechoices are correct.
50)
Which term is not used to describe bacterial cell shapes?
A) Coccus B) Tetrad C) Vibrio D) Rod E) Spirochete
51) Cells form a arrangement when cells in a chain snap back upon each other forming a row of cells oriented side by side.
A) tetrad B) strep C) staph D) sarcina E) palisade
52) If you looked at a cluster of spherical cells.
Version 1
under the microscope, you would likely see an irregular
16
A) palisade B) sarcina C) staphylococcus D) streptococcus E) diplococcus
53)
A chain of rod-shaped cells would be called a(an)
.
A) streptobacillus B) staphylobacillus C) streptococcus D) staphylococcus E) sarcina
54) Manual of Systematic Bacteriology is a manual of bacterial descriptions and classifications. A) Pasteur's B) Lister's C) Bergey's D) Leeuwenhoek's E) Koch's
55)
Which of the following is not a phenotypic trait of bacteria?
A) rRNA sequencing B) Shape C) Growth pattern in a culture medium D) Biochemical reaction E) All of these arephenotypic traits.
Version 1
17
56)
Which of the following is mismatched?
A) Gracilicutes -gram-negative cell walls B) Firmicutes -gram-positive cell walls C) Tenericutes - waxy acid-fast cell walls D) Mendosicutes - archaea cell walls E) None of the choices are correct.
57) Which of the following is not a division of bacteria and archaea according to Bergey's Manual of Determinative Bacteriology? A) Gracilicutes B) Scotobacteria C) Firmicutes D) Tenericutes E) Mendosicutes
58)
Which of the following is mismatched?
A) Methanogens – convert CO 2 and H 2 gases into methane B) Extreme halophiles – adapted to salty habitats C) Psychrophiles – adapted to very low temperatures D) Hyperthermophile – adapted to high temperatures E) Thermoplasmas – adapted to frozen environments
59)
Two major structures that allow bacteria to adhere to surfaces are
Version 1
and
.
18
A) pili; ribosomes B) fimbrae;capsules C) lipopolysaccharides;techoic acids D) actin filaments;phospholipid membrane(s) E) actin filaments; ribosomes
60) A client has a serious case of the flu. A random sample of sputum was taken from the patient coughing up blood. The lab tech said they had isolated a bacterium that did not have any peptidoglycan. You hypothesize that the identity of this microbe could possibly be .
A) Mycobacterium tuberculosis B) Borrelia burghdorferi C) Streptococcus pneumoniae D) Mycoplasma pneumoniae E) Staphylococcus aureus
61) Halobacterium salinarium lives in and requires salt. This is an example of an archaeabacterium described as a(n) . A) osmophile B) halophile C) thermophile D) psychrophile E) methanogen
62)
Spirochetes are able to move due to
Version 1
.
19
A) a periplasmic flagellum B) a membrane-bound flagellum C) cilia serving as walking feet D) pseudopods E) fimbriae
63)
Which of the following species of bacteria is not closely related to the others?
A) Staphylococcusaureus B) Staphylococcusepidermidis C) Staphylococcussaprophyticus D) Escherichia coli E) Staphylococcus capitis
64)
Lipopolysaccharide is an important cell envelope component of
.
A) gram-negative bacteria B) gram-positive bacteria C) acid-fast bacteria D) mycoplasmas E) protoplasts
65) Which statement below supports the argument that gram-negative cells are structurally weaker than gram-positive cells?
Version 1
20
A) Gram-negative cells have a thinner layer of peptidoglycan in their cell wall compared to gram-positive cells. B) Gram-positive cells contain teichoic acids, whereas gram-negative cells lack teichoic acids in their cell wall. C) The envelope of gram-negative cells comprises a hydrophobic outer membrane. D) Gram-negative cells contain lipopolysaccharides, the lipid component of which acts as an endotoxin.
66) Which of the following statements is not true regarding the differences between bacteria, archaea, and eukaryotes?
A) The cell walls of bacteria and archaea both contain the complex macromolecule peptidoglycan, whereas only the organisms in kingdom Plantae contain cell walls, which are made of cellulose. B) Bacteria and archaea have circular chromosomes in a nucleoid region whereas eukaryotes have linear chromosomes that are enclosed in a nuclear membrane. C) Flagella are found on bacteria, archaea and eukaryotes yet they all vary in structure. D) Ribosomal RNA sequencing revealed that the structure of the ribosomal subunits in archaea more closely resemble those of eukaryotes than bacteria.
67) The organism Mycoplasma pneumoniae that causes "walking pneumonia" lacks a cell wall. In which of the following taxonomic divisions is it classified?
A) B) C) D)
Version 1
Tenericutes Mendosicutes Gracilicutes Firmicutes
21
68) A 23 year-old female presents at a walk-in clinic with a fever, swollen lymph nodes and yellow spots on the throat. A rapid strep test comes back positive, confirming an infection with Streptococcus pyogenes, which has a thick layer of peptidoglycan in its cell wall. In which of the following taxonomic divisions does S. pyogenes belong?
A) B) C) D)
Firmicutes Gracilicutes Mendosicutes Tenericutes
69) The typical definition of a species is one with distinct characteristics that produce viable offspring when mated with another organism in the same species. Bacterial species cannot be defined as readily, due to the fact that .
A) they do not reproduce sexually by way of gametes B) they can receive genetic material from others in their generation, regardless of species C) they have a short generation time and evolve rapidly D) All of the statements are true regarding the indistinct defining of bacterial species.
70)
Rods that are slightly curved are called A) B) C) D)
.
vibrio spirochetes coccobacilli filaments
71) The presence of variation in the size and shape of bacterial cells of the same species is known as .
Version 1
22
A) B) C) D)
pleomorphism chemotaxis pseudomorphology sarcina
72) Choose the bacterial appendage that enables bacteria to stick to one another, to inanimate surfaces, and to host cells.
A) Pili B) Fimbriae C) Flagella D) Axial filaments E) Cell wall
73) During bacterial conjugation, DNA can be transferred through a to the recipient cell. A) B) C) D)
74)
pilus flagellum axial filament fimbria
The movement of bacteria toward or away from a chemical stimulus is A) B) C) D)
Version 1
from the donor
.
chemotaxis phototaxis peritrichous monotrichous
23
75)
Choose the sentence that best describes how bacteria respond to chemical attractants.
A) Chemical attractants cause bacterial cells to grow additional flagella B) Chemical attractants speed up the rotation rate of flagella C) Chemical attractants inhibit tumbles, enabling bacteria to make faster progress toward the stimulus D) Chemical attractants cause flagella to rotate clockwise E) Chemical attractants stop rotation of flagella
76) Choose the component that provides for a stronger cell wall structure in gram-positive cells as compared to gram-negative cells.
A) Thick peptidoglycan layer B) Lipopolysaccharide C) Outer membrane D) Teichoic acids E) Porin proteins
77) Lipopolysaccharide is a major component of gram-negative outer membranes, can stimulate shock and fever, and is also referred to as . A) B) C) D)
endotoxin exotoxin peptidoglycan teichoic acid
78) Choose the substance or condition that triggers sporogenesis among endospore-forming bacteria.
Version 1
24
A) Lack of water B) Presence of toxic chemicals C) Ultraviolet light exposure D) Excess heat E) Nutrient depletion
79) A unicellular microorganism was recovered from a hot spring (95°C) in Wyoming. The cells lack a nucleus, have a cell wall that lacks peptidoglycan, and have 70S ribosomes.Analysis of the plasma membrane reveals that the lipids contain long-chained branched hydrocarbons with ether linkages. Please choose the group in which this organism would be classified, based on the description provided.
A) Protista B) Archaea C) Bacteria D) Fungi E) Viruses
80) To find listings and charts of biochemical tests that can be used to identify unknown bacteria, you could use Bergey’s Manual of Bacteriology. A) B) C) D)
81)
Determinative Systematic Phenotypic Morphological
Choose the division that includes the archaea (archaebacteria).
Version 1
25
A) Gracilicutes B) Firmicutes C) Temericutes D) Mendosicutes E) Actinobacteria
82) A collection of bacterial cells that shares an overall similar pattern of traits different from other groups of bacteria is defined as a bacterial . A) B) C) D)
species serotype subspecies domain
83) NCLEX Prep - Test Bank Question: Please read the clinical scenario, and then answer the questions that follow to become familiar with the traditional NCLEX question format. A 3-year-old patient presents to the ER after a sudden onset of high fever and chills, a productive cough, and shortness of breath.The parents indicate that the child had been healthy up until that morning.An initial assessment of the child shows an elevated temperature (103oF), rapid pulse rate (140 bpm), and low oxygen saturation level (82%).Supplemental oxygen and acetaminophen are administered immediately, and specimens are obtained from the patient for microbial analysis.
83.1) The specimens are Gram-stained and analyzed microscopically.The laboratory technician observes spherical purple cells arranged in short chains.Based upon this evidence, which of the following microbes is potentially the pathogen affecting the patient?
Version 1
26
A) B) C) D)
Bacillus cereus Vibrio cholerae Staphylococcus aureus Streptococcuspneumoniae
83.2) Further microscopic analysis of negatively stained specimens reveals a colorless ring around each of the spherical cells against a dark background.This clearing indicates that the pathogen possesses which structure(s)? A) B) C) D)
Endospores A capsule A nucleoid Pili
83.3) The RN provides education for the family about the laboratory findings. Which of the following statements correctly describes the medical importance of a bacterial capsule?
A) A capsule is a slime layer that causes a bacterium to attach to body tissues, making it invasive and difficult to treat. B) A capsule allows a bacterium to survive in a dormant state, creating a prolonged course of illness. C) A capsule decreases a bacterium’s pathogenicity and makes it susceptible to antibiotic treatment. D) A capsule can protect a bacterium from the phagocytic activities of white blood cells, enhancing its ability to cause disease.
Version 1
27
84) NCLEX Prep - Test Bank Question: Please read the clinical scenario, and then answer the questions that follow to become familiar with the traditional NCLEX question format. A triage nurse in the ER begins the assessment of an elderly patient complaining of bloody diarrhea and severe stomach cramps.The patient has a fever of 104 oF as well, and is showing signs of dehydration.IV fluids are administered while further workup of the patient continues.
84.1) Specimen collection from the patient is ordered for microbial analysis in the hospital laboratory.Based upon the patient’s symptoms, which of the following samples would the RN expect to be ordered for collection?
A) B) C) D)
Aseptic collection of cerebrospinal fluid Clean catch urine sample Stool sample Aseptic collection of sputum
84.2) Testing is performed on the specimen in the laboratory, including the Gram stain.This test separates bacteria into two broad classes: gram-positive and gramnegative.Which step in the staining process is the differential step, for it only acts upon one type of cell? A) B) C) D)
Addition of Gram’s iodine (mordant) Addition of crystal violet Decolorization with alcohol Addition of safranin (counterstain)
84.3) Gram staining reveals the presence of gram-negative bacilli in the patient’s stool.The RN understands how this bacterium would cause the patient’s spike in temperature.Which of the following is the correct explanation for this patient’s fever?
Version 1
28
A) Gram-negative bacteria have a thick peptidoglycan layer which confers theirability to cause fever. B) Gram-negative bacteria have two periplasmic spaces, which enhance their ability to cause fever. C) Gram-negative bacteria have porin proteins in their outer membrane, which confer their ability to cause fever. D) Gram-negative bacteria have lipopolysaccharide in their outer membrane, and the lipid portion confers their ability to cause fever.
84.4) The provider educates the patient about his condition, based on information from the laboratory findings and clinical presentation.Later, the RN reinforces the teaching and explains why this infection may be difficult to treat with antibiotics.Which part of the cell envelope restricts the passage of many of drugs into gram-negative bacteria?
A) B) C) D)
Peptidoglycan Cell membrane Outer membrane layer Lipopolysaccharides
85) NCLEX Prep - Test Bank Question: Please read the clinical scenario, and then answer the questions that follow to become familiar with the traditional NCLEX question format. An RN in a primary care office is caring for a 15-year-old male with severe gastrointestinal distress.The patient reports four days of diarrhea, vomiting, and intermittent fever. A stool culture was ordered in an attempt to identify a disease-causing pathogen.
85.1) Microscopic analysis revealed the presence of gram-negative bacilli in the patient’s stool.Based upon this information, which of the following accurately describes the appearance of the bacterium under magnification?
Version 1
29
A) B) C) D)
Red spheres Purple rods Purple spheres Red rods
85.2) The laboratory report shows that a motility test was performed on the sample with positive results.Thinking about bacterial movement, the RN is aware that this bacterium could possess which structure(s) for motility? A) B) C) D)
Endospores Pili One or more flagella Pseudopodia
85.3) The laboratory report concludes that the diagnosis is salmonellosis caused by Salmonella infection.Remembering resources available for prokaryotic identification, the RN speculates that the laboratory technicians may have used .
A) Bergey’s Manual of Systematic Bacteriology because rRNA analysis was required in the identification of this pathogen B) Bergey’s Manual of Systematic Bacteriology because its five volumes of information were necessary for phenotypic identification of this pathogen C) Bergey’s Manual of Determinative Bacteriology because they needed to fully understand the evolutionary background of the pathogen D) Bergey’s Manual of Determinative Bacteriology because it classifies prokaryotes based on phenotypic traits typically tested for in clinical settings
85.4) The patient is diagnosed with Salmonella enterica serovar Enteritidis infection, a foodborne illness.The identification of the bacterial subspecies is important for developing his treatment plan and was possible due to .
Version 1
30
A) B) C) D)
86)
the analysis of antibody reactions against the pathogen the analysis of bacterial growth on a solid agar culture the analysis of the pathogen’s Gram-stain the analysis of the pathogen’s motility
Archaea do not have the typical peptidoglycan structure found in bacterial cell walls. ⊚ ⊚
true false
87)
Cellular organisms without a true nucleus include bacteria and viruses. ⊚ true ⊚ false
88)
Pili used for conjugation are only found on gram-negative bacteria. ⊚ true ⊚ false
89)
The slime layer provides bacteria greater pathogenicity as compared to the capsule. ⊚ ⊚
true false
90) If during the Gram stain procedure, the bacterial cells were viewed immediately after crystal violet was applied, gram-positive cells would be purple but gram-negative cells would be colorless. ⊚ ⊚
Version 1
true false
31
91)
Iodine is the decolorizer in the Gram stain. ⊚ ⊚
true false
92) The cell envelope of gram-positive bacteria has two layers: a thick cell wall and the cell membrane. ⊚ ⊚
93)
Gram-negative bacteria do not have peptidoglycan in their cell walls. ⊚ ⊚
94)
true false
Bacteria in the genus Mycoplasma and bacteria called L-forms lack cell walls. ⊚ ⊚
95)
true false
true false
The bacterial cell membrane is a site for many enzymes and metabolic reactions. ⊚ true ⊚ false
96) If you observe rod-shaped, pink cells on a slide that had just been Gram-stained, you can assume that the cell envelopes contain endotoxin. ⊚ true ⊚ false Version 1
32
97)
Both gram-positive and gram-negative cells have outer membranes. ⊚ ⊚
98)
Alcohol-based compounds can weaken the outer membrane. ⊚ ⊚
99)
true false
The cell envelope or its parts can interact with human tissue and cause disease. ⊚ ⊚
100)
true false
true false
Some bacteria have a cytoskeleton of protein polymers to help maintain their shape. ⊚ true ⊚ false
101) Endospores of certain bacterial species can enter tissues in the human body, germinate, and cause an infectious disease. ⊚ ⊚
102)
true false
Boiling water (100°C) can normally destroy endospores. ⊚ ⊚
Version 1
true false
33
103)
The term diplococcus refers to an irregular cluster of spherical bacterial cells. true ⊚ false ⊚
104) Each bacterial species represents a distinct organism that can produce viable offspring when mating with other bacteria of its kind. ⊚ true ⊚ false
105) The gram-positive cell wall is considered stronger than that of gram-negative cells since its hydrophobic outer membrane contains an endotoxin. ⊚ ⊚
true false
106) When an endospore germinates, it develops into multiple vegetative cells ensuring reproductive success. ⊚ ⊚
Version 1
true false
34
Answer Key Test name: Ch 3 4e 1) [A, D, H] 2) [A, C, E, F, G] 3) [B, C] 4) [A, C, D, F] 5) [B, C, E, G, H] 6) [A, D, E, F, G, H, I] 7) [B, C, D, E, F] 8) D 9) B 10) B 11) D 12) E 13) E 14) C 15) A 16) B 17) D 18) C 19) C 20) C 21) E 22) D 23) C 24) C 25) B 26) B Version 1
35
27) B 28) A 29) B 30) A 31) B 32) A 33) B 34) B 35) A 36) E 37) E 38) B 39) D 40) A 41) B 42) C 43) C 44) E 45) C 46) C 47) E 48) E 49) A 50) B 51) E 52) C 53) A 54) C 55) A 56) C Version 1
36
57) B 58) E 59) B 60) D 61) B 62) A 63) D 64) A 65) A 66) A 67) A 68) A 69) D 70) A 71) A 72) B 73) A 74) A 75) C 76) A 77) A 78) E 79) B 80) A 81) D 82) A 83) Section Break 83.1) D 83.2) B 83.3) D Version 1
37
84) Section Break 84.1) C 84.2) C 84.3) D 84.4) C 85) Section Break 85.1) D 85.2) C 85.3) D 85.4) A 86) TRUE 87) FALSE 88) TRUE 89) FALSE 90) FALSE 91) FALSE 92) TRUE 93) FALSE 94) TRUE 95) TRUE 96) TRUE 97) FALSE 98) TRUE 99) TRUE 100) TRUE 101) TRUE 102) FALSE 103) FALSE 104) FALSE 105) FALSE Version 1
38
106) FALSE
Version 1
39
CHAPTER 4 1)
Select all of the groups on this list that contain eukaryotic microorganisms.
A) Bacteria B) Algae C) Protozoa D) Archaea E) Helminths F) Fungi G) Viruses
2) Select all of the characteristics of eukaryotic flagella that distinguish them from bacterial flagella. A) Rotate either clockwise or counterclockwise B) Whip back and forth C) Consist of a 9 + 2 microtubule arrangement D) Consist of a hook, filament, and basal body E) 200 nm in diameter
3)
Select the organisms that typically have cell walls.
A) Animals B) Fungi C) Protozoa D) Algae E) Plants
4) Select characteristics exhibited by BOTH bacterial and eukaryotic cytoplasmic membranes.
Version 1
1
A) Phospholipid bilayer B) Embedded proteins C) Sterols D) Selectively permeable barrier E) Transport systems
5)
Select those activities of fungi considered detrimental from a human perspective.
A) Produce toxins such as aflatoxin B) Form associations with plant roots to enhance uptake of water and nutrients C) Cause decomposition of fresh produce D) Cause disease in plants and animals E) Contribute to allergies F) Add flavor to foods such as blue cheese and soy sauce
6)
Select those activities of fungi considered beneficial from a human perspective.
A) Produce toxins such as aflatoxin B) Form associations with plant roots to enhance uptake of water and nutrients C) Produce antibiotics D) Enable fermentation to produce alcohol E) Contribute to allergies F) Contribute to the carbon cycle by decomposing organic matter
7)
Select characteristics exhibited by most protozoans.
Version 1
2
A) Unicellular B) Cell walls containing cellulose C) Motile by means of flagella, cilia, or pseudopodia D) Heterotrophic E) 3 µm—300 µm size range F) Contain a nucleus and variety of organelles
8)
Protists include
.
A) yeasts and molds B) algae and protozoa C) helminths D) All of the choices are correct. E) None of the choices are correct.
9)
The first primitive eukaryotic cells likely evolved from
.
A) archaea B) bacteria C) prokaryotes D) the last common ancestor E) None of the choices are correct.
10)
Which of the following is found in eukaryotic cells but not in the cells of bacteria? A) Nucleus B) Mitochondria C) Endoplasmic reticulum D) Lysosomes E) All of the choicesare correct.
Version 1
3
11)
Eukaryotic flagella differ from bacterial flagella because only eukaryotic flagella . A) are used for cell motility B) facilitate chemotaxis C) facilitate phototaxis D) are long, whiplike structures E) contain microtubules
12)
Cilia are structures for motility found primarily in
.
A) protozoa B) algae C) fungi D) bacteria E) All of the choicesare correct.
13) There are nine peripheral pairs and one central pair of flagella and cilia.
found inside eukaryotic
A) filaments B) microtubules C) flagella D) cilia E) None of the choices are correct.
14)
Cell walls are not usually found in
Version 1
.
4
A) protozoa B) algae C) fungi D) bacteria E) All of the choicesare correct.
15)
The eukaryotic cell's glycocalyx is
.
A) mostly polysaccharide B) the site where many metabolic reactions occur C) also called the cell wall D) composed of lipids E) protection against osmotic lysis
16)
Which of the following is not a function of the eukaryote glycocalyx?
A) Protection B) Adherence C) Movement D) Reception of chemical signals E) All of thechoices are functions.
17)
Chitin is a chemical component of the cell walls of
.
A) protozoa B) algae C) fungi D) bacteria E) All of thechoices are correct.
Version 1
5
18)
The eukaryote cell membrane is composed of
.
A) sterols B) proteins C) phospholipids D) cholesterol E) All of thechoices are correct.
19)
The site for ribosomal RNA synthesis is the
.
A) ribosome B) nucleolus C) nucleus D) Golgi apparatus E) lysosome
20) When a eukaryotic cell is not undergoing mitosis, the DNA and its associated proteins appear as a visible network of dark fibers called the . A) nuclear envelope B) nucleosome C) nucleolus D) nucleoplasm E) chromatin
21)
Histones are
Version 1
.
6
A) found in polyribosomes B) enzymes found in lysosomes C) proteins of the cytoskeleton D) proteins associated with DNA in the nucleus E) on the surface of rough endoplasmic reticulum
22) The passageways in the nuclear envelope for movement of substances to and from the nucleus and cytoplasm are called nuclear .
A) histones B) chromatin C) pores D) endoplasmic reticulum E) inclusions
23)
The cell's series of tunnel-like membranes functioning in transport and storage are the .
A) mitochondria B) lysosomes C) Golgi apparatus D) chloroplasts E) endoplasmic reticulum
24) An organelle that is a stack of flattened, membranous sacs and functions to receive, modify, and package proteins for cell secretion is the .
Version 1
7
A) mitochondria B) lysosome C) Golgi apparatus D) chloroplast E) endoplasmic reticulum
25)
Protists with contractile vacuoles
.
A) are algae B) use them to expel excess water from the cell C) typically live in salty seawater D) use them for motility E) All of the choices are correct.
26) A(n) originates from the Golgi apparatus as one type of vesicle that contains a variety of enzymes for intracellular digestion. A) perixosome B) lysosome C) magnetosome D) inclusion E) ribosome
27) Which organelle contains cristae where enzymes and electron carriers for aerobic respiration are found?
Version 1
8
A) Mitochondria B) Lysosomes C) Golgi apparatus D) Chloroplasts E) Endoplasmic reticulum
28)
Mitochondria possess all of the following except
.
A) enzymes for metabolism B) cristae C) electron transport chain proteins D) enzymes for photosynthesis E) 70S ribosomes (prokaryote)
29)
Which organelle is found in algae but not found in protozoa or fungi?
A) Mitochondria B) Lysosome C) Golgi apparatus D) Chloroplast E) Endoplasmic reticulum
30)
The size of a eukaryotic cell ribosome is
.
A) 30S B) 40S C) 50S D) 70S E) 80S
Version 1
9
31)
Which of the following is not true of the cytoskeleton?
A) Structural framework for the cell B) Anchor points for organelles C) Made up of microfilaments D) Made up of microtubules E) Made up of cilia
32) In eukaryotic cells, ribosomes have two locations: scattered in the surface of the . A) B) C) D)
33)
and on the
cytoplasm; Golgi apparatus nucleus; Golgi apparatus cytoplasm; endoplasmic reticulum nucleus; endoplasmic reticulum
The cytoskeleton
.
A) anchors organelles B) provides support C) functions in movements of the cytoplasm D) helps maintain cell shape E) All of the choices are correct.
34)
Filamentous fungi are called
Version 1
.
10
A) pseudohyphae B) septa C) molds D) dimorphic E) mycelium
35)
When buds remain attached, they form a chain of yeast cells called
.
A) pseudohyphae B) septa C) molds D) dimorphic E) mycelium
36) Fungi that grow as yeast at one temperature but will grow as mold at another temperature are called .
A) dimorphic B) saprobes C) pseudohyphae D) spores E) parasites
37)
Which is not a characteristic of fungi?
Version 1
11
A) Have cell walls B) Photosynthetic C) Include single-celled and filamentous forms D) Heterotrophic nutrition E) Can use a wide variety of nutrients
38)
The long, threadlike branching cells of molds are called
.
A) conidiophores B) pseudohyphae C) hyphae D) septate E) ascus
39)
Most fungi obtain nutrients from dead plants and animals. These fungi are called .
A) saprobes B) parasites C) substrates D) nonseptate E) dimorphic
40)
The woven, intertwining mass of hyphae that makes up the body of a mold is a
.
A) septum B) rhizoid C) spore D) bud E) mycelium
Version 1
12
41) Fungal infections are known as clinical sources.
and can be acquired from environmental or
A) secondary infections B) vegetative infections C) mycoses D) saprobic infections E) parasitoses
42) During unfavorable growth conditions, many protozoa can convert to a resistant, dormant stage called a(n) . A) endospore B) cyst C) seed D) trophozoite E) sporozoa
43)
All of the following are found in some or all protozoa except
.
A) motility B) ectoplasm and endoplasm C) heterotrophic nutrition D) formation of a cyst stage E) cell wall
44)
The motile, feeding stage of protozoa is called the
Version 1
.
13
A) trophozoite B) cyst C) sporozoite D) oocyst E) food vacuole
45)
The group of protozoa that have flagella are the
.
A) sarcodina B) ciliophora C) mastigophora D) apicomplexa E) None of the choices are correct.
46)
The group of protozoa that have gliding motility are the
.
A) amoeba/sarcodina B) ciliophora C) mastigophora D) apicomplexa/sporozoa E) None of the choices are correct.
47)
The group of protozoa that use pseudopodia to move are the
.
A) sarcodina B) ciliophora C) mastigophora D) apicomplexa E) None of thechoices is correct.
Version 1
14
48)
Which is mismatched?
A) Giardia— causes intestinal distress; transmitted by feces in drinking water B) Histoplasma—fungus that causes Ohio Valley fever C) Trichomonas—sexually transmitted vaginal infection D) Plasmodium—protozoan that causes Chagas disease E) Naegleria—amoeba that causes brain infection
49)
Protozoan endoplasm contains
.
A) ectoplasm B) mitochondria C) flagella D) oral groves E) None of the choices are correct.
50)
Protozoan cysts
.
A) are part of all protozoan life cycles B) are necessary for transmission to a new host C) are helpful in surviving unfavorable conditions D) are the primary form of replication E) All of the choices are correct.
51)
Amoebiasis (amoebic dysentery) is most commonly contracted through the
Version 1
.
15
A) fecal-oral route from contaminated food or water B) direct transmission from one host to another C) puncture wounds D) insect bites E) None of the choices are correct.
52)
All of the following are helminths except
.
A) pinworms B) flukes C) trypanosomes D) roundworms E) tapeworms
53)
Which of the following does not pertain to helminths?
A) In kingdom Protista B) Parasitic worms C) Eggs and sperm used for reproduction D) Often alternate hosts in complex life cycles E) Have various organ systems
54)
Larvae and eggs are developmental forms of
.
A) protozoa B) algae C) helminths D) fungi E) None of the choices are correct.
Version 1
16
55)
Which of the following is not classified as a helminth?
A) Tapeworm B) Fluke C) Flatworm D) Roundworm E) All of thechoices are helminths.
56)
Adulthood and mating of helminths occur in which host?
A) Primary host B) Secondary host C) Definitive host D) Transport host E) Mating takesplace in all hosts.
57)
Larval development of helminths occurs in which host?
A) Primary host B) Intermediate (secondary) host C) Definitive host D) Transport host E) Mating takesplace in all hosts.
58)
Parasitic worms have a highly developed
Version 1
system.
17
A) digestive B) nervous C) respiratory D) muscular E) reproductive
59)
In humans, helminths generally infect the
.
A) digestive tract B) urinary tract C) nervous system D) muscular system E) skin
60) After returning from a trip to Africa, Tom begins to feel very tired and weak. He has severe anemia. A blood smear reveals a protozoan is present in his blood. The health care provider tells Tom he has malaria. Which of the following could be the causative agent of his disease?
A) HIV B) Nagleriafowleri C) Plasmodiumfalciparum D) Trichophyton E) Histoplasma capsulatum
61) Which of the following statements is true concerning the evolution of prokaryotes and eukaryotes?
Version 1
18
A) Rather than eukaryotes evolving from prokaryotes, evidence suggests that both prokaryotes and eukaryotes evolved from a precursor of these called the Last Common Ancestor. B) Studies suggest that eukaryotes evolved directly from prokaryotes C) Genetic evidence has shown that archaea were the precursor to prokaryotes, which in turn were the precursor to eukaryotes. D) Evolutionary biologists have shown that eukaryotes appeared first and led to the evolution of prokaryotic cells.
62)
The best definition of endosymbiosis is
.
A) A photosynthetic bacterial cell and an aerobic bacterial cell fused to become a larger, eukaryotic cell B) A eukaryotic cell ingested a bacterial cell and the organelles fused producing a larger cell C) A pre-eukaryotic cell was infected by a prokaryote, and symbiosis between the two cell types gave rise to the modern-day eukaryotic cell D) A DNA virus, a bacteria and a photosynthetic prokaryote fused to form a nucleus, a chloroplast and a mitochondria
63) Which of the following lines of evidence suggest that mitochondria evolved from the endosymbiosis of a pre-eukaryotic cell with a prokaryote?
A) B) C) D)
64)
They contain circular molecules of DNA They contain 70S ribosomes They divide independently from the cell All of these suggest that mitochondria evolved from prokaryotes.
Eukaryotic flagella differ from prokaryotic flagella in that
Version 1
.
19
A) they are thicker, they are covered by membrane and they contain microtubules B) they are thinner, they contain microfilaments and they are not involved in motility C) they are thicker, they are made of the protein flagellin and they move in a 360 o rotation D) they are thinner, they have a basal body and a hook, and they contain a 9 + 2 arrangement of microtubules
65) Eukaryotic cell membranes contain large lipid molecules called sterols, whereas prokaryotic membranes lack sterols. One argument supporting the presence of these sterols in eukaryotes is that .
A) unlike prokaryotes, many eukaryotes lack the protection of a cell wall and therefore require sterols to add some strength and rigidity to the membrane B) eukaryotic nutrients are rich in sterols, which then become embedded in the membrane C) sterols are essential catalysts for the efficient functioning of the cell membrane D) sterols are necessary membrane molecules that mediate the transport of dissolved nutrients from the cell environment to the cytosol
66) Which fiber of the cytoskeleton is a hollow tube that is also a component of flagella and the spindle fibers that form during mitosis?
A) B) C) D)
Microtubules Actin filaments Intermediate filaments Hyphae
67) Which of the following describes the relationship between the fungal hyphae and mycelia?
Version 1
20
A) Hyphae are filamentous cells that grow together in an intertwined mass called a mycelium. B) Mycelia are filamentous cells that grow together to form hyphae. C) Colonies of yeasts grow together to form hyphae, which then extend to form a mycelium. D) Septate hyphae aggregate to form mycelia and nonseptate hyphae form yeasts through budding.
68)
Fungal spores differ from bacterial endospores in that
.
A) fungal spores are reproductive, whereas bacterial endospores are for survival B) fungal spores are a result of sexual reproduction only, whereas bacterial endospores are a result of binary fission C) bacterial endospores are reproductive, whereas fungal spores are for survival D) fungal spores result from binary fission and bacterial endospores result from meiosis
69)
Fungal spores arise
.
A) when asexual reproduction produces sporangiospores and conidiospores, and sexual reproduction produces spores, often from a fruiting body B) when sexual reproduction produces sporangiospores and conidiospores, and asexual reproduction produces spores when a male hyphae mates with a female hyphae C) when sexual reproduction produces sporangiospores and conidiospores, and asexual reproduction produces spores when two fertile hyphae fuse D) as a result of asexual reproduction only E) as a result of sexual reproduction only
70)
A saprobe differs from a parasite in that
Version 1
.
21
A) a saprobe acquires its nutrients from the remains of dead animals or plants whereas a parasite acquires its nutrients from living tissue B) a parasite acquires its nutrients from the remains of dead animals or plants whereas a saprobe acquires its nutrients from living tissue C) a saprobe synthesizes its nutrients using light energy and CO 2 whereas a parasite acquires its nutrients from living tissue D) a parasite synthesizes its nutrients using light energy and CO 2 whereas a saprobe acquires its nutrients from living tissue
71)
Which of the following are mismatched?
A) Saprobe - obtain nutrients from the remains of dead plants and animals B) Heterotroph - synthesize organic nutrients using light energy and CO 2 C) Parasite - obtain nutrients from a living organism to the detriment of that organism
72) Bacteria, archaea, and eukaryotes are believed to have evolved from a different kind of cell, known as the . A) B) C) D)
last common ancestor prokaryote first primitive cell aggregate
73) Two different kinds of appendages may be found on eukaryotic cells that enable them to move. are long slender locomotor appendages that are usually single and few in number, whereas are appendages which are numerous and short.
Version 1
22
A) B) C) D)
74)
Flagella; cilia Cilia; flagella Flagella; fimbriae Pili; flagella
Choose the statement that best describes the role of mitochondria.
A) Containhydrolytic enzymes to digest macromolecules B) Provide cell withenergy through cell respiration C) Control the transport of materials into and out of the cell D) Store the geneticinformation of the cell E) Assemble amino acids intopolypeptides
75) Chloroplasts are the site of called .
and are located in plants and plantlike organisms
A) photosynthesis; algae B) photosynthesis; yeasts C) cellular respiration; algae D) cellular respiration; yeasts E) fermentation; algae F) fermentation; yeasts
76) Choose the statement that correctly describes the major difference between bacterial and eukaryotic ribosomes.
Version 1
23
A) Bacterial and eukaryotic ribosomes contain different types of nucleotides. B) Bacterial and eukaryotic ribosomes differ in the type of nucleic acid that makes up much of their structure. C) Bacterial and eukaryotic ribosomes differ in function. D) Bacterial and eukaryotic ribosomes differ in the number of subunits. E) Bacterial and eukaryotic ribosomes differ in size.
77) Long, threadlike fungal cells are called called a . A) B) C) D)
78)
hyphae; mycelium mycelia; hyphae dimorphic; mycelium hyphae; saprobe
The three structures utilized for locomotion by protozoa are flagella, cilia, and A) B) C) D)
79)
, and an intertwined mass of these cells is
.
pseudopods pili fimbriae actin filaments
Choose the term that best describes the role of protozoan cysts.
A) Protozoan cysts area reproductive stage. B) Protozoan cystsare necessary for the infection of intermediate hosts while the trophozoitesare the stage that infects definitive hosts. C) Protozoan cysts are a survival form that allows them to survive adverse environmental conditions between hosts. D) Protozoan cystsare the active, growing stage of the organism.
Version 1
24
80) The two major types of helminths based on body type are the tapeworms,and the , which are also known as nematodes. A) B) C) D)
81)
, which include the
flatworms; roundworms roundworms; flatworms flatworms; flukes flatworms, cestodes
Choose the most accurate description of a typical helminth lifestyle.
A) The helminthreproduces sexually and spends most of its life in either a single animal hostor a series of animal hosts. B) The helminthreproduces asexually in its definitive host and sexually in its intermediatehost. C) The helminthreproduces asexually and is facultatively parasitic, which means it can liveindependently or can live parasitically within a host. D) The helminthreproduces sexually and alternates between free living and parasitic stages ofits life cycle.
82) NCLEX Prep - Test Bank Question: Please read the clinical scenario, and then answer the questions that follow to become familiar with the traditional NCLEX question format. The county public health department is notified that an organism has been identified in a local lake, which is popular for water recreation and swimming. Acanthamoeba is familiar to the public health officials as an amoeba that can cause a fatal brain disease in both normal and immunocompromised individuals. An RN is a member of the taskforce assigned to the development and dissemination of community education. The team prepares information for the public to be released to media outlets and posted at the lake.
82.1) The public health information flyer explains that an amoeba is which type of microorganism?
Version 1
25
A) B) C) D)
82.2)
A) B) C) D)
A prokaryotic alga A eukaryotic fungus A eukaryotic protozoan A prokaryotic bacterium
Which type(s) of eukaryotic organisms can cause infections in humans?
Helminths Fungi Protozoans Helminths, fungi, and protozoans
82.3) At the lakeside, public health officials post signs warning the public to avoid swimming in the lake. Acanthamoebaand other protozoa are able to enter a resting state when environmental conditions are unfavorable. This stage of the life cycle is called a(n) . A) B) C) D)
trophozoite endospore cyst spore
82.4) The public health team informs the community that Acanthamoeba may be transmitted in which stage(s) of its life cycle? A) Merozoite B) Trophozoite C) Encystment D) Merozoite and encystment E) Trophozoite and encystment
Version 1
26
83) NCLEX Prep - Test Bank Question: Please read the clinical scenario, and then answer the questions that follow to become familiar with the traditional NCLEX question format. An RN begins his shift in the infusion center at the local oncology center. His first patient is a 45-year-old female who has been coming in weekly for the past month for her infusion of chemotherapy. She reports oral pain and difficulty eating that has been worsening since her last appointment. The nurse’s assessment reveals a white film covering the inside of her cheeks and tongue. The RN swabs the inside of her mouth for microscopic analysis.
83.1) After staining, the specimen is inspected. The laboratory technician identifies single-celled organisms that possess a nucleus, and many of them are undergoing the process of budding. Based on this finding, it is most likely that the patient is suffering from an infection caused by a .
A) helminth B) protozoan C) fungus D) bacterium E) Unable to determine with this information
83.2) In the specimen, it is noted that some of the cells are producing filamentous structures called hyphae. The RN educates the patient about the microscopic analysis. A term referring to the ability of a yeast able to exist in either yeast or hyphal form is .
A) B) C) D)
Version 1
dioecious dimorphic dichotomous pseudohypha
27
83.3) The RN suspects the patient is suffering from oral candidiasis, caused by Candida albicans, and knows that treatment may require drugs that produce toxic side effects.Which of the following statements explains why drugs used to treat this type of infection can be toxic to humans?
A) Antifungals target bacteria only seen in humans. B) Antifungals target eukaryotic structures seen in both Candida albicans and human cells. C) Antifungals target peptidoglycan which occurs in the cell wall of in both Candida albicans and human cells. D) Antifungals target the process of sporulation exhibited by both Candida albicans and human cells.
84) NCLEX Prep - Test Bank Question: Please read the clinical scenario, and then answer the questions that follow to become familiar with the traditional NCLEX question format. An RN at an elementary school notices a second grade female scratching and pulling at her pants. The student complains that her buttocks itch. Upon further assessment, the patient reports that the itching worsened in the morning hours. Since another child in the student’s class was recently diagnosed with Enterobius vermicularis, or pinworm infection, the RN. contacts the girl’s mother and recommends follow up with her pediatrician.
84.1) The RN explains to the patient’s mother that Enterobius vermicularis is a helminth. He explains that disease-causing helminths are .
A) B) C) D)
84.2)
Version 1
usually only visualized by microscope unicellular organisms primarily found in the gastrointestinal tract functionally similar to viruses
Pinworm is characterized as which type of helminth?
28
A) B) C) D)
Nematode Flatworm Trematode Cestode
84.3) The RN explains the pinworm life cycle to the student’s mother. Which of the following statements by the mother demonstrates a need for further education and reinforcement of the nurse’s teaching?
A) B) C) D)
Practicing good handwashing can prevent reinfection. Pinworm is transmitted through droplets in the air. Changing my daughter’s underwear in the morning can prevent reinfection. Cleaning my daughter’s toys will help stop the spread of pinworm.
84.4) Most helminths are macroscopic, yet they are studied in the field of microbiology. This classification is due to the fact that during their lifecycle, they may produce which microscopic structures?
A) B) C) D)
Eggs or larvae Pili Proglottids Spores
84.5) The RN educates the student’s mother that anal scratching, followed by putting the fingers in the mouth, may allow the pinworm infection to persist in the host. Which of the following terms best describes this phenomenon?
Version 1
29
A) B) C) D)
85)
Cross-infection Contamination Cross-contamination Reinfection
The eukaryotic cell membrane is a bilayer of sterols. ⊚ ⊚
true false
86)
The cell wall of fungi and algae are chemically identical to the bacterial cell wall. ⊚ true ⊚ false
87)
The nuclear envelope is a single layer. ⊚ ⊚
88)
Rough endoplasmic reticulum is studded with ribosomes. ⊚ ⊚
89)
true false
true false
Eukaryotic mitochondria have their own 70S ribosomes and circular DNA. ⊚ ⊚
Version 1
true false
30
90)
Algae are classified into divisions based principally on their type of motility. ⊚ ⊚
91)
Infections caused by fungi are called mycoses. ⊚ ⊚
92)
true false
All fungi cause some kind of disease in plants and animals. ⊚ ⊚
95)
true false
In humans, fungi can only infect the skin. ⊚ ⊚
94)
true false
All fungi have hyphae. ⊚ ⊚
93)
true false
true false
Fungi can reproduce both sexually and asexually. ⊚ ⊚
Version 1
true false
31
96) The Last Common Ancestor gave rise to the cells in the three domains that we recognize today; bacteria archaea and eukaryotes. ⊚ ⊚
true false
97) Scientists believe that the modern eukaryotic cell evolved 3.5 billion years ago when a photosynthetic bacteria and an aerobic bacteria fused together. ⊚ ⊚
98)
true false
All algae have chloroplasts. ⊚ ⊚
Version 1
true false
32
Answer Key Test name: Ch 4 4e 1) [B, C, E, F] 2) [B, C, E] 3) [B, D, E] 4) [A, B, D, E] 5) [A, C, D, E] 6) [B, C, D, F] 7) [A, C, D, E, F] 8) B 9) D 10) E 11) E 12) A 13) B 14) A 15) A 16) C 17) C 18) E 19) B 20) E 21) D 22) C 23) E 24) C 25) B 26) B Version 1
33
27) A 28) D 29) D 30) E 31) E 32) C 33) E 34) C 35) A 36) A 37) B 38) C 39) A 40) E 41) C 42) B 43) E 44) A 45) C 46) D 47) A 48) D 49) B 50) C 51) A 52) C 53) A 54) C 55) E 56) C Version 1
34
57) B 58) E 59) A 60) C 61) A 62) C 63) D 64) A 65) A 66) A 67) A 68) A 69) A 70) A 71) B 72) A 73) A 74) B 75) A 76) E 77) A 78) A 79) C 80) A 81) A 82) Section Break 82.1) C 82.2) D 82.3) C 82.4) E Version 1
35
83) Section Break 83.1) C 83.2) B 83.3) B 84) Section Break 84.1) C 84.2) A 84.3) B 84.4) A 84.5) D 85) FALSE 86) FALSE 87) FALSE 88) TRUE 89) TRUE 90) FALSE 91) TRUE 92) FALSE 93) FALSE 94) FALSE 95) TRUE 96) TRUE 97) FALSE 98) TRUE
Version 1
36
CHAPTER 5 1)
Select characteristics exhibited by viral capsids.
A) Composed entirely of protein B) May be an icosahedral shape C) Contain a significant amount of phospholipid D) May be spherical in shape E) May be helical in shape F) Protect the viral nucleic acid G) Composed of subunits called capsomers H) May bind to the surface of the host cell
2)
Select the terms that describe the possible configurations of viral nucleic acids.
A) Single-stranded DNA B) Double-stranded DNA, linear C) Double-stranded DNA-RNA hybrid D) Double-stranded DNA, circular E) Single-stranded RNA, positive sense F) Single-stranded RNA, negative sense G) Double-stranded RNA
3)
Select the three choices that are the primary purposes of cultivating viruses.
A) To isolateviruses from clinical specimens B) To producechemicals such as alcohol and acetone C) To produce virusfor bioweapons use D) To prepareviruses for vaccines E) To research the biology of viruses
Version 1
1
4)
Select the three methods that are used to cultivate viruses.
A) Use of cellculture techniques B) Use of enrichedagar media C) Use of animalinoculation D) Inoculation ofembryonated eggs E) Use of enriched broth media
5)
Select the infectious agents that are noncellular.
A) Bacteria B) Prions C) Satellite viruses D) Protozoa E) Viroids
6)
Viruses have all the following except
.
A) definite shape B) metabolism C) genes D) the ability to infect host cells E) ultramicroscopic size
7)
Who developed a rabies vaccine by separating bacteria from virus using a filter?
Version 1
2
A) Leeuwenhoek B) Koch C) Pasteur D) Cohn E) Semmelwise
8)
Host cells of viruses include
.
A) humans and other animals B) plants and fungi C) bacteria D) protozoa and algae E) All of the choices are correct
9)
Viruses
.
A) cannot be seen in a light microscope B) are prokaryotic C) contain 70S ribosomes D) undergo binary fission E) All of the choices are correct
10)
Virus capsids are made from subunits called
.
A) envelopes B) spikes C) capsomeres D) prophages E) peplomers
Version 1
3
11)
Helical and icosahedral are terms used to describe the shapes of a virus
.
A) spike B) capsomere C) envelope D) capsid E) core
12)
A(n)
is the protein shell around the nucleic acid core of a virus.
A) capsomere B) capsid C) spike D) envelope E) monolayer
13) One of the principal capsid shapes is a 20-sided figure with 12 evenly spaced corners referred to as a(n) capsid. A) spiked B) complex C) icosahedral D) helical E) buckeyball
14)
A naked virus hasonly a(n)
Version 1
.
4
A) capsid B) capsomere C) nucleocapsid D) envelope E) antigenic surface
15)
Which of the following is not a typical capsid shape?
A) Tetrahedral B) Complex C) Helical D) Icosahedron E) All of thechoices are capsid shapes.
16)
All of the following pertain to virus envelopes except
.
A) gained as a virus leaves the host cell membrane B) are comprised primarily of lipids C) contain special virus proteins D) help the virus particle attach to host cells E) are located between the capsid and nucleic acid
17)
Viral spikes
.
A) are present on all viruses B) protrude from the envelope C) block attachment between virus and host D) are derived from host proteins E) All of the choices are correct
Version 1
5
18)
The core of every virus particle always contains
.
A) DNA B) capsomeres C) enzymes D) DNA and RNA E) either DNA or RNA
19)
Which of the following is not associated with every virus?
A) Envelope B) Capsomeres C) Capsid D) Nucleic acid E) Genome
20)
Viral nucleic acids include which of the following?
A) Double-stranded DNA B) Single-stranded DNA C) Double-stranded RNA D) Single-stranded RNA E) All of the choices are correct
21)
Reverse transcriptase synthesizes
Version 1
.
6
A) the positive RNA strand from a negative RNA strand B) a negative RNA strand from a positive RNA strand C) RNA from DNA D) DNA from RNA E) None of the choices are correct
22)
A negative-sense RNA virus
.
A) is ready for immediate translation B) must synthesize a negative RNA copy of its genome C) must synthesize a positive RNA copy of its genome D) is a special form of tRNA used by viruses E) always codes for RNA polymerase
23) Viruses with -sense RNA contain the correct message for translation, while viruses with -sense RNA must first be converted into a correct message. A) positive; negative B) negative; positive C) primary; secondary D) secondary; primary E) None of the choices are correct
24)
Viruses acquire envelopes around their nucleocapsids during
Version 1
.
7
A) replication B) assembly C) adsorption D) release E) penetration
25) In general, most DNA viruses multiply in the host cell's viruses multiply in the host cell's .
, while most RNA
A) nucleus; cytoplasm B) cytoplasm; cell membrane C) cell membrane; cytoplasm D) cytoplasm; nucleus E) nucleus; endoplasmic reticulum
26)
Host range is limited by
.
A) type of nucleic acid in the virus B) age of the host cell C) type of host cell receptors on cell membrane D) size of the host cell E) All of the choices are correct
27)
Oncoviruses include all the following except
Version 1
.
8
A) hepatitis B virus B) measles virus C) papillomavirus D) HTLV I E) Epstein-Barr virus
28)
Which of the following is a type of cytopathic effect?
A) Inclusions in the nucleus B) Multinucleated giant cells C) Inclusions in the cytoplasm D) Cells change shape E) All of thechoices are correct.
29)
The envelope of enveloped viruses
.
A) is identical to the host plasma membrane B) is only composed of host endomembrane C) does not contain spikes D) is obtained by viral budding or exocytosis E) None of the choices are correct
30)
Viruses attach to their hosts via
.
A) host glycoproteins B) host phospholipids C) viral phospholipids D) viral flagella E) All of the choices are correct
Version 1
9
31)
Viral tissue specificities are called
.
A) ranges B) virions C) receptacles D) tropisms E) uncoating
32)
The process of dissolving the envelope and capsid to release the viral nucleic acid is .
A) adsorption B) penetration C) uncoating D) synthesis E) assembly
33)
Which of the following occurs during assembly of an enveloped virus?
A) Nucleocapsid isformed. B) New viralnucleic acid is formed. C) Viral spikesinsert in host cell membrane. D) A nucleocapsid is formed and viral spikes insert in host cell membrane. E) All of thechoices occur.
34)
Mammalian viruses capable of starting tumors are
Version 1
.
10
A) chronic latent viruses B) oncoviruses C) syncytia D) inclusion bodies E) cytopathic
35)
Persistent viruses that can reactivate periodically are
.
A) chronic latent viruses B) oncoviruses C) syncytia D) inclusion bodies E) cytopathic
36)
Which of the following is not a characteristic of a transformed cell?
A) Viral nucleic acid integrated into host DNA B) Decreased growth rate C) Alterations in chromosomes D) Changes in cell surface molecules E) Capacity to divide indefinitely
37)
New, nonenveloped virus release occurs by
.
A) lysis B) budding C) exocytosis D) both lysis and budding E) both budding and exocytosis
Version 1
11
38)
What structures are used by bacteriophages to attach to host cell receptors?
A) Sheath B) Tail fibers C) Nucleic acid D) Capsid head E) None of the choices are correct
39)
Which of the following is incorrect about prophages?
A) Present when the virus is in lysogeny B) Formed when viral DNA enters the bacterial chromosome C) Replicated with host DNA and passed on to progeny D) Cause lysis of host cells E) Occur when temperate phages enter host cells
40)
T-even phages
.
A) include the poxviruses B) infect Escherichia coli cells C) enter host cells by engulfment D) have helical capsids E) All of the choices are correct
41) The event that occurs in bacteriophage multiplication that does not occur in animal virus replication is .
Version 1
12
A) adsorption to the host cells B) injection of only the viral nucleic acid into the host cell C) host cell synthesis of viral enzymes and capsid proteins D) assembly of nucleocapsids E) replication of viral nucleic acid
42) Viruses that cause infection resulting in alternating periods of activity with symptoms and inactivity without symptoms are called .
A) latent B) oncogenic C) prions D) viroids E) delta agents
43)
Uncoating of viral nucleic acid
.
A) does not occur in bacteriophage multiplication B) involves enzymatic destruction of the capsid C) releases viral nucleic acid into the cell D) occurs before replication E) All of the choices are correct
44)
Lysogeny refers to
Version 1
.
13
A) altering the host range of a virus B) latent state of herpes infections C) virion exiting host cell D) viral genome inserting into bacterial host chromosome E) None of the choices are correct
45)
Viruses that infect bacteria are specifically called
.
A) viroids B) prions C) bacteriophages D) satellite viruses E) All of thechoices infect bacteria.
46) the
During lysogeny, an inactive prophage state occurs when the viral DNA is inserted into .
A) host cytoplasm B) host nucleus C) host nucleolus D) host DNA E) host cell membrane
47)
What type of phage enters an inactive prophage stage?
Version 1
14
A) Primary B) Secondary C) Temperate D) Temporary E) Transformed
48)
The activation of a prophage is called
.
A) activation B) lysogeny C) transformation D) induction E) adsorption
49)
When a bacterium acquires a trait from its temperate phage, it is called
.
A) transformation B) lysogenic conversion C) viral persistence D) transcription E) translation
50)
Which of the following will not support viral cultivation?
A) Live lab animals B) Embryonated bird eggs C) Primary cell cultures D) Continuous cell cultures E) All of thechoices will support viral cultivation.
Version 1
15
51) Visible, clear, well-defined patches in a monolayer of virus-infected cells in a culture are called .
A) lysogeny B) budding C) plaques D) cytopathic effects E) pocks
52)
Cells grown in culture form a(n)
.
A) monolayer B) bilayer C) aggregate D) plaque E) None of the choices are correct.
53)
A common method for cultivating viruses in the lab is to use in vitro systems called cultures.
A) embryo B) cell C) plaque D) bacteriophage E) egg
54)
Infectious protein particles are called
Version 1
.
16
A) viroids B) phages C) prions D) oncogenic viruses E) spikes
55)
Infectious naked strands of RNA that affect plants are called
.
A) viroids B) phages C) prions D) oncogenic viruses E) spikes
56)
Creutzfeldt-Jacob disease is
.
A) caused by a chronic latent virus B) initiated by an oncogenic virus C) caused by a viroid D) a spongiform encephalopathy of humans E) also called "mad cow disease"
57)
Satellite viruses are
.
A) also called viroids B) dependent on other viruses for replication C) the cause of spongiform encephalopathies D) significant pathogens of plants E) All of the choices are correct
Version 1
17
58) Two noncellular agents, smaller than viruses, are the infectious proteins called and the infectious RNA strands called .
A) prions; capsomeres B) virions; prions C) viroids; phages D) prions; phages E) prions; viroids
59)
The development of antiviral drug therapy is difficult because A) B) C) D)
60)
viruses are obligate intracellular parasites so the host cell can be harmed by the drug viruses do not have specific replication cycles that can be targeted viruses do not contain genetic material their life cycles do not have distinctive stages
The primary purpose of viral cultivation is
A) B) C) D)
.
.
to isolate and identify viruses in clinical specimens to prepare viruses for vaccines to do detailed research on viral structure, lifestyle, genetics, and effects on host cells All of the choices are correct
61) You are running an experiment in calf serum, which cannot be autoclaved because proteins essential to your protocol will be denatured. You decide to filter sterilize the serum since the 0.22μm filter is small enough to block any bacteria that may contaminate your tissue culture. The success of this procedure hinges on the fact that .
Version 1
18
A) B) C) D)
62)
the presence of viruses in your serum is inconsequential to your experiment some of the bacteria will still be allowed to filter through viruses, as well as bacteria, will also be blocked by the 0.22μm pore size the proteins in the serum are also blocked by the pore size
Which of the following is not true regarding the structure and function of viral spikes?
A) They are found on both enveloped and naked viruses B) They are coded for by the host genome C) They are coded for by the viral genome D) They mediate the docking process of virus to host cell E) They consist of proteins and carbohydrates
63) Respiratory syncytial virus (RSV) is a common respiratory virus that causes mild coldlike symptoms in most individuals, but can be more serious in infants and the elderly. RSV is sonamed because .
A) B) C) D)
it causes the fusion of damaged host cells, forming a large, multinucleated cell it causes the proliferation of inclusion bodies within the host cell it causes a persistent infection in the host it transforms the host cell, causing cancer
64) A patient undergoing chemotherapy for cancer develops an infection with cytomegalovirus, conclusively diagnosed by the presence of "owl's eye" viral nuclear inclusions in a liver biopsy. This is an example of a(n) .
Version 1
19
A) B) C) D)
cytopathic effect by cytomegalovirus immune response by the host cytomegalovirus becoming a provirus transformation of the host cell by cytomegalovirus
65) Viral genetic studies, vaccine development and clinical identification would not be possible without the ability to .
A) B) C) D)
66)
culture viruses in vivo and in vitro transform human cells into cancerous cells studying prions in neurological tissue of animals activate lysogenic induction in bacteriophages
An ideal antiviral drug would be one that
.
A) disrupted an integral viral process while causing little damage to the host cell B) killed the host cell thereby depriving the virus of its means to translate proteins C) killed both the virus and the host cell D) prevents translation by the host cell ribosome, depriving the virus of a means to replicate
67)
What is the rationale for providing intravenous fluids and oxygen support for this patient?
A) These measures increase the patient’s immune response. B) These measures augment antiviral treatment for RSV disease. C) These measures prevent secondary respiratory infection while the patient is compromised. D) These measures are supportive therapies while the disease runs its course
Version 1
20
68)
Rather than to argue that a virus is “alive,” it is preferred to say a virus is A) B) C) D)
.
active weak attenuated abiotic
69) The polio vaccine currently used in the United States, the Salk vaccine, consists of polioviruses that have been treated with chemicals sothat they can no longer reproduce within host cells and thus cannot establish an infection. They are referred to as , rather than dead, viruses. A) B) C) D)
70)
inactive active inattentive abiotic
Viruses are usually microscopy. A) B) C) D)
in size, and their structure is best viewed through the use of
nanometers; electron nanometers; bright-field micrometers; electron micrometers; bright-field
71) Choose the term that describes a virus that has a membranous outer covering over its capsid, partially derived from a host cell.
Version 1
21
A) Enveloped virus B) Naked virus C) Encapsulated virus D) Icosahedral virus E) Coated virus
72)
Choose the statement that best describes the role of viral surface proteins or spikes.
A) Inject viral nucleic acid into host cell B) Allow viruses to bind to each other C) Provide means of attachment to host cell surface D) Enable replication of the viral nucleic acid E) Provide means for viruses to exchange nucleic acid
73)
Choose the statement that indicates how cytopathic effects are detected.
A) B) C) D)
Perform biochemical tests on infected cells Examine infected cells with a microscope Perform immunological tests on infected cells Grow infected cells in a variety of different media
74) A viral infection in which the virus can remain latent in the host cell for weeks toyears is described as a(n) infection, and may lead to cancer if host DNA is altered. A) B) C) D)
Version 1
persistent cytopathic syncytial acute
22
75) Choose the term that describes a bacteriophage infection in which the bacteriophage DNA is inserted into the host cell DNA as a prophage, and the infected bacteria can continue to grow and divide.
A) Lysogenic B) Hemorrhagic C) Lytic D) Oncogenic E) Transforming
76) When growing in cell cultures, viral infection may be indicated by a clear well-defined patch where host cells have been lysed.This patch of destroyed cells is known as a . A) B) C) D)
plaque colony syncytium transformation
77) Choose the number that best estimates the annual number of human viral infections worldwide. A) 100,000 –900,000 B) 1 million – 50million C) 50 million – 200million D) 200 million – 900million E) It is nearlyimpossible to measure this number accurately.
78) Choose the primary reason why it is more difficult to design antiviral drugs than antibacterial drugs.
Version 1
23
A) B) thehost. C) D)
79)
Viral cellstructure is different than bacterial cell structure. Antiviral drugsoften must target host cell functions which results in side effects in Antiviral drugscan’t readily penetrate host cells. Antiviral drugsmust target viral envelopes and capsids.
Choose the statement that correctly explains why viruses are described as “filterable.”
A) Viruses are able topass through a filter because of their high density. B) Viruses are able topass through a filter because of their small size. C) Viruses are able topass through a filter because of the negative charge on their surface. D) Viruses are able topass through a filter because they lack peptidoglycan in their envelope. E) Viruses are not able to pass throughfilters.
80) Choose the term that describes a bacteriophage infection in which the host bacteria are lysed and killed by the infection.
A) Lysogenic B) Hemorrhagic C) Lytic D) Oncogenic E) Transforming
81) HPV can cause cellular changes that lead to the development of cervical cancer.The virus is said to cause transformation as it changes the growth rate and properties of the host cells. Because of this, HPV can be referred to as a(n) virus.
Version 1
24
A) B) C) D)
oncogenic subclinical syncytial latent
82) NCLEX Prep - Test Bank Question: Please read the clinical scenario, and then answer the questions that follow to become familiar with the traditional NCLEX question format. An RN is in a long-term care facility is caring for a 79-year-old female. The patient has been a resident on in the facility for the past 5 years. She has been complaining of pain, burning, and tingling on her left side. Upon further assessment, the RN observes fluid-filled blisters wrapping around the left side of her torso in a concentrated strip. The facility physician diagnoses her with shingles and orders the appropriate isolation.
82.1) The patient asks the RN how she contracted this viral infection since she has not come in contact with anyone with anyone infected with shingles. Which of the following statements is an appropriate response?
A) Varicella zoster virus can remain in a chronic latent state by integrating into a bacteriophage. B) Varicella zoster virus can remain in a chronic latent state by turning on oncogenes. C) Varicella zoster virus can remain in a chronic latent state by attacking T cells. D) Varicella zoster virus can hide from the immune system within nerve cells.
82.2) Viruses contain the necessary tools to invade and control a host cell. These tools may consist of a .
Version 1
25
A) B) C) D)
nucleus, nucleic acid strands of DNA and RNA, and enzymes capsid or envelope, nucleic acid strands of DNA or RNA, and enzymes nucleus, nucleic acid strands of RNA, capsid, and enzymes capsid or envelope, nucleic acid strands of DNA, and enzymes
82.3)
The viral genome is carried by
A) B) C) D)
DNA only RNA only both RNA and DNA either DNA or RNA but not both
.
82.4) Which of the following is the priority intervention to prevent the spread of shingles to other residents of the facility?
A) B) C) D)
Institute droplet precautions for the patient. Perform wound care twice a day. Observe strict hand hygiene practices. Administer antivirals as soon as possible.
83) NCLEX Prep - Test Bank Question: Please read the clinical scenario, and then answer the questions that follow to become familiar with the traditional NCLEX question format. An RN in a pediatric emergency department performs a triage patient on a 13-month-old male. The patient’s father states he had difficulty waking his sonfrom a nap that afternoon. Additionally, the infant had difficulty crawling and had difficulty with feeding the following morning. The RN observes that the patient has a weak cry. Upon further assessment, the patient’s father noted that the infant had ingested fresh honey on his toast at breakfast yesterday. The medical team suspects Clostridium botulinum infection due to the ingestion of bacterial endospores.
Version 1
26
83.1) Even though it is a bacterium, Clostridium botulinum can be made more pathogenic once infected with a(n) . A) B) C) D)
83.2) be
A) B) C) D)
bacteriophage oncogene plaque retrovirus
The RN anticipates that the priority intervention for this patient’s care will .
antibiotic administration oxygen administration airway support blood pressure support
83.3) Which of the following statements best explains how a bacteriophage can impair human health?
A) Toxins or enzymes are produced by the infected bacterial pathogen that cause pathology in humans. B) Invasion by the prophage causes cessation of protein synthesis in human cells. C) Lysis of infected human cells results in the release of numerous phage particles. D) The entry of the prophage mutates the genome of human cells.
Version 1
27
84) NCLEX Prep - Test Bank Question: Please read the clinical scenario, and then answer the questions that follow to become familiar with the traditional NCLEX question format. A 5-month-old female, accompanied by her parents, presents to the pediatrician’s office with a fever, dry mucous membranes, labored breathing, wheezing, and a harsh cough. Rapid testing of nasal swabs reveals respiratory syncytial virus infection. The RN arranges for transport to the hospital, where she will receive intravenous fluids and oxygen therapy.
84.1) Which of the following statements by the patient’s mother demonstrates a proper understanding of the cause of RSV disease? A) B) C) D)
RSV is caused by a DNA virus. RSV is caused by an RNA virus. RSV is caused by a prion. RSV is caused by a bacteriophage.
84.2) The patient’s parents question which antibiotic will be given to treat their daughter’s infection. You inform her that .
A) B) C) D)
antibiotics target eukaryotic cells and do not work on viruses antibiotics target bacterial cells and do not work on viruses her daughter will be tested for antibiotic sensitivity before they are given her daughter is too young to receive antibiotic therapy
84.3) If the patient receives antibiotic therapy at the hospital, the RN would expect the patient’s duration of symptoms to be .
Version 1
28
A) B) C) D)
increased decreased unchanged indeterminate with the information provided
85) When a virus enters a host cell, the viral genes redirect the genetic and metabolic activities of the host cell. ⊚ ⊚
86)
Viruses are used to produce vaccines for prevention of certain viral infections. ⊚ ⊚
87)
true false
Viruses are ultramicroscopic because they range in size from 2 mm to 450 mm. ⊚ ⊚
88)
true false
true false
A fully formed virus that can cause an infection in a host cell is called a virion. ⊚ ⊚
true false
89) Spikes are glycoproteins projecting outward from the surface of some viruses and may be used for attachment to a host cell. ⊚ true ⊚ false
Version 1
29
90)
A specific animal virus has the ability to attach to and enter almost any animal host cell. ⊚ ⊚
91)
Viral spikes are inserted into the host cell membrane before budding or exocytosis. ⊚ ⊚
92)
true false
true false
Prophages can be activated into viral replication and enter the lytic cycle. ⊚ ⊚
true false
93) Viruses are the most common cause of acute infections that do not result in hospitalization. ⊚ ⊚
94)
The adeno-associated virus (AAV) is a prion. ⊚ ⊚
95)
true false
true false
Viruses are simple, noncellular, and lack ribosomes. ⊚ true ⊚ false
Version 1
30
96)
Viruses mutate and some have not been discovered. ⊚ ⊚
97)
Viruses are not filterable. ⊚ ⊚
98)
true false
Viruses are unable to multiply outside of a host cell. ⊚ ⊚
99)
true false
true false
Glycoprotein spikes are essential for mediating the release stage of the viral life cycle. ⊚ ⊚
true false
100) It is necessary to culture viruses both in vivo and in vitro so that vaccines can be developed, analysis of viral genomes can be made and viruses in clinical specimens can be identified. ⊚ ⊚
true false
101) An ideal antiviral drug would be one that killed the host cell, effectively preventing the virus from replicating.
Version 1
31
⊚ ⊚
true false
102) Cellular inclusions, cell enlargement, cell fusion, cell lysis, and syncytia formation are all examples of cytopathic effects demonstrated by viral-infected cells. ⊚ true ⊚ false
Version 1
32
Answer Key Test name: Ch 5 4e 1) [A, B, E, F, G, H] 2) [A, B, D, E, F, G] 3) [A, D, E] 4) [A, C, D] 5) [B, C, E] 6) B 7) C 8) E 9) A 10) C 11) D 12) B 13) C 14) C 15) A 16) E 17) B 18) E 19) A 20) E 21) D 22) C 23) A 24) D 25) A 26) C Version 1
33
27) B 28) E 29) D 30) A 31) D 32) C 33) D 34) B 35) A 36) B 37) A 38) B 39) D 40) B 41) B 42) A 43) E 44) D 45) C 46) D 47) C 48) D 49) B 50) E 51) C 52) A 53) B 54) C 55) A 56) D Version 1
34
57) B 58) E 59) A 60) D 61) A 62) B 63) A 64) A 65) A 66) A 67) D 68) A 69) A 70) A 71) A 72) C 73) B 74) A 75) A 76) A 77) E 78) B 79) B 80) C 81) A 82) Section Break 82.1) D 82.2) B 82.3) D 82.4) B Version 1
35
83) Section Break 83.1) A 83.2) C 83.3) A 84) Section Break 84.1) B 84.2) B 84.3) C 85) TRUE 86) TRUE 87) FALSE 88) TRUE 89) TRUE 90) FALSE 91) TRUE 92) TRUE 93) TRUE 94) FALSE 95) TRUE 96) TRUE 97) FALSE 98) TRUE 99) FALSE 100) TRUE 101) FALSE 102) TRUE
Version 1
36
CHAPTER 6 1) Select all of the nutrients that are essential for growth, reproduction, and maintenance of a bacterial cell.
A) DNA B) Carbon C) Silver D) Hydrogen E) Oxygen F) Phospholipids G) Nitrogen H) Phosphorus I) Sulfur
2)
Which of the following descriptions of IV fluids are correct?
A) Isotonic IV solutions include saline and Ringer's solution because they have an osmolarity very close to blood plasma. B) Isotonic IV solutions have a potential side effect of red blood cell rupture. C) Hypertonic IV solutions will reduce extracellular tissue fluid by moving water into the bloodstream. D) Hypotonic IV solutions will cause water to move into the bloodstream, reducing tissue edema. E) Hypotonic IV solutions may result in lowered blood volume as water moves into tissue cells.
3) Select all of the passive transport mechanisms used by cells to test your understanding of active and passive transport.
Version 1
1
A) Simple diffusion B) Carrier-mediated active transport C) Group translocation D) Facilitated diffusion E) Bulk transport (phagocytosis, pinocytosis)
4)
Select all of the cellular transport mechanisms that require energy.
A) B) C) D)
5)
Select the physical factors below that can affect microbial growth.
A) B) C) D)
6)
Simple diffusion Carrier-mediated active transport Facilitated diffusion Bulk transport (phagocytosis, pinocytosis)
Temperature Radiation (visible light, UV, gamma) Barometric pressure Extreme drying/dehydration
Select characteristics of biofilms that differentiate them from independent-livingbacteria. A) Microorganismslive singly, independently. B) Microorganismsparticipate in quorum sensing. C) Microorganisms secrete extracellular material, usually polysaccharide, that forms a
matrix. D) Microorganismsare readily eradicated using antibiotics. E) Microorganisms exist as part of adiverse interdependent community.
Version 1
2
7)
Select all of the cell-counting methods that enable only living cells to be counted. A) Viable plate count B) Direct cell count C) Flow cytometry D) Coulter counter E) Turbidometry
8) Microorganisms require large quantities of metabolism.
for use in cell structure and
A) elements B) macronutrients C) water D) growth factors E) trace elements
9) Microorganisms require small quantities of maintenance of protein structure.
for enzyme function and
A) carbon B) macronutrients C) water D) growth factors E) trace elements
10)
What compound is the most abundant in a cell?
Version 1
3
A) CO 2 B) CH 4 C) H 2O D) Glucose E) NH 3
11)
Most of the dry weight of a microbial cell is from
.
A) inorganic compounds B) minerals C) water D) organic compounds E) salts
12)
Which of the following is not a major element of a microbial cell?
A) Copper B) Carbon C) Hydrogen D) Nitrogen E) Oxygen
13)
An important mineral ion of the cytochrome pigments of cellular respiration is
.
A) iron B) zinc C) calcium D) magnesium E) potassium
Version 1
4
14) An important mineral ion that is a component of chloroplasts and stabilizer of membranes and ribosomes is .
A) iron B) zinc C) calcium D) magnesium E) potassium
15)
The term autotroph refers to an organism that
.
A) uses CO 2 for its carbon source B) must obtain organic compounds for its carbon needs C) gets energy from sunlight D) gets energy by oxidizing chemical compounds E) does not need a carbon source
16)
The term photoheterotroph refers to an organism that
.
A) uses CO 2 for its carbon source B) utilizes inorganic compounds for its carbon needs C) gets energy from sunlight D) gets energy by metabolizing organic matter of dead organisms E) does not need a carbon source
17)
Calcium is required for bacteria because it
Version 1
.
5
A) stabilizes the cell wall B) stabilizes the ribosomes C) stabilizes the nucleoid D) maintains cellular pH E) makes strong bones
18)
An organism that uses CO2 for its carbon needs would be called a(n)
.
A) heterotroph B) autotroph C) chemoheterotroph D) saprobe E) halotroph
19)
The term photoautotroph refers to an organism that
.
A) is a decomposer B) must obtain organic compounds for its carbon needs C) gets energy from sunlight D) gets energy by oxidizing chemical compounds E) does not need a carbon source
20)
The term chemoheterotroph refers to an organism that
.
A) uses CO 2 for its carbon source B) must obtain inorganic compounds for its energy source C) gets energy from sunlight D) gets energy by oxidizing chemical compounds E) does not need a carbon source
Version 1
6
21)
Organisms that feed on dead organisms for nutrients are called
.
A) saprobes B) parasites C) autotrophs D) lithoautotrophs E) phototrophs
22)
Archaea, as a group, are not pathogens. This is because
.
A) they evolved without mammals B) mammals evolved special defenses against them C) mammalian hosts do not meet their environmental requirements D) they are outcompeted by natural flora E) None of the choices are correct
23)
Aerobic respiration is an example of
.
A) photosynthesis B) methanoheterophy C) photoheterotrophy D) chemoheterotrophy E) photoautotrophy
24)
Organisms called
Version 1
live on or in the body of a host and cause some degree of harm.
7
A) mesophiles B) thermophiles C) commensals D) parasites E) halophiles
25)
The term obligate refers to
.
A) the ability to exist in a wide range of conditions B) existing in a very narrow niche C) using chemicals for energy production D) using light for energy production E) using oxygen for metabolism
26)
The term facultative refers to
.
A) the ability to exist in a wide range of conditions B) existing in a very narrow niche C) using chemicals for energy production D) using light for energy production E) using oxygen for metabolism
27) The movement of molecules from an area of high concentration to an area of lower concentration is called .
Version 1
8
A) facilitated diffusion B) diffusion C) active transport D) osmosis E) endocytosis
28)
Diffusion of water through a semipermeable membrane is called
.
A) facilitated diffusion B) diffusion C) active transport D) osmosis E) endocytosis
29) The movement of substances from lower to higher concentration across a semipermeable membrane that must have a specific protein carrier and cell expenditure of energy is called . A) facilitated diffusion B) diffusion C) active transport D) osmosis E) endocytosis
30) The movement of substances from higher to lower concentration across a semipermeable membrane that must have a specific protein carrier but no energy expenditure is called .
Version 1
9
A) facilitated diffusion B) diffusion C) active transport D) osmosis E) endocytosis
31) The use of energy by a cell to enclose a substance in its membrane by forming a vacuole and engulfing it is called .
A) facilitated diffusion B) diffusion C) active transport D) osmosis E) endocytosis
32)
Bacteria living in a freshwater stream that are moved to salty seawater would
.
A) be in a hypotonic solution B) gain water C) be in an isotonic solution D) shrivel E) None of the choices are correct.
33)
Which of the following require the cell to use ATP?
Version 1
10
A) Facilitated diffusion B) Diffusion C) Endocytosis D) Osmosis E) None of the choices are correct.
34)
Nutrient absorption is mediated by the
.
A) cell wall B) peptidoglycan layer C) proteins in the periplasmic space D) cell membrane E) nuclear membrane
35)
Facilitated diffusion is limited by
.
A) substrate concentration B) carrier proteins in the membrane C) size of the pores in the membrane D) osmotic pressure E) the size of the cell
36) When whole cells or large molecules in solution are engulfed by a cell, this endocytosis is specifically termed .
Version 1
11
A) pinocytosis B) phagocytosis C) facilitated transport D) facilitated diffusion E) exocytosis
37)
In
A) B) C) D)
conditions, the cell wall will help prevent the cell from bursting.
hypertonic hypotonic isotonic All of thechoices are correct.
38) Cultures of a bacterial species were incubated on the shelf of a refrigerator, out on a lab bench top, on the shelf of a 37°C incubator, and on the shelf of a 50°C incubator. After incubation, there was no growth at 37°C and 50°C, very slight growth out on the bench top, and abundant growth at refrigeration. What term could be used for this species? A) Halophile B) Mesophile C) Anaerobe D) Psychrophile E) Capnophile
39) Cultures of a bacterial species were incubated out on the incubator shelf, in an anaerobic jar, and in a candle jar. After incubation, there was moderate growth of cultures in the candle and anaerobic jars, but heavy growth of the culture on the incubator shelf. This species is a(n) .
Version 1
12
A) obligate aerobe B) obligate anaerobe C) facultative anaerobe D) microaerophile E) capnophile
40) An organism that grows slowly in the cold but has an optimum growth temperature of 25° C is called a(n) . A) extremophile B) thermophile C) psychrophile D) psychrotroph E) thermoduric
41)
An organism with a temperature growth range of 45°C to 60°C would be called a(n) . A) extremophile B) thermophile C) psychrophile D) psychrotroph E) thermoduric
42)
Human pathogens fall into the group termed
Version 1
.
13
A) psychrophiles B) thermophiles C) halophiles D) mesophiles E) acidophiles
43)
All of the following could find a location in or on body tissues suitable for growth except .
A) psychrophiles B) anaerobes C) facultative anaerobes D) mesophiles E) capnophiles
44) A microorganism that does not have catalase or superoxide dismutase would find it difficult to live in an environment with . A) carbon dioxide B) oxygen C) high salt D) temperatures above 37°C E) high acidity
45)
A microaerophile
Version 1
.
14
A) grows best in an anaerobic jar B) grows with or without oxygen C) needs normal atmospheric levels of oxygen D) requires a small amount of oxygen but won't grow at normal atmospheric levels E) None of the choices are correct
46)
The toxic superoxide ion is converted to harmless oxygen by two enzymes,
.
A) catalase and hydrogen peroxidases B) superoxide dismutase and hydrogen peroxidases C) superoxide dismutase and catalase D) catalase and oxidase E) superoxide dismutase and oxidase
47) An organism that does not use oxygen in its metabolic pathways but can survive in its presence is a(n) . A) aerotolerant anaerobe B) obligate aerobe C) facultative anaerobe D) microaerophile E) obligate anaerobe
48) An organism that respires using oxygen when it is present but can switch to an alternative anaerobic pathways when it is absent is a(n) . A) aerotolerant anaerobe B) obligate aerobe C) facultative anaerobe D) microaerophile E) obligate anaerobe
Version 1
15
49)
An organism that cannot tolerate an oxygen environment is a(n)
.
A) aerotolerant anaerobe B) obligate aerobe C) facultative anaerobe D) microaerophile E) obligate anaerobe
50)
An organism that cannot grow without oxygen is a(n)
.
A) aerotolerant anaerobe B) obligate aerobe C) facultative anaerobe D) microaerophile E) obligate anaerobe
51)
What type of media is used to demonstrate oxygen requirements of microbes?
A) Blood agar B) Thioglycollate C) Sulfite polymyxin sulfadiazine D) Sodium chloride E) None of the choices are correct.
52)
A halophile would grow best in
Version 1
.
16
A) acid pools B) freshwater ponds C) hot geyser springs D) arid, desert soil E) salt lakes
53)
A barophile would grow best in
.
A) acid pools B) the deep oceans C) hot geyser springs D) arid, desert soil E) salt lakes
54) The E. coli that normally live in the human large intestines and produce vitamin K for the body to usewould best be termed a relationship.
A) parasitic B) saprobic C) commensal D) mutualistic E) None of the choices are correct.
55)
The production of antibiotics is a form of antagonism called
Version 1
.
17
A) symbiosis B) satellitism C) antibiosis D) mutualism E) synergism
56)
When microbes live independently but cooperate and share nutrients, it is called
.
A) symbiosis B) satellitism C) antibiosis D) mutualism E) synergism
57) When microbes are in a close nutritional relationship, and one benefits but the other is not harmed, it is called .
A) symbiosis B) satellitism C) commensalism D) mutualism E) synergism
58) The time interval going from a single bacterial parent cell to two new daughter cells is called the .
Version 1
18
A) binary fission B) growth curve C) generation time D) death phase E) culture time
59) The phase of the bacterial growth curve in which the rate of cell division equals the rate of cell death is the .
A) lag phase B) log phase C) stationary phase D) death phase E) telophase
60) The phase of the bacterial growth curve in which newly inoculated cells are adjusting to their new environment, metabolizing but not growing, is the .
A) lag phase B) log phase C) stationary phase D) death phase E) prophase
61) the
The phase of the bacterial growth curve that shows the maximum rate of cell division is .
Version 1
19
A) lag phase B) exponential (log) phase C) stationary phase D) death phase E) prophase
62) In the viable plate count method, a measured sample of a culture is evenly spread across an agar surface and incubated. Each represents one from the sample.
A) cell; cell B) cell; colony C) colony; cell D) species; colony E) generation; cell
63) When it is important to count the number of cells, determine cell size, and differentiate between dead and live cells, a is used. A) B) C) D)
Coultercounter flow cytometer SEM methylene dye indicator
64) Atoms and molecules are in constant, random motion. Movement of any molecule along its concentration gradient is termed , and movement of water in the same manner, across a selectively permeable membrane, is called .
Version 1
20
A) diffusion; osmosis B) osmosis; diffusion C) hypertonic; hypotonic D) isotonic; hypertonic E) isotonic; osmosis
65) For a physician, treating a biofilm infection in a patient poses a significant challenge compared to treating an infection with planktonic bacteria. This is due to the fact that
.
A) the biofilm likely contains several different species of bacteria living in a thick, extracellular matrix that cannot be treated by routine antibiotic therapy B) planktonic bacteria and biofilm bacteria activate the same genes C) despite being a thick layer, the pH remains constant throughout the biofilm making the deep layers hard to penetrate D) planktonic bacteria secrete quorum-sensing proteins that interfere with the actions of the antibiotic
66)
Organisms in a biofilm differ from planktonic bacteria in that
.
A) biofilms typically contain multiple species of bacteria living in a complex, whereas planktonic bacteria are single, free-floating organisms B) organisms in a biofilm secrete quorum-sensing chemicals to monitor the population; planktonic bacteria do not produce these chemicals C) an organism in a biofilm will activate different genes compared to a planktonic organism of the same genus and species D) organisms in a biofilm secrete extracellular materials to keep them anchored to a surface; these substances are not produced by free-floating bacteria E) All of the choices reflect differences between biofilms and planktonic bacteria
67)
Which of the following reflects the stages of binary fission in the correct order?
Version 1
21
A) Replication of the chromosome, formation of central protein band, septum divides daughter cells B) Replication of the chromosome, septum divides daughter cells, formation of central protein band C) Septum divides daughter cells, replication of the chromosome, formation of central protein band D) Formation of central protein band, septum divides daughter cells, replication of the chromosome E) Septum divides daughter cells, formation of central protein band, replication of the chromosome
68)
If a parent cell divides through 8 generations, how many cells will result?
A) 8 B) 64 C) 128 D) 256 E) 100,000,000
69) At 12:35 pm, 20 Salmonella enteritidis cells were present in the remains of a Caesar salad at a picnic. Five hours later, the picnic organizer ate the rest of the salad at home. S. enteritidis divides every 20 minutes; how many cells (N t) were consumed if N t = N(2 n)?
A) 655,360 cells B) 100 cells C) 327,680 cells D) 1,000,000 cells E) 1 x 10 15 cells
Version 1
22
70) Performing a standard plate count experiment for Bacillus cereus would allow a microbiologist to .
A) B) C) D)
generate a population growth curve for that particular organism generate a population growth curve for any organism calculate the number of cells in the initial inoculum count the number of cells per colony
71) Attime = 0, 200 ml of broth receives an initial inoculum of bacteria. 0.1 ml of the bacterial culture is taken from the culture at repeated intervals, diluted in liquid agar and spread on a plate containing solid media. After incubation, the plate from time = 420 min yields 62 colonies from the 0.1 ml sample. The total estimated number of cells in the flask at t = 420 min is .
A) 124,000 B) 12,400 C) 62 D) 26,040
72) Carbon, nitrogen, oxygen, and hydrogen are considered nutrients because they are required in relatively large quantities and cannot be manufactured by microbes themselves. A) B) C) D)
essential phototrophic trace exogenous
73) A derives its nutrients and energy from the organic matter of dead organisms, whereas a lives in or on another living organism and derives its nutrients and energy from that living organism.
Version 1
23
A) saprobe; parasite B) parasite; saprobe C) phototroph; autotroph D) autotroph; chemotroph E) phototroph; parasite
74)
Diffusion of water through a selectively permeable membrane is A) B) C) D)
.
osmosis pinocytosis facilitated diffusion active transport
75) Diffusion is a general term referring to the net movement of atoms and molecules along a concentration gradient, from an area of concentration to an area of concentration. A) high; low B) low; high C) diluted; thick
76) Choose the term that describes an organism that lacks superoxide dismutase and catalase enzymes to test your understanding of the role of these enzymes in microbial growth. A) Obligate aerobe B) Facultative anaerobe C) Microaerophile D) Obligate anaerobe E) Capnophile
Version 1
24
77) The time required for one complete cycle of binary fission is known as the time or generation time. A) B) C) D)
doubling log exponential binary
78) Consider the following hypothetical conditions: A population of 10,000 bacteria has a doubling time of 20 minutes, under a given set of growth conditions.The culture is allowed to grow for 2 hours.Select the number of bacteria that best predicts the population size at the end of the 2 hours incubation. Assume there is no lag phase. A) 40,000 B) 640,000 C) 1,280,000 D) 320,000 E) 160,000
79) Choose the phase of the bacterial growth curve during which a bacterial population has the briefest doubling time.
A) B) C) D)
Lag phase Exponential growth phase Stationary growth phase Death phase
80) The generation time for Enterobacter aerogenes during the exponential growth phase is 30 minutes. If a single E. aerogenes cell begins dividing during this phase, how many cells would be present if counted after 5 hours? Version 1
25
A) 10 cells B) 30 cells C) 512 cells D) 1024 cells E) 1 x 10 10 cells
81) NCLEX Prep—Test Bank Question: Please read the clinical scenario, and then answer the questions that follow to become familiar with the traditional NCLEX question format. A 24-year-old male arrives at the emergency department via air transport after sustaining a fall while rock climbing. The patient is unresponsive with sluggishly reactive pupils upon arrival. His right arm and bilateral lower extremities have obvious fractures. He was intubated by EMS and requires mechanical ventilation. A CT scan reveals multiple head bleeds and intracranial swelling with a midline shift. The patient is rushed to surgery with the plan to relieve the pressure of the intracranial swelling.
81.1) The physician orders administration of an intravenous osmotic diuretic as the patient is en route to surgery. As theRN, you are familiar with the action of the drug, which increases the osmolality of the blood. Which of the following do youanticipate as thephysiologic response to the administration of this medication?
A) B) C) D)
Decreased extravascular volume and increased intravascular volume Decreasedextravascular volume and decreased intravascular volume Increased extravascular volume and increased intravascular volume Increased extravascular volume and decreased intravascular volume
81.2) The RN administers the osmotic diuretic as ordered. Which of the following responses by the patient would indicate the therapeutic effectiveness of the drug?
Version 1
26
A) B) C) D)
No response to painful stimuli Minimal response to painful stimuli Withdrawal to painful stimuli Spontaneous movement
81.3) Following the patient’s surgical procedure, he is admitted to the neurological intensive care unit. Interventions are implemented to decrease intracranial pressure, including the administration of intravenous hypertonic saline. Which of the following do you anticipate as thetherapeutic response to the administration of this medication?
A) B) C) D)
Decreased extravascular volume and increased intravascular volume Decreasedextravascular volume and decreased intravascular volume Increased extravascular volume and increased intravascular volume Increased extravascular volume and decreased intravascular volume
81.4) The RN prepares to administer hypertonic saline. Which of the following solutions would you expect the physician to order to produce the desired physiologic effect in the patient?
A) B) C) D)
0.45% sodiumchloride 0.9% sodiumchloride 0.9% sodiumchloride with dextrose 3% sodiumchloride
81.5) Because a hypertonic environment forces water to move out of a cell, it is said to have high osmotic .
Version 1
27
A) B) C) D)
pressure diffusion tension leverage
82) NCLEX Prep—Test Bank Question: Please read the clinical scenario, and then answer the questions that follow to become familiar with the traditional NCLEX question format. An 85-year-old female in a long-term care facility is prescribed verapamil for blood pressure management twice daily. The medication is classified as a calcium channel blocker.
82.1) The RN is familiar with the action of calcium channel blockers. By inhibiting the passage of calcium through ion channels, the drug targets which of the following processes?
A) B) C) D)
Diffusion Active transport Passive transport Osmosis
82.2) Which of the following describes the mechanism of action of calcium channel blockers?
A) B) C) D)
Version 1
Relaxation of arterial smooth muscle Relaxation of venous smooth muscle Relaxation of venous smooth muscle and reduction of impulse conduction Relaxation of arterial smooth muscle and reduction of impulse conduction
28
82.3)
Which of the following is a characteristic of active transport?
A) Involves the movement of water B) Does not require the input of energy C) Involves the movement of substances from areas of high concentration to areas of low concentration D) Involves the use of carrier proteins
82.4) Prior to administration of verapamil, which of the following is most important to assess?
A) B) C) D)
Pupillary light reflex Blood oxygen levels Level of consciousness Vital signs
82.5) Prior to administration of the scheduled morning dose of verapamil, the patient’s blood pressure is found to be low. What is the most appropriate action by the RN?
A) B) C) D)
Version 1
Administer the medication as ordered Administer the medication and notify the physician Reassess the blood pressure in 30 minutes Administer a double dose of verapamil for the scheduled evening dose
29
83) NCLEX Prep—Test Bank Question: Please read the clinical scenario, and then answer the questions that follow to become familiar with the traditional NCLEX question format. The hospital infectious disease team is involved with the study of a healthcare-associated infection that has spread to 10% of inpatients. The infected patients are exhibiting symptoms of severe abdominal cramping, diarrhea, nausea, and vomiting. The RN on the infectious disease team provides education to staff regarding the pattern of bacterial growth and methods of infection control.
83.1) The period of growth between inoculation and when the cells adjust to their environment and begin growth at their maximum rate is called the .
A) B) C) D)
83.2)
A) B) C) D)
lag phase exponential growth phase stationary growth phase death phase
Which of the following is a characteristic of the death phase of bacterial growth?
The patient isexperiencing the most severe stage of infection. The cells are not reproducing. The population ofcells is rapidly increasing. The population ofcells is slowly increasing.
83.3) The most rapid increase in bacterial growth occurs during which of the following stages?
Version 1
30
A) B) C) D)
Lag phase Exponential growth phase Stationary growth phase Death phase
83.4) The RN educates the staff about the importance of timely symptom recognition and treatment of the infection. Which of the following statements by a hospital provider demonstrates a proper understanding of antimicrobial agents?
A) It is most effective to treat patients with antimicrobials in the lag phase of bacterial growth. B) It is most effective to treat patients with antimicrobials in the exponential growth phase. C) It is most effective to treat patients with antimicrobials during the stationary growth phase. D) It is most effective to treat patients with antimicrobials during the death phase of bacterial growth.
83.5) During which stages of bacterial growth is an infected patient most at risk of spreading the organism to others?
A) B) C) D)
84)
Lag phase and exponential growth phase Exponential growth phase and stationary growth phase Stationary growth phase and death phase Lag phase, stationary growth phase, and death phase
Phosphorus is one of the major elements needed in larger quantities by microorganisms. ⊚ ⊚
Version 1
true false
31
85) Most microorganisms on earth can only live and survive in habitats that are similar to human body conditions. ⊚ ⊚
86)
Whether an organism is an autotroph or heterotroph depends on its source of nitrogen. ⊚ ⊚
87)
true false
Saprobes do not need a carbon source for growth and metabolism. ⊚ ⊚
90)
true false
Obligate saprobes can adapt to a living host. ⊚ ⊚
89)
true false
Whether an organism is a phototroph or a chemotroph depends on its source of energy. ⊚ ⊚
88)
true false
true false
Lithoautotrophs use inorganic nutrients for carbon and energy sources.
Version 1
32
⊚ ⊚
true false
91) Facilitated diffusion and active transport require a carrier protein to mediate the movement of molecules across the plasma membrane. ⊚ ⊚
true false
92) In a commensal relationship, the commensal benefits but the cohabitant is neither harmed nor benefited. ⊚ ⊚
93)
Anaerobes can be cultured in a CO 2 environment. ⊚ ⊚
94)
true false
The majority of microbes live and grow in habitats between pH 7 and 9. ⊚ ⊚
95)
true false
true false
Bacteria have an average generation time of 24 hours. ⊚ ⊚
Version 1
true false
33
96) The time that it takes for a freshly inoculated agar culture to develop visible colonies is principally governed by the generation time of the inoculated species. ⊚ true ⊚ false
97)
Binary fission results in four daughter cells from one parent cell. ⊚ true ⊚ false
98)
After binary fission, daughter cells will differ genetically. ⊚ ⊚
99)
A closed culture system is used to determine a population growth curve. ⊚ ⊚
100)
true false
The direct cell count, using a cytometer, can calculate viable cell numbers from a sample. ⊚ ⊚
102)
true false
The degree of turbidity in a culture correlates to the amount of cell growth. ⊚ ⊚
101)
true false
true false
A Coulter counter can count viable cells as well as determine the size.
Version 1
34
⊚ ⊚
true false
103) Bacterial biofilms stimulate the immune system differently than free (planktonic) bacteria. ⊚ true ⊚ false
Version 1
35
Answer Key Test name: Ch 6 4e 1) [B, D, E, G, H, I] 2) [A, C, E] 3) [A, D] 4) [B, D] 5) [A, B, C, D] 6) [B, C, E] 7) [A, C] 8) B 9) E 10) C 11) D 12) A 13) A 14) D 15) A 16) C 17) A 18) B 19) C 20) D 21) A 22) C 23) D 24) D 25) B 26) A Version 1
36
27) B 28) D 29) C 30) A 31) E 32) D 33) C 34) D 35) B 36) B 37) B 38) D 39) C 40) D 41) B 42) D 43) A 44) B 45) D 46) C 47) A 48) C 49) E 50) B 51) B 52) E 53) B 54) D 55) C 56) E Version 1
37
57) C 58) C 59) C 60) A 61) B 62) C 63) B 64) A 65) A 66) E 67) A 68) D 69) A 70) A 71) A 72) A 73) A 74) A 75) A 76) D 77) A 78) B 79) B 80) D 81) Section Break 81.1) A 81.2) D 81.3) A 81.4) D 81.5) A Version 1
38
82) Section Break 82.1) C 82.2) D 82.3) D 82.4) D 82.5) C 83) Section Break 83.1) A 83.2) B 83.3) B 83.4) B 83.5) B 84) TRUE 85) FALSE 86) FALSE 87) TRUE 88) FALSE 89) FALSE 90) TRUE 91) TRUE 92) TRUE 93) TRUE 94) FALSE 95) FALSE 96) TRUE 97) FALSE 98) FALSE 99) TRUE 100) TRUE 101) FALSE Version 1
39
102) FALSE 103) TRUE
Version 1
40
CHAPTER 7 1) Select all of the descriptions that are true of enzymes to test your understanding of their structure and function.
A) Composed ofprotein B) Increase the rateof chemical reactions C) Are used up inchemical reactions D) High concentrationsare necessary for activity E) Activity isaffected by temperature and pH F) Activity isaffected by regulatory mechanisms G) Becomeincorporated into the reaction product H) Have active sites with which substratesinteract I) Increase the activation energy of a reaction
2)
Select each of the phrases that describe the Krebs cycle.
A) Pyruvic acid must be converted to acetyl CoA prior to entering the cycle B) Most of the ATP generated during the aerobic respiration of glucose is generated during the Krebs cycle C) There are 10 steps each catalyzed by a different enzyme D) There are four steps involving redox reactions E) Electrons removed during oxidation steps are picked up by NAD or FAD F) CO2 is released at two steps G) The “starting material,” oxaloacetic acid, is regenerated at the “end” of the cycle
3) Select all of the correct descriptions of fermentation to test your understanding of this process.
Version 1
1
A) Does not requireO 2 B) Uses glycolysis,Krebs cycle, and an electron transport system C) Two ATP are produced (per molecule of glucose) D) No NADH is produced E) The final electron acceptor is an organic molecule, usually pyruvic acid or a derivative F) Different fermentation pathways can produce ethanol, CO 2, lactic acid, or other organic acids G) Is only done by yeast
4)
Which of the following are involved in the process of protein catabolism?
A) Lipases B) Deamination C) Proteases D) Beta oxidation E) Glycolysis
5)
Select the phrases that correctly describe anabolic activities.
A) Synthesis ofproteins from amino acids B) Beta oxidation offatty acids C) Formation of cellwall structures from monomer building blocks D) Assembly of DNAand RNA from nucleotides E) Synthesis of amino acids via aminationreactions F) Conversion of glucose to pyruvic acidvia glycolysis
6)
All of the chemical reactions of the cell are called
Version 1
.
2
A) catabolism B) redox reactions C) phosphorylation D) metabolism E) cellular respiration
7) The breakdown of peptidoglycan to N-acetylmuramic acid, N-acetylglucosamine, and peptides is an example of .
A) anabolism B) catabolism C) phosphorylation D) fermentation E) synthesis
8)
Enzymes are
.
A) broken down in reactions that require energy input B) proteins that function as catalysts C) used up in chemical reactions D) not needed for catabolic reactions E) All of the choices are correct.
9)
Formation of peptide bonds between amino acids to build a polypeptide would be called .
Version 1
3
A) anabolism B) phosphorylation C) fermentation D) exergonic E) glycolysis
10)
Reactants are converted to products by
.
A) enzymes releasing energy B) breaking and forming bonds C) enzymes binding to reactants D) reactants releasing energy E) None of the choices are correct.
11)
Each of the following is true of enzymes except
.
A) they can be used over and over B) they may or may not require cofactors C) their active site is specific to the substrate D) they increase the initial energy required for a reaction to start E) All of the choices are true of enzymes.
12) The cell's metabolic reactions involve the participation of activation energy needed for the initiation of a reaction.
Version 1
that lower the
4
A) cofactors B) vitamins C) enzymes D) ATP E) coenzymes
13)
An apoenzyme is
.
A) part of a simple enzyme B) also called a coenzyme C) the protein part of a holoenzyme D) often an inorganic metal ion E) an RNA molecule
14)
A holoenzyme is a combination of a protein and one or more substances called
.
A) substrates B) apoenzymes C) catalysts D) cofactors E) reactants
15)
Important components of coenzymes are
.
A) vitamins B) metallic ions C) active sites D) substrates E) ribozymes
Version 1
5
16)
Enzymes that are only produced when substrate is present are termed
.
A) exoenzymes B) endoenzymes C) constitutive enzymes D) induced enzymes E) conjugated enzymes
17)
Enzymes that hydrolyze a peptide bond are called
.
A) phosphotransferases B) oxidoreductases C) peptidases D) aminotransferases E) ligases
18)
During aerobic cellular respiration, the final electron acceptor is
.
A) pyruvic acid B) oxygen C) nitrate D) cytochrome c E) FAD
19)
Enzymes that are always present, regardless of the amount of substrate, are
Version 1
.
6
A) apoenzymes B) axoenzymes C) constitutive enzymes D) regulated enzymes E) endoenzymes
20)
When enzyme action stops due to a buildup of end product, this control is called .
A) feedback inhibition (negative feedback) B) competitive inhibition C) enzyme induction D) enzyme repression E) None of the choices are correct.
21)
Each of the following are denaturing agents except
.
A) high temperature B) low temperature C) high pH D) low pH E) All of the choices are correct.
22) Feedback inhibition, a negative feedback mechanism that regulates enzymes, is best described as .
Version 1
7
A) substrate binding to DNA, blocking enzyme transcription B) product binding to DNA, blocking enzyme transcription C) substrate binding to enzyme in noncompetitive site D) product binding to enzyme in noncompetitive site E) None of the choices are correct.
23)
Metabolic pathways that regenerate their starting point are called
pathways.
A) linear B) bidirectional C) convergent D) cyclic E) divergent
24) When the product of reaction A becomes the reactant of reaction B, the metabolic pathway is .
A) linear B) bidirectional C) convergent D) cyclic E) divergent
25)
Most electron carriers are
Version 1
.
8
A) coenzymes B) enzymes C) hydrogens D) inorganic phosphates E) polysaccharides
26)
Exergonic reactions
.
A) include synthesis of large carbohydrates B) only occur in heterotrophs C) release energy D) do not occur in anaerobic cellular respiration E) occur when ADP binds to inorganic phosphate to form ATP
27) In the electron transport chain, the collective, captured energy released by electrons is used to phosphorylate .
A) ATP B) ADP C) pyruvic acid D) oxygen E) NAD
28) In addition to electrons, which of the following can also be transferred, exchanging energy in the process?
Version 1
9
A) ADP B) Glucose C) Carbon D) Hydrogen ion (proton) E) Carbon dioxide
29)
FAD, NAD, and coenzyme A are all
carriers.
A) hydrogen B) electron C) ATP D) hydrogen andelectron E) None of the choices are correct.
30)
Which of the following statements about glycolysis is false? A) It occurs without oxygen B) It ends with the formation of pyruvic acid C) It occurs during fermentation D) It degrades glucose to CO 2 and H 2O E) It involves the reduction of NAD +
31)
Glycolysis
.
A) uses 2 ATPs, produces 2 ATPs, and requires oxygen B) uses 2 ATPs, produces 4 ATPs, and requires oxygen C) uses 2 ATPs, produces 4 ATPs, without using oxygen D) uses 2 ATPs, produces 2 ATPs, without using oxygen E) None of the choices are correct.
Version 1
10
32)
The formation of citrate from oxaloacetate and an acetyl-CoA begins
.
A) glycolysis B) the electron transport system C) the Krebs cycle D) fermentation E) oxidative phosphorylation
33) Which of the following is not involved in the step that occurs between glycolysis and the Krebs cycle?
A) B) C) D)
34)
NADH is formed CO2 is released Coenzyme A attaches to an acetyl group Pyruvic acid accepts electrons from NADH
In aerobic respiration, the majority of the reduced coenzyme NADH is produced in .
A) glycolysis B) the Krebs cycle C) the electron transport chain D) photosynthesis E) the cell membrane
35)
In bacterial cells, the electron transport system is located in the
Version 1
.
11
A) cell membrane B) mitochondria C) chloroplasts D) ribosomes E) cytoplasm
36)
Each NADH that enters the electron transport system gives rise to a maximum of ATP. A) 2 B) 3 C) 24 D) 36 E) 38
37) As the electron transport carriers shuttle electrons, they actively pump into the outer membrane compartment setting up a concentration gradient called the proton motive force.
A) ATP B) phosphate C) hydrogen ions D) oxygen E) NADH
38) The carrier molecules in the electron transport chain that have a tightly bound metal atom responsible for accepting and donating electrons are .
Version 1
12
A) NAD + B) FAD C) NADH D) cytochromes E) flavoproteins
Each FADH2 from the Krebs cycle enters the electron transport system and gives rise to a 39) maximum of ATP(s). A) 1 B) 2 C) 3 D) 4 E) 5
NADH and FADH2 molecules are oxidized in which step of the aerobic respiration 40) process? A) B) C) D)
41)
Electron transport system Krebs cycle Glycolysis Preparation for Krebs cycle
During which phase of cellular respiration is the majority of ATP formed?
A) Krebs cycle B) Glycolysis C) Processing of pyruvic acid for the Krebs cycle D) Electron transport E) All phasesproduce the same number of ATP molecules.
Version 1
13
42) In bacterial cells, when glucose is completely oxidized by all the pathways of aerobic cellular respiration, what is the maximum number of ATP generated?
A) 2 ATP B) 3 ATP C) 24 ATP D) 38 ATP E) 42 ATP
43) How many carbon dioxide molecules are produced by the complete aerobic breakdown of one glucose molecule?
A) 2 B) 3 C) 4 D) 5 E) 6
44)
Which of the following is not true of anaerobic respiration?
A) B) C) D)
45)
Involves glycolysis Generates some ATP Utilizes an electron transport system Uses the same final electron acceptor as aerobic respiration
Reduction of nitrogen-oxygen ions, such as nitrate, by some bacteria is called
Version 1
.
14
A) aerobic respiration B) denitrification C) nitrification D) fermentation E) deamination
46) The reactions of fermentation function to produce glycolysis.
molecules for use in
A) pyruvic acid B) ATP C) NAD+ D) NADH E) glucose
47) When glucose is broken down by glycolysis during bacterial fermentation, how many ATP are generated?
A) 2 ATP B) 3 ATP C) 24 ATP D) 36 ATP E) 38 ATP
48)
Fermentation
Version 1
.
15
A) requires an organic electron acceptor B) requires oxygen C) only occurs in aerobic organisms D) can be equivalent to aerobic respiration in ATP production E) is the same as anaerobic respiration
49)
Deamination is the removal of a(n)
group in proteins.
A) carboxyl B) amino C) phosphate D) hydroxyl E) glycerol
50) The property of organisms to integrate catabolic and anabolic pathways to improve cell efficiency is called .
A) metabolism B) amphibolism C) anabolism D) catabolism E) biosynthesis
51)
The principal sites of amphibolic interaction occur during
.
A) glycolysis and photosynthesis B) the Krebs cycle and electron transport system C) glycolysis and the Krebs cycle D) fermentation and the Krebs cycle E) fermentation and glycolysis
Version 1
16
52)
Fatty acids can be converted to acetyl-CoA for entering the Krebs cycle by the process of . A) amination B) deamination C) phosphorylation D) beta oxidation E) gluconeogenesis
53) In a bacterial cell, synthesis of an essential amino acid would take place continually unless the amino acid was abundant in the growth media, in which case .
A) the genes that code for the enzymes that catalyze the synthesis of the amino acid would be repressed B) the genes that code for the enzymes that catalyze the synthesis of the amino acid would be induced C) the enzymes that catalyze the reactions to hydrolyze the amino acid would be repressed D) the enzymes that catalyze the reactions to hydrolyze the amino acid would be induced
54) In a bacterial cell, the genes that code for enzymes that catalyze the hydrolysis of an unusual carbohydrate source would be turned off unless that carbohydrate was the only fuel source available in the media, in which case .
A) B) C) D)
Version 1
the genes coding for the hydrolytic enzymes would be induced the genes coding for the hydrolytic enzymes would be repressed the genes coding for the hydrolytic enzymes would be constitutive the enzymes would be denatured
17
55) A metabolic pathway begins as a series oflinear reactions then, at a specific point in the pathway, two reactants are produced, each of which proceed in a linear, yet different chain of reactions. This is an example of a .
A) B) C) D)
divergent pathway convergent pathway cyclical pathway linear pathway
56) Catabolic reactions are coupled with ATP coupled with ATP .
A) B) C) D)
, whereas anabolic reactions are
synthesis; hydrolysis hydrolysis; synthesis amphibolism; repression deamination; beta oxidation
The complete combustion of methane is written CH 4 + 2O 2 → CO 2 + 2H 2O In this 57) reaction, the methane is and the oxygen is .
A) B) C) D)
oxidized; reduced reduced; oxidized phosphorylated; oxidized endergonic; exergonic
58) In bacterial cells, the electron carriers NAD + and FAD are found in the whereas thecytochromes are located in the .
,
Version 1
18
A) B) C) D)
59)
cytoplasm; cell membrane mitochondria; cytoplasm cell membrane; cytoplasm cell membrane; inner mitochondrial membrane
The hydrolysis of two ATP molecules at the start of glycolysis is analogous to
A) B) C) D)
.
striking a match to ignite a pile of burning firewood combining butter, sugar, flour and eggs to make a cake building a house kicking a soccer ball and scoring a goal
60) Intermediates in the glycolysis pathway feed into additional pathways that make amino acids and complex carbohydrates. These latter pathways are .
A) B) C) D)
61)
biosynthetic and endergonic hydrolytic and exergonic biosynthetic and exergonic hydrolytic and endergonic
Which of the following does not reflect anabolism?
A) B) C) D)
Version 1
Beta oxidation of fatty acids to acetyl CoA. Enzyme production from amino acids. DNA replication from nucleotides. Fatty acid assimilation to form lipids.
19
62)
An example of amphibolism is when
.
A) pyruvate is converted to acetyl CoA to enter the Krebs cycle, and is also used as a precursor for amino acid production B) acetyl CoA enters the Krebs cycle, and is also used for fatty acid production C) intermediates of glucose oxidation are further oxidized during glycolysis, and are also harnessed for the production of complex carbohydrates D) All of the above are examples of amphibolism
63) All of the chemical reactions that occur within a cell are collectively referred to as metabolism. This includes , which are those reactions that use energy and synthesize building blocks and macromolecules, and , which are the cellular reactions that break larger molecules into smaller molecules and release energy. A) B) C) D)
anabolism; catabolism catabolism; anabolism amphibolism; catabolism anabolism; amphibolism
64) When the product of a metabolic pathway accumulates, it may be able to slow down or turn off the pathway by inhibiting the activity of an enzyme that catalyzes an early step in the pathway. This is referred to as inhibition. A) B) C) D)
feedback competitive redox amphibolic
65) inhibition of an enzyme occurs when the inhibitor mimics the substrate and occupies the active site of the enzyme, whereas inhibition occurs when an inhibitor binds to a regulatory site on the enzyme that is separate and distinct from the active site
Version 1
20
A) Competitive; noncompetitive B) Noncompetitive; competitive C) Allosteric; competitive
66) The three basic catabolic pathways are respiration (oxygen is the final electron acceptor), respiration (non-oxygen electron acceptors), and (organic molecules as final electron acceptors). A) aerobic; anaerobic; fermentation B) anaerobic; aerobic; fermentation C) aerobic; fermentative; anaerobic metabolism
67)
Choose the phrase that best describes glycolysis.
A) A cyclic series of redox reactions that process pyruvic acid and release CO 2 and generate ATP and NADH B) A series ofredox reactions used to generate ATP using energy harvested from the transport of electrons from NADH and FADH 2 C) A multistep process resulting in the enzymatic splitting and oxidation of glucose to form pyruvic acid, with a net gain of 2 ATP D) The series of catabolic reactions that break down pyruvate to form acetyl CoA and CO 2
68) Choose the answer that most accurately expresses the significance of the electron transport system.
Version 1
21
A) The electron transport system transforms the energy of electrons into the potential energy of an electrochemical gradient (proton gradient) across the cell membrane (bacteria) or inner mitochondrial membrane (eukaryotes). This proton gradient provides the energy used by ATP synthase to phosphorylate ADP, producing ATP. B) The movement ofelectrons from one carrier to the next provides the energy necessary to splitglucose. C) The role of the electron transport system is to dispose of electrons that were removed during glycolysis and Krebs cycle and use these electrons to make water. D) The electron transport system is instrumental in the creation of an electrochemical gradient of electrons across the cell membrane (bacteria) or inner mitochondrial membrane (eukaryotes). The electron gradient drives the phosphorylation of ADP by the ATP synthase enzyme.
69) Both aerobic respiration and anaerobic respiration utilize glycolysis, Krebs cycle, and an electron transport system, but in respiration, O2 is the final electron acceptor at the end of the electron transport system, whereas in , an inorganic molecule other than O2 is the final electron acceptor. A) B) C) D)
aerobic; anaerobic anaerobic; aerobic aerobic; fermentative fermentative; anaerobic
70) In the first step of lipid catabolism, acids to the glycerol. A) B) C) D)
Version 1
enzymes break the bonds joining the fatty
lipase restriction protease glycerase
22
71) In later steps of lipid metabolism, fatty acids are converted to a number of acetyl CoA molecules in a process known as . A) B) C) D)
beta oxidation chemiosmosis deamination pyruvate oxidation
72) The property of some metabolic pathways that enables them to serve multiple functions in the breakdown, synthesis, and conversion of metabolites is known as . A) B) C) D)
amphibolism catabolism beta oxidation chemiosmosis
73) NCLEX Prep—Test Bank Question: Please read the clinical scenario, and then answer the questions that follow to become familiar with the traditional NCLEX question format. A patient with anorexia nervosa is admitted to the hospital for medical treatment. The patient presents with severe malnutrition and evidence of organ dysfunction. Labs are drawn and continuous intravenous (IV) fluids are initiated.
73.1) The RN prepares to initiate IV fluid therapy for the patient. Knowing the importance of cellular energy production, which of the following maintenance IV fluids would be most appropriate for the physician to order?
A) B) C) D)
Version 1
0.9% sodiumchloride Lactated ringers 0.45% sodiumchloride 0.9% sodiumchloride with dextrose
23
73.2) The RN educates the patient and her family about cellular metabolism. Which of the following processes is common to fermentation, anaerobic respiration, and aerobic respiration?
A) B) C) D)
Glycolysis The Krebs cycle The electron transport system Glycosylation
73.3) The RN is familiar with ATP production in bacterial and eukaryotic cells. How many ATP per glucose molecule would aerobic respiration yield in the patient’s cells? A) B) C) D)
2 ATP 36 ATPs 38 ATP ATP yield is unpredictable.
73.4) The RN instructs the patient that although bacterial cells can also undergo aerobic respiration, they differ from eukaryotic cells in that . A) B) C) D)
Version 1
they make less ATP in this process than eukaryotic cells the Krebs cycle takes place in the mitochondrion in these cells glycolysis takes place in the mitochondrion in these cells the electron transport chain is present in the plasma membrane of these cells
24
74) NCLEX Prep—Test Bank Question: Please read the clinical scenario, and then answer the questions that follow to become familiar with the traditional NCLEX question format. A newborn infant screening yields a positive result for biotinidase deficiency. The RN prepares education for the patient’s parents regarding the role of enzymes in metabolism and this enzyme deficiency disorder.
74.1) Which of the following statements, by the patient’s mother, best represents an understanding of metabolism?
A) B) C) D)
74.2)
A) B) C) D)
74.3)
A) B) C) D)
Version 1
Metabolism involves the production of enzymes. Metabolism involves all chemical reactions and physical workings of the cell. Metabolism involves the consumption of energy for workings of the cell. Metabolism involves the production of energy for workings of the cell.
All of the following are true of enzymes except
.
enzymes increase the rate of a chemical reaction enzymes are not part of the products of a chemical reaction enzymes create a chemical reaction enzymes are not consumed in a chemical reaction
Inborn errors of metabolism
.
are caused by infectious agents produce very specific signs and symptoms, making them easy to diagnose cannot be diagnosed by DNA testing may not become evident until adulthood
25
74.4) Based upon knowledge of enzymes and metabolic processes, which of the following outcomes should the parents expect if the deficiency is not treated?
A) B) C) D)
No effect on the patient’s health Reduced immunity in adulthood in the patient Mildly impaired vision in the patient Permanent damage to organ systems and/or death of the patient
75) NCLEX Prep—Test Bank Question: Please read the clinical scenario, and then answer the questions that follow to become familiar with the traditional NCLEX question format. A 67-year-old patient with severe sepsis undergoes a full infectious workup upon admittance to the intensive care unit. Cultures of urine, blood, sputum, and cerebral spinal fluid are obtained. The cultures do not yield a specific organism, so further analysis of the sample is done to determine the characteristics of the microorganism. The organism’s metabolic processes are analyzed in the lab.
75.1) The organism is found to grow throughout the entire tube of broth media. Which of the following terms best describes the oxygen requirement of this organism? A) B) C) D)
Obligate aerobe Microaerophile Facultative anaerobe Obligate anaerobe
75.2) The organism sample is further tested and found to grow in the absence of oxygen. Which of three main pathways of catabolism may this organism utilize to produce energy under these circumstances?
Version 1
26
A) B) C) D)
Aerobic respiration and fermentation Anaerobic respiration and aerobic respiration Fermentation and anaerobic respiration Aerobic respiration, anaerobic respiration, and fermentation
75.3) Biochemical testing reveals that the organism further reduces nitrite to produce nitrous oxide. Based on this byproduct, which of the following is the most likely catabolic pathway utilized by this organism? A) Aerobic respiration B) Anaerobic respiration C) Fermentation D) Aerobic respiration, anaerobic respiration, and fermentation can all lead to the production of nitrous oxide
75.4) Which of the following statements properly characterizes the ATP yield from anaerobic respiration?
A) Anaerobic respiration produces less ATP per glucose molecule than aerobic respiration at maximum net yield. B) Anaerobic respiration at maximum net yield produces more ATP per glucose molecule than fermentation. C) Anaerobic respiration produces 2-36 ATPs per glucose molecule. D) All of the choices are correct.
76)
ATP molecules are catalysts that lower the activation energy needed to initiate a reaction. ⊚ ⊚
Version 1
true false
27
77) Building block molecules for biosynthetic pathways come from the cell's catabolic pathways and from the environment. ⊚ ⊚
true false
78) Denaturing an apoenzyme will destroy the three-dimensional shape of the protein, making it nonfunctional. ⊚ ⊚
true false
79)
ATP is composed of deoxyribose, adenine, and three phosphate groups. ⊚ true ⊚ false
80)
ATP synthase is a complex enzyme needed for oxidative phosphorylation. ⊚ ⊚
true false
81) All aerobic bacterial species have identical electron acceptors in their electron transport systems. ⊚ ⊚
82)
true false
The fermentation process always produces alcohol as at least one of its products.
Version 1
28
⊚ ⊚
true false
83)
Oxygen-containing ions are used by some bacteria as final electron acceptors. ⊚ true ⊚ false
84)
Microbes can utilize only carbohydrates for energy production. ⊚ ⊚
true false
85) Anabolic pathways are associated with a net gain of ATP, whereas catabolic pathways, for example, glycolysis, are associated with a net ATP breakdown. ⊚ ⊚
Version 1
true false
29
Answer Key Test name: Ch 7 4e 1) [A, B, E, F, H] 2) [A, D, E, F, G] 3) [A, C, E, F] 4) [B, C] 5) [A, C, D, E] 6) D 7) B 8) B 9) A 10) B 11) D 12) C 13) C 14) D 15) A 16) D 17) C 18) B 19) C 20) A 21) B 22) D 23) D 24) A 25) A 26) C Version 1
30
27) B 28) D 29) D 30) D 31) C 32) C 33) D 34) B 35) A 36) B 37) C 38) D 39) B 40) A 41) D 42) D 43) E 44) D 45) B 46) C 47) A 48) A 49) B 50) B 51) C 52) D 53) A 54) A 55) A 56) A Version 1
31
57) A 58) A 59) A 60) A 61) A 62) D 63) A 64) A 65) A 66) A 67) C 68) A 69) A 70) A 71) A 72) A 73) Section Break 73.1) D 73.2) A 73.3) B 73.4) D 74) Section Break 74.1) B 74.2) C 74.3) D 74.4) D 75) Section Break 75.1) C 75.2) C 75.3) B Version 1
32
75.4) D 76) FALSE 77) TRUE 78) TRUE 79) FALSE 80) TRUE 81) FALSE 82) FALSE 83) TRUE 84) FALSE 85) FALSE
Version 1
33
CHAPTER 8 1)
Select all of the features that you would include if you drew a short segment of DNA.
A) Double-stranded B) Single-stranded C) Helical shape D) Chains of nucleotides E) The backbone of the chains are created by bonds between sugars and phosphates of nucleotides F) Nucleotides are base-paired with T bonded to A and C bonded to G through hydrogen bonds G) Nucleotides contain ribose
2) Select all of the differences between DNA and RNA to test your understanding of these nucleic acids. A) DNA is double stranded, RNA is single stranded. B) DNA nucleotides contain deoxyribose whereas RNA nucleotides contain ribose. C) DNA contains the following nitrogenous bases: adenine (A), cytosine (C), guanine (G), and uracil (U); RNA contains adenine, cytosine, guanine, and thymine. D) There are multiple types of DNA with different functions whereas there is only one type of RNA.
3)
The three types of RNA directly involved in translation include
.
A) siRNA B) mRNA C) tRNA D) primer RNA E) rRNA
Version 1
1
4) Select thephrases that describe features of transcription and translationin eukaryotes but not in bacteria. A) Transcription and translation both occur in the cytoplasm B) Genes are interrupted by intervening sequences C) mRNA is ready to translate directly as it is produced by RNA polymerase D) A tail of multiple adenine ribonucleotides is added to the mRNA transcript E) mRNA transcript must pass through pores to get out of the nucleus F) RNA transcript is processed to remove introns and splice together exons G) Genes are organized into functional units called operons
5)
Select the statements that describe the main points of lac operon operation.
A) The regulator geneencoding the repressor lies upstream of the operator region and is transcribedconstitutively. B) The repressor isallosteric, meaning it has only onebinding site. C) In the lac operon, transcription is only initiated when lactose isabsent. D) The lac operon only functions in the absence of glucose. E) DNA polymerase binds to the promoter to begin transcription of thestructural genes. F) The structural genes are transcribed in a single unbroken transcript codingfor all three enzymes needed for lactose metabolism.
6) Select the mechanisms below which enable the horizontal transfer of antibiotic resistance genes.
A) Transformation B) Transduction C) Transcription D) Translation E) Conjugation
Version 1
2
7)
Select characteristics exhibited by an ideal cloning host.
A) Grows easily B) Is pathogenic C) Readily accepts recombinant DNA D) Will synthesize large quantities of desired gene products E) Exhibits a high rate of mutation
8) Select the products developed by recombinant DNA technology that have contributed to human health.
A) B) C) D)
9)
Hormones, such ashuman growth hormone Enzymes, such astPA Vaccines, such asHepatitis B vaccine Antibiotics, such as penicillin
Select the components and equipment that would be necessary for doing PCR.
A) Sample containingthe DNA to be amplified B) dNTPs(deoxyribonucleotide triphosphates) C) Taq polymerase (thermostable polymerase) D) RNA polymerase E) Restriction endonucleases F) Primers specificto the DNA sequence to be amplified G) Thermocycler
10)
Among the microorganisms, various genomes can include
Version 1
.
3
A) chromosomes B) plasmids C) mitochondrial DNA D) chloroplast DNA E) All of the choices are correct.
11) have
Eukaryotic chromosomes differ from bacterial chromosomes because only eukaryotes . A) histone proteins B) chromosomes in a nucleus C) several to many chromosomes D) elongated, not circular, chromosomes E) All of the choices are correct
12)
Which of the following is not true of an organism's genotype?
A) Is inherited B) Are structural genes coding for proteins C) Are genes coding for RNA D) Are regulatory genes controlling gene expression E) Are the expressed traits governed by the genes
13)
The
Version 1
is all of the genetic material of a cell.
4
A) chromosome B) plasmid C) prophage D) genome E) proteome
14) Each functional product.
is a specific segment of the DNA with the code for production of one
A) intron B) exon C) gene D) operator E) triplet
15)
The expression of genetic traits is the
.
A) genome B) genotype C) proteome D) phenotype E) proteotype
16)
The DNA of microorganisms is made up of subunits called
Version 1
.
5
A) histones B) amino acids C) nucleotides D) mRNA E) polymerases
17)
Which is incorrect about purines?
A) Only found in DNA, not in RNA B) Are nitrogenous bases C) Always paired with a specific pyrimidine D) Include adenine and guanine E) Found within nucleotides
18)
The antiparallel arrangement within DNA molecules refers to
.
A) each base bonding at the 1' position of the sugar B) a purine always bonding to a pyrimidine C) one helix strand that runs from the 5' to 3' direction and the other strand runs from the 3' to 5' direction D) an original parent DNA strand and one newly synthesized DNA strand comprising a new DNA molecule E) None of the choices are correct.
19)
All of the following pertain to nitrogenous bases except
Version 1
.
6
A) form pairs by hydrogen bonding B) guanine pairs with uracil C) adenine pairs with thymine D) cytosine and thymine are pyrimidines E) they are only present in the form of purines
20)
Each nucleotide is composed of
.
A) one phosphate, one nitrogenous base, and one sugar B) one phosphate, one nitrogenous base, and two sugars C) two phosphates, one nitrogenous base, and one sugar D) two phosphates, one nitrogenous base, and two sugars E) one phosphate, two nitrogenous bases, and one sugar
21)
Base pairs in DNA are held together by
bonds.
A) peptide B) nonpolarcovalent C) polarcovalent D) hydrogen E) sulfhydryl
22)
The duplication of a cell's DNA is called
.
A) mitosis B) replication C) transcription D) translation E) mutation
Version 1
7
23) During replication, each parent DNA strand serves as a DNA strands.
for synthesis of new
A) copy point B) template C) comparisonmolecule D) scaffold E) reservoir
24)
Semiconservative replication refers to
.
A) each base bonding at the 1' position of the sugar B) a purine always bonding to a pyrimidine C) one helix strand that runs from the 5' to 3' direction and the other strand runs from the 3' to 5' direction D) an original parent DNA strand and one newly synthesized DNA strand comprising a new DNA molecule E) None of the choices are correct.
25)
DNA polymerase III
.
A) is needed for adding nucleotides during mRNA synthesis B) synthesizes new DNA only in the 5' to 3' direction C) cannot add nucleotides to the lagging strand D) synthesizes an RNA primer E) All of thechoices are correct.
26)
Okazaki fragments are attached to the growing end of the lagging strand by
Version 1
.
8
A) DNA ligases B) DNA polymerases C) DNA helicases D) DNA gyrases E) primases
27) The enzymes that can proofread replicating DNA, detect incorrect bases, excise them, and correctly replace them are .
A) DNA ligases B) DNA polymerases C) DNA helicases D) DNA gyrases E) primases
28) The site where the old DNA strands separate and new DNA strands will be synthesized is called the .
A) primer B) Okazaki fragment C) template D) rolling circle E) replication fork
29)
Helicase
Version 1
.
9
A) supercoils DNA B) unzips DNA C) unwinds RNA D) winds RNA E) None of the choices are correct.
30)
DNA Polymerase I
.
A) adds bases to new DNA chain B) supercoils DNA C) removes primers D) unzips DNA E) synthesizes RNA primer
31)
DNA Polymerase III
.
A) synthesizes primer B) removes primer C) joins Okazaki fragments D) unzips the DNA helix E) adds new bases and proofreads new DNA
32)
A permanent, inheritable change in the genetic information is called
.
A) translation B) transcription C) mutation D) alteration E) regeneration
Version 1
10
33)
Replication of DNA begins at a(n)
rich area.
A) guanine-cytosine B) uracil-adenine C) adenine-thymine D) adenine-cytosine E) guanine-adenine
34)
For DNA replication, a primer of
is needed at the origin of nucleotide addition.
A) polymeraseIII B) polymerase I C) helicase D) RNA E) DNA
35)
Three consecutive bases in the DNA of a gene represent the code for one
.
A) protein B) nucleotide C) amino acid D) purine E) pyrimidine
36)
A three base sequence on mRNA is called a(n) A) B) C) D)
Version 1
.
codon exon anticodon intron
11
37) The RNA molecules that carry amino acids to the ribosomes during protein synthesis are called .
A) ribosomal RNA B) messenger RNA C) transfer RNA D) primer RNA E) ribozymes
38)
Which molecule is synthesized as a copy of a gene on the DNA template strand? A) Ribosomal RNA B) Messenger RNA C) Transfer RNA D) Primer RNA E) Ribozymes
39)
RNA molecules differ from DNA molecules because only RNA
.
A) has ribose B) has uracil C) is typically one strand of nucleotides D) does not have thymine E) All of the choices are correct.
40)
All of the following pertain to transcription except
Version 1
.
12
A) occurs on a ribosome in the cytoplasm B) occurs before translation C) requires RNA polymerase D) requires a template DNA strand E) proceeds in the 5' to 3' direction of the growing mRNA molecule
41) The nontranscribed region of DNA to which RNA polymerase binds to initiate transcription is called the .
A) promoter B) operator C) operon D) exon E) intron
42)
Which of the following is incorrect about transfer RNA?
A) It has a bottom hairpin loop with an anticodon. B) An anticodon is complementary to a codon. C) It contains a binding site for an amino acid. D) The initiator tRNA that binds to the P site has the anticodon UAC. E) The initiator tRNA in bacteria carries tryptophan.
43)
If the mRNA codon for alanine is GCA, then the correct anticodon would be
Version 1
.
13
A) GCA B) CGT C) ACG D) CGU E) UGC
44)
The following pertain to ribosomes during protein synthesis except
.
A) they participate in both transcription and translation B) they participate only in translation C) they scans the mRNA for an AUG codon D) they are composed of ribosomal RNA and protein E) they shift to the right along the mRNA strand from one codon to the next
45)
Which of the following is incorrect about termination codons?
A) Where the bond between the final tRNA and the finished polypeptide chain is broken B) Include AUG C) Include UAA, UAG, and UGA D) Do not have corresponding tRNA E) Can also be called nonsense codons
46)
Which of the following is not associated with a bacterial ribosome? A) Is a 70S B) Has a peptide (P) site C) Has an exit site D) Small unit provides the enzymes for making peptide bonds E) Has an amino acid (A) site
Version 1
14
47)
A sequence of bases on a eukaryotic gene that does not code for protein is called a(n) . A) promoter B) operator C) operon D) exon E) intron
48)
Split genes
.
A) are common in bacteria and eukaryotes B) only have exons initially transcribed to mRNA C) have introns located only at the beginning and end of a coding region D) use spliceosomes to excise introns and then join exons E) All of thechoices are correct.
49)
Which is incorrect about inducible operons?
A) Have genes turned off by a buildup of end product B) Are often for catabolic pathways C) Are normally turned off D) Are turned on by the substrate of the enzyme E) Include the lac operon
50)
The gene that codes for a protein capable of repressing an operon is called the
Version 1
.
15
A) operator B) structural locus C) regulator D) promoter E) terminator
51)
Synthesis of an inducible enzyme requires
.
A) repressor alone bound to operator B) substrate bound to repressor C) substrate bound to promoter D) corepressor and repressor binding to operator E) None of the choices are correct.
52)
Full induction of the lactose operon requires
.
A) lactose present B) lactose and glucose present C) lactose present without glucose D) lactose and arabinose present E) All of the choices induce the lactose operon
53)
The lactose repressor protein
.
A) is transcribed with the structural lac genes B) is activated by binding lactose C) is inactivated by binding lactose D) requires lactose for its transcription E) None of the choices are correct.
Version 1
16
54) Gene regulation can involve a protein repressor that blocks transcription.
from initiating
A) DNA polymerase I B) DNA polymerase III C) RNA polymerase D) mRNA E) rRNA
55)
A mutation that changes a normal codon to a stop codon is called a
.
A) point mutation B) silent mutation C) back mutation D) missense mutation E) nonsense mutation
56)
The most serious type of mutation is a
.
A) point mutation B) silent mutation C) frameshift mutation D) back mutation E) All the choices are equally serious
57) If the wild-type DNA sequence reads THE CAT ATE THE BIG RAT, what type of mutation would change the sequence to THE CAT ATA ETH EBI GRA T?
Version 1
17
A) Missense B) Nonsense C) Insertion D) Deletion E) Silent
58) If the wild-type DNA sequence reads THE CAT ATE THE BIG RAT, what type of mutation would change the sequence to THE CAT ATE THE BAG RAT? A) Missense B) Nonsense C) Insertion D) Deletion E) Silent
59)
A frameshift is caused by
mutations.
A) missense andinsertion B) missense andnonsense C) nonsense anddeletion D) deletion andinsertion E) insertion andnonsense
60) If the wild-type DNA sequence reads THE CAT ATE THE BIG RAT, what type of mutation would change the sequence to THE CAT ATE (stop)?
Version 1
18
A) Missense B) Nonsense C) Insertion D) Deletion E) Silent
61)
What type of mutation alters the base, but not the amino acid being coded for?
A) Silent B) Back C) Point D) Nonsense E) Missense
62)
The process in which mutations are removed and the correct bases added is called .
A) transduction B) excision repair C) frameshift D) back-mutation E) transformation
63)
Bacterial conjugation involves
Version 1
.
19
A) bacteriophage carrying donor DNA to the recipient cell B) a donor cell with a plasmid that synthesizes a pilus C) naked DNA fragments from a lysed donor cell taken up by a recipient cell D) new progeny cells with genes from two parent bacterial cells E) None of the choices are correct.
64) Which cell can transfer chromosomal and plasmid genes to a recipient cell by conjugation? A) F + cell B) F - cell C) Hfr cell D) R cell E) B cell
65)
Which of the following is not true of conjugation?
A) It involves direct contact between cells. B) It transfers genes for drug resistance. C) It transfers genes for resistance to heavy metals. D) It transfers genes for a polysaccharide capsule. E) The donor retains a copy of the transferred genes.
66) The transfer of DNA fragments from a dead cell to a live, competent recipient cell that results in a changed recipient cell is .
Version 1
20
A) transduction B) conjugation C) transformation D) transmission E) mitosis
67) The development of virulent, toxin-producing bacterial strains due to the presence of a temperate phage can occur in .
A) bacterial conjugation B) transformation C) generalized transduction D) specialized transduction E) All of thechoices are correct.
68) A bacteriophage transfers DNA of the previous host to the current host. This is an example of .
A) conjugation B) generalized transduction C) specialized transduction D) a reaction of an Hfr cell E) None of the choices are correct.
69)
The jumping of a gene from one location to another is done by
Version 1
.
21
A) conjugation B) transposons C) transformation D) transduction E) transmission
70)
Which of the following is not true of transposons?
A) Can change pigmentation B) Can replace damaged DNA C) Can transfer drug resistance D) Can change the genome E) Are always part of plasmids
71)
The various techniques by which scientists manipulate DNA in the lab are called .
A) genetic engineering B) biotechnology C) recombinant DNA D) gel electrophoresis E) gene probes
72)
A technique that separates a readable pattern of DNA fragments is
Version 1
.
22
A) genetic engineering B) biotechnology C) recombinant DNA D) gel electrophoresis E) gene probes
73)
DNA strands can be clipped crosswise at selected positions by using enzymes called .
A) palindromes B) reverse transcriptase C) restriction endonucleases D) ligases E) DNA polymerases
74) Geneticists can make complimentary DNA from messenger, transfer, and ribosomal RNA by using . A) palindromes B) reverse transcriptase C) restriction endonucleases D) ligases E) DNA polymerases
75)
EcoRI and HindIII are
Version 1
.
23
A) palindromes B) reverse transcriptase C) restriction endonucleases D) ligases E) DNA polymerases
76) Sequences of DNA that are identical when read from the 5' to 3' direction on one strand and the 3' to 5' direction on the other strand are .
A) palindromes B) reverse transcriptase C) restriction endonucleases D) ligases E) DNA polymerases
77)
Analysis of DNA fragments in gel electrophoresis involves
.
A) larger fragments that move slowly and remain closer to the wells B) DNA that has an overall negative charge and moves to the positive pole C) DNA fragments that are stained to see them D) an electric current through the gel that causes DNA fragments to migrate E) All of the choices are correct
78)
DNA is denatured at
Version 1
.
24
A) 37oC B) 42oC C) 60oC D) 94oC E) 100oC
79)
DNA fragments can be separated in gel electrophoresis because
.
A) nitrogenous bases have a net positive charge B) nitrogenous bases have a net negative charge C) phosphate groups have a net positive charge D) phosphate groups have a net negative charge E) None of the choices are correct
80)
Restriction endonucleases recognize and clip at DNA base sequences called
.
A) codons B) palindromes C) introns D) exons E) genes
81) In recombinant DNA technique, what enzyme is needed to seal the sticky ends of genes into plasmids or chromosomes?
Version 1
25
A) DNA polymerase I B) DNA polymerase II C) DNA helicase D) DNA ligase E) Primase
82)
Amplification of DNA is accomplished by
.
A) polymerase chain reaction B) DNA sequencing C) gene probes D) southern blot E) western blot
83)
The size of DNA is often given in the number of
that it contains.
A) genes B) codons C) base pairs D) proteins E) triplets
84)
DNA polymerases used in PCR
.
A) use an RNA template to make complementary DNA B) must remain active at very cold temperatures C) include Taq polymerase D) are labeled with fluorescent dyes E) All of the choices are correct
Version 1
26
85)
Which PCR step causes the denaturation of double-stranded DNA?
A) Add DNA polymerase and nucleotides at 72°C B) Cool DNA to between 50°C and 65°C C) Add primers D) Heat target DNA to 94°C E) Repeat the cycle of heating and cooling
86)
Which PCR step synthesizes complimentary DNA strands?
A) Add DNA polymerase and nucleotides at 72°C B) Cool DNA to between 50°C and 65°C C) Add primers D) Heat target DNA to 94°C E) Repeat the cycle of heating and cooling
87)
The primers in PCR are
.
A) synthetic short strands of DNA (oligonucleotides) B) bacterial enzymes C) short RNA strands D) DNA polymerases E) reverse transcriptases
88) have
If you start with three double-stranded DNA fragments, after four cycles of PCR you will fragments.
Version 1
27
A) 12 B) 24 C) 27 D) 48 E) 81
89) The deliberate removal of genetic material from one organism and combining it with the genetic material of another organism is a specific technique called .
A) polymerase chain reaction B) biotechnology C) recombinant DNA technology D) gel electrophoresis E) gene probes
90)
Common vectors used to transfer a piece of DNA into a cloning host are
.
A) plasmids B) viruses C) bacteriophages D) artificial chromosomes E) All of the choices are correct
91)
Genomic
Version 1
are collections of isolated genes maintained in a cloning host.
28
A) DNA B) libraries C) clones D) digests E) books
92) During DNA replication, synthesis of the leading strand of DNA differs from the synthesis of the lagging strand in that .
A) the leading strand is synthesized continuously toward the replication fork, whereas the lagging strand extends away from the fork and is synthesized in short segments called Okazaki fragments B) the leading strand is synthesized away from the fork and is made in short segments called Okazaki fragments whereas the lagging strand extends continuously toward the replication fork C) the leading strand is made in the 5'-3' direction by DNA polymerase III, whereas the lagging strand is synthesized in the opposite direction, 3'-5' D) only the lagging strand requires a primer for each of the Okazaki fragments; the leading strand does not require a primer to begin synthesis of the daughter strand
93)
Which statement best describes the flow of genetic information in a cell?
A) DNA codes for RNA, which codes for proteins and some information from RNA flows back to DNA in a regulatory role. B) DNA codes for RNA, which codes for proteins. C) DNA synthesizes proteins. D) RNA directs the synthesis of DNA, which codes for proteins.
94)
Which of the following statements applies to both DNA and RNA?
Version 1
29
A) The molecule is synthesized in the 5'–3' direction when a polymerase adds a nucleotide onto the 3' end of the existing polymer. B) The molecule is synthesized in the 5'–3' direction when a polymerase adds a nucleotide onto the 3' end of the existing polymer, creating a bond between the phosphate group and the ribose. C) The molecule is synthesized in the 5'–3' direction when a polymerase adds A, T C, and G nucleotides onto the 3' end of the existing polymer. D) The molecule is synthesized in the 5'–3' direction then, on completion, the single strand folds to form secondary and tertiary level structures.
95)
Which of the following statements best describes the initiation of translation?
A) The mRNA containing the start codon, AUG, sits at the P site and forms a complex with the corresponding tRNA, and the large and small ribosomal subunits. B) A tRNA with the anticodon, AUG, enters the ribosomal complex and binds to the mRNA at the A site. C) The large and small ribosomal subunits scan the mRNA in the 3'–5' direction until the promoter is reached. D) The mRNA attaches to the large ribosomal subunit and once the start codon reaches the A site, the tRNA binds and the small subunit completes the complex.
96)
The benefit of the bacterial operon is that
.
A) since the proteins coded for by the operon structural genes work as a team, they are regulated (i.e. induced or repressed) as a team, thus saving energy B) the structural genes can be regulated individually, affording the cell more control over the protein products C) the split genes can be cut and spliced back together with greater precision than with single genes D) they are much less prone to mutation than individual genes
Version 1
30
97)
A recombinant organism is one that
.
A) has obtained genetic material from another organism that has integrated into the genome, combining the two DNA molecules B) induces a gene that was previously turned off, thereby producing a new phenotype C) obtains genes by conjugation, but not transduction D) has sustained a mutation in the genetic material such that a new phenotype results, for example, the organism develops increased virulence or antibiotic resistance
98) A new field of research called nutrigenomics is based on the premise that individuals gain or lose weight according to differences in their individual genomes, making "going on a diet" no longer a one-size-fits-all solution. This is based on what genomic principle?
A) B) C) D)
99)
The human genome contains 10 million SNPs. All human genes code for the same proteins. Each human genome differs because of frameshift mutations. Human genes and consumption of nutrients are completely unrelated.
The discovery of single nucleotide polymorphisms in the human genome means that .
A) individual risk for specific diseases can be analyzed, and personalized medicine designed as a result of the analysis B) the human species is in grave danger of dying out due to the threat of mutation across the species C) genes are not important in determining the individual phenotypes of individuals, since we are all similar in our growth, metabolism and reproduction D) mutations are far more dangerous than we thought, prior to the Human Genome Project
Version 1
31
100)
Mastery of synthetic biology will allow humans to
.
A) make new organisms using only chemicals as the starting materials B) synthesize chemicals in cells that are missing due to a genetic anomaly C) construct complex molecules in situ that will target cancerous cells in tissues D) design alternative forms of energy production E) All of the choices are possibilities from synthetic biology
101) The emerging field of synthetic biology, while offering promising breakthroughs in medicine and energy production, is not without concern. Apprehension by ethicists is understandable since .
A) B) C) D)
the ability to create life from scratch raises a multitude of issues medical insurance costs will likely skyrocket mutations in manufactured cells will cause problems for existing organisms oil and gas prices will increase as a result of the decreased demand for fossil fuels
102) When bacteriophages invade bacterial cells, they leave behind DNA that is recognized by the bacteria as foreign and can be eliminated in the event of another attack, reminiscent of adaptive immunity. This natural process can be adapted in the lab so that a combination of guide DNA and nucleases facilitate the cutting of DNA at a precise location in any organism. Here we are describing . A) CRISPR B) gene therapy C) recombinant DNA technology D) synthetic biology E) metagenomics
Version 1
32
103)
The basic premise of gene therapy is that
.
A) the error in protein production from a defective gene is overcome by the insertion of the correct gene from a healthy cell into the patient's tissue B) the defective gene is permanently excised from the patient's cells C) genetically engineering the correct gene into the patient's tissues has had no reported safety issues D) the correct gene, once inserted into the affected tissue, will be passed on to the individual's offspring, thus eliminating the disease from future generations
104)
The sum total of genetic material of a cell is referred to as its A) B) C) D)
.
genome phenotype chromosome DNA
105) A specific segment of DNA that contains the necessary code to make a protein or RNA molecule is a . A) B) C) D)
gene chromosome phenotype genotype
106) The genetic makeup of an organism is referred to as its observable traits are referred to as its .
Version 1
, whereas the
33
A) B) C) D)
genotype; phenotype phenotype; genotype chromosomes; gentoype phenotype; traits
107) The strand is synthesized continuously while the in a series of short fragments called Okazaki fragments. A) B) C) D)
strand is synthesized
leading; lagging lagging; leading primary; secondary original; primary
108)
The specific sequence of three bases in mRNA that encodes a particular amino acid is the whereas the complementary sequence of three bases in tRNA that binds to the mRNA is best known as the . A) B) C) D)
109) as a
codon; anticodon anticodon; codon codon; DNA triplet gene; complement
Any organism that contains genes that originated in another organism is best referred to . A) B) C) D)
Version 1
recombinant clone vector conjugate
34
110)
Select the statement that best describes the purpose of gel electrophoresis.
A) Gelelectrophoresis is able to cut DNA in very specific locations. B) Gelelectrophoresis enables the amplification of specific DNA sequences. C) Gelelectrophoresis separates DNA fragments based on size. D) Gelelectrophoresis identifies specific segments of DNA. E) Gelelectrophoresis joins different segments of DNA.
111) A type of recombinant DNA technology procedurewhich involves removing a selected gene of interest from an animal, plant, or microorganism and then inserting and propagating it within a different host organism is known as . A) B) C) D)
112)
cloning conjugation induced mutation transgeneration
Choose the phrase that best describes the role of a cloning vector.
A) Isolates DNA from a donor organism B) Introduces DNA into a cloning host C) Separates fragments of DNA D) Detects the presence of specific DNA sample E) Screens for recombinants that have been successfully transformed
113) Choose the statement that best describes the role of restriction enzymes in genetic engineering.
Version 1
35
A) Restrictionenzymes are used to replicate specific sequences of DNA. B) Restrictionenzymes are used to cut DNA at specific locations. C) Restriction enzymesare used to join different segments of DNA. D) Restrictionenzymes are used to sequence long stretches of DNA. E) Restriction enzymes are used to purifyDNA.
114) NCLEX Prep—Test Bank Question: Please read the clinical scenario, and then answer the questions that follow to become familiar with the traditional NCLEX question format. An 18-month-old male is admitted to the hospital with symptoms of a respiratory infection. The patient’s mother reports that since birth, the toddler has frequently had a cough and copious nasal secretions. His weight gain has been poor, despite his excellent appetite. The patient appears in moderate respiratory distress, so oxygen therapy and intravenous fluids are initiated. The physician orders tests for cystic fibrosis due to the patient’s history and clinical presentation. When the tests confirm the diagnosis, as the RN, youprovide education to the mother regarding this autosomal recessive disease.
114.1) You educate the patient’s mother that the most common cause of cystic fibrosis is a deletion of three nucleotides. Which of the following statements, by the mother, demonstrates an understanding of the disease etiology? A) B) C) D)
"My child’sdisease is caused by a missing chromosome." "My child’sdisease is caused by a missing gene." "My child’sdisease is caused by a missing amino acid." "My child’sdisease is caused by an infectious process."
114.2) Which of the following is the most appropriate term to describe the patient’s clinical manifestations of cough, rhinorrhea, and poor weight gain due to this genetic disease?
Version 1
36
A) B) C) D)
Genotype Phenotype Structural genes Regulatory genes
114.3) The RN is aware that many different types of mutations have been identified in the gene that causes cystic fibrosis. Which of the following terms would best describe a mutation that leads to the deletion of two bases from a DNA sequence? A) B) C) D)
Frameshift mutation Back mutation Silent mutation Point mutation
114.4) The RN is aware that many different types of mutations have been identified in the gene that causes cystic fibrosis. Which of the following terms would best describe a mutation that changes a normal codon into a stop codon, usually resulting in a nonfunctional protein?
A) B) C) D)
Version 1
Missense mutation Nonsense mutation Point mutation Frameshift mutation
37
115) NCLEX Prep - Test Bank Question: Please read the clinical scenario, and then answer the questions that follow to become familiar with the traditional NCLEX question format. A 59-year-old female has been diagnosed with Escherichia coli infection following the consumption of contaminated meat. The patient is exhibiting a high fever, severe abdominal cramping, and diarrhea. The RNprovides supportive care and fluid resuscitation as ordered. The nurse and medical team provide education to the patient and her family regarding the genetics of the E. coli bacterium.
115.1) The RN instructs the family that the transfer of genetic material can involve contact between two bacterial cells. DNA can pass from one cell to the other through a pilus during this process, which is called . A) B) C) D)
transformation conjugation specialized transduction generalized transduction
115.2) In gram-negative bacteria, like E. coli, the fertility factor is a plasmid possessed by the bacterium that will donate its DNA to a recipient cell. Which of the following statements by the patient demonstrates an understanding of this plasmid?
A) B) C) D)
It is a large, double-stranded linear piece of DNA. It is found in F- cells only. It controls the process of transduction in bacterial cells. It encodes the information to create a pilus.
115.3) The RN educates the patient that genetic information can be acquired by bacterial cells in other ways. Which of the following is a correct statement regarding these processes?
Version 1
38
A) Transformation requires the formation of a pilus for genetic exchange to occur. B) Transposition involves the transfer of genetic information to bacterial cells directly from their environment. C) Genetic transfer of transposons involves a random fragment of DNA being taken up by a phage. D) Transduction involves the transfer of genetic information from one bacterial cell to another using a bacteriophage as the vector.
116) NCLEX Prep—Test Bank Question: Please read the clinical scenario, and then answer the questions that follow to become familiar with the traditional NCLEX question format. An RN working in a research unit is responsible for data and specimen collection for multiple research studies. A patient is admitted as a participant in a study focused on the role of tRNA mutations in Alzheimer’s disease. The 78-year-old male is accompanied by his wife and daughter. Prior to obtaining the first blood specimen, the study coordinator provides education regarding the importance of the study and the overall process of protein synthesis.
116.1) Which of the following statements by the patient’s daughter, demonstrates her understanding of the function of tRNA?
A) B) C) D)
tRNA possesses an anticodon complementary to an mRNA codon. tRNA is an exact copy of a gene’s DNA sequence. tRNA is a structural component of the ribosome. tRNA is responsible for regulating transcription and translation.
116.2)
Version 1
tRNA plays a role in which of the following processes of protein synthesis?
39
A) B) C) D)
Translation Transcription Replication Transduction
116.3) Which tRNA anticodon would properly base-pair with an mRNA codon of cytosine-uracil-guanine (CUG)? A) B) C) D)
GTC ATC GAC UCA
116.4) During protein synthesis, the final mRNA codon on the strand, UAA, recruits which of the following tRNA molecules to the ribosomal complex?
A) B) C) D)
117)
The smallest unit of heredity is a chromosome. ⊚ ⊚
118)
tRNA carryingmethionine tRNA carryingproline tRNA carryingarginine No correspondingtRNA exists for this codon.
true false
DNA replication proceeds in one direction around the bacterial chromosome. ⊚ ⊚
Version 1
true false 40
119)
Transcription occurs in the nucleus or at the nucleoid. ⊚ ⊚
120)
In bacteria, initiator tRNA carries formyl methionine. ⊚ ⊚
121)
true false
Repressible operons are normally turned on in the cell. ⊚ ⊚
124)
true false
Lactose is a corepressor in the lac operon. ⊚ ⊚
123)
true false
Excised introns form structures called lariats. ⊚ ⊚
122)
true false
true false
Induced mutations result from errors in DNA replication. ⊚ ⊚
Version 1
true false
41
125) DNA photolyase, in the presence of visible light, can repair DNA damage due to ultraviolet radiation. ⊚ ⊚
true false
126)
All mutations are detrimental to the cell. ⊚ true ⊚ false
127)
Restriction endonucleases are obtained from various species of bacteria. ⊚ ⊚
128)
When DNA is heated to 94 oC, the two strands will separate. ⊚ ⊚
129)
true false
true false
Reverse transcriptase is used to make cDNA from an RNA template. ⊚ ⊚
true false
130) After three replication cycles in PCR, there will be a total of three double-stranded DNA molecules. ⊚ true ⊚ false Version 1
42
131)
Viruses are often used as cloning hosts in recombinant DNA methods. ⊚ ⊚
true false
132) Vectors often contain a gene conferring drug resistance to their cloning host, in order to detect cells harboring the plasmid. ⊚ ⊚
true false
133) The lagging strand of DNA is made in small segments called Okazaki fragments, each one requiring a primer to begin synthesis, whereas the leading strand, synthesized continuously, does not require a primer to initiate the daughter strand. ⊚ ⊚
true false
134) A recombinant organism is one that has sustained a change in its genome through receipt of DNA from a different organism. ⊚ ⊚
true false
135) Synthetic biology is an emerging field of science whereby researchers are creating new molecules and organisms from an array of chemicals, rather than as a result of cellular reproduction. ⊚ ⊚
Version 1
true false
43
136) In the case of somatic cell gene therapy, the corrected gene will be passed on through the gametes, preventing the manifestation of the disease from recurring in the patient's offspring. ⊚ ⊚
137)
true false
Mutations are the primary source of genetic variation that makes evolution possible. ⊚ ⊚
Version 1
true false
44
Answer Key Test name: Ch 8 4e 1) [A, C, D, E, F] 2) [A, B] 3) [B, C, E] 4) [B, D, E, F] 5) [A, D, F] 6) [A, B, E] 7) [A, C, D] 8) [A, B, C] 9) [A, B, C, F, G] 10) E 11) E 12) E 13) D 14) C 15) D 16) C 17) A 18) C 19) B 20) A 21) D 22) B 23) B 24) D 25) B 26) A Version 1
45
27) B 28) E 29) B 30) C 31) E 32) C 33) C 34) D 35) C 36) A 37) C 38) B 39) E 40) A 41) A 42) E 43) D 44) A 45) B 46) D 47) E 48) D 49) A 50) C 51) B 52) C 53) C 54) C 55) E 56) C Version 1
46
57) C 58) A 59) D 60) B 61) A 62) B 63) B 64) C 65) D 66) C 67) D 68) B 69) B 70) E 71) A 72) D 73) C 74) B 75) C 76) A 77) E 78) D 79) D 80) B 81) D 82) A 83) C 84) C 85) D 86) A Version 1
47
87) A 88) D 89) C 90) E 91) B 92) A 93) A 94) A 95) A 96) A 97) A 98) A 99) A 100) E 101) A 102) A 103) A 104) A 105) A 106) A 107) A 108) A 109) A 110) C 111) A 112) B 113) B 114) Section Break 114.1) C 114.2) B Version 1
48
114.3) A 114.4) B 115) Section Break 115.1) B 115.2) D 115.3) D 116) Section Break 116.1) A 116.2) A 116.3) C 116.4) D 117) FALSE 118) FALSE 119) TRUE 120) TRUE 121) TRUE 122) FALSE 123) TRUE 124) FALSE 125) TRUE 126) FALSE 127) TRUE 128) TRUE 129) TRUE 130) FALSE 131) FALSE 132) TRUE 133) FALSE 134) TRUE 135) TRUE Version 1
49
136) FALSE 137) TRUE
Version 1
50
CHAPTER 9 1)
From the list below, choose the two most resistant microbial forms. A) Prions B) Vegetative bacteria C) Fungal spores D) Endospores E) Enveloped viruses
2)
Which of the following are correctly matched? Check all that apply. A) B) C) D)
Bactericidal—inhibits the growth of bacteria Bactericidal—kills bacteria Bacteriostatic—inhibits the growth of bacteria Bacteriostatic—kills bacteria
3) Which of the following represent the cellular targets for physical and chemical control agents? A) Cell membrane B) Cellular synthesis of DNA and RNA C) Proteins D) Lysosomes E) Cell wall
4)
Which of the following are physical control methods? Check all that apply.
Version 1
1
A) Phenolics B) Cold C) Dry and moist heat D) Radiation E) Halogens F) Desiccation
5)
Select the methods below that illustrate the use of moist heat to control microbes.
A) Steam autoclave B) Incineration C) Pasteurization D) Desiccation E) Boiling water
6)
Select two methods that can be used for long-term preservation of microbial cultures.
A) Irradiation B) Deep freezing (-70oC to -135oC) C) Filtration D) Pasteurization E) Lyophilization
7) Ionizing and nonionizing radiation can be used in microbial control. Which of the following statements are true regarding the use of radiation?
Version 1
2
A) Ionizing radiation has good penetrating power. B) Ionizing radiation can only disinfect materials because it cannot penetrate solid substances. C) Ionizing radiation can be used to sterilize heat-sensitive materials. D) UV light is an example of nonionizing radiation. E) UV light is an example of ionizing radiation. F) Nonionizing radiation cannot penetrate solid objects.
8)
Select those items in which microbial growth is controlled by means of osmotic pressure.
A) Honey B) Apple cider C) Salt-cured meats D) Milk E) Yogurt F) Jellies
9)
Select those characteristics of a chemical control agent that make it a desirable agent.
A) Corrosive to metals and plastics B) Water or alcohol soluble and stable C) Microbicidal to a wide range of microbes, but nontoxic to humans and animal tissue D) Rapidly effective in low concentrations E) Ineffective in the presence of organic matter F) Inexpensive G) Short lived, spontaneously inactivate with time
10)
Select the advantages of phenolic compounds as antimicrobial agents.
Version 1
3
A) B) C) D)
11)
Remain effective in the presence of organic matter Generally nontoxic Strongly microbicidal Effective against prions
Select the appropriate uses of hydrogen peroxide for antimicrobial purposes.
A) Mouthwashes B) Skin antisepsis prior to surgery C) Sterilization of laboratory glassware D) Low temperature sterilizing cabinets for colonoscopes E) Wound cleansing F) Disinfection of countertops and other environmental surfaces
12)
Select the applications of ethylene oxide sterilization.
A) B) C) D)
Plastics Prepackaged medical devices Bacteriologic growth media Milk and dairy products
13) Which of the following microbial forms have the highest resistance to physical and chemical controls?
A) Naked viruses B) Protozoan cysts C) Fungal spores D) Bacterial endospores E) Yeast
Version 1
4
14) The process that destroys or removes all microorganisms and microbial forms including bacterial endospores on inanimate objects is .
A) disinfection B) sterilization C) antisepsis D) sanitization E) degermation
15) The process of using a cleansing technique to mechanically remove and reduce microorganisms and debris to safe levels is .
A) disinfection B) sterilization C) antisepsis D) sanitization E) degermation
16)
The use of a physical or chemical process to destroy vegetative pathogens is
.
A) disinfection B) sterilization C) antisepsis D) sanitization E) degermation
17) The use of chemical agents directly on exposed body surfaces to destroy or inhibit vegetative pathogens is .
Version 1
5
A) disinfection B) sterilization C) antisepsis D) sanitization E) ionization
18) Scrubbing or immersing the skin in chemicals to reduce the numbers of microbes on the skin is .
A) disinfection B) sterilization C) antisepsis D) sanitization E) ionization
19)
Which of the following types of control agents would be used to achieve sterility?
A) Virucide B) Bactericide C) Germicide D) Sporicide E) Fungicide
20)
The easiest microbial forms to kill or inhibit are
Version 1
.
6
A) prions B) vegetative bacteria and fungi C) endospores D) protozoan cysts E) mycobacteria and staphylococci
21)
The method of removing vegetative microbial life forms from inanimate objects is termed .
A) antisepsis B) disinfection C) sterilization D) decontamination E) degerming
22)
The method of removing vegetative life forms from living surfaces is termed
.
A) antisepsis B) disinfection C) sterilization D) decontamination E) degerming
23)
The removal of all life forms from inanimate objects is termed
Version 1
.
7
A) antisepsis B) disinfection C) sterilization D) decontamination E) degerming
24)
The betadine swab before blood donation is an example of
.
A) antisepsis B) disinfection C) sterilization D) decontamination E) sanitation
25)
The alcohol wipe before an injection is an example of
.
A) antisepsis B) disinfection C) sterilization D) decontamination E) virilization
26) A cleansing method that mechanically removes microbes and other debris to reduce contamination is .
Version 1
8
A) disinfection B) sterilization. C) antisepsis D) sanitization E) degermation
27)
Which is correct regarding the rate of microbial death?
A) Cells die atincreasingly greater rates. B) Only older cellsdie in a culture. C) Cells in aculture die at a constant rate. D) Upon contact withthe control agent, all cells die at one time. E) Cells becomemetabolically inactive, but are never killed.
28)
Which of the following factors will influence the action of antimicrobial agents?
A) The number of microorganisms B) The kind of microorganisms C) Temperature and pH D) Mode and dosage of the agent E) All of these willinfluence the action.
29)
Microbial death occurs when there is
.
A) no movement B) permanent loss of reproduction C) a change in appearance D) a decrease in size E) All of these occur
Version 1
9
30)
Each of the following is the target of antimicrobial agents except
.
A) cell walls B) cell membranes C) ribosomes D) cellular proteins E) cytoplasm
31)
Surfactants work by
.
A) coating the organism, preventing interaction with its environment B) blocking transport into the organism C) blocking transport out from the organism D) disrupting membrane integrity E) All of the choices are correct
32) Some microbial control agents are able to cell proteins by breaking bonds that maintain the native state (three-dimensional configuration) of the proteins.
A) denature B) bind C) dissolve D) activate E) All of thechoices are correct.
33)
Agents that can denature microbial proteins include all of the following except
Version 1
.
10
A) moist heat B) alcohol C) acids D) metallic ions E) X rays
34)
Which of the following does not affect microbial nucleic acids?
A) Moist heat B) Ultraviolet light C) X rays D) Ethylene dioxide E) Formaldehyde
35)
Physical agents for controlling microbial growth include all the following except .
A) ultraviolet radiation B) boiling water C) HEPA filters D) pasteurization E) hydrogen peroxide
36)
Sterilization is achieved by
Version 1
.
11
A) flash pasteurization B) hot water C) boiling water D) steam autoclave E) All of the choices are correct
37)
Dry heat
.
A) is less efficient than moist heat B) cannot sterilize C) includes tyndallization D) is used in devices called autoclaves E) will sterilize at 121°C for 15 minutes
38)
The most efficient sterilizing conditions in a steam autoclave are
.
A) 121°C at 15 psi for 15 minutes B) 63°C for 30 minutes C) 160°C for 2 hours D) 71.6°C for 15 seconds E) 100°C for 30 minutes
39) The shortest time required to kill all the microbes in a sample at a specified temperature is called the .
Version 1
12
A) thermal death point (TDP) B) thermal death time (TDT) C) sporicidal time D) death phase point E) None of the choices are correct
40)
The lowest temperature needed to kill all microbes in 10 minutes is the
.
A) thermal death point (TDP) B) thermal death time (TDT) C) sporicidal time D) death phase point E) None of the choices are correct
41)
Disinfection of beverages, such as apple juice, milk, and wine, is optimally achieved by . A) pasteurization B) chlorination C) moist heat autoclave D) hydrogen peroxide E) boiling water
42)
Placing organisms at 4
Version 1
o
C is
.
13
A) bactericidal B) bacteriostatic C) decontamination D) sterilization E) None of the choices are correct
43)
Pasteurization
.
A) kills all vegetative forms B) reduces the number of vegetative forms C) reduces the number of endospores D) increases food nutrient value E) None of the choices are correct
44)
heat is more rapidly effective and efficient compared to
heat.
A) High; dry B) High; moist C) Dry; moist D) Moist; dry E) Moist; high
45)
A method for sterilizing milk, called
treatment, uses 134°C for 1 to 2 seconds.
A) pasteurization B) batchpasteurization C) flashpasteurization D) ultra hightemperature E) None of the choices are correct
Version 1
14
46)
What instrument is most effective for pressure-temperature sterilization?
A) Oven B) Autoclave C) Water bath D) Bunsen burner E) Incubator
47) Vials of microorganisms that undergo the freeze-drying process called remain preserved and viable for years.
will
A) desiccation B) flash freeze C) lyophilization D) pasteurization E) sterilization
48)
Removal of moisture by dehydration is called
.
A) desiccation B) flash freeze C) lyophilization D) pasteurization E) sterilization
49) radiation excites atoms to a higher energy state within molecules such as DNA that then leads to the formation of pyrimidine dimers.
Version 1
15
A) Infrared B) Ultraviolet C) Gamma D) Particle E) Ionizing
50)
Which of the following items are typically irradiated in order to kill microbes?
A) Meats like ground beef and pork B) Human tissues such as heart valves and skin C) Operating room air D) Surgical gloves E) All of thechoices are correct.
51) Which control method would not be a suitable choice for killing Mycobacterium in a capped culture tube?
A) Ultraviolet (germicidal) light B) Gamma rays C) 121°C at 15 psi for 15 minutes D) 160°C for 2 hours E) All of the choices would kill Mycobacterium in a culture tube.
52) Which ofthe following forms of radiation is in order from the most penetrating to the least penetrating?
Version 1
16
A) Gamma, cathode, X rays B) Gamma, X rays, cathode C) Cathode, gamma, X ray D) Cathode, X ray, gamma E) X ray, gamma,cathode
53)
HEPA filters are used to remove microbes from
.
A) air B) liquids C) human tissues D) medical instruments E) All of the choices are correct
54) killing them.
is a control method that removes microorganisms rather than inhibiting or
A) Boiling B) Sterilization C) Radiation D) Filtration E) Disinfection
55)
Which of the following is not a factor that affects germicidal activity?
Version 1
17
A) The material being treated B) The length of exposure C) The strength of the germicide D) The microorganism being treated E) All of these choices are factors.
56)
All of the following pertain to hypochlorites except
.
A) release hypochlorous acid in solution B) cause denaturation of enzymes C) found in iodophors D) used to disinfect water, restaurant, and medical equipment E) found in common household bleach
57) is a halogen used in gaseous and liquid form for large-scale disinfection of drinking water and sewage. A) Iodine B) Chlorine C) Bromine D) Fluorine E) All of thechoices are correct.
58) The compound that is an organic base containing chlorine and two phenolic rings, and is used increasingly for mucous membrane irrigation, obstetrical washes, hand scrubbing, and prepping surgical skin sites is .
Version 1
18
A) carbolic acid B) chlorhexidine C) triclosan. D) formalin E) quarternary ammonium compounds
59)
Alcohols
.
A) dissolve membrane lipids at concentrations of greater than 50% B) can be used for disinfection or antisepsis C) are skin degerming agents D) are limited in effectiveness due to rapid evaporation E) All of the choices are correct
60)
The chemical agent that produces highly toxic and reactive free radicals is
.
A) cidex B) cationic detergents C) hydrogen peroxide D) chlorhexidine E) iodophors
61)
Hydrogen peroxide can be
.
A) sporicidal B) fungicidal C) bactericidal D) virucidal E) All of the choicesare correct
Version 1
19
62)
All of the following disrupt the cytoplasmic membrane except A) B) C) D)
63)
.
detergents quaternary ammonium compounds high concentration phenols iodine
Heavy metals work by
.
A) rupturing the cell membrane B) inactivating proteins C) binding to DNA D) dissolving the cell wall E) mutating DNA
64)
Which of the following is not used as an antiseptic?
A) Iodophor B) Chlorhexidine C) 3% hydrogen peroxide D) Betadine (povidone-iodine) E) Glutaraldehyde
65)
Ethylene oxide is
Version 1
.
20
A) sporicidal B) only effective with high heat C) the active agent in household bleach D) used as an antiseptic against anaerobes E) a halogen
66)
Which of the following acids is not used to destroy or inhibit microbial cells in food?
A) Acetic acid B) Benzoic acid C) Lactic acid D) Phosphoric acid E) Propionic acid
67)
In lab, inoculating loops are sterilized using
.
A) moist heat B) chemicals C) incineration D) filtration E) ethylene oxide
68)
Antimicrobial agents can target the cell wall by
A) B) C) D)
Version 1
.
blocking its synthesis digesting it inhibiting peptidoglycan synthesis All of the choices are correct.
21
69) Which of the following represents the use of osmotic pressure as a microbial control method?
A) B) C) D)
70)
Bleaching a kitchen counter Salting meat Rinsing a cut with Betadine Both bleaching akitchen counter and rinsing a cut with Betadine are correct.
Which pair of physical control methods will achieve complete sterilization?
A) Autoclaving and ionizing radiation B) Ultraviolet radiation and filtration C) Freezing and filtration D) Osmotic pressure and incineration E) Pasteurization and ultraviolet radiation
71) Which of the following statements is true when comparing moist heat methods of microbial control to dry heat control?
A) B) C) D)
Both moist heat and dry heat methods can achieve complete sterilization. Only moist heat (autoclaving) can sterilize objects, dry heat is limited to disinfection. Dry heat is more efficient than moist heat, requiring less time to sterilize objects. Boiling is a more successful method of sterilizing instruments than using a dry heat
oven.
72) Temperature and time must be considered when determining susceptibility of microbes to lethal conditions. At a specific temperature, the thermal death can be determined, alternatively, the thermal death can be assessed as the lowest temperature that will kill a test organism in 10 minutes. Version 1
22
A) B) C) D)
time; point point; time exposure, time point; exposure
73) All of the following are accurate statements concerning microbial control using dry heat except .
A) it is the preferred method of control to sterilize cottons, plastics and paper B) it takes up to 4 hours to sterilize instruments in a hot-air oven, whereas the Bunsen burner will incinerate an inoculating loop almost immediately C) the Bunsen burner can achieve temperatures up to 1,870 oC, whereas the hot-air oven reaches temperatures of up to 180 oC D) while fast and effective, the Bunsen burner is limited to metals and glassware, whereas powders and oils can also be sterilized in the hot-air oven
74) Beginning Fall 2017, the FDA banned the use of phenolic compounds, such as triclosan, in consumer soaps and lotions. This is due to .
A) B) C) D)
75)
increased microbial resistance to these chemicals they only work against bacterial cells, not fungi or viruses theinefficiency of phenolics as antimicrobials a worldwide shortage of triclosan
Chlorhexidine kills microbes by
Version 1
.
23
A) targeting cell membranes, cell walls and denaturing proteins B) disrupting cell surface tension C) inactivating enzymes by binding to the active site D) blocking DNA replication E) forming free radicals that create thymine dimers in the DNA
76) All of the following are disadvantages to using compounds containing heavy metals, except .
A) they can only be taken in pill form, not applied as a lotion or ointment B) they may cause allergic reactions in susceptible individuals C) organisms can develop resistance to the compounds D) they are not effective against endospores E) they can be toxic
77)
The use of aldehydes as antimicrobial agents is limited due to
A) B) C) D)
.
their instability when pH or temperature is increased their explosive nature they will disinfect, but not sterilize they are ineffective in the presence of organic matter
78) One disadvantage to using ortho-phthalaldehyde vs. glutaraldehyde as an antimicrobial agent is .
A) B) C) D)
Version 1
increased cost decreased efficacy decreased safety increased toxicity
24
79) The advantage of using ethylene oxide as a sterilant over moist heat sterilization in an autoclave is that .
A) B) C) D)
80)
it kills endospores, whereas the autoclave will only kill vegetative cells it is much safer for humans to work with ethylene oxide than to operate an autoclave it can be used to sterilize plastics that would melt in an autoclave All of the choices are reasons to preferentially use ethylene oxide as a sterilant
A(n) would be used to destroy bacteria on a countertop whereas a(n) would be used on skin prior to making an incision. A) B) C) D)
disinfectant; antiseptic antiseptic; disinfectant antiseptic; sterilant sterilant; disinfectant
81) Select the microorganism that is least resistant to chemical and physical control measures.
A) Protozoan cysts B) Enveloped viruses C) Gram-negative bacteria D) Nonenveloped (naked) viruses E) Gram-positive bacteria
82) The shortest length of time required to kill all test microbes at a specified temperature is the thermal death whereas the lowest temperature required to kill all microbes in a sample within 10 minutes is the thermal death .
Version 1
25
A) B) C) D)
time; point window; point time; temperature point; time
83) Choose the statement that identifies the major disadvantage of both cold and desiccation in terms of microbial control.
A) B) C) D)
These methods are technically difficult and labor intensive. These methods mayencourage growth of microbial populations. These methods arebacteriostatic rather than bactericidal. These methods produce harmful by-products and are costly.
84) Choose the microbial control method that neither inhibits nor kills microbes, but instead physically removes them from liquids or air.
A) Desiccation B) Lyophilization C) Filtration D) Irradiation E) Cold sterilization
85)
Select the most disadvantageous characteristic of phenolics.
A) Inactivated by organic matter B) Limited in their antimicrobial spectrum C) Toxicity D) Expensive E) Difficult to use
Version 1
26
86)
Select the statement that describes the antimicrobial activity of chlorhexidine.
A) B) C) D)
Targets cell membrane components and denatures proteins Disrupts peptidoglycan structure Destroys endospores Interfere with nucleic acid replication and transcription
87) Choose the method used to sterilize a loop before and after inoculating a culture in the laboratory. A) Hot air B) Steamunder pressure C) Incineration D) Radiation E) Pasteurization
88) NCLEX Prep—Test Bank Question: Please read the clinical scenario, and then answer the questions that follow to become familiar with the traditional NCLEX question format. An RN in a long-term care facility oversees the care of 15 patients. He observes precautions to control and prevent the spread of infection.
88.1) The RN practices proper handwashing technique when his hands are visibly dirty. Which of the following terms most appropriately describes the type of antimicrobial control provided by handwashing with germicidal soap and water?
Version 1
27
A) B) C) D)
Sterilization Disinfection Decontamination Antisepsis
88.2) The RN utilizes an alcohol-based hand cleanser with friction rub when his hands are not visibly dirty. Which of the following terms appropriately describes the type of antimicrobial control provided by alcohol-based hand cleansers?
A) B) C) D)
Sterilization Disinfection Decontamination Antisepsis
88.3) Which of the following scenarios are most appropriate for handwashing with an alcohol-based hand cleanser instead of soap and water?
A) B) C) D)
Following a diaper change with visible soiling on the hands after glove removal Following a wound dressing change when a glove tore during the procedure Prior to applying gloves Prior to returning to work after eating lunch
88.4)
Which of the following microbes are targeted by alcohol-based hand cleaners?
A) B) C) D)
Version 1
Bacteria Viruses Fungi Bacteria, viruses, and fungi
28
89) NCLEX Prep—Test Bank Question: Please read the clinical scenario, and then answer the questions that follow to become familiar with the traditional NCLEX question format. A 13-year-old male is admitted to the pediatric intensive care unit. The patient has a history of intellectual disability and cerebral palsy following an anoxic injury as an infant. The patient was admitted two months earlier due to aspiration pneumonia, necessitating a lengthy hospital stay, mechanical ventilation, and intravenous antibiotics. His caregiver reports another aspiration event two days prior. For the past 24 hours, the patient has been lethargic with a frequent cough producing thick sputum. Upon presentation to the hospital, the patient is noted to have a fever, low oxygen saturations, and low blood pressure. Upon arrival to the intensive care unit, the patient is cool, pale, and has poor pulses and capillary refill. Following administration of intravenous fluid boluses, it is determined the patient needs central venous access for fluid and medication administration. The RN prepares for the sterile procedure.
89.1) To reduce the patient’s risk of infection, the RN assists the physician with applying all of the following prior to the procedure except .
A) B) C) D)
sterile gloves clean gown cap mask
89.2) Prior to placement of the central venous line, the patient’s skin is scrubbed with chlorhexidine. Which of the following microorganisms are targeted by this chemical agent?
A) B) C) D)
Version 1
Bacteria Viruses Fungi Bacteria, viruses, and fungi
29
89.3) If chlorhexidine is unavailable for the procedure, which of the following chemical agents would also be appropriate as a topical antiseptic?
A) B) C) D)
2% iodine 10% potassiumiodide Formaldehyde Phenol
89.4) Following successful placement of the central venous line, the RN is diligent to prevent infection when accessing the catheter and administering medications. Which of the following actions should be completed prior to use of the line?
A) Hand washing B) Disinfection of the access port with alcohol prior to use C) Application of gloves D) Hand washing, application of gloves, and disinfection of the access port with alcohol should all be completed prior to use of the line.
90) NCLEX Prep—Test Bank Question: Please read the clinical scenario, and then answer the questions that follow to become familiar with the traditional NCLEX question format. A woman who is 38 weeks pregnant is admitted for a scheduled cesarean section. The RN performs an assessment of the patient and prepares her for the surgery. Prior to the procedure, an indwelling urinary catheter is placed.
90.1) The RN prepares the patient for the sterile procedure. Which of the following statements made by the patient, best demonstrates understanding of the need for sterile equipment and aseptic procedures?
Version 1
30
A) Sterile equipment and aseptic procedures are utilized so there are no microorganisms introduced that could cause infection. B) Sterile equipment and aseptic procedures are utilized to reduce trauma to the urinary tract. C) Sterile equipment and aseptic procedures are utilized to protect the nurse from infection. D) Sterile equipment and aseptic procedures are utilized so that microorganisms on the skin that could cause infection are destroyed.
90.2) The RN cleanses the urethral opening with an iodine swab. Which of the following terms most appropriately describes this method of microbial control?
A) B) C) D)
Disinfection Antisepsis Decontamination Sterilization
90.3) After preparing the patient’s skin, the RN brushes against the sterile field with her arm. What is the most appropriate action by the RN?
A) B) C) D)
91)
Continue the procedure and place the catheter. Clean the catheter with alcohol, and then continue the procedure. Discard the sterile field and obtain new supplies. Clean the catheter with iodine, and then continue the procedure.
Microbicidal agents are sterilants. ⊚ ⊚
Version 1
true false
31
92)
Bacteristatic agents kill bacterial cells. ⊚ ⊚
true false
93) Prions require more extensive methods of sterilization than are needed for bacterial endospores. ⊚ ⊚
true false
94) The presence of organic matter such as saliva and pus can interfere with the actions of disinfectants. ⊚ ⊚
true false
95) When a control agent targets the metabolic processes of microbial cells, active younger cells typically die more rapidly than older cells. ⊚ ⊚
true false
96) A microorganism that is not motile and has stopped metabolizing could be considered dead. ⊚ ⊚
Version 1
true false
32
97)
Most microbial contaminants of food are killed at freezing temperatures. ⊚ ⊚
98)
Pasteurization will make milk sterile. ⊚ ⊚
99)
true false
true false
Pasteurization does not kill endospores or thermoduric microbes. ⊚ ⊚
true false
100)
Ionizing radiation is more effective than nonionizing radiation in killing microbes. ⊚ true ⊚ false
101)
Chlorine compounds remain stable and effective in the presence of excess organic matter. ⊚ ⊚
102)
Alcohols are more effective at inactivating enveloped viruses than naked viruses. ⊚ ⊚
103)
true false
true false
Isopropyl alcohol wiped across a skin site can sterilize it.
Version 1
33
⊚ ⊚
104)
Hydrogen peroxide can be used to sterilize instruments such as endoscopes. ⊚ ⊚
105)
true false
Pseudomonas is resistant to soaps. ⊚ ⊚
107)
true false
Soaps and detergents are very effective as sterilants. ⊚ ⊚
106)
true false
true false
Ethylene oxide is a sterilizing gas. ⊚ true ⊚ false
108) Analine dyes, like crystal violet, have antimicrobial activity particularly against grampositive bacteria and some fungi. ⊚ ⊚
Version 1
true false
34
109) Of the six methods of controlling growth by physical means (heat, cold, desiccation, radiation, filtration and osmotic pressure), the only method that is capable of complete sterilization is radiation. ⊚ ⊚
true false
110) When comparing the Bunsen burner and a hot-air oven, the Bunsen burner is more versatile since it can sterilize a wide range of materials, including plastics, cottons and oils. ⊚ ⊚
true false
111) Phenols are useful antimicrobial compounds in that they are effective against bacteria fungi and viruses, disrupting several cellular targets, however, they have been removed from many consumer products because of their high toxicity and propensity to select for resistant strains of microorganisms. ⊚ ⊚
true false
112) Heavy metals such as silver, mercury and copper are effective antimicrobials due to their ability to bind to the regulatory site of enzymes, thus acting as an irreversible non-competitive inhibitor. ⊚ ⊚
113)
true false
Aldehyde-based compounds, while effective sterilants, can be unstable and costly to use. ⊚ ⊚
Version 1
true false
35
114) Regarding use of heat to control microbial growth, dry heat at lower temperatures is as effective as moist heat at the same temperature. ⊚ ⊚
Version 1
true false
36
Answer Key Test name: Ch 9 4e 1) [A, D] 2) [B, C] 3) [A, B, C, E] 4) [B, C, D, F] 5) [A, C, E] 6) [B, E] 7) [A, C, D, F] 8) [A, C, F] 9) [B, C, D, F] 10) [A, C] 11) [A, D, E] 12) [A, B] 13) D 14) B 15) D 16) A 17) C 18) C 19) D 20) B 21) B 22) A 23) C 24) A 25) A 26) D Version 1
37
27) C 28) E 29) B 30) E 31) D 32) A 33) E 34) A 35) E 36) D 37) A 38) A 39) B 40) A 41) A 42) B 43) B 44) D 45) D 46) B 47) C 48) A 49) B 50) E 51) A 52) B 53) A 54) D 55) E 56) C Version 1
38
57) B 58) B 59) E 60) C 61) E 62) D 63) B 64) E 65) A 66) D 67) C 68) D 69) B 70) A 71) A 72) A 73) A 74) A 75) A 76) A 77) A 78) A 79) C 80) A 81) B 82) A 83) C 84) C 85) C 86) A Version 1
39
87) C 88) Section Break 88.1) D 88.2) D 88.3) C 88.4) D 89) Section Break 89.1) B 89.2) D 89.3) A 89.4) D 90) Section Break 90.1) A 90.2) B 90.3) C 91) FALSE 92) FALSE 93) TRUE 94) TRUE 95) TRUE 96) FALSE 97) FALSE 98) FALSE 99) TRUE 100) TRUE 101) FALSE 102) TRUE 103) FALSE 104) TRUE 105) FALSE Version 1
40
106) TRUE 107) TRUE 108) TRUE 109) FALSE 110) FALSE 111) TRUE 112) FALSE 113) TRUE 114) FALSE
Version 1
41
CHAPTER 10 1) Select the two correct answers to test your understanding of the primary goals of antimicrobial treatment.
A) To destroy the infective agent B) To kill malignant cells C) To be nontoxic to the host and produce no side effects D) To identify the cause of infection E) To alter the normal microbiota of the patient
2)
Most antibiotics are derived from which bacterial and fungal genera? A) Bacillus B) Streptococcus C) Streptomyces D) Penicillium E) Cephalosporium F) Staphylococcus
3) Select the methods of antimicrobial susceptibility testing that can be used to determine minimum inhibitory concentration(MIC). A) B) C) D)
4)
Kirby-Bauertest E-test Tube dilution test Beta-lactamasetesting
What is the therapeutic index? Check all that apply.
Version 1
1
A) B) C) D)
5)
A ratio of the toxic dose of a drug over the minimum effective dose A ratio of the minimum effective dose over the toxic dose of a drug A number that can indicate the safety of a drug A list of drugs most effective for a particular microbial infection
Choose the five main targets of antimicrobial drugs. A) Cell wall synthesis B) Cell membrane structure/function C) Protein synthesis D) Synthesis of flagellar components E) DNA and RNA synthesis F) Folic acid synthesis G) Oxidation of NAD+
6)
Select all of the correct statements that apply to beta-lactamases.
A) Beta-lactamasesare made by bacteria. B) Beta-lactamases can inactivate nafcillin. C) Beta-lactamasesare made by humans and other mammals. D) Beta-lactamasescan inactivate penicillin. E) Beta-lactamases are inactivated by a chemical called clavulanic acid.
7)
Select the correct statements about linezolid and synercid.
Version 1
2
A) Synercid and linezolid are two of the newer antibiotics that inhibit protein synthesis. B) Synercid and linezolid are used to treat some of the very resistant gram-positive bacteria. C) Synercid andlinezolid are relatively toxic to the kidneys. D) Synercid andlinezolid are most often used to treat gram-negative bacteria. E) Synercid and linezolid are effective antibiotics for TB.
8)
Which drugs target folic acid synthesis? A) B) C) D)
9)
Trimethoprim Tetracyclines Ciprofloxacin Sulfonamides
Which statements are true regarding the fluoroquinolones?
A) They inhibit DNA synthesis and transcription. B) They interfere with ribosomes. C) Ciprofloxacin is a commonly used fluoroquinolone. D) Bacitracin is an example of a fluoroquinolone. E) These drugs are broad spectrum, targeting both gram-negative and gram-positive bacteria as well as viruses and yeasts.
10)
What two drugs target the cell membrane? A) Ciprofloxacin B) Polymyxin C) Daptomycin D) Trimethoprim E) Penicillin
Version 1
3
11)
Select those statements that are true about the treatment of biofilm infections.
A) Bacteria in biofilms have different susceptibility patterns than their free-living counterparts. B) Aminoglycosidesgenerally are very effective against bacteria in biofilms. C) Bacteria inbiofilms may be inhibited by drugs that interfere with quorum sensing. D) Antibiotic penetration of biofilms may be improved by adding DNase with the antibiotic. E) Bacteria in biofilms tend to be more antibiotic sensitive. F) Antibiotic-impregnated biomaterials that are inserted into the body preventbiofilm development on those materials.
12)
Select the four main categories of antifungal agents.
A) Aminoglycosides B) Macrolide polyenes C) Beta-lactams D) Azoles E) Echinocandins F) Allylamines
13)
Select all of the true statements about azoles. A) B) C) D)
14)
They are used to treat fungal infections. They are used to treat viral infections. They include fluconazole which is used to treat AIDS-related mycoses. They include topical medications that can be used to treat cutaneous infections.
Select the antiprotozoal drugs from this list.
Version 1
4
A) Chloroquine B) Primaquine C) Ivermectin D) Mebendazole E) Fluconazole F) Metranidazole
15)
Select the antihelminthic drugs from this list. A) Chloroquine B) Primaquine C) Ivermectin D) Mebendazole E) Fluconazole F) Metranidazole G) Praziquantel
16)
Select the three primary mechanisms by which antiviral medications work. A) Preventing viral cell wall synthesis B) Preventing entry of the virus into the host cell C) Directly disrupting viral capsids D) Blocking transcription and translation of viral proteins E) Preventing the maturation of viral particles F) Directly disrupting viral envelopes
17)
Select the three ways in which microorganisms acquire antimicrobial resistance.
Version 1
5
A) Spontaneous mutation B) Antibiotic-induced mutations C) Transfer of genes from their human or animal host D) Transfer of genes from other microorganisms E) Entering a dormant state in the presence of antibiotics, and resuming normal metabolic functions in the absence of antibiotics
18)
Select the five major mechanisms of antimicrobial resistance. A) A microbial enzyme inactivates the antimicrobial agent B) Microbes use the antimicrobial agent as an energy source C) Microbial efflux pumps remove the antimicrobial agent by pumping it out of the cell D) An altered target site prevents the antimicrobial agent from binding to its target E) The microbe uses an alternative pathway to circumvent the blocked pathway F) The permeability of the microbial envelope to the antimicrobial agent is decreased
19)
Which of the following bacteria are classified as "urgent" threats due to drug resistance? A) Carbapenem-resistant Enterobacteriaceae (CRE) B) Drug-resistant Mycobacterium tuberculosis C) Erythromycin-resistant Streptococcus D) Drug-resistant Neisseria gonorrhoeae E) Carbapenem-resistant Enterobacteriaceae (CRE)
20)
Select potential new antimicrobial therapies that are under investigation.
Version 1
6
A) CRISPR B) Methods that directly interfere with cell respiration in bacteria C) Bacteriophage therapy D) Making use of host and bacterial defense peptides E) RNA interference–directed at bacterial mRNAs F) Interfering with synthesis of bacterial capsules/glycocalyx
21) Substances that are naturally produced by certain microorganisms that can inhibit or destroy other microorganisms are called .
A) antibiotics B) narrow-spectrum drugs C) semisynthetic drugs D) synthetic drugs E) broad-spectrum drugs
22)
Antimicrobials that are effective against a wide variety of microbial types are termed . A) antibiotics B) narrow-spectrum drugs C) semisynthetic drugs D) synthetic drugs E) broad-spectrum drugs
23)
Antibiotics are derived from all of the following except
Version 1
.
7
A) Penicillium B) Bacillus C) Staphylococcus D) Streptomyces E) Cephalosporium
24)
Important characteristics of antimicrobial drugs include
.
A) low toxicity for human tissues B) high toxicity against microbial cells C) do not cause serious side effects in humans D) stable and soluble in body tissues and fluids E) All of the choices are correct.
25)
The use of a drug to prevent a person at risk of an imminent infection is called
.
A) competitive inhibition B) synergism C) prebiotics D) prophylaxis E) lantibiotics
26)
The use of any chemical in the treatment, relief, or prophylaxis of a disease is called .
Version 1
8
A) prophylaxis B) chemotherapy C) selective toxicity D) nephrotoxicity E) synergism
27)
Penicillins and cephalosporins
A) B) C) D)
28)
.
block folic acid synthesis attach to the 30S ribosomal subunit and disrupt protein synthesis damage cell membranes block peptidases that cross-link glycan molecules
Selective toxicity refers to
.
A) damage to pathogenic organisms B) damage to prokaryotic cell membranes C) damage to the target organisms but not host cells D) damage to nucleic acids E) None of the choices are correct.
29)
Each of the following affect cell walls except
.
A) penicillin B) isoniazid C) vancomycin D) erythromycin E) cephalosporin
Version 1
9
30)
Aminoglycosides
.
A) interfere with elongation of peptidoglycan B) block folic acid synthesis C) attach to the 30S ribosomal subunit and disrupt protein synthesis D) damage cell membranes E) block peptidases that cross-link glycan molecules
31)
Each of the following target bacterial ribosomes except
.
A) streptomycin B) gentamicin C) polymyxins D) tetracycline E) erythromycin
32)
Sulfonamides
.
A) interfere with elongation of peptidoglycan B) block folic acid synthesis C) attach to the 30S ribosomal subunit and disrupt protein synthesis D) damage cell membranes E) block peptidases that cross-link glycan molecules
33)
Sulfa drugs work on
Version 1
.
10
A) nucleic acid biosynthesis B) ribosome biosynthesis C) peptidoglycan biosynthesis D) folic acid biosynthesis E) None of the choices are correct.
34)
Ampicillin, amoxicillin, mezlocillin, and penicillin G all
.
A) target the cell wall B) have resistance to the action of penicillinase C) are semisynthetic D) have an expanded spectrum of activity E) All of the choices are correct.
35)
A chemical that inhibits beta-lactamase enzymes is
.
A) synercid B) penicillinase C) aztreonam D) clavulanic acid E) imipenem
36)
What type of chemical will allow some bacteria to be resistant to many penicillins?
A) Synercid B) Penicillinase C) Aztreonam D) Clavulanic acid E) Imipenem
Version 1
11
37)
Which antimicrobial does not inhibit cell wall synthesis?
A) Gentamicin B) Vancomycin C) Cephalosporins D) Penicillins E) Bacitracin
38)
Which drug is used to treat cases of tuberculosis?
A) Penicillin G B) Vancomycin C) Tetracycline D) Synercid E) Isoniazid
39)
What cell wall inhibiting drug is used in cases of penicillin and methicillin resistance? A) Penicillin G B) Vancomycin C) Tetracycline D) Erythromycin E) Isoniazid
40)
Clavulanic acid
Version 1
.
12
A) inhibits B-lactamase activity B) inhibits peptidoglycan synthesis C) inhibits formation of peptidoglycan cross linkages D) inhibits cell membrane synthesis E) None of the choices are correct.
41)
Gram-negative rods are often treated with
.
A) penicillin G B) vancomycin C) aminoglycosides D) synercid E) isoniazid
42)
Which antimicrobial does not interfere with protein synthesis?
A) Aminoglycosides B) Tetracyclines C) Erythromycin D) Trimethroprim E) Gentamicin
43)
Which of these drugs has the most narrow spectrum? A) Tetracycline B) Isoniazid C) Erythromycin D) Aminoglycosides E) Cephalosporins
Version 1
13
44)
Antimicrobials that are macrolides
.
A) disrupt cell membrane function B) include tetracyclines C) include azithromycin, clarithromycin and erythromycin D) are narrow-spectrum drugs E) are hepatotoxic
45)
The drug that can cause injury to red blood cells and white blood cells is
.
A) chloramphenicol B) clindamycin C) ciprofloxacin D) bacitracin E) gentamicin
46)
Which of the following antibacterial drug groups does not target protein synthesis?
A) Clindamycin B) Erythromycin C) Aminoglycosides D) Sulfonamides E) Tetracycline
47)
Which newer synthetic drug is used to treat MRSA and VRE infections?
Version 1
14
A) Synercid B) Clindamycin C) Linezolid D) Azithromycin E) Clarithromycin
48)
Which of the following is not true of polymyxins?
A) Disrupt the cell membrane B) Have a narrow spectrum C) Toxic to kidneys D) Target peptidoglycan E) Can treat severe urinary tract infections
49) The antifungal drug that can be injected to treat serious systemic fungal infections, like histoplasmosis, is . A) nystatin B) griseofulvin C) amphotericin B D) sulfa drugs E) metronidazole
50) treat
Ketoconazole, fluconazole, clotrimazole, miconazole are broad-spectrum azoles used to infections.
Version 1
15
A) bacterial B) fungal C) protozoan D) helminthic E) viral
51)
Which of the following is not a drug group used to treat fungal infections?
A) Quinolones B) Macrolide polyenes C) Echinocandins D) Synthetic azoles E) Allylamines
52)
The drug used for several protozoan infections is
.
A) nystatin B) griseofulvin C) amphotericin B D) sulfa drugs E) metronidazole
53)
Mebendazole is adrug used to treat
infections.
A) bacterial B) fungal C) protozoan D) helminthic E) viral
Version 1
16
54) There are fewer antifungal, antiprotozoan, and antihelminth drugs compared to antibacterial drugs because fungi, protozoa, and helminths . A) do not cause many human infections B) are not affected by antimicrobials C) are so similar to human cells that drug selective toxicity is difficult D) are parasites found inside human cells E) have cells with fewer target sites compared to bacteria
55)
Primaquine and chloroquine are drugs used in the treatment of
.
A) gram-positive infections B) gram-negative infections C) fungal infections D) protozoan infections E) viral infections
56)
Which of the following is not a mode of action of antiviral drugs?
A) Block penetration B) Block transcription and translation C) Inhibit DNA synthesis D) Block maturation E) Bond to ergosterol in the cell membrane
57)
An antiviral that is a guanine analog would have an antiviral mode of action that .
Version 1
17
A) blocks penetration B) blocks transcription and translation C) inhibits DNA synthesis D) blocks maturation E) bonds to ergosterol in the cell membrane
58)
Antiviral drugs that target reverse transcriptase would be used to treat
.
A) influenza A virus B) HIV C) herpes zoster virus D) respiratory syncytial virus E) hepatitis C virus
59)
Acyclovir is used to treat
.
A) influenza A virus B) HIV C) herpes simplex virus D) respiratory syncytial virus E) hepatitis C virus
60)
Which of the following block HIV binding to host cell receptors?
A) AZT B) Acyclovir C) Nevirapine D) Fuzeon E) Amantidine
Version 1
18
61)
The cellular basis for bacterial resistance to antimicrobials include
.
A) bacterial chromosomal mutations B) synthesis of enzymes that alter drug structure C) prevention of drug entry into the cell D) alteration of drug receptors on cell targets E) All of the choices are correct.
62)
The multidrug-resistant pumps in many bacterial cell membranes function by
.
A) bacterial chromosomal mutations B) synthesis of enzymes that alter drug structure C) removing the drug from the cell when it enters D) alteration of drug receptors on cell targets E) All of the choices are correct.
63)
Microbial resistance resulting from mutation occurs because
.
A) bacterial genomes undergo mutation rapidly B) bacterial genomes undergo mutation often C) short generation times accumulate mutations in populations D) mutations are passed between organisms E) All of the choices are correct.
64)
Each of the following result in drug resistance except
Version 1
.
19
A) drug pumped out of the cell B) drug used as a nutrient by the cell C) drug binding site altered D) drug inactivated E) drug blocked from entering cell
65) Each of the following is a mechanism for drug resistance transfer between microorganisms except .
A) transduction B) R-plasmids C) conjugation D) mutation E) transformation
66)
Each of the following contributes to emerging drug resistance except
.
A) overuse of antibiotics B) improper use of antibiotics C) multiple drug therapy D) ingestion of antibiotics with animal feed E) addition of antibiotics to common household products
67)
Nutrients that encourage the growth of beneficial microbes in the intestines are known as .
Version 1
20
A) prebiotics B) probiotics. C) lantibiotics D) phytobiotics E) riboswitches
68) of
The use of vaginal inserts of Lactobacillus to restore healthy vaginal biota is an example . A) prebiotics B) probiotics C) lantibiotics D) phytobiotics E) riboswitches
69)
Broad-spectrum drugs that disrupt the body's microbiota often cause
.
A) nephrotoxicity B) superinfections C) allergic reactions D) drug toxicity E) mutation
70) Side effects that occur in patient's tissues while on antimicrobial drugs include all the following except .
Version 1
21
A) development of resistance to the drug B) hepatotoxicity C) nephrotoxicity D) diarrhea E) deafness
71)
A superinfection results from
A) B) C) D)
.
buildup of a drug to toxic levels in the patient the wrong drug administered to the patient an immune system reaction to the drug decrease in the microbiota with overgrowth of an unaffected species
72) The discoloration.
are drugs that deposit in developing teeth and cause a permanent brown
A) streptomycins B) cephalosporins C) macrolides D) tetracyclines E) penicillins
73)
Drug susceptibility testing determines
.
A) the patient's response to various antimicrobials B) the pathogen's response to various antimicrobials C) if normal flora will be affected by antimicrobials D) if the drug is increasing to toxic levels in a patient E) None of the choices are correct.
Version 1
22
74) A clinical microbiologist makes serial dilutions of several antimicrobials in broth, and then incubates each drug dilution series with a standard amount of a patient's isolated pathogen. What is this microbiologist setting up?
A) Kirby-Bauer B) Antibiogram C) E-test D) MIC E) Therapeutic index (TI)
75) A ratio of the dose of a drug that is toxic to humans versus the minimum effective dose for that pathogen is assessed to predict the potential for toxic drug reactions. This is called the . A) Kirby-Bauer B) antibiogram C) E-test D) MIC E) therapeutic index (TI)
76) If pathogen A is more resistant to an erythromycin disc on a Kirby-Bauer plate compared to pathogen B, then pathogen A will have a(n) zone of inhibition compared to pathogen B.
A) smaller B) equal C) larger
77)
Which therapeutic index value would be the drug of choice?
Version 1
23
A) 20 B) 10 C) 1 D) 0.1 E) Any value would be equally effective.
78)
Which two antibiotics affect the DNA and RNA of bacteria?
A) B) C) D)
Tetracycline and amphotericin B Trimethoprim and sulfonamides Rifampin and quinolones Tetracycline and bacitracin
79) Which of the following statements does not accurately reflect the origins of antimicrobial drugs?
A) All antimicrobial drugs are synthesized by medicinal chemists in a laboratory environment. B) Bacteria and fungi are natural producers of antimicrobial drugs; the molecules are synthesized to maintain competition for nutrients and space to a minimum. C) Scientists have learned how to modify antimicrobial molecules naturally produced by organisms to enhance their efficacy and range. D) Some antimicrobial agents are synthesized wholly in the laboratory.
80)
An example of an antibacterial drug with high selective toxicity would be one that .
Version 1
24
A) B) C) D)
81)
Drugs with high selective toxicity include
A) B) C) D)
82)
targets the synthesis of the bacterial cell wall targets the permeability of the cell membrane targets DNA replication targets RNA transcription
.
ampicillin and bacitracin amphotericin B and fluconazole mebendazole and albendazole quinine and metronidazole
Naturally-produced penicillins are most effective against gram-positive bacteria because .
A) they prevent the synthesis of peptidoglycan which comprises the gram-positive cell wall B) they target the enzymes needed by gram-positive cells for ATP production C) they inhibit the permeability of the gram-positive cell membrane D) they target the 30s subunit of the gram-positive ribosome
83) that
Semisynthetic penicillins have an advantage over their naturally occurring counterparts in .
A) B) ribosome C) D)
Version 1
they have a broader target range and are less susceptible to penicillinases they target both cell wall synthesis and protein synthesis by binding to the 30s they can target both bacterial and eukaryotic organisms they cause fewer allergic reactions than naturally occurring penicillins
25
84) Fluoroquinolones such as ciprofloxacin, are synthetic drugs that interfere with the action of DNA helicases. This means that .
A) the bacterial cell will die since it cannot replicate or transcribe its DNA B) binary fission cannot take place because the cell cannot make proteins C) the bacterial cell will die since it cannot take in nutrients across the cell membrane D) the cell will die because it cannot make folic acid, which is a precursor to DNA and RNA, as well as amino acids
85) Polymyxin B is a large molecule with a hydrophobic tail that can disrupt phospholipids, making it particularly effective against .
A) gram-negative cells since they have both an outer and inner membrane B) gram-positive cells since the peptidoglycan is easily dissolved by detergent-like molecules C) fungi since the drug can traverse the complex chitin cell wall to dissolve the cell membrane D) both gram-positive and gram-negative cells since they both have a membrane
86)
Biofilm infections are harder to treat than the same free-living organisms because .
A) when part of a biofilm community, the bacterial cells express different genes thus changing their antibiotic susceptibility B) bacteria in a biofilm exist only on synthetic surfaces such as catheters, so they cannot be treated with antibiotics C) the organisms in the biofilm degrade the antibiotics at a much faster rate than their free-living counterparts D) organisms in a biofilm develop antibiotic resistance more rapidly than free-living cells
Version 1
26
87) Penicillinases are enzymes produced by bacterial cells, usually after acquiring a new gene, that .
A) hydrolyze the β-lactam ring of penicillins and cephalosporins, rendering them ineffective B) pump the penicillins out of the cell as soon as they enter C) prevent the drug from crossing the cell membrane D) block the target binding site of the drug, rendering it ineffective
88) Micafungin and caspofungin are antifungal drugs that inhibit cell wall synthesis. They belong to the group of drugs known as .
A) B) C) D)
echinocandins macrolide polyenes azoles allylamines
89) In light of increasing antibiotic resistance, alternative antimicrobial therapies are being explored. These include the CRISPR system, which .
A) when delivered with an antibiotic, can splice out the genes causing resistance to the drug, which is then able to kill the bacteria B) directly kill the bacteria instead of requiring an antibiotic C) is comprised of immune peptides that are not susceptible to bacterial resistance D) extracts drugs from bacteria that are not cultivable in a laboratory environment
90) Choose the method of antimicrobial susceptibility testing that involves measuring and evaluating zones of inhibition around antibiotic disks placed on a culture of bacteria prior to incubation. Version 1
27
A) ELISA test B) Kirby-Bauertest C) Tube dilutiontest D) Gram staintest E) Beta-lactamase test
91) The antibiotics listed here all have similar MICs. Please choose the antibiotic that would be the best option for treatment.
A) B) C) D)
Antibiotic W Therapeutic index = 5 Antibiotic X Therapeutic index = 20 Antibiotic Y Therapeutic index = 100 Antibiotic Z Therapeutic index = 10
92) The feature of an antimicrobial that makes it kill or inhibit microbes yet be harmless to the host is referred to as . A) B) C) D)
selective toxicity the therapeutic index the thermal death point minimum inhibition
93) Choose the statement that correctly explains how an antibiotic that inhibits protein synthesis can be selective.
Version 1
28
A) Antibiotics prevent the synthesis of bacterial tRNAs but not eukaryotic tRNAs. B) Antibiotics bind to the 70S bacterial ribosomes but not to the 80S eukaryotic ribosomes. C) Eukaryotic cells inactivate these antibiotics. D) Eukaryotic cells are impermeable to these antibiotics so the antibiotics cannot enter them.
94)
Choose the antimicrobial category that is most selectively toxic. A) Antimicrobials that inhibit cell wall synthesis B) Antimicrobials that inhibit protein synthesis C) Antimicrobials that interfere with nucleic acid structure and function D) Antimicrobials that inhibit folic acid synthesis E) Antimicrobials that interfere with cell membrane structure and function
95) An antimicrobial drug that is effective against a very diverse array of bacteria is described as spectrum. A) broad B) sensitive C) medium D) narrow E) wide
96)
Streptomycin, tetracycline, clindamycin, and erythromycin are examples of
.
A) aminoglycosides B) antibiotics that inhibit folic acid synthesis C) antibiotics that inhibit protein synthesis D) beta-lactamase susceptible antibiotics E) antibiotics that are associated with a high incidence of liver toxicity
Version 1
29
97) An example of a macrolide polyene drug used to treat systemic fungal infections and lesions caused by Candida albicans is . A) B) C) D)
amphotericin B flucytosine fluoroquinolone clotrimazole
98) Direct damage to tissue or an organ due to the effect of an antimicrobial on that tissue or organ is known as drug , whereas an inappropriate immune reaction to an antimicrobial is known as drug-induced . A) B) C) D)
99)
toxicity; allergy allergy; toxicity resistance; allergy superinfection; resistance
Choose the statement that best describes superinfection.
A) An infection with an extensively drug-resistant microbe B) An infection that occurs due to the overgrowth of other potentially pathogenic microbes during or after initial antimicrobial therapy C) An infection by an extremely virulent microbe D) An infection caused by multiple species of microbes, synergistically contributing to the infectious process
100) Yeast infections of the vaginal tract and Clostridium difficile colitis following antibiotic treatment are examples of .
Version 1
30
A) B) C) D)
superinfections drug resistance systemic infections drug allergies
101) NCLEX Prep - Test Bank Question: Please read the clinical scenario, and then answer the questions that follow to become familiar with the traditional NCLEX question format. A 76-year-old male visits his primary care physician with complaints of burning with urination and a frequent urge to void. The RNaids the patient in collecting a urine specimen to be sent for analysis. While awaiting the final results of drug susceptibility testing in the lab, the patient is prescribed oral sulfasoxazole.
101.1) In the lab, the urine specimen is spread on an agar plate with antibiotic discs placed to determine antimicrobial susceptibility. Following incubation, the clear area around each disc is measured. What term describes the observed area of antimicrobial action around each disc?
A) B) C) D)
Zone of inhibition Zone of susceptibility Zone of resistance No growth zone
101.2) You provide education to the patient regarding sulfasoxazole, a sulfonamide that targets folic acid synthesis. The synthesis of which of the following products is affected by inhibition of folate metabolism?
Version 1
31
A) B) C) D)
DNA RNA DNA and RNA DNA, RNA, and aminoacids
101.3) Which of the following reactions by the patient could demonstrate an allergic response to the drug?
A) Anaphylaxis B) Rash C) Difficulty breathing D) Anaphylaxis, rash development, and difficulty breathing could all indicate an allergic response.
101.4) The in vitro susceptibility of the pathogen is established and the patient is maintained on sulfasoxazole. If no in vivo effect of the drug is observed by the patient following the first 7 days of antibiotics, what is the most appropriate action by the medical team? A) B) C) D)
Version 1
Increase the dose of sulfasoxazole Discontinue antibiotic therapy Discontinue sulfasoxazole and initiate a new antibiotic Initiate antiviral therapy
32
102) NCLEX Prep - Test Bank Question: Please read the clinical scenario, and then answer the questions that follow to become familiar with the traditional NCLEX question format. A 12-year-old male is admitted to the pediatric unit due to persistent upper respiratory symptoms and respiratory distress. The patient has a history of developmental delay, chronic lung disease, and frequent respiratory infections. Upon assessment, the RN notes the patient to have a fever, moderate respiratory rate, productive cough, and large amount of nasal secretions. Upon report from the patient’s mother, the patient has been on a 3-week course of antibiotics with no improvement in symptoms. The medical team completes a respiratory infection workup and the patient is diagnosed with bacterial pneumonia and methicillin-resistant Staphylococcus aureus (MRSA) superinfection.
102.1) The RN provides education to the patient and his mother regarding the diagnosis. Which of the following statements by the mother demonstrates a proper understanding of the teaching?
A) My son has an extremely resistant form of MRSA causing his prolonged illness. B) My son developed simultaneous bacterial infections and his antibiotics were not treating his MRSA infection. C) My son developed a MRSA superinfection following suppression of normal resident species in his lungs by the antibiotics he was taking to treat his initial infection. D) My son has pneumonia caused by a virus.
102.2) Based upon the patient’s history, what is the most likely cause of the superinfection?
A) B) C) D)
Version 1
Poor hygiene Long-term antibiotic therapy Fecal-oral contamination Community-acquired MRSA
33
102.3) The use of which of the following drugs results in the highest risk of superinfection in a patient receiving antimicrobial chemotherapy?
A) B) C) D)
Broad-spectrum antibiotics Narrow-spectrum antibiotics Antifungals Antivirals
102.4) Treatment of a urinary tract infection with antimicrobials is most likely to lead to a superinfection caused by which microbe?
A) B) C) D)
Giardialamblia Lactobacillusacidophilus Escherichiacoli Candidaalbicans
103) NCLEX Prep - Test Bank Question: Please read the clinical scenario, and then answer the questions that follow to become familiar with the traditional NCLEX question format. An RN in a medical intensive care unit is leading a research study regarding administration of probiotics to critically ill patients receiving antibiotic therapy. She provides education about normal flora and probiotics as part of the consent process to patients and families.
103.1) Following the teaching to patients and families, the RN assesses understanding prior to obtaining consent. Which of the following demonstrates a lack of understanding by the patient?
Version 1
34
A) B) C) D)
Probiotics are live microorganisms. Probiotics are nutrients to encourage microbe growth. Probiotics augment the microbes in the intestine. Probiotics can replace microbes killed during antibiotic therapy.
103.2)
A) B) C) D)
Which of the following is an example of a probiotic?
Penicillin Micafungin Yogurt Fructans
103.3) The RN conducting the research study most appropriately hypothesizes that probiotics may alleviate which of the following side effects of antimicrobial therapy?
A) B) C) D)
Headache Nausea Diarrhea Skin rash
104) Antimicrobial drugs that inhibit folic acid synthesis work with very few side effects because mammals must get folic acid from their diet. ⊚ true ⊚ false
105)
An antibiotic of the penicillin family is amoxicillin. ⊚ ⊚
Version 1
true false 35
106)
Ciprofloxacin is used to treat viral respiratory infections. ⊚ ⊚
true false
107) Resistance factor plasmids are transferred to other bacterial cells during transformation, transduction, and conjugation. ⊚ ⊚
108)
true false
Bacteria can have a natural resistance to a drug that it has never been exposed to. ⊚ ⊚
true false
109) When a patient's immune system reacts adversely to a drug, this serious side effect is called a superinfection. ⊚ ⊚
110)
true false
Drugs that are hepatotoxic cause damage to a patient's kidneys. ⊚ ⊚
true false
111) The MIC is the smallest concentration of an antimicrobial required to inhibit the growth of the microbe.
Version 1
36
⊚ ⊚
true false
112) The Kirby-Bauer test uses an agar surface, seeded with the test bacterium, to which small discs containing a specific concentration of several drugs are placed on the surface. ⊚ ⊚
true false
113) An antimicrobial with a low therapeutic index is a safer choice compared to a drug with a high therapeutic index. ⊚ ⊚
114)
true false
It is better to use a broad-spectrum drug instead of a more specific narrow-spectrum drug. ⊚ ⊚
true false
115) Indwelling catheter biofilm infections are more resistant to antibiotics than nonbiofilm infections. ⊚ true ⊚ false
116) Drug toxicity occurs when an antimicrobial drug acts as antigen and stimulates an allergic response. ⊚ ⊚
Version 1
true false
37
117) Aminoglycosides demonstrate selective toxicity since they target the bacterial 30s subunit, leaving the host 40s subunit largely unaffected. ⊚ ⊚
true false
118) Polymyxin B and daptomycin are both narrow spectrum drugs that target the cell membrane. They differ in that polymyxin B is effective against gram-negative organisms since it disrupts both the inner and outer membranes whereas daptomycin integrates only into the grampositive cell wall creating leakage. ⊚ ⊚
true false
119) The greatest hurdle to overcome when treating biofilm infections is finding a drug that can penetrate the extracellular material. ⊚ ⊚
true false
120) A narrow-spectrum antimicrobial would be an appropriate choice to treat an abscess caused by several different microbe species, including both gram-negative and gram-positive bacteria. ⊚ ⊚
true false
121) Antiprotozoal and antihelminthic drugs are often less toxic to human tissues because of the high degree of similarity between human cells and various pathogenic protozoa and helminths.
Version 1
38
⊚ ⊚
Version 1
true false
39
Answer Key Test name: Ch 10 4e 1) [A, C] 2) [A, C, D, E] 3) [B, C] 4) [A, C] 5) [A, B, C, E, F] 6) [A, D, E] 7) [A, B] 8) [A, D] 9) [A, C] 10) [B, C] 11) [A, C, D, F] 12) [B, D, E, F] 13) [A, C, D] 14) [A, B, F] 15) [C, D, G] 16) [B, D, E] 17) [A, D, E] 18) [A, C, D, E, F] 19) [A, D, E] 20) [A, C, D, E] 21) A 22) E 23) C 24) E 25) D 26) B Version 1
40
27) D 28) C 29) D 30) C 31) C 32) B 33) D 34) A 35) D 36) B 37) A 38) E 39) B 40) A 41) C 42) D 43) B 44) C 45) A 46) D 47) C 48) D 49) C 50) B 51) A 52) E 53) D 54) C 55) D 56) E Version 1
41
57) C 58) B 59) C 60) D 61) E 62) C 63) E 64) B 65) D 66) C 67) A 68) B 69) B 70) A 71) D 72) D 73) B 74) D 75) E 76) A 77) A 78) C 79) A 80) A 81) A 82) A 83) A 84) A 85) A 86) A Version 1
42
87) A 88) A 89) A 90) B 91) C 92) A 93) B 94) A 95) A 96) C 97) A 98) A 99) B 100) A 101) Section Break 101.1) A 101.2) D 101.3) D 101.4) C 102) Section Break 102.1) C 102.2) B 102.3) A 102.4) D 103) Section Break 103.1) B 103.2) C 103.3) C 104) TRUE 105) TRUE Version 1
43
106) FALSE 107) TRUE 108) TRUE 109) FALSE 110) FALSE 111) TRUE 112) TRUE 113) FALSE 114) FALSE 115) TRUE 116) FALSE 117) TRUE 118) TRUE 119) FALSE 120) FALSE 121) FALSE
Version 1
44
CHAPTER 11 1)
Select the two major goals of the Human Microbiome Project.
A) To determine antibiotic susceptibility of the most prevalent microorganisms among human microbiota B) To enumerate allof the members of the human microbiota, both those that can be cultured andthose that are nonculturable C) To eradicate thosemembers of the normal microbiota that may potentially cause disease D) To determine therole the human microbiota plays in health and disease
2)
Select the body sites that serve as portals of entry for microbes.
A) Vagina B) Nose C) Mouth D) Endocrine glands E) Brain F) Liver G) Urethra H) Skin
3)
Select all of the statements that correctly describe the infectious dose.
A) The infectious dose is the minimum number of organisms that must be present for an infection to proceed. B) The infectious dose is the maximum number of organisms that a host can harbor without resulting in a fatal infection. C) Microbes with low infectious doses are usually highly virulent. D) Microbes with high infectious doses are usually the most virulent. E) Certain infections, like tuberculosis and giardiasis, may proceed even if only a small number of microbial cells are present.
Version 1
1
4)
Select the basic ways in which microbes cause damage to the host.
A) Microbes releasetoxins that cause tissue damage. B) Microbes blockhost cell respiration pathways. C) Microbes alterhost blood glucose levels. D) Microbes releaseenzymes that break down host tissue. E) Microbes activate a host response thatis itself destructive to host tissue. F) Microbes disrupt the host’s endocrinesystem. G) Microbes induce epigenetic changes in host cells.
5)
Select characteristics exhibited by exotoxins, but not exhibited by endotoxin.
A) Composed of lipopolysaccharide B) Require very small doses to causetoxic effects C) Secreted from a living cell D) Heat stable E) Have very specific targets F) Fever inducing
6)
Select characteristics exhibited by endotoxin, but not exhibited by exotoxins.
A) Composed oflipopolysaccharide B) Require verysmall doses to cause toxic effects C) Secreted from aliving cell D) Heat-stable E) Have very specific targets F) Fever-inducing
7)
Select the patterns of indirect transmission of infectious disease.
Version 1
2
A) Food, water B) Kissing, touching C) Fomites D) Mother to fetus (vertical) E) Droplet F) Air
8)
Select the four most prevalent types of healthcare-associated infections.
A) Septicemia B) Surgical site infections C) Skin infections D) Urinary tract infections E) Respiratory infections F) Reproductive tract infections G) Gastrointestinal infections
9) Select those circumstances in which Koch’s postulates cannot be readily applied or would be inappropriate to establish the cause of a disease.
A) The suspectedpathogen infects animals but not humans. B) The suspectedpathogen cannot be cultured in the laboratory. C) There is not asuitable experimental host for the suspected pathogen. D) The suspectedpathogen produces disease both in humans and a suitable test animal. E) The disease is polymicrobial, caused bymore than one pathogen.
10) Select all of the sites where it was previously known that normal microbiota existed in large populations in/on the human body.
Version 1
3
A) Brain B) Mouth C) Liver D) Throat E) Large intestine F) Skin G) Vagina H) Lungs
11)
Infection occurs when
.
A) contaminants are present on the skin B) a person swallows microbes in/on food C) a person inhales microbes in the air D) pathogens enter and multiply in body tissues E) All of thechoices are correct.
12)
The term infection refers to
.
A) microorganisms colonizing the body B) contact with microorganisms C) contact with pathogens D) pathogens penetrating host defenses E) None of the choices are correct.
13)
Which terminology is not used to describe members of the resident biota?
Version 1
4
A) Pathogenic biota B) Normal biota C) Indigenous biota D) Normal microbiota E) Commensals
14)
Endogenous infectious agents arise from microbes that are
.
A) in food B) the patient's own normal biota C) on fomites D) in the air E) transmitted from one person to another
15)
The human body typically begins to be colonized by its normal biota
.
A) before birth, in utero B) during and immediately after birth C) when a child first goes to school D) when an infant gets its first infectious disease E) during puberty
16)
Resident biota are found in/on the
.
A) skin B) mouth C) nasal passages D) large intestine E) All of the choices are correct.
Version 1
5
17)
Normal biota includes each of the following except
.
A) bacteria B) fungi C) protozoans D) viruses E) All of the choices are correct.
18)
Each of the following bring about inoculation of normal biota to a newborn except . A) the birth process through the birth canal B) bottle feeding C) breast feeding D) contact with hospital staff E) All of the choices are correct.
19)
The effect of "good" microbes against invading microbes is called
.
A) microbial antagonism B) endogenous infection C) infectious disease D) axenic E) gnotobiotism
20)
Opportunistic pathogens
Version 1
.
6
A) cause disease in every individual B) cause disease in compromised individuals C) are always pathogens D) have well-developed virulence factors E) None of thechoices is correct.
21)
Pathogenic microbes that cause disease in healthy people are called
.
A) opportunistic pathogens B) normal biota C) indigenous biota D) true pathogens E) micropathogens
22)
Which of the following is not a factor that weakens host defenses against infections?
A) Genetic defects in immunity B) Physical and mental stress C) Strong, healthy body D) Chemotherapy E) Old age
23)
The greatest number of pathogens enter the body through the
.
A) respiratory system B) gastrointestinal system C) urinary system D) genital system E) skin
Version 1
7
24)
An infectious agent that originates from outside the body is called
.
A) exogenous B) an exotoxin C) an enterotoxin D) endogenous E) axenic
25)
An infectious agent already existing on or in the body is called
.
A) exogenous B) an exotoxin C) an enterotoxin D) endogenous E) axenic
26)
The minimum amount of microbes in the inoculating dose is the
.
A) virulence factor B) indigenous biota C) infectious dose D) endotoxin E) minimal dose
27)
Which of the following is not a method of adhesion?
Version 1
8
A) Fimbriae B) Surface proteins C) Specialized receptors D) Adhesive slime or capsules E) Cilia
28) Once a microbe has entered a host, what process performed by certain white blood cells will attempt to destroy the microbes?
A) Phagocytosis B) Adhesion C) Encapsulation D) Margination E) Exocytosis
29)
Which of the following is not an antiphagocytic factor?
A) Secretion of slime B) Production of leukocidins C) Adhering to the host D) Secretion of a capsule E) Ability to survive intracellularly
30)
Virulence factors include all the following except
Version 1
.
9
A) capsules B) ribosomes C) exoenzymes D) endotoxins E) exotoxins
31)
Microbial hyaluronidase, coagulase, and streptokinase are examples of
.
A) adhesive factors B) exotoxins C) hemolysins D) antiphagocytic factors E) exoenzymes
32)
Exotoxins are
.
A) proteins B) only released after a cell is damaged or lysed C) antiphagocytic factors D) secretions that always target nervous tissue E) lipopolysaccharides
33)
Enterotoxins are
.
A) virulence factors B) toxins that target the intestines C) proteins D) exotoxins E) All of thechoices are correct.
Version 1
10
34)
Which of the following pairs is mismatched?
A) Fimbriae - adherence to substrate B) Capsules - antiphagocytic factor C) Coagulase - dissolve fibrin clots D) Leukocidins - damage white blood cells E) Hemolysins - damage red blood cells
35)
Mucinase has the greatest effect on the
.
A) respiratory system B) gastrointestinal system C) urinary system D) genital system E) skin
36)
Exotoxins
.
A) are secreted by pathogenic organisms B) are bound to the membrane of pathogenic organisms C) are bound to the cell wall of pathogenic organisms D) cause more damage than endotoxins E) are host specific
37)
An endotoxin is
Version 1
.
11
A) secreted by pathogenic organisms B) indicative of gram-negative organisms C) indicative of gram-positive organisms D) indicative of fungal infections E) indicative of viral infections
38) are toxins that are the lipopolysaccharide of the outer membrane of gramnegative cell walls.
A) Exotoxins B) Endotoxins C) Enterotoxins D) Leukocidins E) Hemolysins
39)
are bacterial enzymes that dissolve fibrin clots.
A) Coagulases B) Mucinases C) Keratinases D) Kinases E) Hyaluronidases
40)
Which of the following is the endotoxin?
Version 1
12
A) Hemolysin B) Hyaluronidase C) Streptokinase D) Collagenase E) Lipopolysaccharide
41)
Hyaluronidase is a virulence factor in
.
A) amoebic dysentery B) ringworm C) clostridia D) cold virus E) diphtheria
42)
The suffix - emia means
.
A) blood B) a disease or morbid process C) an inflammation D) tumor E) pertaining to
43) The stage of an infectious disease when specific signs and symptoms are seen and the pathogen is at peak activity is the .
Version 1
13
A) prodromal stage B) convalescent stage C) incubation period D) acute phase E) None of the choices are correct.
44) The time from when pathogen first enters the body and begins to multiply, until symptoms first appear is the .
A) prodromal stage B) convalescent stage C) incubation period D) acute phase E) None of the choices are correct.
45) the
The initial, brief period of early, general symptoms such as fatigue and muscle aches is .
A) prodromal stage B) convalescent stage C) incubation period D) acute phase E) None of the choices are correct.
46)
Which of the following pairs is mismatched?
Version 1
14
A) Secondary infection - infection spreads to several tissue sites B) Mixed infection - several agents established at infection site C) Acute infection - rapid onset of severe, short-lived symptoms D) Local infection - pathogen remains at or near entry site E) Toxemia - pathogen's toxins carried by the blood to target tissues
47)
The subjective evidence of disease sensed by the patient is termed a(n)
.
A) syndrome B) symptom C) sign D) pathology E) inflammation
48)
The objective, measurable evidence of disease evaluated by an observer is termed a(n) . A) syndrome B) symptom C) sign D) pathology E) inflammation
49)
Local edema, swollen lymph nodes, fever, soreness, and abscesses are indications of .
Version 1
15
A) toxemia B) inflammation C) sequelae D) a syndrome E) latency
50)
A symptom is
.
A) an objective indication of disease B) a subjective indication of disease C) measurable by health care personnel D) temperature E) None of the choices are correct.
51)
A sign is
.
A) an objective indication of disease B) a subjective indication of disease C) measurable by health care personnel D) temperature E) Both objective indication of diseaseand measurable by health care personnel are correct.
52)
Leukopenia is the
in the level of white blood cells in a patient.
A) elevation B) stabilization C) decrease
Version 1
16
53)
A
is the presence of small numbers of bacteria in the blood.
A) bacteremia B) septicemia C) viremia D) toxemia E) None of the choices are correct.
54)
Infections that go unnoticed because there are no symptoms are called
.
A) syndromes B) malaises C) inflammation D) asymptomatic E) secondary infections
55)
Which of the following is not a normal portal of exit for an infectious disease?
A) Removal of blood B) Urogenital tract and feces C) Coughing and sneezing D) Skin E) All of these arenormal exit portals.
56)
Which of the following is an example of sequelae?
Version 1
17
A) Headache from meningitis B) Difficulty swallowing from a Streptococcus infection C) Arthritis from Lyme disease D) Diarrhea from Salmonella enteritidis infection E) All of the choicesare correct.
57) Long-term or permanent damage to tissues or organs resulting from a specific disease are called .
A) symptoms B) sequelae C) infections D) latencies E) dormancies
58)
The primary, natural habitat of a pathogen where it continues to exist is called the .
A) fomite B) carrier C) vector D) reservoir E) source
59)
Someone who inconspicuously harbors a pathogen and spreads it to others is a
Version 1
.
18
A) fomite B) carrier C) vector D) reservoir E) source
60)
An animal, such as an arthropod, that transmits a pathogen from one host to another is a .
A) fomite B) carrier C) vector D) reservoir E) source
61) The intermediary object or individual from which the infectious agent is actually acquired is termed the .
A) fomite B) carrier C) vector D) reservoir E) source
62)
Reservoirs include
Version 1
.
19
A) humans B) animals C) soil D) water E) All of the choices are correct.
63) Animals that participate in the life cycles of pathogens and transmit pathogens from host to host are .
A) fomites B) aerosols C) mechanical vectors D) droplet nuclei E) biological vectors
64) A humans.
is an infection indigenous to animals that can, on occasion, be transmitted to
A) secondary infection B) sequelae C) nosocomialinfection D) zoonosis E) None of the choices are correct.
65) carriers are shedding and transmitting pathogens while they are recovering from an infectious disease.
Version 1
20
A) Asymptomatic B) Passive C) Incubation D) Chronic E) Convalescent
66) carriers are shedding and transmitting pathogens a long time after they have recovered from an infectious disease.
A) Asymptomatic B) Passive C) Incubation D) Chronic E) Convalescent
67)
All infectious diseases
.
A) are contagious B) occur only in humans C) are caused by microorganisms or their products D) are caused by vectors E) involve viruses as the pathogen
68)
An inanimate object that harbors and transmits a pathogen is a
Version 1
.
21
A) fomite B) carrier C) vector D) reservoir E) source
69)
Which of the following is a direct contact method of microbe transmission?
A) Fomites B) Water C) Vectors D) Aerosols E) Droplets
70)
Which of the following is transmission of disease from mother to fetus?
A) Vertical B) Direct C) Vector D) Horizontal E) Fomites
71)
Healthcare-associated infections involve all the following except
.
A) they are only transmitted by medical personnel B) they often involve the patient's urinary tract and surgical incisions C) the patient's resident biota can be the infectious agent D) Escherichia coli and staphylococci are common infectious agents E) medical and surgical asepsis help lower their occurrence
Version 1
22
72)
When would Koch's postulates be utilized?
A) Determination of the cause of a patient's illness in a hospital microbiology lab B) Development of a new antibiotic in a pharmaceutical lab C) Determination of the cause of a new disease in a microbiology research lab D) Formulation of a vaccine against a new pathogen in a genetic engineering lab E) Whenever the scientific method cannot be used to investigate a microbiological problem
73)
The study of the frequency and distribution of a disease in a defined population is .
A) pathology B) clinical microbiology C) medicine D) immunology E) epidemiology
74) The principal government agency responsible for tracking infectious diseases in the United States is the . A) Centers for Disease Control and Prevention B) World Health Organization C) National Institutes of Health D) United States Department of Agriculture E) Infection Control Committee
75) The number of new cases of a disease in a population over a specific period of time compared with the healthy population is the .
Version 1
23
A) mortality rate B) morbidity rate C) incidence rate D) prevalence rate E) epidemic rate
76)
A disease that has a steady frequency over time in a particular geographic location is .
A) an epidemic B) endemic C) pandemic D) sporadic E) chronic
77)
The total number of deaths in a population due to a disease is the
rate.
A) morbidity B) mortality C) incidence D) endemic E) pandemic
78)
The number of persons afflicted with an infectious disease is the
Version 1
rate.
24
A) morbidity B) mortality C) incidence D) endemic E) pandemic
79)
What is one goal of the Human MicrobiomeProject?
A) To sequence the DNA of all microorganisms B) To study the prevalence of disease C) To study microorganisms in their natural habitat D) To provide comprehensive characterization of microbiota relating to human health and disease E) To discover new organisms in extreme habitats
80) All of the following statements are true regarding discoveries made by the Human Microbiome Project except .
A) the genes coded by our microbiome number in the millions, whereas human cells contain approximately 21,000 genes B) areas of the body once believed to be sterile, are now known to harbor microbes C) the microbiota does not contain pathogens; all of the microbes in our microbiome are non-pathogenic D) the composition of the microbiome has implications for the development of other, non-infectious diseases such as diabetes, obesity and asthma E) bacteria, as well as viruses and fungi, make up the microbiome
81) The virus that causes rabies, and the rhinovirus that causes the common cold are both considered true pathogens; the degree of pathogenicity is determined by their .
Version 1
25
A) B) C) D)
virulence protein coat envelope infectivity
82) Tooth decay develops when human saliva and proteins aggregate on the tooth and provide a hospitable surface for a range of microbial species to develop into a biofilm. These organisms ferment carbohydrates and the acid byproducts erode the enamel, forming caries. Tooth decay is therefore considered to be .
A) B) C) D)
a polymicrobial infection a state of well-being nonharmful noninfectious
83) In an experiment to determine the infectious dose for a specific organism, 50% of the test subjects must develop the disease following a specific dose. This is quantified as the ID50and is a measure of the pathogenicity of that organism. Assess the following data from an experiment involving 200 mice and determine the ID50 for the infectious agent. Infectious Dose Living Mice Dead Mice 103 190 10 104 150 50 105 120 80 106 100 100 107 0 200 A) 10 3 organisms B) 10 4 organisms C) 10 5 organisms D) 10 6 organisms E) 10 7organisms
Version 1
26
84) Robert Koch is credited with proving the germ theory of disease; that infectious diseases are caused by specific microorganisms. In developing a series of proofs (Koch's Postulates), he was able to determine the causative organism of anthrax was the bacteria Bacillus anthracis. Which of the following statements does not contribute to Koch's Postulates and the germ theory of disease?
A) The causative organism must be evidenced in each subject presenting with the disease manifestations. B) Inoculating a healthy subject with the isolated organism must result in the reported disease manifestations. C) The organism must be isolated, purified and characterized from an infected individual. D) Each organism from a polymicrobial infection must be isolated and inoculated into a single host. E) The same species of organism must be isolated from the second inoculated individual.
85) If an infectious disease emerged that was highly contagious and virulent, resulting in certain death with no known cure, epidemiological data would show .
A) B) C) D)
a high incidence and low prevalence a low incidence and high prevalence both incidence and prevalence to be high both incidence and prevalence to be low
86) Nowadays, HIV infection is considered a chronic disease, rather than the death sentence it was when it first emerged in the 1980s. Educational strategies to limit transmission, coupled with advances in treatment, means that compared to 1985, .
Version 1
27
A) B) C) D)
the incidence has dropped and the prevalence has increased the incidence has increased and the prevalence has dropped both incidence and prevalence have increased both incidence and prevalence have dropped
87) Ninety people developed diarrhea, fever. and abdominal cramps in the aftermath of a wedding. On diagnosis it was determined that they were suffering from salmonella, and after taking their histories, they all reported that they had eaten the chicken at the reception. This is an example of .
A) B) C) D)
a point-source epidemic a propagated epidemic a pandemic an endemic disease
88) In 2003 over 500 people throughout the United States became ill with hepatitis A that was traced to green onions grown in Mexico. This is an example of .
A) B) C) D)
a common-source epidemic a propagated epidemic a pandemic an endemic disease
89) Choose the statement that best describes the benefits of microbial antagonism to the human host.
Version 1
28
A) Microbialantagonism refers to the mechanisms by which a pathogen damages the host. B) Microbialantagonism keeps normal microbiota from colonizing some parts of the humanbody. C) Microbialantagonism occurs when members of the normal microbiota prevent pathogens fromcolonizing and becoming established in the body. D) Microbialantagonism refers to the effects that antibiotics have on the normalmicrobiota. E) Microbial antagonism refers to theeffects that the human immune system has on the normal microbiota.
90) Please choose the answer that best completes the blanks of this sentence in the correct order. is the ability of a microbe to cause disease, whereas refers to the relative severity of a disease caused by a particular microorganism. A) B) C) D)
Pathogenicity; virulence Virulence; pathogenicity Pathogenicity; opportunistic Virulence; opportunistic
91) Candida or Pseudomonas are pathogens, meaning theyare not usually pathogenic to healthy persons with normal immune system functions because these microbes do not have well-developed virulence factors. A) B) C) D)
92)
opportunistic true virulent temperate
Choose the best definition of virulence factors.
Version 1
29
A) Factors that enable a microorganism to be resistant to antimicrobial medications B) Characteristics of a microorganism, such as its Gram stain morphology, type of media it grows on, and colony morphology C) The site within a host in which a microorganism resides and causes disease D) Characteristics of a microorganism that enable it to establish infection and cause disease E) The natural habitat of the microorganism or the source from which a host can become infected
93) A teacher walking through her first-grade classroom pauses to pick up a used tissue that had ended up on the floor instead of in the waste basket. Unfortunately, after discarding the tissue, she doesn’t immediately wash her hands, and acquires a strain of rhinovirus. Several days later she begins to experience symptoms of a cold. Based upon this scenario, please select the mode of disease transmission demonstrated here.
A) Direct contact (touching) B) Droplet C) Air D) Fomite E) Vector
94)
Epidemiology is the study of disease in the A) B) C) D)
.
community immunocompromised individual environment
95) By law, certain diseases must be recorded with the public health authorities in order to maintain proper surveillance at the local, state, national, and international levels.
Version 1
30
A) B) C) D)
reportable propagated chronic hypersensitive
96)
The percentage of the population with a particular disease at any point in time is the of disease, whereas the number of new cases over a given time period is the disease. A) B) C) D)
of
prevalence; incidence incidence; prevalence incidence; propagation mortality rate; incidence
97) During a recent survey, 0.014% of the population showed evidence of a particular respiratory infection. Please select the term that is illustrated by this statistic.
A) Mortality B) Prevalence C) Incidence D) Epidemic rate E) Endemic rate
98) During 2007, there were 3.6 new cases of pertussis per 100,000 susceptible individuals in the United States, part of a steady increase that has been occurring since the 1980s. Please select the term that is illustrated by this statistic.
Version 1
31
A) Mortality B) Prevalence C) Incidence D) Epidemic rate E) Endemic rate
99) What is believed to be the primary route of infection with SARS-CoV-2, the virus leading to COVID-19?
A) Respiratory droplets B) Fomites C) Contaminated food D) Sexual contact E) Biological vector
100) The cytokine storm seen in patients with severe COVID-19 is an example of a microbe causing disease in which way?
A) B) C) D)
Epigenetic changes Secretion of exotoxins Causing an overreaction of the host immune system Causing direct damage through exoenzymes
101) Many people tested positive for COVID-19, but they did not have any of the signs or symptoms of the disease. Infections like this are referred to as infections.
Version 1
32
A) B) C) D)
asymptomatic persistent latent chronic
102) Many researchers believe that SARS-CoV-2 originated in bats. Which of the following is a true statement?
A) B) C) D)
Bats are the reservoir for SARS-CoV-2. Bats are the source of most cases of SARS-CoV-2 infection. Bats are the reservoir and the major source for SARS-CoV-2 in this pandemic. Bats do not represent either the source or the reservoir for SARS-CoV-2.
103) BecauseSARS-CoV-2 likely originated in bats and then became transmissible to humans, COVID-19 is considered a(n) infection.
A) B) C) D)
zoonotic noncommunicable vector-borne polymicrobial
104) NCLEX Prep - Test Bank Question: Please read the clinical scenario, and then answer the questions that follow to become familiar with the traditional NCLEX question format. A 78-year-old male presents to the emergency department reporting a 2-day history of difficulty breathing, cough, and chest pain. Upon triage assessment, the patient is found to have a fever, rapid rate of breathing, low oxygen saturation, and crackles upon auscultation of the lungs. A chest X-ray reveals consolidation in the right upper lung field. The patient is diagnosed with pneumonia, initiated on oxygen therapy, and admitted for observation and treatment.
Version 1
33
104.1) As the emergency department RN, youdocument the care and assessment of the patient in the chart prior to transfer to an inpatient unit. What is the most appropriate term for the clinical manifestations of disease as reported by the patient?
A) B) C) D)
Signs Symptoms Syndromes Signs and symptoms
104.2) As the RN, you document the care and assessment of the patient in the chart prior to transfer to an inpatient unit. What is the most appropriate term for the clinical manifestations of disease as noted in your assessment?
A) B) C) D)
Signs Symptoms Syndromes Signs and symptoms
104.3) Upon admission to the inpatient unit, a respiratory sputum culture and blood culture are sent. Prior to obtaining the blood culture, you provide education to the patient. Which of the following statements, by the patient, best demonstrates understanding of the rationale for the blood culture?
Version 1
34
A) "The bloodculture will be able to determine which bacterium is causing the infection inmy lungs." B) "The bloodculture will be able to determine which virus is causing the infection in mylungs." C) "The bloodculture will be able to determine if the microbe causing the infection ispresent and multiplying in my blood, putting me at risk forsepticemia." D) "The bloodculture will be able to determine if the infection has infected other bodytissues."
104.4) The patient’s sputum culture returns a positive result for Streptococcus pneumoniae. As the RN, you are aware this capsule-forming bacterium possesses virulence factors to make it difficult to treat. In providing education to the patient regarding the disease process, which of the following statements is most appropriate?
A) "Fluid isbuilding up in your lungs because of the body’s persistent inflammatoryresponse to the microbe." B) "Thisbacterium secretes toxins that directly damage your lungs." C) "Fragments ofthis bacterium’s cell wall directly harm your lungs." D) "Fluid isbuilding up in your lungs because white blood cells are effectively lysing theencapsulated bacteria."
104.5) You arediligent to prevent the spread of disease to other patients. Which of the following is the most appropriate type of isolation for this patient, considering the portal of exit for the pathogen in this case?
A) B) C) D)
Version 1
Standard isolation (gloves) Contact isolation (gown and gloves) Droplet isolation (gown, gloves, and mask) Airborne isolation (gown, gloves, mask, and negative airflow room)
35
105) NCLEX Prep - Test Bank Question: Please read the clinical scenario, and then answer the questions that follow to become familiar with the traditional NCLEX question format. As an RN with the infectious disease team, you are providing an education course to new graduate nurses regarding the process of infection control and prevention.
105.1) Which of the following statements by a new nurse demonstrates understanding of the cause of infectious disease?
A) "All contactwith infectious material leads to colonization." B) "Allcolonization leads to infection." C) "Allinfections lead to disease." D) "Host resistance and organism pathogenicity determine the type and severity of infection an individual will experience.”
105.2) As the RN, youeducate the new nurses about the prevalence of healthcareassociated infections. All of the following are frequent sources of patient infection, except .
A) B) C) D)
urinary catheter endotracheal tube surgical incision hospital beds
105.3) The infectious disease RN discusses scenarios that require hand washing with germicidal soap and water instead of alcohol-based hand cleanser. Which of the following statements by a new nurse is most accurate regarding when hands should be washed with soap and water?
Version 1
36
A) "I should washmy hands when they are visibly contaminated." B) "I should washmy hands before applying gloves." C) "I should washmy hands after removing gloves." D) "I should washmy hands when they are visibly contaminated, but also before and after usinggloves."
105.4) As the infectious disease RN, youare familiar with the concept of normal biota. A positive Escherichia coli culture would be unremarkable for which of the following samples?
A) B) C) D)
Stool Sputum Blood Cerebrospinal fluid
106) NCLEX Prep - Test Bank Question: Please read the clinical scenario, and then answer the questions that follow to become familiar with the traditional NCLEX question format. A 36-year-old female is bit by a raccoon and presents to the emergency department with mild trauma to her right lower extremity. Following a triage assessment, the patient’s wound is cleaned and dressed. The physician verbalizes concern that the patient may have been exposed to rabies from the bite. One role of the RN is to provide education to the patient regarding rabies infection and its transmission.
106.1) Rabies is an example of this type of infection, which is indigenous to animals but naturally transmissible to humans .
Version 1
37
A) B) C) D)
mixed infection focal infection zoonotic infection latent infection
106.2) Which of the following terms most appropriately describes the role of the raccoon in the patient’s exposure to infection?
A) B) C) D)
Living reservoir Nonliving reservoir Carrier Biological vector
106.3) If the patient develops rabies infection, the time from the bite to the appearance of symptoms is referred to as the .
A) B) C) D)
incubation period prodromal stage period of invasion convalescent period
106.4) If the patient develops rabies infection, you know that it must be reported to local and state public health officials. Which of the following diseases is also reportable in the United States? A) B) C) D)
Version 1
Pinworm infection Fifth disease EBV infection Cholera
38
107) Under certain circumstances, members of a person's resident biota can become opportunistic pathogens. ⊚ true ⊚ false
108) Most of the skin's resident biota are found in the uppermost, superficial layers of the epidermis. ⊚ ⊚
true false
109) The virulence factors of a pathogen are established by how strong or weak a patient's body defenses are at the time of infection. ⊚ ⊚
true false
110) A fetus can get an infection when a pathogen in the mother's blood is capable of crossing the placenta to the fetal circulation and tissues. ⊚ ⊚
true false
111) When an infected person is in the incubation period, that person cannot transmit the pathogen to others. ⊚ ⊚
112)
true false
Septicemia means that a pathogen is present and multiplying in the blood.
Version 1
39
⊚ ⊚
113)
Latency is a dormant state of an infectious agent. ⊚ ⊚
114)
true false
Fomites, food, and air serve as indirect transmission routes of pathogens. ⊚ ⊚
115)
true false
true false
Koch's postulates are easily satisfied for viral diseases. ⊚ ⊚
true false
116) While one person's microbiome differs when compared to another person, one individual's microbiome is uniform throughout the body; for example, the organisms on the left hand will be the same as the organisms on the right. ⊚ ⊚
true false
117) Virulence differs from pathogenicity in that pathogenicity describes the ability of an organism to cause disease, whereas virulence describes the degree of pathogenicity as the disease develops. ⊚ ⊚
Version 1
true false
40
118) It is important to understand the extent and significance of polymicrobial infections, since treating an infection with a single antibiotic for an assumed causative organism may not adequately eliminate the infection. ⊚ ⊚
true false
119) In a comparison of two pathogens, organism A has an infectious dose of 200,000 organisms, whereas organism B has an infectious dose of 15 organisms. This means that organism A is much more pathogenic than organism B. ⊚ ⊚
true false
120) The spread of influenza, which is particularly widespread during the winter months, is an example of a propagated epidemic. ⊚ ⊚
Version 1
true false
41
Answer Key Test name: Ch 11 4e 1) [B, D] 2) [A, B, C, G, H] 3) [A, C, E] 4) [A, D, E, G] 5) [B, C, E] 6) [A, D, F] 7) [A, C, F] 8) [B, D, E, G] 9) [B, C, E] 10) [B, D, E, F, G] 11) D 12) D 13) A 14) B 15) A 16) E 17) E 18) E 19) A 20) B 21) D 22) C 23) A 24) A 25) D 26) C Version 1
42
27) E 28) A 29) C 30) B 31) E 32) A 33) B 34) C 35) B 36) A 37) B 38) B 39) D 40) E 41) C 42) A 43) D 44) C 45) A 46) A 47) B 48) C 49) B 50) B 51) E 52) C 53) A 54) D 55) E 56) C Version 1
43
57) B 58) D 59) B 60) C 61) E 62) E 63) E 64) D 65) E 66) D 67) C 68) A 69) E 70) A 71) A 72) C 73) E 74) A 75) C 76) B 77) B 78) A 79) D 80) C 81) A 82) A 83) D 84) D 85) A 86) A Version 1
44
87) A 88) A 89) C 90) A 91) A 92) D 93) D 94) A 95) A 96) A 97) B 98) C 99) A 100) C 101) A 102) A 103) A 104) Section Break 104.1) B 104.2) A 104.3) C 104.4) A 104.5) C 105) Section Break 105.1) D 105.2) D 105.3) D 105.4) A 106) Section Break 106.1) C Version 1
45
106.2) A 106.3) A 106.4) D 107) TRUE 108) TRUE 109) FALSE 110) TRUE 111) FALSE 112) TRUE 113) TRUE 114) TRUE 115) FALSE 116) FALSE 117) TRUE 118) TRUE 119) FALSE 120) TRUE
Version 1
46
CHAPTER 12 1) Select examples of innate host defense mechanisms in order to test your understanding of host defenses.
A) Anatomical and physiological barriers B) Internal cellular and chemical defenses C) Host defenses due to B and T cells and their products D) Use of drugs and antibiotics E) Phagocytosis and inflammation
2)
Which of the following are correct descriptions of the first line of defense?
A) Involves barriers at portals of entry B) Includes chemicals such as lysozyme and lactic acid C) Involves phagocytic cells (neutrophils and macrophages) D) Involves recognition of foreign agents by their pathogen-associated molecular patterns E) Examples include the ciliated epithelium of the respiratory passages, excretion of urine, and production of tears
3)
Innate, nonspecific physical barriers of host defenses
. (Check all that apply.)
A) include the skin B) include the mucous membranes C) all contain keratin which makes the barriers highly impervious D) are part of the second line of host defenses E) are all covered with mucus that adds another layer of protection
4) Select all of the components of the mononuclear phagocyte system to test your understanding of the components of this system.
Version 1
1
A) Thymus B) Lymph nodes, spleen, GALT C) Heart (circulates components) D) Macrophages E) Extracellular fluid-filled spaces
5) Select all that represent characteristics of the lymphatic system to test your understanding of its structure and major functions.
A) Provides a route to extract fluid from circulation in order to diffuse into extracellular spaces B) Helps to regulate the inflammatory response by acting as a “drain-off” system C) Moves like the blood in response to pumping by the heart D) Lymph contains a number of immune cells and their products to defend against pathogens E) Lymph can flow to and from the heart
6) Select all of the characteristics of T lymphocytes, which are involved in adaptive immunity.
A) Mature in the bonemarrow B) Mature in thethymus C) Move freely amonglymphoid tissues and connective tissue D) Responsible for cell mediated immunity E) Form specialized plasma cells thatproduce antibodies
7) Select all of the characteristics of B lymphocytes, which are involved in adaptive immunity.
Version 1
2
A) Mature in the bone marrow B) Mature in the thymus C) Move freely among lymphoid tissues and connective tissue D) Responsible for cell mediated immunity E) Form specialized plasma cells that produce antibodies
8) Select all of the innateprocesses below to test your understanding of the various categories of innate immunity.
A) Inflammatory response B) Complement cascade C) Phagocytosis D) Interferon production E) Antibody production
9)
Components of the first line of defense include all the following except
.
A) the tough cell sheet of the upper epidermis of the skin B) nasal hairs C) flushing action of tears and blinking D) flushing action of urine E) phagocytic white blood cells
10) The chemical found in tears and saliva that hydrolyzes the peptidoglycan in certain bacterial cell walls is .
Version 1
3
A) lactic acid B) hydrochloric acid C) lysozyme D) histamine E) bile
11)
Innate chemical defenses include
.
A) lysozyme B) lactic acid and electrolytes of sweat C) skin's acidic pH and fatty acids D) stomach hydrochloric acid E) All of thechoices are correct.
12) The body region where a ciliary escalator helps to sweep microbes trapped in mucus away from that body site is the .
A) skin B) respiratory tract C) digestive tract D) urinary tract E) eyes
13) This body region is protected by fatty acids, acidic pH, lactic acid, and a tough cell barrier with its own normal biota.
Version 1
4
A) Skin B) Respiratory tract C) Digestive tract D) Urinary tract E) Eyes
14)
Keratin is an important aspect of nonspecific defense because it
.
A) is toxic to pathogens B) creates a physical barrier against pathogens C) destroys pathogens D) physically restricts pathogens to a specific region E) None of the choices are correct.
15)
Each of the following are physical barriers to pathogens except
.
A) unbroken skin B) mucus C) hairs D) tears E) T cells
16)
Lysozyme is found in
.
A) mucous membranes of the respiratory tract B) mucous membranes of the gastrointestinal tract C) salivary secretions D) sweat from sweat glands E) All of thechoices are correct.
Version 1
5
17)
A properly functioning immune system is responsible for
.
A) surveillance of the body B) recognition of foreign material C) destruction of foreign material D) All of the choices are correct. E) Only recognitionof foreign materialand destruction of foreign materialarecorrect.
18)
Immune system cells differentiate between self and foreign cells by their
.
A) cell walls B) biota C) markers D) skin E) cell processes
19) The term system response.
is given to any foreign substance that stimulates a specific immune
A) antibody B) allergen C) antigen D) foreign body E) None of the choices are correct.
20)
Which of the following fluid compartments is not a partner in immune function?
Version 1
6
A) B) C) D)
21)
Bloodstream Extracellular fluid Lymphatic system Intracellular fluid
Plasma
.
A) is the liquid portion of blood in which blood cells are suspended B) is mostly water C) contains albumin and globulins D) contains fibrinogen E) All of thechoices are correct.
22)
Which of the following is incorrect about circulating blood cells?
A) After birth, produced in red bone marrow sites B) Develop from undifferentiated stem cells C) Include Kupffer cells D) Include leukocytes that are either granulocytes or agranulocytes E) Include erythrocytes that, when mature, lose their nuclei
23) The blood cells that function in allergic reactions and inflammation, contain peroxidase and lysozyme, and particularly target parasitic worms and fungi are .
A) basophils B) eosinophils C) neutrophils D) monocytes E) lymphocytes
Version 1
7
24) The least numerous of all white blood cells that release histamine during inflammation and allergic reactions are .
A) basophils B) eosinophils C) neutrophils D) monocytes E) lymphocytes
25) Which white blood cells comprise 20% to 30% of the circulating WBCs and are the cells that function in the body's third line of defense?
A) Basophils B) Eosinophils C) Neutrophils D) Monocytes E) Lymphocytes
26)
The most numerous WBCs that have multilobed nuclei and are very phagocytic are . A) basophils B) eosinophils C) neutrophils D) monocytes E) lymphocytes
27) Which white blood cells comprise 3% to 7% of circulating WBCs, are phagocytic, and can migrate out into body tissues to differentiate into macrophages?
Version 1
8
A) Basophils B) Eosinophils C) Neutrophils D) Monocytes E) Lymphocytes
28)
All of the following pertain to platelets except they
.
A) contain hemoglobin to transport oxygen and carbon dioxide B) function primarily in hemostasis C) originate from giant multinucleate cells called megakaryocytes D) function in blood clotting and inflammation E) are not whole cells but are pieces of cells
29)
Hematopoiesis is the
.
A) loss of blood due to hemorrhaging B) production of only red blood cells C) production of white blood cells, red blood cells, and platelets D) plugging of broken vessels to stop bleeding E) migration of white blood cells from the blood out to the tissues
30)
Plasma cells
.
A) function in cell-mediated immunity B) are derived from T lymphocytes C) function in blood clotting D) produce and secrete antibodies E) All of thechoices are correct.
Version 1
9
31) Which type of white blood cells are particularly attracted to sites of parasitic worm infections?
A) Monocytes B) Eosinophils C) Basophils D) Neutrophils E) Lymphocytes
32)
Each of the following are granulocytes except
.
A) neutrophils B) eosinophils C) lymphocytes D) basophils E) All of thechoices are granulocytes.
33)
The granules of basophils contain
.
A) antibodies B) antigens C) digestive enzymes D) histamine E) lysozyme
34)
Which of the following is in the correct order from greatest to least phagocytic activity?
Version 1
10
A) Neutrophils, eosinophils, macrophages B) Neutrophils, macrophages, eosinophils C) Eosinophils, macrophages, neutrophils D) Macrophages, neutrophils, eosinophils E) Macrophages, eosinophils, neutrophils
35) Neutrophils, eosinophils, and basophils are called because they have prominent cytoplasmic inclusions that, in a stained blood smear, appear with identifying, characteristic colors. A) leukocytes B) granulocytes C) agranulocytes D) monocytes E) None of the choices are correct.
36)
function in humoral immunity, while
function in cell-mediated immunity.
A) B cells; T cells B) T cells; B cells C) Monocytes; basophils D) Basophils; T cells E) B cells; neutrophils
37) When monocytes migrate from the blood out to the tissues they are transformed by inflammatory mediators to develop into .
Version 1
11
A) primary phagocytes B) neutrophils C) killer T cells D) cytotoxic T cells E) macrophages
38) Leukocytes that are derived from monocytes and have long, thin processes to trap pathogens are .
A) dendritic cells B) eosinophils C) macrophages D) platelets E) mast cells
39) Which of the following lymphoid organs or tissues has the immunological function of filtering pathogens from the blood?
A) Lymph nodes B) Thymus C) Spleen D) GALT E) Tonsils
40) What structures are found along lymphatic vessels and are heavily clustered in the armpit, groin, and neck?
Version 1
12
A) Lymph nodes B) Thymus C) Spleen D) GALT E) Tonsils
41) Which gland shrinks in size during adulthood and has hormones that function in maturation of T lymphocytes?
A) Lymph nodes B) Thymus C) Spleen D) GALT E) Tonsils
42)
The lymphoid tissues of the intestinal tract are collectively referred to as
.
A) lymph nodes B) thymus C) spleen D) GALT E) tonsils
43)
Diapedesis is the
Version 1
.
13
A) loss of blood due to hemorrhaging B) production of only red blood cells C) production of white blood cells, red blood cells, and platelets D) plugging of broken vessels to stop bleeding E) migration of white blood cells from the blood out to the tissues
44)
The four classic signs and symptoms of inflammation include all the following except .
A) redness B) warmth C) swelling D) pain E) chills
45)
All the following are events of early inflammation except
.
A) macrophages appear first and begin phagocytosis B) chemical mediators and cytokines are released C) brief vasoconstriction is followed by vasodilation D) exudate and pus can accumulate E) capillaries become more permeable resulting in edema
46)
Which of the following is incorrect about inflammation?
Version 1
14
A) It can last hours to years. B) Pyrogens cause vasodilation and increased capillary permeability. C) Serotonin causes smooth muscle contraction. D) Fever could be beneficial to inhibiting the pathogen. E) Basophils and mast cells release histamine.
47)
Which of the following is mismatched?
A) Interferon alpha and beta—inhibits viral replication B) Interleukin-2—stimulates T-cell mitosis and B-cell antibody production C) Serotonin—causes smooth muscle contraction D) Prostaglandins—activate eosinophils and B cells E) Tumor necrosis factor—increases chemotaxis and phagocytosis
48)
Histamine, serotonin, and bradykinin are all
.
A) vasoactive mediators B) mediators of B-cell activity C) mediators of T-cell activity D) mediators that increase chemotaxis E) fever inducers
49)
Each of the following is involved in the migration of white blood cells except
Version 1
.
15
A) chemotaxis B) diapedesis C) motility D) vasodilation E) phagocytosis
50)
The leakage of excess fluid into tissues is called
.
A) chemotaxis B) edema C) diapedesis D) vasoactivity E) pus
51)
Which of the following is mismatched dealing with inflammation?
A) Rubor—redness B) Calor—warmth C) Tumor—cancer D) Dolar—pain E) None of these are mismatched.
52)
Which of the following is not a chief function of inflammation?
A) Start tissue repair B) Destroy microbes C) Mobilize and attract immune components to injury site D) Block further invasion E) Cause a fever
Version 1
16
53)
The circulating substances that affect the hypothalamus and initiate fever are
.
A) complement B) interferons C) leukotrines D) pyrogens E) lysozymes
54) A person will typically experience the sensation of occur in the body.
when fever is starting to
A) heat B) pain C) chills D) sweat E) All of thechoices are correct.
55)
Each of the following are benefits of fever except it
.
A) reduces the ability of temperature-sensitive organisms to multiply B) increases the availability of iron C) increases phagocytosis D) increases metabolism E) stimulates hematopoiesis
56)
Which of the following is not a type of phagocyte?
Version 1
17
A) Neutrophil B) Macrophage C) Kupffer cell D) Monocyte E) Lymphocyte
57)
Which of the following is not an event of phagocytosis?
A) Diapedesis B) Chemotaxis C) Phagolysosome formation D) Destruction E) Ingestion
58)
What type of molecules act to draw phagocytes to foreign substances?
A) MALTs B) RES C) PAMPs D) BALTs E) Lysosomes
59) Which of the following is a chemical used by phagocytes to destroy ingested foreign substances?
Version 1
18
A) Hydrogen peroxide B) Nitric oxide C) Lactic acid D) Superoxide anion E) All of these areused.
60)
All of the following pertain to interferon except it
.
A) is a protein B) is produced by certain white blood cells and tissue cells C) includes alpha, beta, and gamma types D) inhibits viruses, tumors, and cancer gene expression E) increases capillary permeability and vasodilation
61)
Which of the following is incorrect about complement?
A) Is composed of at least 20 blood proteins B) Only appears in the blood during a response to a pathogen C) Acts in a cascade reaction D) Involves a classical pathway E) Involves an alternate pathway
62)
The membrane attack stage of the complement cascade involves
Version 1
.
19
A) initiation of the cascade B) production of inflammatory cytokines C) a ring-shaped protein that digests holes in bacterial cell membranes and virus envelopes D) cleaving of C3 to yield C3a and C3b E) C1q binding to surface receptors on a membrane
63)
The
system of blood proteins acts to lyse foreign cells and viruses.
A) humoralimmunity B) cell-mediated C) complement D) lymphoid E) None of the choices are correct.
64)
What is the first step in the major events of the inflammation process?
A) Formation of pus and edema B) Scar formation and/or resolution C) Vascular reactions D) Injury and mast cells releasing chemical mediators E) Activation of the complement system
65) During what process are hypochlorite and hydrogen peroxide produced to destroy bacteria and inhibit viral replication?
Version 1
20
A) Inflammation B) Phagocytosis C) Interferon production D) Complement production E) Bradykinin
66) Which protein can be produced by a virus-infected cell, in order to communicate with other cells the need to produce antiviral proteins?
A) B) C) D)
Complement Albumin Interferon Histamine
67) Cytokines are chemicals and proteins that are critical components of the immune system. Which of the following statements is incorrect regarding the function of cytokines?
A) All cytokines are synthesized to stimulate the inflammatory response; their purpose is to mediate vascular reactions and diapedesis. B) Cytokines are secreted by a wide variety of cell types, including immune cells, epithelial and endothelial cells. C) Cytokines play a role in both inflammatory stimulation, as well as being antiinflammatory mediators. D) Cytokines mediate communication between cells; they are crucial for stimulation, suppression and control of all aspects of the immune response.
68) Cytokines mediate a variety of processes in the immune system, for example, encourages inflammatory responses and suppresses the actions of immune cells.
Version 1
21
A) TNF-β; IL-10 B) IL-10; TNF-β C) erythropoietin; TNF-β D) seratonin; histamine E) IL-10; IL-7
69) The human microbiome is considered an important component of the first line of defense. Which statement adequately reflects its role as a defensive barrier?
A) The human microbiota competes with pathogens for space and nutrients limiting their ability to cause infection, and trains the immune system to recognize antigens. B) The organisms that comprise the microbiota are able to phagocytose potential pathogens, thereby eliminating them from the human host. C) The human microbiome has evolved with the species so it is able to actively destroy pathogens by raising the body temperature when an infection by a pathogen occurs. D) Cells in the human microbiome secrete cytokines that play a role in stimulating the immune response when a pathogen has entered the system.
70) Which of the following is not a mechanism employed by the human microbiota to contribute to the first line of defense?
A) It can phagocytose pathogens B) It can lower the pH making the environment too acidic for pathogens to survive C) It can out-compete pathogens for limited nutrients D) It can train the immune cells to recognize antigenic markers E) It can block potential binding sites on epithelial cells restricting adhesion, which is a necessary step for infection
71)
Which of the following is not a component of the second line of defense?
Version 1
22
A) Keratin B) Complement C) Inflammation D) Phagocytosis E) Fever
72) Inflammation, fever, phagocytosis, and antimicrobial molecules are considered innate because .
A) they are not antigen-specific; they contribute to host defenses regardless of the source of activation B) they are activated only if the system has encountered the antigen previously C) they are physical barriers that block the binding of pathogens to host tissues D) they operate independently of the first line of defense and the B and T lymphocytes in the third line of defense
73)
Which statement below characterizes the complement system?
A) Complement comprises a team of blood proteins that are activated in a cascade, ultimately leading to pathogen cytolysis, inflammation and enhanced phagocytosis. B) Complement pyrogens stimulate the hypothalamus to raise the body temperature, increasing metabolism and preventing the growth of temperature-sensitive pathogens. C) Complement proteins bind to cell surfaces and induce changes in genetic expression, eliminating not only pathogens, but cancer cells also. D) Complement proteins, produced by a host cell in response to viral attack, are secreted and enter neighboring cells, stimulating the production of antiviral proteins in anticipation of an invasion.
74)
Which statement below characterizes the action of interferon?
Version 1
23
A) Interferon, produced by a host cell in response to viral attack, is secreted and enters neighboring cells, stimulating the production of antiviral proteins in anticipation of an invasion. B) Interferon stimulates the hypothalamus to raise the body temperature, increasing metabolism and preventing the growth of temperature-sensitive pathogens. C) Interferon comprises a team of blood proteins that are activated in a cascade, ultimately leading to pathogen cytolysis, inflammation and enhanced phagocytosis. D) Interferon binds to cell surfaces and induces changes in genetic expression, eliminating not only pathogens, but cancer cells also.
75)
Choose the statement that best describes the characteristics of lymph.
A) B) C) D)
It contains red and white blood cells It is formed by fluid that leaves circulation into the surrounding extracellular spaces It moves due to stimulation of the lymph vessels by nerve impulses One of its main functions is oxygenation of the tissues
76) Similar to macrophages, are products of the monocytic cell line that reside throughout the tissues and present antigen to lymphocytes. A) B) C) D)
77)
dendritic cells natural killer cells mast cells neutrophils
Choose the statement that describes the first stage of phagocytosis.
Version 1
24
A) Phagocytes maketheir way to an inflammatory site by random chance B) Phagocytes move in response to chemicals from the site of injury or inflammation C) Phagocytesrelease substances to attack the extracellular microbe D) Phagocytes formpseudopodia at the site of inflammation in order to grab the invadingmicrobe
78) Fever is initiated when a substance in circulation, called a(n) hypothalamus causing it to reset body temperature to a higher setting. A) B) C) D)
, acts on the
pyrogen prostaglandin interferon inflammasome
79) When a cell is infected by a virus, is produced and is secreted from the infected cell to protect nearby uninfected cells by binding to surface receptors and initiating synthesis of antiviral proteins. A) B) C) D)
Version 1
interferon pyrogen interleukin-2 exudate
25
80) NCLEX Prep - Test Bank Question: Please read the clinical scenario, and then answer the questions that follow to become familiar with the traditional NCLEX question format. A 16-year-old male with cystic fibrosis is admitted to the inpatient pulmonary unit. He has been hospitalized several times in the past year with respiratory infections, including Pseudomonas and H. influenza. Because of his thick mucous, airway inflammation, and immobile mucosal cilia, microorganisms are unable to be effectively cleared from his lungs. He is supposed to perform vigorous airway mucous clearance measures at home with a vibrating vest and IPV (intrapulmonary percussive ventilation) device. However, the patient states that his treatment takes hours and he does not always feel sick. He estimates his compliance with home airway clearance regimen to be less than 50%.
80.1) If the patient’s cilia are unable to clear his lungs of mucus, which line of host defense is breached? A) B) C) D)
First line of defense Second line of defense Third line of defense First and second lines of defense
80.2) The patient asks the nurse to explain to him why he keeps getting sick from the same bacteria. Which of the following statements is the most appropriate response?
A) Your lungs produce these organisms because your cilia are permanently damaged. B) Your mucus traps and harbors the microorganisms and when they can’t be cleared, they begin to breach your second and third lines of defense. C) Your disease stunts your immunity, so you are unable to fight those bacteria. D) You don’t have second and third line defenses because of your cystic fibrosis.
80.3) The patient asks the nurse to explain what cilia are. All of the following statements are correct except .
Version 1
26
A) B) C) D)
cilia form a microscopic brush-like lining cilia help move and expel foreign invaders cilia compose a sort of microscopic "escalator" to expel invading organisms cilia are chemical-excreting natural killer cells
80.4) Following teaching by the medical team, the patient asks “can I just do my airway clearance exercises when I am not feeling well and on the weekends, even though the doctor tells me I have to do them every day?” What is a correct response by the nurse? A) It is okay to skip your airway clearance if you are not feeling sick as long as you are compliant on the weekends. B) Your body does not have the ability to protect itself because of its damaged immunity so you must do airway clearance regularly. C) As long as you perform airway clearance twice as often when you start to feel sick, you will make up the difference. D) Your airway clearance activities can be performed at your convenience.
81) NCLEX Prep - Test Bank Question: Please read the clinical scenario, and then answer the questions that follow to become familiar with the traditional NCLEX question format. A 35-year-old male presents to the ER with a laceration on his left hand. The patient reports an hour ago he was working on renovations in his 1945 home, when an old board fell and injured his hand. The laceration is approximately 3 inches long and moderately deep. It appears that the cut was bleeding heavily, but he has stopped the bleeding by holding pressure on the site with an old t-shirt.
81.1) At this phase of injury, how would the nurse expect the area surrounding the laceration to present?
Version 1
27
A) B) C) D)
Cool and blue Pink, warm, and dry Red and hot Dusky, gray, and clammy
81.2) The patient asks why the laceration is puffy, and has become itchy. Which of the following statements by the nurse is the most appropriate response?
A) This is a normal response to injury. Blood has rushed to the area, fluid is leaking from the damaged vessels, and chemical mediators called cytokines are causing you to itch. B) This is a normal response to injury. Your body is building up pressure around the wound to stop the bleeding, and the wood likely left internal splinters that are causing you to itch. C) This is a concerning sign. I will call the doctor now to lance the swollen area, and in the meantime, I will apply hydrogen peroxide to the wound. D) This is a concerning sign. Edema and pruritus are early signs of gangrene. To help the swelling, hold your hand above your heart to decrease blood flow to the area.
81.3) The patient states, “There has to be a way for me to reduce this swelling. There is no way this can be good for me because it is so uncomfortable.” Which of the following statements by the nurse is not an appropriate response? A) Swelling helps trap foreign microbes to prevent them from getting into your main circulation. B) Inflammation is a sign that your immune system is fighting the effects of your skin barrier being broken. C) The influx of fluid into the area is helping to dilute toxins. D) Swelling is a result of your bones responding to the influx of calcium-fighting toxins.
81.4) Version 1
What is the natural progression of healing for this wound? 28
A) B) C) D)
Injury, vascular reactions, secondary injury, scar formation Injury, edema/pus formation, vascular reactions, scar formation Injury, edema/pus formation, scar formation, exudative leaking Injury, vascular reactions, edema/pus formation, scar formation
81.5)
Which line(s) of defense has been breached for this injury?
A) B) C) D)
First line of defense Second line of defense First and second lines of defense First, second, and third lines of defense
82) NCLEX Prep - Test Bank Question: Please read the clinical scenario, and then answer the questions that follow to become familiar with the traditional NCLEX question format. The mother of a 6-year-old child brings her daughter to the pediatrician’s office because she has had a sore throat, cough, and fever for several days. She reports a fever up to 104°F that worsens at night, but seems to respond to treatment with acetaminophen. The child has not been eating or drinking well because she says, “it hurts to swallow.” She is ill appearing, and does not seem interested in playing with the toys that are in the examination room. The physician notes a thick, white exudate on both of the patient’s tonsils, which also seem enlarged. A throat culture and complete blood count with differential is ordered.
82.1) The nurse examines the child and finds that the lymph nodes along of her neck are warm, tender, and slightly enlarged. Her other lymph nodes are nonpalpable or small, nontender, rubbery, and less than 2 mm in size. What is the most accurate interpretation of this finding?
Version 1
29
A) B) C) D)
This is a normal examination of the posterior cervical chain lymph nodes. This examination is consistent with a systemic infection. This examination is consistent with a localized infection. This examination is definitive for an oncologic diagnosis.
82.2) The patient’s mother expresses concern about her daughter’s fevers. She states that the fevers are a sign that her child is unable to fight this infection. What is a proper response to these concerns?
A) It is impossible to determine exactly why she is having these fevers. Doctors call this a fever of unknown origin. B) Fevers are a natural response by the body to help fight infection. C) These fevers are very dangerous, and you have strong reason for concern. D) High fever could be a sign of a tumor of the hypothalamus.
82.3) The nurse examines the child’s tonsils while preparing to swab them with the culture swab. A white pustule coating is noted to be covering the tonsils. What is this pus composed of?
A) B) C) D)
Waste from red blood cells Live bacteria Fibrin threads White blood cells and debris from phagocytosis
82.4) The lab performs a complete blood count with cell differential while the patient is still in the office. The results show an elevated white blood cell count, as well as a high neutrophil count. What does this indicate?
Version 1
30
A) B) C) D)
82.5)
Bacterial infection Viral infection Protozoan infection Helminth infection
What is the most likely cause of the patient’s swollen tonsils?
A) Her intense cough caused rupture of the tonsils, and swelling is part of the normal wound-healing process. B) Tonsils consist of lymphatic tissue, and became swollen by filtering out microbes and their toxic products. C) She is dehydrated from poor fluid intake, so the tonsils are retaining fluid. D) Tonsils are sensitive to stimulation, so they became swollen from her constant coughing.
83)
Plasma is also called lymph. ⊚ ⊚
84)
B lymphocytes and T lymphocytes mature in the thymus gland. ⊚ ⊚
85)
true false
true false
Monocytes and lymphocytes are agranular leukocytes. ⊚ ⊚
Version 1
true false
31
86)
During inflammation, a high neutrophil count is a common sign of bacterial infection. ⊚ ⊚
true false
87) Inflammatory responses are orchestrated by the immune system and are part of the body's third line of defense. ⊚ ⊚
88)
true false
Endotoxin is an exogenous pyrogen. ⊚ ⊚
true false
89) A low to moderate fever in an otherwise healthy person should be treated immediately with antipyretic drugs. ⊚ ⊚
90)
During phagocytosis, intracellular digestion begins as soon as the phagosome is formed. ⊚ ⊚
91)
true false
true false
The production of interferon will protect the host cell from a viral infection.
Version 1
32
⊚ ⊚
92)
Complement proteins are produced by the spleen. ⊚ ⊚
93)
true false
true false
Certain complement components stimulate inflammation and phagocytosis. ⊚ ⊚
true false
94) Overuse of disinfectants and antiseptics, as well as antibiotics, can result in inflammatory diseases stemming from an inadequate microbiome. ⊚ ⊚
true false
95) The second line of defense is a series of systems that involves both cells and chemicals working together. ⊚ ⊚
true false
96) The second and third lines of host defenses determine whether cells are self or nonself by examining cell surfaces for identifying molecular markers. ⊚ ⊚
Version 1
true false
33
97) The reticuloendothelial system, also known as the mononuclear phagocyte system, is an intricate passageway within and between tissues and organs and is an area full of lymphocytes, which are phagocytic cells used to attack foreign invaders. ⊚ ⊚
true false
98) The three complement cascades are activated by different mechanisms but share the same end results of the membrane attack complex formation, enhanced phagocytic capabilities, and amplification of inflammation. ⊚ ⊚
Version 1
true false
34
Answer Key Test name: Ch 12 4e 1) [A, B, E] 2) [A, B, E] 3) [A, B] 4) [A, B, D] 5) [B, D] 6) [B, C, D] 7) [A, C, E] 8) [A, B, C, D] 9) E 10) C 11) E 12) B 13) A 14) B 15) E 16) C 17) D 18) C 19) C 20) D 21) E 22) C 23) B 24) A 25) E 26) C Version 1
35
27) D 28) A 29) C 30) D 31) B 32) C 33) D 34) D 35) B 36) A 37) E 38) A 39) C 40) A 41) B 42) D 43) E 44) E 45) A 46) B 47) D 48) A 49) E 50) B 51) C 52) E 53) D 54) C 55) B 56) E Version 1
36
57) A 58) C 59) E 60) E 61) B 62) C 63) C 64) D 65) B 66) C 67) A 68) A 69) A 70) A 71) A 72) A 73) A 74) A 75) B 76) A 77) B 78) A 79) A 80) Section Break 80.1) A 80.2) B 80.3) D 80.4) B 81) Section Break 81.1) C Version 1
37
81.2) A 81.3) D 81.4) D 81.5) C 82) Section Break 82.1) C 82.2) B 82.3) D 82.4) A 82.5) B 83) FALSE 84) FALSE 85) TRUE 86) TRUE 87) FALSE 88) TRUE 89) FALSE 90) FALSE 91) FALSE 92) FALSE 93) TRUE 94) TRUE 95) TRUE 96) TRUE 97) FALSE 98) TRUE
Version 1
38
CHAPTER 13 1) How is the third line of defense different than the first and second lines of defense? Check all that apply. A) B) C) D)
2)
Antigen specificity Development of memory Response to pathogen-associated molecular patterns Release of inflammatory mediators
Select all of the statements that apply to B-cell maturation.
A) B cells form and mature in the bone marrow B) B cells form in the bone marrow and mature in the thymus C) Naive B cells circulate for lymphatic organs where they come into contact with antigens D) Hormones guide the maturation process E) B cells become responsive to MHC molecules
3)
Select all of the characteristics shared by both B- and T-cell receptors. A) Receptors are formed by genetic recombination B) Receptors contain only constant regions C) Receptors are inserted into the cell membrane D) Antigen binding site formed by one polypeptide chain E) B- and T-cell receptors are also secreted
4)
Select all cell types below that can serve as antigen-presenting cells.
Version 1
1
A) Macrophages B) B cells C) Dendriticcells D) Neutrophils E) Eosinophils
5)
Select all of the characteristics exhibited by effective vaccines.
A) Little or no adverse side effects B) Protect against exposure to wild-type, natural pathogens C) Requires numerous doses or boosters D) Stimulate a B-cell or T-cell response E) Relatively long shelf life
6)
Select all of the correct statements that illustrate advantages of live, attenuated vaccines.
A) They produce long-lasting protection due to the memory response B) They require special storage facilities C) They can revert to a more virulent form due to mutation D) They can be spread to nonimmune individuals E) Viable attenuated microbes multiply, inducing robust immune responses
7)
A foreign molecule that causes an adaptive immune response is a(n)
.
A) PAMP B) marker C) hapten D) antibody E) antigen
Version 1
2
8)
Adaptive immunity involves the response of
.
A) skin barriers B) B and T lymphocytes C) lysozyme D) mucous membranes E) interferon
9)
The progeny cells of a B-cell clone are called
.
A) antibodies B) sensitized T cells C) activated macrophages D) plasma cells E) bursa cells
10)
Helper T cells
.
A) secrete antibodies B) function in allergic reactions C) directly destroy target cells D) suppress immune reactions E) activate B cells and other T cells
11)
Plasma cells
Version 1
.
3
A) secrete antibodies B) function in allergic reactions C) directly destroy target cells D) suppress immune reactions E) activate B cells and other T cells
12)
Lymphocyte maturation involves
.
A) hormonal signals that initiate development B) B cells maturing in bone marrow sites C) T cells maturing in the thymus D) release of mature lymphocytes to begin migration to various lymphoid organs E) All of the choices are correct.
13)
What type of cells secrete antibodies?
A) B cells B) Cytotoxic T cells C) Plasma cells D) Helper T cells E) Antigen-presenting cells
14)
Cell surface markers involved in immune reactions
.
A) are the result of genetic expression B) function in recognition of self molecules C) receive and transmit chemical messages among other cells of the system D) aid in cellular development E) All of thechoices are correct.
Version 1
4
15)
The major histocompatibility complex is
.
A) glycoproteins, called MHC antigens, found on all body cells B) a set of genes that code for MHC glycoproteins C) found on the third chromosome D) located in the thymus gland E) All of thechoices are correct.
16)
Class II MHC genes code for
.
A) certain secreted complement components B) self receptors recognized by T lymphocytes C) all HLA antigens D) receptors located primarily on macrophages and B cells E) All of thechoices are correct.
17)
Class I MHC genes code for
.
A) certain secreted complement components B) self receptors recognized by T lymphocytes C) all HLA antigens D) receptors located primarily on macrophages and B cells E) All of thechoices are correct.
18)
The histocompatibility complex proteins function in
Version 1
.
5
A) recognition of self B) antibody proliferation C) B-cell maturation D) T-cell maturation E) None of the choices are correct.
19)
MHC molecules are found on each of the following cells except
.
A) leukocytes B) eosinophils C) epithelial cells D) red blood cells E) islet of Langerhans cells
20)
Lymphocytes
.
A) possess MHC antigens for recognizing self B) have membrane receptors that recognize foreign antigens C) gain tolerance to self by destruction of lymphocytes that could react against self D) develop into clones of B and T cells with extreme variations of specificity E) All of thechoices are correct.
21)
The monomer subunit of immunoglobulin molecules has all the following except .
Version 1
6
A) two identical heavy polypeptide chains B) two identical light polypeptide chains C) disulfide bonds between polypeptide chains D) four antigen binding sites E) a variable and constant region on each polypeptide chain
22) The region of each antibody molecule where amino acid composition is highly varied from one clone of B lymphocytes to another is the .
A) variable region B) joining region C) constant region D) light region E) hinge region
23)
Destruction of lymphocytes with self specificity is called
.
A) clonal deletion B) proliferation C) clonal selection D) differentiation E) hypersensitivity
24)
Properties of effective antigens include all the following except
Version 1
.
7
A) are foreign to the immune system B) have molecular complexity C) are large molecules with a minimum molecular weight of 1,000 D) have large polymers made up of repeating subunits E) are cells or large, complex molecules
25) The molecular fragment on an antigen molecule that a lymphocyte recognizes and responds to is called a(n) . A) epitope B) hapten C) antigen binding site D) variable region E) None of the choices are correct.
26) Small foreign molecules that are too small by themselves to elicit an immune response are termed .
A) epitope B) hapten C) antigen binding site D) variable region E) None of the choices are correct.
27)
Superantigens are
Version 1
.
8
A) body tissues that the immune system mistakes as foreign B) cell markers found in some member of a species but not in other members C) bacterial toxins that activate T cells at a 100 times greater rate than other antigens D) those that evoke allergic reactions E) None of the choices are correct.
28)
Antigens that elicit allergic reactions are called
.
A) superantigens B) heterophilic antigens C) allergens D) autoantigens E) None of the choices are correct.
29)
Which of the following is not a property of B cells?
A) Produce plasma cells and memory cells B) Low numbers circulating in the blood C) Require antigen presented with MHC proteins D) Receptors called immunoglobins E) Mature in the bone marrow
30)
Antigen-presenting cells
.
A) include dendritic cells B) include macrophages C) engulf and modify antigen to be more immunogenic D) hold and present processed antigen on their cell membrane surface E) All of thechoices are correct.
Version 1
9
31)
T-cell response to T-cell-dependent antigens requires
.
A) typically a protein antigen B) binding of T-cell to a class II MHC receptor on a macrophage C) binding of T-cell to a site on the antigen D) interleukin-1 activating the T helper cell E) All of thechoices are correct.
32) During presentation of APC-bound antigen, macrophages and dendritic cells secrete the cytokine, , that activates T helper cells.
A) interferon B) interleukin-2 C) interleukin-1 D) histamine E) None of the choices are correct.
33)
Which is incorrect about the Fc region of an immunoglobulin?
A) Is called the crystallizable fragment B) Forms the antigen binding sites C) Contains an effector molecule that can bind to cells such as macrophages and mast cells D) Contains an effector molecule that can fix complement E) Determines the class to which the immunoglobulin belongs
34) Which process involves antibodies covering surface receptors on a virus or toxin molecule thereby disrupting their activity?
Version 1
10
A) Neutralization B) Opsonization C) Complement fixation D) Agglutination E) Anamnestic response
35)
Which process involves antibodies cross-linking cells or particles into large aggregates?
A) Neutralization B) Opsonization C) Complement fixation D) Agglutination E) Anamnestic response
36) Which process involves antibodies coating microorganisms in order to facilitate phagocytosis?
A) Neutralization B) Opsonization C) Complement fixation D) Agglutination E) Anamnestic response
37) Which process involves a more rapid synthesis and greatly increased titer of antibody when the immune system is subsequently exposed to the same antigen?
Version 1
11
A) Neutralization B) Opsonization C) Complement fixation D) Agglutination E) Memory response
38) The immunoglobulin class that has a dimer form found in mucus, saliva, colostrum, and other body secretions is .
A) IgA B) IgD C) IgE D) IgG E) IgM
39)
The immunoglobulin class that is the only one capable of crossing the placenta is .
A) IgA B) IgD C) IgE D) IgG E) IgM
40) The immunoglobulin class that has an Fc region that binds to receptors on basophils and mast cells is .
Version 1
12
A) IgA B) IgD C) IgE D) IgG E) IgM
41)
All of the following are characteristics of IgM except
.
A) has 10 antigen binding sites B) contains a central J chain C) is the first class synthesized by a plasma cell D) can serve as a B-cell receptor E) is a dimer
42)
Which immunoglobulin class(es) can fix complement?
A) IgM only B) IgG only C) IgD only D) IgM and IgG E) IgE and IgA
43)
The immunoglobulins found on the surface of B cells are
.
A) IgM only B) IgG only C) IgD only D) IgM and IgD E) IgD and IgE
Version 1
13
44)
is the most abundant class of antibodies in serum.
A) IgG B) IgM C) IgA D) IgD E) IgE
45) Each fragment of an antibody molecule contains the variable regions of a heavy and light chain that folds into a groove for one epitope.
A) variable B) Fab C) Fc D) terminal E) hinge
An activated T 46) cytotoxic T cells.
H cell produces
, which is a growth factor for T helper cells and
A) interleukin-1 B) interleukin-2 C) interleukin-12 D) antiserum E) complement
47) What process provides many B cells and T cells that are activated against specific antigens?
Version 1
14
A) Antigen expression B) Antibody production C) Clonal expansion D) Antigen presentation E) Opsonization
48)
The most significant cells in graft rejection are
.
A) helper T cells B) suppressor T cells C) cytotoxic T cells D) delayed hypersensitivity T cells E) natural killer (NK) cells
49) Which lymphocytes lack specificity for antigen and are cells that attack cancer cells and virus-infected cells?
A) Helper T cells B) Suppressor T cells C) Cytotoxic T cells D) Delayed hypersensitivity T cells E) Natural killer (NK) cells
50)
Cytotoxic T cells
Version 1
.
15
A) are directly activated by antigens B) lack specificity for antigen C) secrete granzymes and perforins that damage target cells D) secrete interleukin-2 to stimulate B and T cells E) All of thechoices are correct.
51)
Which of the following is not a target for T C cells?
A) Bacteria B) Virus-infected cells C) Cancer cells D) Human transplanted liver E) Pig transplanted heart
52) The benefit of adaptive immunity is the production of protection.
that provide long-lasting
A) antibodies B) plasma cells C) T helper cells D) memory cells E) phagocytoticcells
53)
An example of artificial passive immunity would be
Version 1
.
16
A) chickenpox infection followed by lifelong immunity B) chickenpox vaccine which triggers extended immunity to chickenpox C) giving a person immune serum globulins to chickenpox virus after exposure to the disease D) a fetus acquiring maternal IgG to the chickenpox virus across the placenta E) None of the choices are correct.
54)
An example of natural passive immunity would be
.
A) chickenpox infection followed by lifelong immunity B) chickenpox vaccine which triggers extended immunity to chickenpox C) giving a person immune serum globulins to chickenpox virus after exposure to the disease D) a fetus acquiring maternal IgG to the chickenpox virus across the placenta E) None of the choices are correct.
55)
An example of artificial active immunity would be
.
A) chickenpox infection followed by lifelong immunity B) chickenpox vaccine which triggers extended immunity to chickenpox C) giving a person immune serum globulins to chickenpox virus after exposure to the disease D) a fetus acquiring maternal IgG to the chickenpox virus across the placenta E) None of the choices are correct.
56)
An example of natural active immunity would be
Version 1
.
17
A) chickenpox infection followed by lifelong immunity B) chickenpox vaccine, which triggers extended immunity to chickenpox C) giving a person immune serum globulins to chickenpox virus after exposure to the disease D) a fetus acquiring maternal IgG to the chickenpox virus across the placenta E) None of the choices are correct.
57)
Edward Jenner's work involved
.
A) inoculation of dried pus from smallpox pustules into a person to stimulate immunity B) development of passive immunotherapy C) development of an immunization to protect people against cowpox D) immunization using a related, less pathogenic organism to give protection against a more pathogenic one E) All of thechoices are correct.
58)
Immunotherapy is the
.
A) use of antitoxins B) use of immune serum globulin C) conferring of passive immunity D) administering of preformed antibodies E) All of thechoices are correct.
59)
High titers of specific antibodies are components of
Version 1
.
18
A) specific immune globulin (SIG) B) gamma globulin C) immune serum globulin (ISG) D) attenuated vaccines E) toxoids
60)
Killed or inactivated vaccines are prepared by
.
A) removal of virulence genes from the microbe B) treatment with formalin, heat, or radiation C) passage of the pathogen through unnatural hosts or tissue culture D) long-term subculturing of the microbe E) All of thechoices are correct.
61)
Live, attenuated vaccines
.
A) include the Sabin polio vaccine B) include the measles, mumps, rubella vaccine (MMR) C) contain viable microbes that can multiply in the person D) require smaller doses and fewer boosters compared to inactivated vaccines E) All of thechoices are correct.
62)
Acellular vaccines and subunit vaccines
.
A) contain modified bacterial exotoxin molecules B) are always genetically engineered C) contain select antigenic components of a pathogen rather than whole cells or viruses D) confer passive immunity E) utilize DNA strands that will produce the antigen
Version 1
19
63)
Antitoxins
.
A) contain modified bacterial exotoxin molecules B) are always genetically engineered C) contain select antigenic components of a pathogen rather than whole cells or viruses D) confer passive immunity E) utilize DNA strands that will produce the antigen
64) Which of the following is a special binding substance that enhances immunogenicity and prolongs antigen retention at the injection site?
A) Adjuvant B) Booster C) Antibodies to toxin D) Gamma globulin E) "Trojan horse" recombinant vaccine
65) When an antibody is covering a foreign microorganism, complement may be activated via the pathway.
A) B) C) D)
66)
classical lectin alternative interleukin
All nucleated cells contain
Version 1
.
20
A) class I MHC B) class II MHC C) secretoryantibodies D) IgE receptors E) IgM molecules
67) During which response to the antigen do we display a latent period of no secretory antibody synthesis?
A) Primary B) Secondary C) Tertiary D) Quaternary E) Memory response
68) Which of the statements below represents the correct order in the development of an immune response?
A) Development and clonal deletion; antigen presentation; lymphocyte challenge; lymphocyte response B) Antigen presentation; development and clonal deletion; lymphocyte challenge; lymphocyte response C) Lymphocyte challenge; antigen presentation; development and clonal deletion; lymphocyte response D) Development and clonal deletion; lymphocyte challenge; antigen presentation; lymphocyte response
69) Which of the following statements is false regarding the development of adaptive immunity?
Version 1
21
A) The fourth stage of adaptive immunity involves the production of antibodies by B and T cells against the antigen that initiated the response. B) The second stage in the development of adaptive immunity requires the antigen to be presented to the T cells on the surface of an APC. C) During the third stage in the development of adaptive immunity, B and T cells are challenged by the antigen and respond by differentiating and proliferating. D) The initiation of adaptive immunity involves the development of lymphocytes with the deletion of cells that would react to self molecules.
70)
Which statement is true regarding lymphocyte specificity?
A) Genetic rearrangement in the DNA that codes for antigen receptors results in up to 10 trillion permutations in the variable regions of B- and T lymphocyte-receptors. B) There is a different gene for every possible antigen that a lymphocyte may encounter. C) B lymphocytes have specificity for a large number of different antigens, however T lymphocytes can recognize only 500 antigens corresponding to the number of gene segments that code for the antigen receptors. D) B- and T cell-receptors are identical and both recognize up to 500 antigens corresponding to the gene segments that code for the antigen receptor variable regions.
71)
A naive B lymphocyte has receptors bound to its surface that
.
A) are all specific for the same antigen B) have a variable region that is different to the variable regions of B cell-receptors embedded in other naive B lymphocytes C) is one of up to 10 trillion possible protein configurations due to genetic rearrangement D) will have the same variable region as the antibodies that will be secreted following a challenge by the corresponding antigen E) All of the choices are true statements concerning B lymphocyte-receptors.
Version 1
22
72)
B cell-receptors differ from T cell-receptors in that
.
A) B cell-receptors are comprised of four polypeptides, whereas T cell-receptors are made up of two polypeptides B) B cell-receptors are immunoglobulins similar to those that are secreted, whereas T cell-receptors are not immunoglobulins and are never secreted C) T cell-receptors are much smaller than B cell-receptors D) All of the choices reflect differences between B cell- and T cell-receptors.
73)
B cell-receptors are similar to T cell-receptors in that
.
A) both receptors are similar in size B) both receptors have a variable and a constant region C) both receptors can be secreted in response to antigen D) both receptors are comprised of four polypeptide chains E) All of the choices reflect similarities between B cell- and T cell-receptors.
74)
The importance of clonal deletion is that
.
A) lymphocytes with receptors that recognize self markers are eliminated to preserve immune tolerance B) worn-out lymphocytes are destroyed to make way for new cells C) lymphocytes that have been activated during antigen exposure are then removed when the individual recovers D) clones of cells that have the same receptor are deleted to avoid redundancy
75)
To exhibit high immunogenicity, an antigen would preferably
Version 1
.
23
A) originate in another organism B) be large in size C) have chemical complexity D) have cell surface markers not recognized as self E) All of the choicesare features of a good antigen.
76) In addition to dendritic cells and macrophages, B cells can also act as antigen-presenting cells. This occurs when .
A) the antigen is a molecule with a simple, repeating structure and evokes an immune response as a T-cell-independent antigen B) the B cell engulfs the antigen and presents it to the T helper cell C) the B cell engulfs the antigen and presents it to the T cytotoxic cell D) the antigen binds to the B cell and after processing, it is presented to the dendritic cells and macrophages
77)
Gamma-delta T cells
.
A) are considered a bridge between innate and adaptive immune responses B) have T cell-receptors that target a wide range of antigens C) can bind to, and become activated by PAMPs D) differentiate into memory cells E) All of the choices are true.
78)
Gamma-delta T cells differ from other T cells in that
Version 1
.
24
A) they can be directly activated by PAMPs, whereas other T cells have to be presented with the antigen in conjunction with an MHC molecule on an antigen-presenting cell B) they do not have T cell-receptors to bind to antigens C) they do not produce memory cells, unlike the T helper and T cytotoxic cells D) they can react against cancer cells, whereas the other T cells can only target bacteria and viral-infected cells
79)
Herd immunity involves
.
A) maintaining a certain level of immunized individuals in a society to prevent a pathogen from circulating through the population B) keeping bovine diseases to a minimum so they do not spread to the human population C) encouraging individuals to work and live in close proximity to each other to prevent spread of a disease outside of a population D) maintaining a 100% vaccination rate among farm workers to prevent the spread of zoonotic diseases
80)
What are the risks to society when herd immunity is not maintained?
A) Decreasing antibody titers in individuals require booster shots which restricts the number of available vaccines. B) Cattle become increasingly more susceptible to dangerous pathogens, which can spread to humans. C) Dangerous pathogens can reemerge and outbreaks of diseases may develop. D) There are no risks to society when herd immunity is not maintained; it is a precautionary measure only.
81)
Choose the phrase that describes MHC class I molecules.
Version 1
25
A) B) C) D)
82)
Found on only sometypes of white blood cells Expressed on allhuman nucleated cells Are proteinsinvolved in the complement system Are secreted fromall human nucleated cells
The first stage of developing an adaptive immune response involves
A) B) C) D)
.
the presentation of antigens lymphocytedevelopment and differentiation the challenge of B or T cells by antigens the production ofantibodies
83) Substances known as provoke a specific immune response so discriminating that only a single molecular fragment, called an , actually interacts with the lymphocyte’s receptor. A) B) C) D)
84) cell.
haptens; epitope antigens; epitope epitopes; antigen antigens; immunogen
Choose the statement that describes the method of Tc cell-mediated killing of a target
A) B) C) D)
Version 1
Opsonization Neutralization Agglutination Apoptosis
26
85)
Choose the statement that best describes the primary action of B cells.
A) B cells interact with antigen on antigen-presenting cells B) B cells produce interleukin-2, a lymphocyte growth factor C) B cells divide and differentiate into plasma cells that produce antibody and memory cells. D) B cells become active in cell-mediated immunity
86) When a large percentage of a population is vaccinated against an infection, the causative microbe is unable to spread easily through that population. Because of this, susceptible individuals who cannot be vaccinated are protected to some extent by the community around them. This phenomenon is known as . A) B) C) D)
herd immunity passive immunization subunit vaccination adjuvant protection
87) NCLEX Prep - Test Bank Question: Please read the clinical scenario, and then answer the questions that follow to become familiar with the traditional NCLEX question format. An RN working in a general medical-surgical unit is caring for a 57-year-old female with a bladder infection. Two weeks prior to hospitalization, the patient was experiencing slight burning upon urination. She self-treated by increasing her intake of fluids, but her symptoms continued to worsen. One week later, she developed generalized abdominal pain and a fever. She presented to the hospital when the pain became unbearable and fever reached 104°F. She was admitted to the medical-surgical unit and started on multiple IV antibiotics with concern that she could be developing a kidney infection.
87.1)
Version 1
Which line(s) of defense has/have been breached in this patient’s case?
27
A) Only the first line of defense has been breached. Her second line of defense is fighting off this infection. B) No lines of defense have been breached. Her intact barrier system is keeping her kidneys from becoming infected. C) The second line of defense has been breached. Her lymphocytes are now being challenged by this infection. D) There are no lines of defense for this sort of infection.
87.2) Knowing that the causative organism for this patient’s urinary tract infection, E. coli, has a lipopolysaccharide cell wall, which cells are being activated by this infection? A) B) C) D)
87.3)
T helper cells T cytotoxiccells Natural killer cells B cells
What is the proper order of steps for B-cell activation?
A) Antigen binding, antigen processing/presentation, T helper cell cooperation/recognition, activation, differentiation, clonal expansion B) Antigen processing/presentation, activation, differentiation, T helper cell cooperation/recognition, antigen binding, clonal expansion C) Activation, clonal expansion, differentiation, antigen processing/presentation, T helper cell cooperation/recognition, antigen binding D) Antigen binding, T helper cell cooperation/recognition, antigen processing/presentation, differentiation, activation, clonal expansion
87.4)
Version 1
Which part of the IgG molecule will bind with the E. coli antigen?
28
A) B) C) D)
Fab Fc Complement binding site Cellular binding site
88) NCLEX Prep - Test Bank Question: Please read the clinical scenario, and then answer the questions that follow to become familiar with the traditional NCLEX question format. A 6-year-old male is brought to his pediatrician’s office with chickenpox. He has had symptoms for one week, and his mother is concerned about the patient’s 8-year-old sister because she had a long, painful chickenpox infection 2 years prior. Of note, the patient was not vaccinated against chickenpox because his parents did not think it was a serious enough virus to warrant the pain of the injection.
88.1) Which of the following explanations correctly represents how this patient could have been protected from chickenpox infection?
A) He would not have been infected if he were breastfed as a baby because he would have passive natural immunity. B) He would not have been infected if his mother had gamma globulin administered during pregnancy because he would have passive artificial immunity. C) He could have been protected from infection if he received active artificial immunity through the form of a vaccine. D) He may have a primary immunodeficiency disease because healthy humans are not typically infected with chickenpox.
88.2) When the patient’s mother expresses concern about his sister being reinfected with chickenpox, what is the most appropriate response?
Version 1
29
A) Your concerns are valid, but his sister developed an active natural immunity to chickenpox because she has already been infected. B) Your concerns are valid and there is a high likelihood that his sister will become infected again. We should probably administer the vaccine. C) Your concerns are valid and there is no way to protect his sister from becoming reinfected except to keep her at least 15 feet from your son at all times. D) Your concerns are valid and if his sister is reinfected, her symptoms will be less severe than her initial chickenpox infection.
88.3) Which form of adaptive immunity is the patient developing by having this infection?
A) B) C) D)
Active natural immunity Passive natural immunity Active artificial immunity Passive artificial immunity
88.4) If the patient’s mother had previously vaccinated her son against chickenpox, what form of immunity would he have had against the virus?
A) B) C) D)
Active natural immunity Passive natural immunity Active artificial immunity Passive artificial immunity
88.5) After his recovery from chickenpox, which lymphocytes will keep this patient from becoming infected again?
Version 1
30
A) B) C) D)
T helper cells Natural killer cells T memory cells B cells
89) NCLEX Prep - Test Bank Question: Please read the clinical scenario, and then answer the questions that follow to become familiar with the traditional NCLEX question format. A 2-week-old infant is brought to a family practice office for a scheduled visit. The parents appear anxious, as this is their first child. They express multiple concerns to the nurse about their son’s sleep patterns, feeding, and crying. The RN provides empathetic listening and education to calm their fears, and they are receptive to the teaching. When the RN discusses vaccination with the parents, they refuse to vaccinate their son because they have heard vaccines can cause autism in children.
89.1)
A) B) C) D)
Which of the following statements is not true concerning vaccines?
They consist ofkilled or inactivated cells. They consist oflive but weakened microbes. It is possible tobe allergic to vaccine preparations. Autism onlyhappens in rare instances of vaccination.
89.2) The parents are concerned when they hear that vaccines consist of whole microbes, even if they have been inactivated. They state that they do not want whole bacteria being put into their son. What is the most appropriate response by the nurse?
Version 1
31
A) B) C) D)
89.3)
A) B) C) D)
The bacteria is inactive, so there’s no risk of it harming your son. The chances of the bacteria infecting your son are very slim. Currently available vaccines have do not contain whole bacteria. Vaccines are refrigerated and the cold inhibits mutations.
What kind of immunity will be conferred to the infant through vaccination?
Active natural immunity Passive natural immunity Passive artificial immunity Active artificial immunity
89.4) Following education from the RN, the parents decide to have their son vaccinated. Which of the following is the most important part of the family history to elicit?
A) B) C) D)
Incidence of autism Food or drug allergies Dairy sensitivity Acute illness during pregnancy
89.5) The morning after the vaccination, the patient’s mother calls to report that the infant has developed a low-grade fever. She states, “I am afraid that he is developing meningitis from one of the vaccines, because he is also fussy.” What is a proper response by the nurse?
Version 1
32
A) I understand you are frightened. However, site pain and low fever are common side effects. You can help minimize both with acetaminophen, as the physician recommended. B) Bring your son to the closest ER for evaluation. He may be experiencing a lifethreatening reaction to the vaccines. C) You sound upset. Please remember how harmless I told you vaccines were. D) This fever and fussiness are probably from an unrelated illness. It is just a coincidence he received vaccinations around the same time as these symptoms presented.
90)
Antibody molecules circulate in lymph, blood, and tissue fluids. ⊚ ⊚
91)
true false
Human B lymphocytes mature in an intestinal region called the bursa. ⊚ ⊚
true false
92) Activation of B cells occurs when antigen binds to B-cell surface immunoglobulin receptors. ⊚ ⊚
93)
true false
Clonal selection can be stimulated by foreign or self antigens. ⊚ ⊚
Version 1
true false
33
94) Antibodies directed to alloantigen from one organism often cross-react with an alloantigen from another organism. ⊚ ⊚
true false
95) After secreting antibodies during an immune response, plasma cells then differentiate into memory cells. ⊚ ⊚
true false
96) The hinge region of an antibody has a hypervariable amino acid region where the antigenic determinant fits. ⊚ ⊚
true false
97) The structural and functional differences that distinguish immunoglobulin isotypes are due to variations associated with their Fc fragments. ⊚ ⊚
true false
98) One plasma cell will secrete antibodies of various classes but the antibodies will all have the same specificity. ⊚ ⊚
99)
true false
The secondary response to an antigen is faster and bigger than the primary response.
Version 1
34
⊚ ⊚
100)
true false
Gamma globulin can be given as immunotherapy to confer artificial passive immunity. ⊚ ⊚
true false
101) The third line of defense can be summarized in four stages; I—antigen binding to T helper cells, II—antigen binding to B cells, III—phagocytosis of antigen by B and T lymphocytes, IV—killing of antigen. ⊚ ⊚
true false
102) The diversity and specificity of the lymphocyte repertoire against any potential antigen it may encounter is due to the rearrangement of gene segments that code for the antigen receptors. ⊚ ⊚
true false
103) To be an effective antigen, a molecule would ideally be small, and consist of repeating units with similar structure, such as a trisaccharide. ⊚ ⊚
true false
104) Vaccinating a child against a dangerous pathogen not only protects that child from disease, but it also protects society as a whole, since herd immunity in the population is maintained.
Version 1
35
⊚ ⊚
true false
105) B and T cells can only recognize and bind antigen that has been processed and presented on MHC molecules. ⊚ true ⊚ false
106)
There are only two major subsets of T cells. ⊚ ⊚
true false
107) Memory B cells are formed during the primary response so that upon a secondary antigen exposure, a faster and more vigorous antibody response ensues. ⊚ ⊚
true false
108) When receiving a DNA vaccine, human cells accept a plasmid-containing pathogen DNA, and subsequently express foreign proteins on their cell surface that stimulate an inflammatory response. ⊚ ⊚
Version 1
true false
36
Answer Key Test name: Ch 13 4e 1) [A, B] 2) [A, C] 3) [A, C] 4) [A, B, C] 5) [A, B, D, E] 6) [A, E] 7) E 8) B 9) D 10) E 11) A 12) E 13) C 14) E 15) B 16) D 17) B 18) A 19) D 20) E 21) D 22) A 23) A 24) D 25) A 26) B Version 1
37
27) C 28) C 29) C 30) E 31) E 32) C 33) B 34) A 35) D 36) B 37) E 38) A 39) D 40) C 41) E 42) D 43) D 44) A 45) B 46) B 47) C 48) C 49) E 50) C 51) A 52) D 53) C 54) D 55) B 56) A Version 1
38
57) D 58) E 59) A 60) B 61) E 62) C 63) D 64) A 65) A 66) A 67) A 68) A 69) A 70) A 71) E 72) D 73) B 74) A 75) E 76) A 77) E 78) A 79) A 80) C 81) B 82) B 83) B 84) D 85) C 86) A Version 1
39
87) Section Break 87.1) C 87.2) D 87.3) A 87.4) A 88) Section Break 88.1) C 88.2) A 88.3) A 88.4) C 88.5) C 89) Section Break 89.1) D 89.2) C 89.3) D 89.4) B 89.5) A 90) TRUE 91) FALSE 92) TRUE 93) TRUE 94) TRUE 95) FALSE 96) FALSE 97) TRUE 98) TRUE 99) TRUE 100) TRUE 101) FALSE 102) TRUE Version 1
40
103) FALSE 104) TRUE 105) FALSE 106) FALSE 107) TRUE 108) TRUE
Version 1
41
CHAPTER 14 1) Select all of the conditions that result from IgE- and mast-cell-mediated allergic reactions.
A) Hay fever B) Allergic asthma C) Food allergy reactions D) Drug allergy reactions E) Eczema F) Graft rejection of host
2)
Which of the following correctly describe systemic anaphylaxis? Check all that apply. A) It involves sudden respiratory and circulatory issues B) It involves a wheal-and-flare reaction C) It can be fatal very quickly D) Concentration of chemical mediators is very high in response E) Concentration of chemical mediators is very low in response
3) Select all of the correct statements concerning in vitro methods for the diagnosis of allergies.
A) Elevated blood levels of tryptase, an enzyme released by mast cells B) Differential blood cell count for basophil and eosinophil levels C) Leukocyte histamine release test D) RAST assays to measure blood IgE concentrations E) Injection of skin with allergen extracts
4) Select all statements that apply to hemolytic disease of the newborn to test your understanding of this disorder.
Version 1
1
A) Occurs due to Rh incompatibility between an Rh+ mom and Rh- baby B) Fetal RBC leakage sensitizes the mother to make anti-Rh antibodies C) Anti-Rh antibodies that are IgG can cross the placenta and induce complementmediated lysis of fetal RBCs D) After sensitization with Rh+ fetal RBCs, all subsequent pregnancies (Rh+ and Rhfetuses) are at risk for hemolytic disease E) Hemolytic disease is a Type II hypersensitivity reaction
5) Select all of the statements that apply to Type III hypersensitivity reactions to test your understanding of these reactions.
A) Reaction involves IgG, IgM, and IgA B) Antibody binds soluble antigen C) Antibody binds to cell surface antigen D) Damage is due to complement-mediated lysis E) Damage is due to inflammation caused by neutrophil granule release
6) Select all of the statements that apply to both Type II and Type III hypersensitivity reactions.
A) Involve IgG antibodies B) Involve IgM antibodies C) Involve IgA antibodies D) Involve the activation of complement E) Involve the association of antibodies with cell-surface-associated antigens F) Involve the production of free-floating antibody-antigen complexes
7) Select all of the correct statements concerning SLE to test your understanding of the pathogenesis of autoimmune diseases.
Version 1
2
A) Autoantibodies bind self antigens forming immune complexes that accumulate in basement membranes of various organs. B) SLE represents a type II hypersensitivity reaction. C) Autoantibodies include those against white and red blood cells, and platelets. D) SLE results in an autoimmune disease that targets the thyroid. E) SLE results in muscle weakness as its principle symptom.
8) Select all of the correct statements concerning SCIDs to test your understanding of severe combined immunodeficiency disease. A) Most serious and lethal of the primary immunodeficiency diseases, caused by genetic defects B) Involves loss or dysfunction of only the B-cell response C) Involves loss or dysfunction of both humoral and cell-mediated immunity D) Easily treated with gene replacement therapy E) Requires vigorous adherence to aseptic techniques to prevent infection
9) The major category(ies) of hypersensitivity that typically involve a B-cell immunoglobulin response is (are) .
A) type I only B) type I and type IV C) type IV only D) type I, type II, and type III E) type I, type II, type III, and type IV
10)
Which is mismatched?
Version 1
3
A) Food allergy - type I hypersensitivity B) Poison ivy dermatitis - type IV hypersensitivity C) Serum sickness - type III hypersensitivity D) Transfusion reaction - type II hypersensitivity E) Hay fever - type IV hypersensitivity
11) a(n)
Any heightened or inappropriate immune response resulting in tissue damage is called . A) autoimmune disease B) immunodeficiency C) hypersensitivity D) transfusion reaction E) desensitization
12)
The study of diseases associated with excesses and deficiencies of the immune system is .
A) humoralpathology B) hemopathology C) epidemiology D) immunopathology E) histopathology
13)
Atopy and anaphylaxis are hypersensitivities in the category
Version 1
.
4
A) type I only B) type I and type IV C) type IV only D) type I, type II, and type III E) type I, type II, type III, and type IV
14)
Bee sting venom is considered to be which type of allergen?
A) Ingestant B) Inhalant C) Injectant D) Contactant E) None of the choices are correct.
15)
Fungal spores and animal dander are considered to be which type of allergen?
A) Ingestant B) Inhalant C) Injectant D) Contactant E) None of the choices are correct.
16)
The initial encounter with an allergen is called the
.
A) sensitizing dose B) provocative dose C) allergic dose D) hypersensitivity dose E) desensitizing dose
Version 1
5
17)
A second encounter with an allergen that causes a response is called the
.
A) sensitizing dose B) provocative dose C) allergic dose D) hypersensitivity dose E) desensitizing dose
18) What will be the immediate action of an allergen when it enters that body for a second time?
A) Degranulation B) Bonding of allergen to adjacent IgE binding sites on mast cells and basophils C) Binding of IgE by the Fc region to mast cells and basophils D) Histamine acts on smooth muscle E) Prostaglandins cause vasodilation and increased vascular permeability
19)
Which event is the process of releasing chemical mediators?
A) Degranulation B) Bonding of allergen to adjacent IgE binding sites on mast cells and basophils C) Binding of IgE by the Fc region to mast cells and basophils D) Histamine acts on smooth muscle E) Prostaglandins cause vasodilation and increased vascular permeability
20)
Which event occurs with the sensitizing dose of allergen?
Version 1
6
A) Degranulation B) Bonding of allergen to adjacent IgE binding sites on mast cells and basophils C) Binding of IgE by the Fc region to mast cells and basophils D) Histamine acts on smooth muscle E) Prostaglandins cause vasodilation and increased vascular permeability
21)
Which of the following is not a possible symptom of type I hypersensitivity?
A) Rhinitis B) Rashes C) Sneezing D) Diarrhea E) Contact dermatitis
22)
Histamine causes all of the following except
.
A) increased sensitivity to pain B) constriction of smooth muscle of bronchi and the intestine C) relaxes vascular smooth muscle D) wheal and flare reaction in skin E) pruritus and headache
23) The chemical mediator that causes prolonged bronchospasm, vascular permeability, and mucus secretion of asthmatic patients is .
Version 1
7
A) prostaglandin B) histamine C) leukotriene D) serotonin E) platelet-activating factor
24)
Which type(s) of hypersensitivity is IgG involved with? A) Anaphylaxis B) Antibody mediated C) Immune complex mediated D) Both anaphylaxis and antibody mediated E) Both antibody mediated and immune complex mediated
25)
Allergic reactions to penicillins are considered a(n)
hypersensitivity.
A) T-cell mediated B) antibody mediated C) immune complex mediated D) immediate E) both T-cell mediated and antibody mediated
26)
Allergies run in families because
.
A) immunoglobulins pass from mother to fetus B) immunoglobulins pass through breast milk C) the variable region of antibodies is genetically determined D) relative production of IgE is inherited E) All of thechoices are correct.
Version 1
8
27)
A chronic, local allergy such as hay fever is considered
.
A) delayed B) T-cell mediated C) atopic D) antibody mediated E) systemic anaphylactic
28)
A systemic, sometimes fatal reaction with airway obstruction and circulatory collapse is .
A) delayed B) T-cell mediated C) atopic D) antibody mediated E) systemic anaphylactic
29)
A seasonal reaction to inhaled allergens is
.
A) atopic dermatitis B) eczema C) allergic rhinitis D) asthma E) anaphylaxis
30)
All of the following are associated with IgE and mast cell-mediated allergy except .
Version 1
9
A) drug allergy B) eczema C) anaphylaxis D) allergic asthma E) systemic lupus erythematosus
31)
Epinephrine
.
A) is an antihistamine B) reverses constriction of airways C) causes desensitization D) inhibits the activity of lymphocytes E) All of thechoices are correct.
32) Allergic patients receiving small, controlled injections of specific allergens are undergoing .
A) desensitization B) sensitization C) tissue matching D) degranulation E) None of thechoices are correct.
33)
An antihistamine will
Version 1
.
10
A) inhibit the activity of lymphocytes B) bind to histamine receptors on target organs C) block synthesis of leukotrienes D) relieve inflammatory symptoms E) reverse spasms of respiratory smooth muscles
34)
Corticosteroids will
.
A) inhibit the activity of lymphocytes B) bind to histamine receptors on target organs C) block synthesis of leukotrienes D) relieve inflammatory symptoms E) reverse spasms of respiratory smooth muscles
35)
All of the following are involved in type 2 hypersensitivity except
.
A) IgM B) IgG C) IgE D) complement E) foreign cells
36)
Human blood types involve all the following except
.
A) MHC genes B) ABO antigen markers C) inheritance of two of three possible alleles D) genetically determined glycoprotein markers E) genes that code for an enzyme that adds a terminal carbohydrate to RBC receptors
Version 1
11
37)
Transfusion of the wrong blood type can cause
.
A) recipient antibody activating the complement cascade to attack the RBCs B) fever and anemia C) systemic shock and kidney failure D) massive hemolysis of the donor RBCs E) All of thechoices are correct.
38)
A female who is Rh
–
.
A) inherited two dominant genes B) is in the majority of the population with regard to Rh status C) is at risk for a pregnancy resulting in hemolytic disease of the newborn D) can never have an Rh+ baby E) All of thechoices are correct.
39)
The potential for hemolytic disease of the newborn occurs when
.
A) maternal Rh + cells enter an Rh -fetus B) fetal Rh + cells enter an Rh - mother C) maternal Rh ˗ cells enter an Rh + fetus D) fetal Rh ˗ cells enter an Rh + mother E) fetal Rh + cells enter an Rh + mother
40)
Once a mother has been sensitized to the Rh factor
Version 1
.
12
A) all other Rh + fetuses are at risk B) she can be given RhoGAM in future pregnancies to prevent hemolytic disease of the newborn C) she can never again have a low risk pregnancy D) only future Rh ˗ fetuses are at risk E) None of the choices are correct.
41)
Which of the following is not true of type III hypersensitivity?
A) Antigen-antibody complexes are deposited in the basement membrane of epithelial tissues B) Involves production of IgG and IgE antibodies C) Involves an immune complex reaction D) The Arthus reaction is a local response E) Serum sickness is a systemic response
42) Which of the following is not a major organ that can be a target of immune complex deposition?
A) Blood vessels and skin B) Heart and lungs C) Brain D) Joints E) Kidneys
43) Large quantities of antibodies that react to the second entry of antigen and lead to formation of antigen-antibody complexes occurs in .
Version 1
13
A) serum sickness B) delayed hypersensitivity C) anaphylaxis D) hemolytic disease of the newborn E) All of thechoices are correct.
44)
Contact dermatitis involves
.
A) a sensitizing and provocative dose B) an allergen entering the skin C) T lymphocytes secreting inflammatory cytokines D) itchy papules and blisters E) All of thechoices are correct.
45) What could result when grafted tissue such as bone marrow contains passenger lymphocytes?
A) Host rejection of graft B) Graft versus host disease C) Formation of autoantibodies D) Hypogammaglobulinemia E) None of the choices are correct.
46) Tissue transplanted from one body site on a patient to a different body site on that patient is called a(n) .
Version 1
14
A) isograft B) autograft C) allograft D) xenograft E) hypograft
47) What involves determination of donor HLA antigens compared to those of the recipient's tissue?
A) Skin graft B) Blood transfusion C) Organ transplantation D) Both skin graft and organ transplantation E) All of thechoices are correct.
48)
Autoimmunity is typically due to
.
A) a transfusion reaction B) IgE and mast cells C) autoantibodies and T cells D) graft rejection E) a deficiency in T-cell development
49)
Autoantibodies cause tissue injury in all of the following diseases except
Version 1
.
15
A) rheumatoid arthritis B) myasthenia gravis C) Graves' disease D) tuberculin reaction E) multiple sclerosis
50)
Each of the following is an autoimmune disease except
.
A) systemic lupus erythematosus B) Graves' disease C) type I diabetes D) metastatic cancer E) rheumatic fever
51)
Myasthenia gravis disease arises from the production of autoantibodies against .
A) myelin sheath cells of the nervous system B) acetylcholine receptors on smooth muscle C) acetylcholine receptors on skeletal muscle D) sodium pump proteins in the cell membrane E) cells in thyroid follicles
52)
In multiple sclerosis, autoantibodies attack
Version 1
.
16
A) myelin sheath cells of the nervous system B) acetylcholine receptors on smooth muscle C) acetylcholine receptors on skeletal muscle D) sodium pump proteins in the cell membrane E) cells in thyroid follicles
53)
Which of the following is not a theory to explain the origin of autoimmune diseases?
A) Sequestered antigen theory B) Clonal selection theory C) Theory of immune deficiency D) Viral infection theory E) All of thechoices are possible theories.
54)
What can be a consequence of a genetic deficiency in B-cell survival and maturity? A) Host rejection of graft B) Graft versus host disease C) Formation of autoantibodies D) Hypogammaglobulinemia E) None of the choices are correct.
55)
The DiGeorge syndrome is the result of
.
A) autoantibodies B) delayed hypersensitivity C) congenital absence or immaturity of the thymus gland D) failure of B-cell development and maturity E) a genetic defect in the development of both T cells and B cells
Version 1
17
56)
Severe combined immunodeficiencies (SCIDs) are due to
.
A) autoantibodies B) delayed hypersensitivity C) congenital absence or immaturity of the thymus gland D) failure of B-cell development and maturity E) a genetic defect in the development of both T cells and B cells
57)
A secondary acquired immunodeficiency is
.
A) AIDS B) adenosine deaminase (ADA) deficiency C) DiGeorge syndrome D) agammaglobulinemia E) SCID
58) In the theory for allergic desensitization, which immunoglobulin blocks the allergen from binding with IgE?
A) IgE B) IgG C) IgA D) IgD E) IgM
59)
Degranulation of mast cells leads to
Version 1
.
18
A) B) C) D)
airway obstruction headache dilated blood vessels All of the choices are correct.
60) A well-functioning immune system should strike a balance between being over- and underreactive. An overreactive immune system results in hypersensitivities such as allergy and , while an underreactive immune system is due to .
A) autoimmunity; immunodeficiency B) immunodeficiency; anaphylaxis C) hyposensitivity; atopy D) inflammation; asthma E) hyposensitivity; autoimmunity
61) A 12-year-old girl received a pair of earrings from her friend for her birthday. The second time she wore them, her earlobes became red, itchy and inflamed, with blisters appearing within 48 hours. What type of hypersensitivity is causing the discomfort?
A) B) C) D)
62)
Type I hypersensitivity Type II hypersensitivity Type III hypersensitivity Type IV hypersensitivity
Type II hypersensitivities differ from type III in that
Version 1
.
19
A) type II hypersensitivity involves the activation of complement proteins that attack and lyse cells, whereas type III involves the formation of immune complexes causing chronic inflammation B) type II hypersensitivity involves the formation of immune complexes causing chronic inflammation, whereas type III involves the activation of complement proteins that attack and lyse cells C) type II hypersensitivity involves the formation of IgM antibodies that attack tissues, whereas type III involves IgE causing degranulation of mast cells D) type II hypersensitivity involves B cells whereas type III involves the activation and sensitization of cytotoxic T cells
63) Desensitization of an individual with allergies by injecting small amounts of the allergen under the skin is effective because it .
A) induces the formation of IgG antibodies that bind the the allergen before it is able to bind to the IgE associated with mast cells B) prevents IgE antibodies from being formed, thus eliminating the degranulation of mast cells C) changes the nature of the allergy from a type I to a type III, which is easier to treat D) induces the formation of IgA antibodies, which are secreted into the tissues, reducing the amount of degranulation by mast cells in the blood
64) The study of disease states (immune dysfunctions) associated with overreactivity or underreactivity of the immune response is known as . A) B) C) D)
Version 1
immunopathology hypersensitivity autoimmunity immunodeficiency
20
65) Immunopathology is the study of disease states or immune dysfunctions, which are divided into two categories: diseases (i.e., allergy) or diseases associated with (i.e., AIDS). A) B) C) D)
hypersensitivity; immunodeficiency hyposensitivity; hypersensitivity hyposensitivity; immunocompetence immunocompetence; hypersensitivity
66) Choose the statement that doesNOT describe the anaphylactic response to test your understanding of anaphylaxis. A) Sudden respiratory and circulatory disruption can be fatal in mere minutes. B) Allergen route of entry is always due to inhalation. C) It is associated with greatly amplified concentration of chemical mediators. D) Allergens do not act directly on the target organ. E) All statements describe the anaphylactic response.
67) Type hypersensitivity results in the lysis of foreign cells due to the actions of complement and antibodies (IgG and IgM), often resulting in transfusion reactions. A) B) C) D)
I II III IV
68) Which type of graft rejection is causedbylymphocytes in grafted tissues causing systemic effects as they travel throughoutthe recipient's body? A) Host versus graft rejection B) Graft versus host disease C) Xenograft rejection
Version 1
21
69) Immunodeficiency diseases are present at birth and usually stem from genetic errors are called immunodeficiencies, whereas immunodeficiency diseases are acquired after birth and are due to agents such as infections, irradiation, or steroids. A) B) C) D)
primary; secondary secondary; primary true; secondary acquired; innate
70) NCLEX Prep - Test Bank Question: Please read the clinical scenario, and then answer the questions that follow to become familiar with the traditional NCLEX question format. A 9-year-old male presents to the emergency room complaining of “chest tightness” and difficulty breathing. The RN notes that he is frequently coughing and wheezes are ausculated in all lung fields. His chest x-ray shows a flattened diaphragm and overexpanded lungs. The patient’s mother reports that he had five episodes like this one over the past few years, but none this bad. Recently, the patient has had more frequent episodes of difficulty breathing and coughing, each lasting for a few hours. He does not take any medications at home.
70.1) Based on the patient’s symptoms, which of the following is the most likely diagnosis?
A) B) C) D)
Asthma Hay fever Seasonal allergy Eczema
70.2) Which of the following conditions would be most pertinent for the nurse to ask about while collecting the patient’s past medical history?
Version 1
22
A) B) C) D)
Hay fever Premature birth Eczema Blood transfusion reaction
70.3) Following a pulmonology consultation, the patient is diagnosed with asthma. Which of the following allergic mediators is responsible for the patient’s reactive airway symptoms?
A) B) C) D)
70.4)
A) B) C) D)
70.5) an
Version 1
Bradykinin Leukotrienes Serotonin All of the choicesare correct.
Asthma is classified as what type of reaction?
Atopy Systemic anaphylaxis Acute allergic Cutaneous anaphylaxis
The RN most accurately describes the pathophysiology of the patients asthma as .
23
A) IgE-mediated response with lung tissue that is hypersensitive to products of mast cell degranulation B) IgE response that causes the body to not recognize its own lung tissue and fight against it by constricting bronchioles C) IgG-mediated response with lung tissue that is hypersensitive to products of mast cell degranulation D) IgG response that causes the body to not recognize its own lung tissue and fight against it by constricting bronchioles
71) NCLEX Prep - Test Bank Question: Please read the clinical scenario, and then answer the questions that follow to become familiar with the traditional NCLEX question format. A 28-year-old woman is 30 weeks pregnant with her second child and is being seen for a scheduled obstetric visit. Her first pregnancy and delivery was uncomplicated, and she has a healthy 2-year-old girl. It is known that the mother is Rh-, and this child could be at risk for hemolytic disease of the newborn. She received RhoGAM with her first pregnancy.
71.1)
A) B) C) D)
Which factor would increase the risk of the unborn child being Rh+? Previous child being Rh Father being Rh+ Maternal history of preexisting type I hypersensitivity Both maternal grandparents being Rh+
71.2)
What intervention would the RN expect during this visit?
A) B) C) D)
Fetal blood type testing to determine if RhoGAM is indicated Administration of first dose of RhoGAM Nothing, since it is not known if the fetus is Rh+ Plan for first dose of RhoGAM immediately after birth
Version 1
24
71.3) After further investigation, it is learned that the father is Rh+, and so was the first child. The patient declines the RhoGAM injection, stating “there were no complications with my first pregnancy and delivery, how can you be sure there will be with this one?” She explains how unpleasant the RhoGAM injection was with her first pregnancy, and she would like to avoid it, if at all possible. What is the most appropriate response by the nurse?
A) Because you received RhoGAM with your first pregnancy, it is not necessary during this pregnancy. B) It is important you receive your RhoGAM injection because this child will probably develop hemolytic disease of the newborn if you do not. C) It is important that we keep you from being sensitized to Rh factor, otherwise future pregnancies will be at risk for developing hemolytic disease of the newborn. D) You were probably sensitized to Rh factor with your first pregnancy, so the chances are decreased that this child will develop hemolytic disease of the newborn. I will ask the doctor if we can forego the injection.
71.4)
A) B) C) D)
What type of hypersensitivity reaction is the fetus at risk for upon delivery?
Type I Type II Type III Type IV
71.5) The patient asks the nurse to explain what an Rh reaction involves with her baby. What is the best explanation?
Version 1
25
A) Your baby’s Rh+ antibodies cross into your system, attacking your red blood cells and making you unable to deliver a healthy baby. B) Your baby has a type II hypersensitivity that will render its liver unable to clear bilirubin released from lysed cells. C) Your Rh antibodies will cross into your baby’s fetal circulation, attacking and lysing his red blood cells. D) As soon as your baby is born, if you do not receive RhoGAM, the baby’s body will reject not having Rh antibody anymore.
72) NCLEX Prep - Test Bank Question: Please read the clinical scenario, and then answer the questions that follow to become familiar with the traditional NCLEX question format. A 46-year-old Caucasian male presented to the hospital in fulminant liver failure secondary to prolonged alcohol abuse. He received a liver transplantation two days ago from an unrelated donor. His post-operative course has been uncomplicated thus far.
72.1)
A) B) C) D)
What type of transplant did the patient receive?
Isograft Autograft Allograft Xenograft
72.2) The patient’s clinical status and lab values will be closely monitored for two weeks following his transplant. If he experiences graft rejection, what type of cells will be primarily responsible?
Version 1
26
A) B) C) D)
72.3) for?
A) B) C) D)
T memorycells Natural killer cells T cytotoxiccells T helpercells
What type of hypersensitivity reaction is an organ transplantation patient at risk
Type I Type II Type III Type IV
72.4) The patient discusses his antirejection medication regiment with the nurse. He asks how they work to prevent graft rejection. What is the best explanation by the RN?
A) Antirejection medications work on your liver to make it less hostile to its new environment, increasing the chances of organ acceptance. B) Antirejection medications work by changing the MHC markers on your new liver to be more like your own. C) Antirejection medications work by suppressing the body’s immune response against foreign MHC markers. D) Antirejection medications work by causing your body to think its MHC markers are similar to your new liver’s.
73)
Systemic anaphylaxis can quickly result in airway blockage, shock, and death. ⊚ ⊚
Version 1
true false
27
74)
Food allergies include gastrointestinal symptoms and often hives. ⊚ ⊚
75)
Allergic rhinitis is also known as asthma. ⊚ ⊚
76)
true false
true false
A person who is Rh˗ will have anti-Rh antibodies in their serum from early infancy. ⊚ true ⊚ false
77) The tuberculin reaction develops within 30 minutes of the skin test in people with prior sensitization due to tuberculosis infection. ⊚ ⊚
true false
78) During graft rejection, cytotoxic T cells of the recipient recognize and respond to foreign class I MHC receptors on the grafted cells. ⊚ ⊚
true false
79) A xenograft is graft tissue from a donor of one species transplanted to a recipient of another species.
Version 1
28
⊚ ⊚
80)
Eczema is an autoimmune disorder. ⊚ ⊚
81)
true false
true false
A viral infection can cause type I diabetes mellitus. ⊚ ⊚
true false
82) AIDS is a secondary immunodeficiency disease that affects several types of immune cells. ⊚ ⊚
true false
83) Immunopathology is the disease state associated with the overreactivity or underreactivity of the immune response. ⊚ ⊚
true false
84) Secondary acquired immunodeficiencies of B cells and T cells can be caused by infections, organic disease, chemotherapy, or radiation. ⊚ ⊚
Version 1
true false
29
Version 1
30
Answer Key Test name: Ch 14 4e 1) [A, B, C, D, E] 2) [A, C, D] 3) [A, B, C, D] 4) [B, C, E] 5) [A, B, E] 6) [A, B, D] 7) [A, C] 8) [A, C, E] 9) D 10) E 11) C 12) D 13) A 14) C 15) B 16) A 17) B 18) B 19) A 20) C 21) E 22) A 23) C 24) E 25) D 26) D Version 1
31
27) C 28) E 29) C 30) E 31) B 32) A 33) B 34) A 35) C 36) A 37) E 38) C 39) B 40) A 41) B 42) C 43) A 44) E 45) B 46) B 47) D 48) C 49) D 50) D 51) C 52) A 53) C 54) D 55) C 56) E Version 1
32
57) A 58) B 59) D 60) A 61) D 62) A 63) A 64) A 65) A 66) B 67) B 68) B 69) A 70) Section Break 70.1) A 70.2) C 70.3) D 70.4) A 70.5) A 71) Section Break 71.1) B 71.2) B 71.3) C 71.4) B 71.5) C 72) Section Break 72.1) C 72.2) C 72.3) D 72.4) C Version 1
33
73) TRUE 74) TRUE 75) FALSE 76) FALSE 77) FALSE 78) TRUE 79) TRUE 80) FALSE 81) TRUE 82) TRUE 83) TRUE 84) TRUE
Version 1
34
CHAPTER 15 1) Please select all of the correct statements regarding sample collection from patients to test your understanding of infectious agent specimen collection.
A) Aseptic techniques, sterile containers, and tools should be used. B) All tissues surrounding the infection site should also be included when taking a sample. C) Urine samples are taken as “first-voided” specimens. D) After proper collection, samples can be stored until use at room temperature. E) Special transport media is used to maintain the specimen for a period of time before clinical analysis.
2) Select all of the correct statements regarding the types of tests performed on microbial isolates versus those performed on patients themselves in the process of microbial identification.
A) In vivo tests for reactions to microbes are performed on microbial isolates obtainedfrom the patient. B) Disease diagnosis begins with patient analysis for signs of microbial infection (i.e., fever, wound exudate, mucus production, abnormal lesion). C) Antibody titers in patient blood can be checked without the isolation of microbes from the patient. D) Phage testing is a type of testing method that can be performed on the patient to aid in the process of microbial identification. E) Biochemical testing is a type of culture test that can be performed on microbial isolates obtained from the patient. F) Acid-fast staining is a direct testing method that can be performed on microbial isolates obtained from the patient.
3)
What are some limiting factors of using culturing techniques to diagnoseinfections?
Version 1
1
A) It can be difficult to isolate the actual causative agent from other bacteria in the collected specimen. B) Culturing takes 18–24 hours for incubation which limits the ability to quickly and effectively treat some fatal infections. C) Culturing requires a patient to be admitted to a clinic or hospital for 18–24 hours of direct testing. D) Many infections are actually caused by more than one organism.
4)
Which of the following is not a phenotypic method of identification?
A) Morphology B) Gram stain reaction C) Acid-fast reaction D) Antibody response E) Endospores
5)
Biochemical tests include all of the following except
.
A) presence of catalase B) presence of oxidase C) colony morphology D) sugar fermentation E) gas production
6)
The primary advantage of genotypic methods of identification is
Version 1
.
2
A) they are easier than any other method B) they areless expensive than other methods C) they arewidely available D) culturing of the organism is not required E) All of the choicesare correct.
7)
Specimen collection
.
A) is always done by a medical professional B) must be done under sterile conditions C) must utilize aseptic techniques D) does not require special handling E) All of the choicesare correct.
8)
Each of the following are appropriate specimens for bacterial culture except
.
A) saliva B) skin C) spinal fluid D) hair E) throat
9)
Which sample is not typically collected by sterile needle aspiration? A) Blood B) Urine C) Cerebrospinal fluid D) Tissue fluids E) All of the choicesare collected by sterile needle aspiration.
Version 1
3
10)
Urine and fecal specimens require
.
A) sterile collection conditions B) incubation in differential media C) incubation in selective media D) both sterile collection conditions and incubation in differential media E) both incubation in differential media and incubation in selective media
11)
Phage typing is useful in identifying
.
A) Treponema pallidum B) Mycobacterium leprae C) Streptococcus D) Salmonella E) Clostridium
12)
Animals are required for the cultivation of
.
A) Treponema pallidum B) Pseudomonas C) Streptococcus D) Salmonella E) Clostridium
13)
Which test is especially good for bacteria that are not readily cultivated in the lab?
Version 1
4
A) Gram stain B) Direct antigen testing C) Dichotomous key D) Direct fluorescence antibody (DFA) testing E) Phage test
14)
Which of the following is not a genotypic method of identification?
A) rRNA sequencing B) PCR C) Biosensor D) Direct antigen testing E) DNA analysis with probes
15)
Which of the following is involved in the in vitro diagnostic testing of serum?
A) Mycology B) Hematology C) Serology D) Histology E) Virology
16) The property of a test to detect even small amounts of antibodies or antigens that are test targets is .
Version 1
5
A) cross reaction B) agglutination C) precipitation D) specificity E) sensitivity
17) The property of a test to detect only a certain antibody or antigen, and not to react with any others is .
A) cross reaction B) agglutination C) precipitation D) specificity E) sensitivity
18)
A serum titer involves
.
A) serially diluting a serum sample B) determining the lowest dilution of serum that produces a visible reaction C) determining the highest dilution of serum that produces a visible reaction D) the Western blot method E) None of the choices are correct.
19)
A positive serological test for tuberculosis indicates that
A) B) C) D)
Version 1
.
the patient has active tuberculosis the patient is an asymptomatic carrier of tuberculosis the patient has been exposed to tuberculosis All of thechoices are correct.
6
20)
Serological testing can be used to test each of the following except
.
A) blood serum B) cerebrospinal fluid C) urine D) saliva E) skin
21)
Titer is the amount of
.
A) antigen in serum B) antibody in serum C) WBC in serum D) complement in serum E) memory cells in serum
22)
Serological testing relies upon
.
A) the specificity of the Fc region of antibodies B) the specificity of the variable regions of antibodies C) a patient who is not immunocompromised D) very high viral or bacterial load in the patient E) None of the choices are correct.
23)
Antibody testing requires
Version 1
.
7
A) B) C) D)
24)
a known antigen a known antibody both a known antigen and a known antibody either a known antigen or a known antibody
What type of test will detect whole antigens?
A) Cross reaction B) Agglutination C) Precipitation D) Specificity E) Sensitivity
25)
Soluble antigens are detected in this type of test.
A) Cross reactions B) Agglutination C) Precipitation D) Specificity E) Sensitivity
26)
Precipitation tests involve all of the following except
.
A) they rely on formation of visible clumps for detection B) they include the VDRL test for syphilis C) they are often performed in agar gels D) they can be done in a test tube by carefully adding antiserum over antigen solution E) a cloudy or opaque zone developing where antigen and antibody react
Version 1
8
27)
In precipitation tests, the antigen
.
A) is a soluble molecule B) is an insoluble molecule C) is a whole cell D) antibody complex settles to the bottom of the tube E) None of the choices are correct.
28)
Syphilis can be diagnosed most easily by
.
A) the Weil-Felix reaction B) direct fluorescence C) ELISA D) agglutination E) immunochromatography
29) Which test is the confirming test for people who initially tested antibody-positive in the screening ELISA test for HIV?
A) Immunochromatography B) Western blot C) Immunelectrophoresis D) Radioimmunoassay (RIA) E) Weil-Felix
30)
The Western blot test is confirmatory for HIV because
Version 1
.
9
A) it is more sensitive than the ELISA B) it has fewer false positives than the ELISA C) it tests for more HIV antibodies than ELISA D) it is easier to interpret than ELISA E) All of thechoices are correct.
31) Which test uses radioactive isotopes to label antibodies or antigens in order to detect minute amounts of corresponding antigen or antibody?
A) Immunochromatography B) Western blot C) Immunelectrophoresis D) Radioimmunoassay (RIA) E) ELISA
32)
Horseradish peroxidase and alkaline phosphatase are enzymes used in
tests.
A) Western blot B) ELISA C) directfluorescent antibody D) indirectfluorescent antibody E) None of the choices are correct.
33)
A positive indirect ELISA result requires
Version 1
.
10
A) one known antibody, one unknown antibody, and an unknown antigen B) a known antigen, an unknown antibody, and a known antibody C) two known antibodies andone known antigen D) two known antibodies andone unknown antigen E) All of the choicesare correct.
34)
The indirect ELISA test is used to diagnose all of the following except
.
A) strep throat B) HIV C) hepatitis A and C D) Helicobacter E) Rickettsia
35)
In vivo testing is useful for diagnosing
.
A) tuberculosis B) rubella virus C) hepatitis A D) HIV E) whooping cough
36)
The tuberculin skin test is read
.
A) within 1 hour B) after 12 hours C) from 12 to 24 hours D) from 24 to 48 hours E) from 48 to 72 hours
Version 1
11
37) When minute samples of DNA need to be genetically analyzed for identification purposes, which test can be effectively used?
A) Direct fluorescence antibody B) Immunofluorescence C) Weil-Felix reaction D) PCR E) Direct antigen reaction
38)
The three main techniques for identifying microbes are
A) B) C) D)
39)
.
phenotypic, immunologic, and genotypic microscopic, macroscopic, and biochemical PCR, electrophoresis, and biochemical DNA, RNA, and antibodies
Which of the following is not one of the main categories for identifying microbes?
A) B) C) D)
Radiologic Phenotypic Genotypic Immunologic
40) Phenotypic methods of identifying microorganisms in a patient sample are characterized as those that .
Version 1
12
A) directly examine the organism's appearance or behavior,which includes its metabolic abilities, environmental preferences and drug susceptibilities B) analyze the genetic makeup of the microorganism, which conclusively diagnoses the infection C) make use of the patient's antibodies to precipitate the microorganism out of solution, or agglutinate the antigens in the sample D) make use of color-changing antibodies directed against the microbe that will become visible to the naked eye when bound
41) Immunologic methods of identifying microorganisms in a patient sample are characterized as those that .
A) rely on the specificity of antibodies to target a single antigen B) directly examine the organism's appearance or behavior,which includes its metabolic abilities, environmental preferences and drug susceptibilities C) analyze the genetic makeup of the microorganism, which conclusively diagnoses the infection D) amplifies the microbial DNA in the patient's sample and during the process, identifies the organism through the use of known primers
42) Microorganisms can be identified from a patient sample using biochemical tests based on the premise that .
A) organisms have a unique combination of metabolic enzymes that are displayed when color-changing dyes are utilized to demonstrate their physiological response to specific substrates B) organisms display specific antigens that can be bound by color-changing antibodies that allow the clinician to locate and identify the pathogen C) organisms will not grow on certain media types and can be selected for on others D) bacteriophages infect certain bacterial cells and the specificity can be used to identify the organism
Version 1
13
43) Microbes can be identified using automated biochemical tests, often without incubation. An example of this type of test would include .
A) a test for the presence of the enzyme urease, that would change the media from yellow to red B) binding of fluorescent antibodies to the specific antigenic determinant in the sample C) exposure to DNA probes that are either fluorescently labeled or will initiate a color change when bound to their complement D) digestion of the genomic DNA by restriction enzymes, followed by separation by gel electrophoresis
44) Techniques that make use of probes that hybridize to specific sequences of a pathogen's genome include .
A) B) C) D)
FISH and microarrays ELISA and FISH FISH and Western blot PFGE and ELISA
45) FISH and microarrays both use probes to hybridize to sequences of the pathogen genome. They differ in that .
Version 1
14
A) FISH involves adding fluorescently labeled probes directly to a patient's sample, whereas microarrays involve the attachment of thousands of potential gene sequences from pathogens to an absorbent plate and adding DNA from the patient's sample to it, allowing matching sequences to hybridize B) microarrays involve adding fluorescently labeled probes directly to a patient's sample, whereas FISH involves the attachment of thousands of potential gene sequences from pathogens to an absorbent plate and adding DNA from the patient's sample to it, allowing matching sequences to hybridize C) FISH involves scanning and analyzing the pathogen genome multiple times, whereas microarrays involve separating DNA samples into fragments using restriction enzymes, then exposing the fragments to varying voltage levels from three different directions D) microarrays involve scanning and analyzing the pathogen genome multiple times, whereas FISH involves separating DNA samples into fragments using restriction enzymes, then exposing the fragments to varying voltage levels from three different directions
46) An outbreak of hepatitis A originating in a popular chain of restaurants serving fresh produce necessitated the rapid analysis of the viral genome in order to trace its source. The genome was scanned and analyzed multiple times to reduce errors. This is an example of .
A) B) C) D)
whole genome sequencing PFGE FISH microarrays
47) An outbreak of E. coli 0157:H7 was suspected to be associated with the petting zoo enclosure at the state fair. Epidemiologists with the CDC created a DNA fingerprint from the E. coli isolated from patients and compared it to bacterial isolates analyzed by national laboratories. They confirmed that the organism was indeed being transmitted by the animals at the fair. The CDC uses what method to generate these fingerprints?
Version 1
15
A) B) C) D)
48)
PFGE FISH Microarray Hybridization
The benefit of "lab on a chip" technology over standard microarrays is that
.
A) lab on a chip technology has miniaturized testing to carry out analyses using minute amounts of reagents, whereas microarrays still require larger volumes of reagents in comparison B) lab on a chip technology requires little technical training compared to analyses using microarrays C) lab on a chip technology is invaluable for developing countries, where the reagents, refrigeration and trained personnel required for microarrays are limited D) All of the choices are benefits of "lab on a chip" technology.
49) A protein fingerprint of a patient's blood sample can be created by adding the sample to a metal plate and striking it with a laser. This causes the sample to become ionized and the ions from the sample are guided into a machine that separates them and identifies them according to their mass-to-charge ratio. This process describes .
A) MALDI-TOF B) "lab on a chip" C) PFGE D) PCR E) FISH
50) Choose the phrase that best describes how most enzymatic reactions are observed in the biochemical testing of microbial specimens.
Version 1
16
A) B) C) D)
Usually visualizedby a breakdown of the media Usually visualizedby increased growth rate of agent Usually visualizedby a color change Usually visualizedby autofluorescence
51) Choose the statement that best describes the probe used in the hybridization method of microbe identification.
A) B) C) D)
Labeledcomplementary DNA Unlabeledcomplementary DNA Labeled identicalDNA Labeledcomplementary RNA
52) Choose the statement that best describes the PCR's usefulness in infectious disease diagnosis.
A) Results inamplification of sample DNA B) Results inamplification of translation C) Results inincreased RNA production D) Uses labeledcomplementary DNA probes E) Uses fluorescent antibody labeling
53) is the branch of immunology that traditionally deals with in vitro diagnostic testing of serum, although the field has expanded to include other types of samples such as urine, saliva, and cerebrospinal fluid.
Version 1
17
A) B) C) D)
Serology Hybridization Immunofluorescence Microscopic morphology
54) The of a test means the test can detect even very small concentrations of the antigen or antibody being analyzed, whereas the of a test focuses on the ability of a test to only detect the desired antigen or antibody without reacting with other unrelated substances. A) B) C) D)
55)
sensitivity; specificity specificity; sensitivity indirectness; directness directness; indirectness
Choose the phrase that best describes the indirect ELISA technique.
A) Detects thepresence of antigen in a sample using known antibody absorbed onto the well B) Detects thepresence of antibody in a sample using a known antigen absorbed onto thewell C) Detects thepresence of antibody using a complement indicator system D) Detects thepresence of antibody using a radioactive isotope labeled antigen
56)
Choose the statement that best describes the direct ELISA technique.
Version 1
18
A) Detects the presence of antigen in a sample using known antibody absorbed into the well B) Detects the presence of antibody in a sample using a known antigen absorbed into the well C) Detects thepresence of antibody using a complement indicator system D) Detects thepresence of antibody using a radioactive isotope labeled antigen
57) The ELISAis the common test used for antibody screening forHIV, various rickettsial species, hepatitis A and C, and Helicobacter,while the ELISA is used for the detection of hantavirus and measles virus in patient samples. A) B) C) D)
58)
indirect; direct direct; indirect sensitive; specific specific; sensitive
Which of the following statements describes a microarray?
A) Absorbent plates ("chips") are loaded with thousands of potential gene sequences from different organisms, and patient specimens are incubated on the plates, allowing a computer to detect regions where there is a match. B) MRIs and PET scans are used to find areas in deep tissues where infections are caused by biofilms. C) Patient samples are ionized when subjected to metal plates and lasers, and then the ions are separated and identified by their mass-to-charge ratio. D) Random primers are used to discover novel sequences in the microbe's DNA.
Version 1
19
59) NCLEX Prep - Test Bank Question: Please read the clinical scenario, and then answer the questions that follow to become familiar with the traditional NCLEX question format. A student nurse on a medical-surgical floor is caring for a 62-year-old previously healthy female. The patient has been admitted for an infectious disease work-up. She has been experiencing generalized pain and nausea for the past three weeks, and recently developed a fever. The student nurse prepares to assist the RN in collecting blood for the following lab tests: complete blood count (CBC), C-reactive protein (CRP), eosinophil sedimentation rate (ESR), blood culture, urine culture, and a lumbar puncture with cerebral spinal fluid (CSF) culture and cell count. The nurse is also collecting a sputum culture, as well as a respiratory PCR by nasopharyngeal suction.
59.1)
The patient’s CRP is markedly elevated. What does this indicate?
A) B) C) D)
An elevated CRP isa highly sensitive urinary tract infection marker. An elevated CRP points to inflammation from any of a variety of sources. An elevated CRP is diagnostic of a fungal infection. An elevated CRP is diagnostic of a bacterial infection.
59.2) The patient’s urine culture was collected using a clean-catch method. After inoculation and incubation, several organisms were identified in amounts that were reported as normal microbial flora. Which of the following statements by the student nurse indicates proper understanding of the findings? A) The patient is superinfected and needs immediate antimicrobial treatment. B) The specimen was contaminated and another clean-catch urine specimen needs to be cultured. C) The results point to an infection, but due to multiple organisms, further testing needs to be done to isolate the offending microbe. D) This is a common finding due to contamination of a clean-catch urine sample with normal microbiota from the urogenital area.
Version 1
20
59.3) The patient’s respiratory PCR results are reported as “insufficient cells available for testing.” Which of the following statements by the student nurse indicates proper understanding of the findings?
A) The patient is free of infection since insufficient cells could be isolated. B) A smaller sample must be sent, since the irrigation solution volume diluted the cells. C) This is a very specific and sensitive finding that points to a viral infection. D) The sample was inadequate, and the nasopharyngeal specimen collection must be repeated.
59.4) The lumbar puncture was especially uncomfortable for the patient. What step listed below is most important to be taken to ensure the procedure does not need to be repeated?
A) The specimens mustbe properly labeled. B) The specimens mustbe properly obtained and stored. C) The specimens mustbe collected in sufficient amounts. D) The specimens mustbe properly transported. E) All of the choicesare equally important.
60) NCLEX Prep - Test Bank Question: Please read the clinical scenario, and then answer the questions that follow to become familiar with the traditional NCLEX question format. The public health department is following a 34-year-old male who has reported several weeks of general malaise and a low-grade fever. The nurse assigned to his case communicates with the patient regarding the results of his laboratory testing.
60.1) An ELISA test sent for HIV screening came back with a positive result. The nurse educates the patient that this test is highly sensitive with low specificity.Which of the following statements by the patient demonstrates understanding of this teaching?
Version 1
21
A) It is possible that this is a false-positive result since the test may have been positive for something other than HIV. B) It is unlikely that this is a false-positive result since the test is so specific it does not miss a positive result. C) It is possible that this is a false-positive result since low specificity means the tests are less accurate. D) It is unlikely that this is a false-positive result since it is highly sensitive.
60.2) A Western blot test will be performed. The nurse educates the patient that the Western blot testhas high sensitivity and specificity. Which of the following statements by the patient demonstrates understanding of this teaching? A) A positive Western blot test may be a false-positive since the test has high specificity and may detect the presence of many antibodies. B) The Western blot test is unlikely to generate a false-positive result. This will be used to confirm or refute the ELISA test results. C) A positive Western blot may be a false-positive since the test has high sensitivity and may detect the presence of many antibodies. D) The Western blot test may generate a false-positive result. A positive test will be confirmed by an HIV PCR.
60.3) The patient reports that he had a negative HIV PCR test about a month ago. What is the most appropriate interpretation of this information by the nurse?
A) PCRs are rarelyaccurate. B) The ELISA was a false-positive. C) It is possible the ELISA was a false-positive, and the Western blot must be performed to confirm the results. D) The HIV PCR is outdated and does not pick up modern strains of the virus.
Version 1
22
61) NCLEX Prep - Test Bank Question: Please read the clinical scenario, and then answer the questions that follow to become familiar with the traditional NCLEX question format. A 16-year-old male with a low-grade fever, and large and painful red sores around his lips is being evaluated at a pediatric clinic. He has a PCR test pending that was swabbed from the sores, as well as a blood culture with Gram stain, and nasopharyngeal specimen for microscopic examination.
61.1) Which of the following can be determined from phenotypic assessment of the nasopharyngeal sample?
A) B) C) D)
61.2)
A) B) C) D)
Cellular shape, size, and arrangement The organism’s DNA The presence of antibodies The organism’s RNA
What kind of data will the PCR results provide?
Biochemical characteristics Immunologic information Genotypic information Microscopic morphology
61.3) Analysis of the laboratory samples yield a diagnosis of herpes simplex virus. The patient’s mother is upset, as she does not believe her son is sexually active. The nurse explains that the presence of HSV does not imply sexual contact and provides education about the accuracy of the results. Which of the following is the best rationale for confidence in the diagnosis?
Version 1
23
A) B) C) D)
62)
Signs and symptoms are diagnostic. PCR tests are highly sensitive and specific. The Gram stain is an excellent screen for HSV. HSV can be definitively identified by light microscopy.
It is necessary to do lab tests to diagnose all diseases. ⊚ ⊚
true false
63) When antibodies or other substances in serum cross-react with the test reagents, a false positive result can occur. ⊚ ⊚
64)
Serological testing always involves reactions between specific antibody and antigen. ⊚ ⊚
65)
true false
Serological tests should have low sensitivity and specificity. ⊚ ⊚
66)
true false
true false
The Weil-Felix test is used for diagnosing salmonellosis. ⊚ ⊚
Version 1
true false
24
67) Fluorescent dye can be used as a label to trace antibody-antigen reactions in an ELISA assay. ⊚ true ⊚ false
68)
The tuberculin test is an example of an in vitro serological test. ⊚ ⊚
true false
69) The three main techniques for identifying microorganisms from patient samples are specificity, sensitivity, and biochemical. ⊚ ⊚
true false
70) If a phenotypic analysis requires culturing the organism before testing can begin, this is problematic for two reasons; i) rapid diagnosis may be critical and culturing can take up to 24 hours, and ii) a successful in vitro culturing method for a specific pathogen may not be feasible. ⊚ ⊚
true false
71) The value of whole genome sequencing is that it can separate larger strands of DNA by slowly applying alternating voltage levels from three different directions. ⊚ ⊚
Version 1
true false
25
72) The CDC operates a national laboratory network called PulseNet, that compiles DNA fingerprinting data and connects foodborne illness cases so outbreaks can be detected rapidly. ⊚ ⊚
true false
73) The ability to miniaturize genetic testing on a chip much smaller than a microarray plate will see its greatest benefit in developed countries so the large corporations can process genetic data much more efficiently. ⊚ ⊚
true false
74) MRI, CT and PET scans are useful diagnostic tools in cases where infection is located in deep tissues, thereby saving the patient from an invasive biopsy. ⊚ ⊚
true false
75) Microbiologists use phenotypic techniques, which consist of both microscopic and macroscopic observations, to help identify bacteria. ⊚ ⊚
true false
76) The success of identification and treatment of a patient’s infection is independent of how specimens are collected, handled, and stored. ⊚ ⊚
Version 1
true false
26
77) The analysis of infectious agents in laboratory specimens generates results that can fall into two categories: presumptive data and confirmatory data. ⊚ ⊚
true false
78) A technique called peptide nucleic acid FISH (fluorescent in situ hybridization) is usedto identify genetic sequences, but is a time-consuming process since the organism must first be cultured. ⊚ true ⊚ false
79) The polymerase chain reaction can only be employed on bacterial samples for pathogen diagnosis. ⊚ ⊚
true false
80) Serological testing is based on the presence of antibodies that bind specifically to antigen, allowing the presence of such antigens to be detected and quantified. ⊚ ⊚
true false
81) Western blotting is the second (verification) test for preliminary positive HIV screening tests because it can detect thepresence of microbial-specific antigens or antibodies in a highly specific and sensitive manner. ⊚ ⊚
Version 1
true false
27
Answer Key Test name: Ch 15 4e 1) [A, E] 2) [B, C, E, F] 3) [A, B, D] 4) D 5) C 6) D 7) C 8) D 9) B 10) E 11) D 12) A 13) D 14) D 15) C 16) E 17) D 18) C 19) C 20) E 21) B 22) B 23) A 24) B 25) C 26) A Version 1
28
27) A 28) B 29) B 30) E 31) D 32) B 33) B 34) A 35) A 36) E 37) D 38) A 39) A 40) A 41) A 42) A 43) A 44) A 45) A 46) A 47) A 48) D 49) A 50) C 51) A 52) A 53) A 54) A 55) B 56) A Version 1
29
57) A 58) A 59) Section Break 59.1) B 59.2) D 59.3) D 59.4) E 60) Section Break 60.1) A 60.2) B 60.3) C 61) Section Break 61.1) A 61.2) C 61.3) B 62) FALSE 63) TRUE 64) TRUE 65) FALSE 66) FALSE 67) FALSE 68) FALSE 69) FALSE 70) TRUE 71) FALSE 72) TRUE 73) FALSE 74) TRUE 75) TRUE 76) FALSE Version 1
30
77) TRUE 78) FALSE 79) FALSE 80) TRUE 81) TRUE
Version 1
31
CHAPTER 16 1)
Select all of the statements that illustrate important anatomical features of the skin.
A) The outermost portion of the skin is the dermis. B) The epidermis is composed of keratinized epithelium. C) Hair follicles are found in the subcutaneous layer. D) Sweat glands are found in the dermis but have openings in the epidermis. E) The stratum basale is the top layer of the dermis and is the source of epidermal cells.
2)
Select all of the answers that illustrate examples of natural defenses present in the skin.
A) Antimicrobialpeptides, which disrupt membranes of bacteria B) High pH of sebum,which makes skin inhospitable to most microbes C) Lysozyme, whichbreaks down peptidoglycan D) Keratin, whichprevents microbe penetration E) Sloughing of skin, which removesattached microbes
3) Select all of the statements that apply to antimicrobial peptides serving as a natural defense present in the skin.
A) They are found in epithelial cells. B) They are negatively charged chemicals. C) They act by disrupting membranes of bacteria. D) They act by breaking down peptidoglycan of the bacterial cell wall. E) They occur in many different types.
4) Select all of the statements that apply to the pathological aspects of maculopapular rash diseases, such as measles, rubella, fifth disease, and roseola.
Version 1
1
A) The causative agents of all the listed diseases are viruses. B) The causative agents of all the listed diseases are bacteria. C) Droplet contact or direct contact is the most common mode of transmission for these diseases if known. D) The listed diseases can all be prevented by live attenuated vaccine. E) Antibiotics are used to treat these diseases.
5) Select all of the statements that apply to the pathological aspects of impetigo, a skin disease.
A) Staphylococcus aureus and Streptococcus pyogenes are the causative organisms. B) The most common modes of transmission are direct and indirect contact. C) No bacterial virulence factors have been found to be associated with the disease. D) Disease is seen only in infants and young children. E) Personal hygiene has no impact on disease prevention.
6)
Select all of the statements that apply to bacterial conjunctivitis.
A) It is caused by more than one type of bacteria. B) In children and adults, prevention is through good hygiene. C) Clear discharge is a distinguishing feature. D) Prevention is through elimination of the vector. E) This infection is relatively rare.
7)
Select all of the statements that apply to keratitis, a serious eye infection.
Version 1
2
A) Viral keratitis canonly be caused by HSV-1. B) Blindness due toherpes is the leading infectious cause of blindness in the United States. C) Keratitis caused byHSV-1 is most often treated with an antibiotic. D) Keratitis can becaused by a eukaryotic organism. E) Acanthamoeba infection can causekeratitis, especially in those who wear contact lenses.
8)
The integument includes all of the following except
.
A) skin B) hair C) nails D) sweat glands E) surface capillaries
9)
Blood vessels are found in the
.
A) stratum corneum B) dermis C) stratum basale D) subcutaneous layer E) dermis and subcutaneous layer
10)
Blisters
Version 1
.
3
A) are confined to the epidermis B) originate in the dermis C) originate in the subcutaneous layer D) result from a separation of epidermis from dermis E) None of the choices are correct.
11)
Protective features of the skin include all but
.
A) keratinized surface B) resident biota C) antimicrobial peptides D) high pH E) lysozyme
12) What material in skin cells provides protection from abrasions, water damage, and microorganism entry?
A) Lysozyme B) Keratin C) Sweat D) Sebum E) Salt
13)
What group of microorganisms do not usually call the skin "home"?
Version 1
4
A) Escherichia B) Corynebacterium C) Propionibacterium D) Yeasts E) All of thechoices are correct.
14) What enzyme is not found in Staphylococcus epidermidis but is found in Staphylococcus aureus?
A) Peroxidase B) Catalase C) Lysozyme D) Coagulase E) Lipase
15)
All species of Staphylococcus
.
A) lack spores B) are motile C) have capsules D) produce coagulase E) All of thechoices are correct.
16)
The enzyme that coagulates plasma is
Version 1
.
5
A) catalase B) coagulase C) hyaluronidase D) staphylokinase E) kinase
17)
Staphylococcus can be differentiated from Streptococcus by the
.
A) Gram stain B) coagulase test C) catalase test D) fermentation of mannitol E) None of the choices are correct.
18) All Staphylococci can be differentiated from all Streptococci because only Staphylococci produce the enzyme that is easily detected in the lab.
A) staphylokinase B) exfoliative toxinA C) exfoliative toxinB D) catalase E) lactase
19)
Which of the following is the most common form of microbe transmission in impetigo?
Version 1
6
A) Fomites B) Direct contact C) Mechanical vectors D) Blood E) Body fluids
20)
Impetigo is caused by
.
A) Propionibacterium acnes B) Corynebacterium diphtheriae C) Staphylococcus aureus D) Streptococcus pyogenes E) both Staphylococcus aureus and Streptococcus pyogenes
21)
MRSAs are Staphylococcus aureus strains that are
.
A) resistant to mupirocin B) responsive to mupirocin C) resistant to penicillin derivatives D) responsive to penicillin derivatives E) related to Streptococcus pyogenes
22) The toxin of Staphylococcus aureus strains that causes blisters and desquamation of skin in scalded skin syndrome is .
Version 1
7
A) enterotoxin B) hemolysin C) toxic shock syndrome toxin D) exfoliative toxin E) erythrogenic toxin
23)
Infections caused by Staphylococcus aureus include all of the following except .
A) meningitis B) furuncles and carbuncles C) impetigo D) scalded skin syndrome E) acne
24)
All of the following pertain to Streptococcus pyogenes except
.
A) is often from an endogenous source B) secretes streptokinase C) causes impetigo D) coats itself with host proteins E) causes gas gangrene
25)
Streptococcus pyogenes causes all of the following except
A) B) C) D)
Version 1
.
necrotizing fasciitis impetigo scarlet fever scalded skin syndrome
8
26)
Which of the following is not true of cellulitis?
A) Caused by Staphylococcus aureus or Streptococcus pyogenes B) Occurs in the epidermis C) Causes pain, tenderness, swelling, and warmth D) Lymphangitis may occur E) Treated with cephalexin
27)
Which of the following is not true of staphylococcal scalded skin syndrome (SSSS)? A) Affects mostly newborns and babies B) An exotoxin-mediated disease C) Toxin causes bullous lesions D) Split in skin occurs between the dermis and epidermis E) Split in skin occurs within the epidermis
28)
The smallpox vaccine uses the
virus.
A) smallpox B) herpessimplex C) vaccinia D) herpes virus6 E) None of the choices are correct.
29)
Smallpox is a disease
Version 1
.
9
A) in which fever, prostration, rash, and possible toxemia and shock occur B) in which the virus becomes latent in ganglia of sensory neurons C) in which recurrent episodes are called shingles D) that can only be transmitted by direct skin contact with skin crusts E) All of thechoices are correct.
30)
Chickenpox
.
A) is transmitted by droplet contact B) lesion distribution is centripetal C) has an incubation of 10 to 20 days D) has fever and vesicular rash that occurs in successive crops E) All of thechoices are correct.
31)
Varicella zoster virus (human herpesvirus-3)
.
A) uses the respiratory epithelium as its portal of entry B) becomes latent in dorsal root ganglia that serve specific dermatomes C) has humans as its reservoir D) causes chickenpox and shingles E) All of thechoices are correct.
32) If a person who has never been infected with the varicella zoster virus comes in contact with a person who has shingles, the first person will come down with . A) herpes labialis B) shingles C) chickenpox D) infectious mononucleosis E) herpes keratitis
Version 1
10
33)
Human herpesvirus 6 (HHV-6) has the following characteristics except
.
A) starts with a high fever B) causes roseola in infants and young children C) appears as a mononucleosis-like illness in adults D) a rash appears on the fourth day E) it is a very rare form of herpesvirus
34)
Oral lesions called Koplik's spots are seen in patients with
.
A) croup B) mumps C) influenza D) measles (rubeola) E) rubella
35)
Which is incorrect about the MMR immunization?
A) Contains attenuated virus B) Contains toxoids C) Is given in early childhood D) Protects against three different viral diseases E) None of the choices are correct.
36)
All of the following pertain to measles (rubeola) except
Version 1
.
11
A) it is transmitted by direct contact with the rash B) humans are the only reservoir for the pathogen C) secondary bacterial otitis media and sinusitis can occur D) it involves a fatal complication called subacute sclerosing panencephalitis (SSPE) E) dry cough, sore throat, fever, and conjunctivitis are symptoms
37)
Measles is described as a
skin lesion.
A) purpura B) bulla C) papule D) macule E) maculopapular
38)
Measles is also known as
.
A) rubella B) shingles C) rubeola D) fifth disease E) varicella
39)
Which of the following is not true of rubeola?
A) Causative agent is a member of the Morbillivirus genus B) It is a single-stranded, nonenveloped RNA virus C) It is in the Paramyxovirus family D) Causes cell membranes to fuse to form syncytia E) Transmitted by respiratory droplets
Version 1
12
40) This febrile disease with a rash has a pathogen that can cross the placenta and cause serious fetal damage. A) Croup B) Mumps C) Influenza D) Measles (rubeola) E) Rubella
41)
Rubella is
.
A) caused by the measles virus (Morbillivirus) B) a very contagious disease C) associated with congenital transmission causing miscarriage, deafness, and cardiac and mental defects D) seen as high fever, severe sore throat, severe cough, and myalgia E) All of thechoices are correct.
42)
Fifth disease
.
A) is caused by Parvovirus B19 B) is a childhood febrile disease with a bright red rash on the cheeks C) cannot be prevented at this time with vaccination D) has a maculopapular rash that lasts for days to weeks E) All of thechoices are correct.
43)
Which form of anthrax involves a black eschar on the skin?
Version 1
13
A) Pulmonary B) Gastrointestinal C) Cutaneous D) All of thechoices are correct. E) None of the choices are correct.
44)
Transmission of the pathogen of Leishmaniasis is by the
.
A) reduviid "kissing" bug B) tsetse fly C) anopheles mosquito D) sand fly E) hard-bodied tick
45)
Which of the following is mismatched?
A) Tinea capitis - ringworm of the beard B) Tinea pedis - ringworm of the foot C) Tinea cruis - ringworm of the groin D) Tinea corporis - ringworm of the body E) Tinea unguium - ringworm of the nails
46)
Transmission of tineas includes
.
A) human to human B) animal to human C) soil to human D) fomites to human E) All of thechoices are correct.
Version 1
14
47)
Which of the following is not a causative agent of cutaneous mycoses?
A) Malassezia B) Trichophyton C) Microsporum D) Epidermophyton E) All of thechoices are correct.
48)
Which of the following is the causative agent of tinea versicolor?
A) Malassezia B) Trichophyton C) Microsporum D) Epidermophyton E) None of the choices are correct.
49)
Which of the following provides the eye with its best defense against microorganisms?
A) Conjunctiva B) Eyelids C) Lymphocytes D) Tears E) Eyelashes
50)
Which of the following is not true of bacterial conjunctivitis?
Version 1
15
A) Transmitted through both direct and indirect contact B) Can be caused by Streptococcus pyogenes or Streptococcus pneumoniae C) Treatment is with a broad-spectrum topical antibiotic D) Has a clear discharge E) Can be caused by Staphylococcus aureus, Haemophilus influenzae, or Neisseria gonorrhoeae
51)
Which is true of viral conjunctivitis?
A) Caused by Chlamydia trachomatis or Neisseria gonorrhoeae B) Has a mucopurulent discharge C) Caused by adenoviruses D) Must be treated with topical and oral antibiotics E) Caused by Moraxella
52)
Keratitis is usually caused by
.
A) Streptococcus pyogenes B) Staphylococcus aureus C) herpes simplex D) Neisseria gonorrhoeae E) All of thechoices are correct.
53)
The eye's best defense is/are the
Version 1
.
16
A) optic nerve B) eyelid C) eyelashes D) tear film E) cornea
54)
What is the only way to prevent conjunctivitis?
A) Wear insect repellent B) Good hygiene C) Trifluridine or acyclovir treatment D) Ivermectin treatment E) Prophylactic antibiotics
55)
The Human Microbiome Project revealed
.
A) vast numbers of organisms are found on expansive, dry areas of the skin B) high salt areas of the skin limit bacterial growth C) the skin microbiota changes constantly over time D) the skin microbiome is much more varied than previously thought E) All of the choices are correct.
56)
Tears are produced in the
and drain into the
.
A) lacrimal gland; lacrimal duct B) lacrimal duct; sclera C) cornea; conjunctiva D) conjunctiva; lacrimal duct E) lacrimal gland; sclera
Version 1
17
57)
The exposed surfaces of the eye that are the most vulnerable to infections are the .
A) conjunctiva and cornea B) sclera and iris C) iris and conjunctiva D) retina and sclera E) pupil and cornea
58)
The eye differs from other parts of the body with reference to defense mechanisms in that .
A) innate defenses, such as inflammation and phagocyte influx, would impair vision and are therefore limited B) they are not vulnerable to infection so they do not require natural defenses C) the primary defense mechanism is the lymphocyte response to epitopes on the pathogen surface D) the third line of defense is the most essential
59) The Human Microbiome Project revealed what important information regarding the microbiota of the eye?
A) Previously thought to be sparsely populated, the eye has been found to have a robust, diverse population of microbiota. B) Previously thought to be sparsely populated, the HMP showed the eye to be sterile. C) The HMP showed that the eye contains only one species of bacteria; Corynebacterium. D) The eye contains a unique population of bacteria, not found anywhere else on the body.
Version 1
18
60)
Which of the following statements is incorrect regarding keratitis?
A) Keratitis can be caused by bacteria or viruses, but not eukaryotic organisms. B) Infection of the cornea by microbes can lead to destruction of the cornea and blindness. C) Infection of the cornea can occur through sexual transmission. D) Latent HSV-1, that reactivates during times of stress, enters the ophthalmic nerve causing keratitis instead of cold sores.
61) The outermost portion of the skin is the , composed of epithelial cells containing keratin which gives this layer the ability to withstand damage, abrasion, and water penetration. A) B) C) D)
62)
epidermis dermis integument subcutaneous layer
Choose the statement that best describes staphylococcal scalded skin syndrome.
A) It is an exotoxin-mediated disease that can be compared to a systemic form of impetigo. B) It is caused by microbial digestion of excess sebum trapped in the pores of the skin. C) Bacteria spread, causing necrotic damage of the tissues and gas production. D) It presents as a generalized pustular rash.
63) occurs when the causative virus, which is strongly teratogenic, is transmitted to the fetus in utero; it can subsequently result in miscarriage or multiple permanent defects in the newborn.
Version 1
19
A) B) C) D)
64)
Tears are formed in the A) B) C) D)
65)
Congenital rubella Scarlet fever Fifth disease Rheumatic fever
gland at the outer and upper corner of each eye.
lacrimal superior nasal conjunctival
Choose the statement that best describes the normal biota of the eye.
A) The normal biota of the eye resembles that found in the mouth. B) The normal biota of the eye only includes bacteria. C) The normal biota of the eye is large in numbers but lacks diversity. D) The normal biota of the eye is larger in numbers and more diverse than previously thought.
66) NCLEX Prep - Test Bank Question: Please read the clinical scenario, and then answer the questions that follow to become familiar with the traditional NCLEX question format. A mother brings her 7-year-old daughter to her pediatrician’s office. It is written in the patient’s chart that she is not immunized. The child has approximately 50 lesions, mainly concentrated on her trunk. Most of the lesions are vesicular, but a few have scabbed over. According to the mother, her daughter was previously healthy and developed this rash about five days ago. She denies any fevers and the rash appears to be persistently itchy. The RN observes the child itch open a vesicle, revealing a clear fluid.
Version 1
20
66.1)
A) B) C) D)
Based upon the patient’s symptoms, the most likely diagnosis is
.
smallpox chickenpox shingles measles
66.2) A staff member in the office is concerned that the state health department needs to be notified of a suspected case of smallpox infection. The RN caring for the child discusses the distinct differences between the symptoms of chickenpox and smallpox. Which of the following is not true concerning the two diseases?
A) Smallpox features deep lesions mainly on the extremities, whereas chickenpox features superficial lesions mainly on the trunk. B) Smallpox patients exhibit a prodromal fever, whereas chickenpox patients do not. C) Smallpox lesions are filled with clear fluid, whereas chickenpox lesions are filled with a thick, opaque liquid and are distinctly indented in the center. D) Smallpox is caused by the variola virus, whereas chickenpox is caused by varicellazoster virus.
66.3) The mother requests antibiotics to treat her daughter’s infection. What is the most appropriate action by the RN?
Version 1
21
A) Advocate that the physician write a prescription for oral amoxicillin because the spread of the infectious lesions can be slowed by a broad-spectrum antibiotic. B) Inform the mother that her daughter needs an antiviral prescription, not an antibiotic, to treat the infection. C) Educate the mother she can purchase a triple-antibiotic ointment to apply to the lesions three times daily. D) Educate the mother that chickenpox does not respond to antibiotics, but is selflimiting and will heal on its own.
66.4) The child shares a bedroom with her younger brother, who has not been vaccinated against or infected with chickenpox. He is supposed to go to summer camp in one week. The mother asks if it is safe to let him go to camp. How would the nurse best respond?
A) Yes, it is safe for him togo. Since chickenpox has an incubation period of 3–5 days, he would have exhibited symptoms of the disease already if he had contracted it. B) Yes, it is safe for him to go. Chickenpox is rarely contracted by children younger than 7 years of age. C) No, it is not safe for him to go. Since chickenpox has an incubation period of 10–20 days, he could begin exhibiting symptoms at camp. D) No, it is not safe for him to go. Since he was exposed to his sister, he will be contagious to other kids at camp even without exhibiting symptoms.
67) NCLEX Prep - Test Bank Question: Please read the clinical scenario, and then answer the questions that follow to become familiar with the traditional NCLEX question format. A mother brings her 8-year-old daughter into an urgent care clinic. The child has a dry, flaky, and peeling rash around her mouth. The mother states she has treated the rash with a number of different lotions, but the lesions persist. She is looking for information on how to make her daughter stop licking her lips, as she thinks this is causing the lesions. Upon assessment, the nurse notes circumoral blisters and honey-colored crusts. In the medical history, the mother reports her daughter has not had any other symptoms, including fever, nausea, vomiting, and malaise.
Version 1
22
67.1) The provider orders a specimen to be obtained for microbial analysis. What is the most appropriate site to culture in this case?
A) B) C) D)
The lesion itself, since there seems to be a superficial infection. The posterior pharynx, since the tonsils are likely the primary source of infection. The blood, since the infection was probably carried systemically. The urine, since the kidneys are probably removing bacterial casts from the body.
67.2) In the lab, the specimen is Gram-stained and inoculated onto blood agar. Microscopic analysis of the slide reveals gram-positive cocci arranged in grapelike clusters, while the blood agar produces round, creamy colonies with clear areas of hemolysis. Which of the following would be the most likely causative organism? A) B) C) D)
Streptococcus pyogenes Staphylococcus aureus Human herpesvirus 3 Pseudomonas aeruginosa
67.3) The lab results indicate that the coagulase test was performed. Which result would be consistent with a diagnosis of S. aureus infection? A) B) C) D)
The sample remains liquid The sample completely coagulates The sample turns yellow and coagulates The sample turns yellow and remains liquid
67.4) The lesions are diagnosed as S. aureus impetigo. The nurse is educating the mother about why her daughter’s affected skin seems so fragile and blisters easily. Which of the following statements is most appropriate? Version 1
23
A) The constant exposure to moisture makes your daughter’s skin very fragile, and blistering is the natural response. B) Your daughter’s scratching irritates the skin, making it very fragile and irritable. C) The blood beneath the surface of the skin is broken down by waste products from S. aureus, and superficial blisters are the clinical sign. D) An exotoxin produced by S. aureus targets a protein responsible for cell-to-cell binding, and blisters are the result.
68) NCLEX Prep - Test Bank Question: Please read the clinical scenario, and then answer the questions that follow to become familiar with the traditional NCLEX question format. A school nurse at a public elementary school notes a number of students presenting to his office with similar symptoms. The students are complaining of itchy, watery eyes with white, milky drainage. They appear to have red and inflamed sclera and conjunctiva. Some students report the symptoms in both eyes, while others are experiencing it in one eye. A number of the students report that they woke up the feeling that their eyes were “glued-shut.”
68.1) Based on the reported symptoms and clinical findings, which of the following is the most likely diagnosis?
A) Keratitis B) Neonatal conjunctivitis C) Bacterial conjunctivitis D) Viral conjunctivitis E) The discharge must be cultured to makea diagnosis.
68.2)
Version 1
What is the nurse’s priority intervention?
24
A) B) C) D)
Send the students home Educate the affected students about hand hygiene practices Obtain prescriptions of antibiotic drops for the affected students Coordinate thorough cleaning of the classrooms
68.3) The nurse calls the parents of the affected students to report the finding of conjunctivitis. One parents questions why antibiotics would be ordered without a culture and identification of a specific organism. Which of the following statements by nurse is correct?
A) Antibiotics are effective at clearing viral and bacterial infections. B) Antibiotics are given prophylactically, even if viral conjunctivitis is suspected because we use clinical signs for diagnosis. C) Antibiotics are given to prevent superinfection while the eye’s natural defenses are compromised. D) Antibiotics are given while we wait for the child to be seen by a physician for more extensive diagnostic workup.
69)
Propionibacterium acnes are part of the microbiome of the skin's sebaceous glands. ⊚ ⊚
70)
Smallpox is caused by an orthopoxvirus. ⊚ ⊚
71)
true false
true false
Varicella and herpes zoster are caused by the same virus.
Version 1
25
⊚ ⊚
72)
true false
Herpesviruses are double-stranded, nonenveloped DNA viruses. ⊚ ⊚
true false
73) The most serious teratogenic effects of intrauterine rubella infection occur if it is acquired during the third trimester. ⊚ ⊚
74)
Mottled, discolored pigmentation is seen in patients with tinea versicolor. ⊚ ⊚
75)
true false
The eye generally has a large percentage of normal biota present. ⊚ ⊚
77)
true false
Impetigo involves itching papules that break and form a very contagious yellow crust. ⊚ ⊚
76)
true false
true false
Herpes keratitis is an infection of the eye.
Version 1
26
⊚ ⊚
78)
true false
Staphylococcus epidermidis is a component of the eye microbiota. ⊚ ⊚
true false
79) The parts of the eye that are the most susceptible to microbial infection are the sclera and the iris. ⊚ ⊚
true false
80) Two bacterial genera found as normal biota on human skin are Pseudomonas and Staphylococcus. ⊚ ⊚
true false
81) MRSA infections thatoccur in individualswho have had no recent history in hospitals or clinics are termed community-acquired or CA-MRSA cases. ⊚ ⊚
82)
true false
Measles (rubeola) is associated with disabling of the host immune response. ⊚ ⊚
Version 1
true false
27
83) Since the eye’s primary function is vision, it is important to have a strong, easily initiated inflammatory response in this region. ⊚ ⊚
true false
84) Keratitis is a more serious eye infection than conjunctivitis, and involves microbial invasion of deeper eye tissues which can lead to corneal destruction. ⊚ ⊚
Version 1
true false
28
Answer Key Test name: Ch 16 4e 1) [B, D] 2) [A, C, D, E] 3) [A, C, E] 4) [A, C] 5) [A, B] 6) [A, B] 7) [B, D, E] 8) E 9) E 10) D 11) D 12) B 13) A 14) D 15) A 16) B 17) C 18) D 19) B 20) E 21) C 22) D 23) E 24) E 25) D 26) B Version 1
29
27) D 28) C 29) A 30) E 31) E 32) C 33) E 34) D 35) B 36) A 37) E 38) C 39) B 40) E 41) C 42) E 43) C 44) D 45) A 46) E 47) A 48) A 49) D 50) D 51) C 52) C 53) D 54) B 55) D 56) A Version 1
30
57) A 58) A 59) A 60) A 61) A 62) A 63) A 64) A 65) D 66) Section Break 66.1) B 66.2) C 66.3) D 66.4) C 67) Section Break 67.1) A 67.2) B 67.3) B 67.4) D 68) Section Break 68.1) C 68.2) C 68.3) B 69) TRUE 70) TRUE 71) TRUE 72) FALSE 73) FALSE 74) TRUE 75) TRUE Version 1
31
76) FALSE 77) TRUE 78) TRUE 79) FALSE 80) TRUE 81) TRUE 82) TRUE 83) FALSE 84) TRUE
Version 1
32
CHAPTER 17 1) Select all of the statements that apply to the natural defenses present in the nervous system.
A) Defenses are mainly humoral immunity. B) Defenses are mainly structural. C) Blood vessels that serve the brain have restricted permeability. D) The immune response is highly exaggerated in the CNS. E) Some nervous system cells have phagocytic capabilities.
2)
Select all of the statements that accurately describe the oral polio vaccine. A) B) C) D)
3)
It is easily administered by mouth. It contains attenuated virus. It is the form of polio vaccine currently used in the United States. It contains inactivated virus.
Select all of the statements that describe the inactivated polio vaccine.
A) It contains attenuated virus. B) It is the form of polio vaccine currently used in the United States. C) It contains inactivated virus.
4) Select all of the statements that apply to meningoencephalitis caused by Naegleria fowleri.
Version 1
1
A) Individuals are usually exposed to the amoeba when swimming in warm, natural bodies of water. B) Naegleria meningoencephalitis is a slowly advancing disease. C) Naegleria meningoencephalitis is a rapidly advancing disease. D) Naegleria meningoencephalitis treatment is usually futile. E) Naegleria meningoencephalitis treatment is very successful.
5)
Select all of the statements that apply to acute encephalitis caused by arboviruses.
A) An arthropod bite is involved in transmission. B) Direct contact is the mode of transmission. C) Insect control is the main method of prevention. D) Good hygiene is the main method of prevention. E) No satisfactory treatment exists.
6)
Cerebrospinal fluid is found in the
.
A) pia mater B) dura mater C) subarachnoid space D) arachnoid mater E) All of thechoices are correct.
7) Place the following in order from skull to brain: 1) subarachnoid space; 2) arachnoid mater; 3) dura mater; 4) pia mater.
Version 1
2
A) Subarachnoid space, arachnoid mater, dura mater, pia mater B) Dura mater, arachnoid mater, subarachnoid space, pia mater C) Dura mater, arachnoid mater, pia mater, subarachnoid space D) Arachnoid mater, subarachnoid space, dura mater, pia mater E) Pia mater, arachnoid mater, subarachnoid space, dura mater
8)
The concept of "immunologically privileged" explains why the CNS
.
A) has a greater immune response than the rest of the body B) has a reduced immune response compared to the rest of the body C) has a completely different set of immune responses D) has more MHC markers than other tissues E) All of thechoices are correct.
9)
Which type of neuroglia has phagocytic capabilities in the CNS?
A) Astrocytes B) Schwann cells C) Macrophages D) Microglial cells E) Neurons
10)
What brain defense restricts substances from entering the brain by the vascular system?
A) Meninges B) Blood-brain barrier C) Macrophages D) Microglia E) Cranium
Version 1
3
11)
The normal biota of the CNS consists of
.
A) Neisseria meningitidis B) herpes simplex I C) herpes simplex II D) Streptococcus agalactiae E) The CNS has nonormal biota.
12)
Meningococcemia is
.
A) best treated with ceftriaxone, penicillin or cefotaxime B) associated with high fever, vascular collapse, hemorrhage, and petechiae C) started from a nasopharyngeal infection D) caused by a gram-negative diplococcus E) All of thechoices are correct.
13)
All of the following pertain to Neisseria meningitidis except
.
A) virulent factors include a capsule, pili, endotoxin, and IgA protease B) causes a serious meningitis C) reservoir is the nasopharynx of human carriers D) more easily transmitted in day care facilities, dorms, and military barracks E) common cause of bacterial pneumonia
14)
Which of the following is a sign of meningitis?
Version 1
4
A) Headache B) Stiff neck C) White blood cells in cerebrospinal fluid D) Fever E) All of thechoices are correct.
15)
Meningococci initially colonize the
.
A) lacrimal ducts B) oral mucosa C) nasopharynx D) eustachian tube E) All of thechoices are correct.
16)
Neisseria meningitidis is transmitted by
.
A) casual contact B) close contact C) intimate contact D) indirect contact E) fomites
17)
Treatment for Neisseria meningitidis consists of
.
A) vancomycin B) penicillin G orally C) penicillin G intravenously D) rifampin E) tetracycline
Version 1
5
18)
Which of the following is not true of Streptococcus pneumoniae?
A) Most frequent cause of community-acquired meningitis B) Has a polysaccharide capsule that resists phagocytosis C) Produces alpha-hemolysin and hydrogen peroxide D) Small gram-positive flattened coccus E) Responds well to penicillin
19)
The virulence of Haemophilus influenzae serotype b is associated with its
.
A) fimbriae B) neurotoxin C) enterotoxin D) capsule E) None of the choices are correct.
20) by
Prophylactic rifampin is indicated for individuals in close contact with patients infected .
A) Neisseria menigitidis B) Listeria monocytogenes C) Cryptococcus neoformans D) Coccidiodes immitis E) Streptococcus agalactiae
21)
Each of the following is true for Listeria monocytogenes except
Version 1
.
6
A) resistant to cold B) fastidious C) resistant to heat D) resistant to salt E) can result in septicemia
22) Which organism is a common cause of meningitis in AIDS patients and can be found in bird droppings?
A) Listeriamonocytogenes B) Haemophilusinfluenzae C) Neisseriameningitidis D) Cryptococcusneoformans E) Streptococcusagalactiae
23)
Cryptococcus neoformans is a
.
A) virus B) bacteria C) prion D) fungus E) helminth
24) Agricultural workers and field archaeologists are exposed to this organism, which, although entering via the respiratory tract, can cause meningitis.
Version 1
7
A) Haemophilusinfluenzae B) Coccidioidesimmitis C) Cryptococcusneoformans D) Streptococcuspneumoniae E) Neisseriameningitidis
25) Arthrospores of Coccidioides immitis develop into into the lungs.
that will release endospores
A) hyphae B) capsules C) spherules D) capsids E) buds
26)
Although many viruses can cause meningitis, the most common viral cause is
.
A) measles B) cytomegalovirus C) arbovirus D) enterovirus E) arenavirus
27)
Neonatal meningitis is most commonly transmitted by
Version 1
.
8
A) breast feeding B) hospital personnel C) transplacental infection D) exposure in birth canal E) exposure from other infants
28)
The organism responsible for the majority of neonatal meningitis is
.
A) Streptococcus agalactiae B) Escherichia coli K1 C) Listeria monocytogenes D) Haemophilus influenzae E) Neisseria meningitidis
29)
Which of the following is not true of meningoencephalitis?
A) Causative organisms are Naegleria fowle ri and Acanthamoeba B) Infections of both the brain and meninges C) Causative organisms are viruses D) Modes of transmission are direct contact and swimming in warm fresh water E) Treatment for primary amoebic meningoencephalitis is mostly ineffective
30)
Which of the following is not a causative agent of acute encephalitis?
A) JC virus B) West Nile virus C) Herpes simplex virus D) Toxoplasmagondii E) California encephalitis
Version 1
9
31)
Encephalitis caused by arboviruses involves
.
A) fever, headache, and rash B) coma, convulsions, and paralysis in severe cases C) myalgia and orbital pain D) muscle aches and joint stiffness E) All of thechoices are correct.
32)
Encephalitis is most commonly caused by a
.
A) bacteria B) protozoan C) virus D) helminth E) All of thechoices are correct.
33)
The best defense against arboviruses is
.
A) prophylactic rifampin B) vector control C) vaccination D) prompt treatment with acyclovir E) All of thechoices are correct.
34)
Which of the following has the highest mortality rate?
Version 1
10
A) Western equineencephalitis B) St. Louisencephalitis C) Californiaencephalitis D) Eastern equineencephalitis E) West Nileencephalitis
35)
Each of the following is true for Toxoplasma gondii except
.
A) is a flagellated parasite B) is often fatal for AIDS patients C) can cause stillbirth D) has a narrow host range E) All of thechoices are true.
36)
Which organism is the definitive host for Toxoplasma gondii?3 A) Cat B) Dog C) Mouse D) Mosquito E) Raccoon
37)
Toxoplasmosis prevention includes
.
A) vaccination of humans B) vaccination of cats C) hygienic precautions D) vector control E) All of thechoices are correct.
Version 1
11
38)
Spongiform encephalopathies are
.
A) associated with abnormal, transmissible protein in the brain B) chronic, fatal infections of the nervous system C) caused by prions D) Creutzfeldt-Jacob disease, kuru, and Gerstmann-Straussler-Scheinker syndrome E) All of thechoices are correct.
39)
Which of the following is not a prion disease?
A) Scrapie B) Creutzfeldt-Jacob disease C) Bovine spongiform encephalopathy D) Subacute sclerosing panencephalitis E) All of thechoices are prion diseases.
40)
Patients with Creutzfeldt-Jacob disease live
.
A) less than 1 year post diagnosis B) 1 to 5 years post diagnosis C) 5 to 10 years post diagnosis D) 10 to 20 years post diagnosis E) greater than 25 years post diagnosis
41)
Subacute encephalitis may be caused by
Version 1
.
12
A) T oxoplasma gondii B) prions C) herpes simplex virus D) persistent measles virus E) All of thechoices are correct.
42)
Which is incorrect about rabies?
A) Is a zoonotic disease B) Wild populations of bats, skunks, raccoons, cats, and canines are primary reservoirs C) Transmission can involve bites, scratches, and inhalation D) Average incubation in human is 1 week E) Symptoms include anxiety, agitation, muscle spasms, convulsions, and paralysis
43)
Treatment of an animal bite for possible rabies includes
.
A) debridement B) washing bite with soap or detergent C) infusing the wound with human rabies immune globulin (HRIG) D) postexposure vaccination with inactive vaccine E) All of thechoices are correct.
44)
All of the following pertain to poliomyelitis except
Version 1
.
13
A) summer outbreaks in the United States have been increasing recently B) transmitted primarily by fecal-contaminated water C) can be asymptomatic or mild with headache, sore throat, fever, and nausea D) if virus enters the central nervous system motor, neurons can be infected and destroyed E) caused by enterovirus
45)
Polio virus initially multiplies in the
.
A) liver B) CNS C) oropharynx and intestine D) nasal mucosa E) None of the choices are correct.
46)
In paralytic polio, what structures are damaged?
A) Peripheral nerves B) Diaphragm C) Anterior horn cells D) Sensory nerves E) Brain stem
47)
The preferred preventative measure for polio in the United States is
A) B) C) D)
Version 1
.
inactivated poliovirus vaccine (IPV) developed by Jonas Salk oral poliovirus vaccine (OPV) developed by Jonas Salk inactivated poliovirus vaccine (IPV) developed by Albert Sabin oral poliovirus vaccine (OPV) developed by Albert Sabin
14
48) Production of a neurotoxin that binds to target sites on spinal cord neurons responsible for inhibiting skeletal muscle contraction is a characteristic of .
A) Clostridium botulinum B) Clostridium perfringens C) Clostridium difficile D) Clostridium tetani E) All of thechoices are correct.
49)
The foodborne disease that involves neurotoxin is
.
A) gastrointestinal anthrax B) bacillus cereus intoxication C) botulism D) clostridium perfringens gastroenteritis E) All of thechoices are correct.
50) Production of a neurotoxin that prevents acetylcholine release from motor neurons at neuromuscular junctions is a characteristic of .
A) Clostridium botulinum B) Clostridium perfringens C) Clostridium difficile D) Clostridium tetani E) All of thechoices are correct.
51)
Which of the following pertains to both tetanus and foodborne botulism?
Version 1
15
A) Occurs when spore-contaminated soil enters deep wounds B) Caused by enterotoxins of the pathogen C) Exotoxin blocks acetylcholine release D) Nausea and diarrhea are symptoms E) Treatment involves antitoxin therapy
52)
Tetanus differs from botulism in that
.
A) tetanus results in flaccid paralysis while botulism paralysis is rigid B) tetanus results from an exotoxin; botulism results from an endotoxin C) there is a vaccine for botulism, but not for tetanus D) muscles cannot relax in tetanus; muscles cannot contract in botulism E) All of thechoices are correct.
53)
Which is the exotoxin of botulism?
A) Hemolysin B) Tetanospasm C) Peroxidase D) Factor V E) Botulinum
54) Which of the following statements regarding the microbiome of the nervous system is true?
Version 1
16
A) In the healthy state, the nervous system is believed to be free from microorganisms. B) In its latent state, the herpesvirus is considered to be part of the microbiome of the nervous system. C) The nervous system develops without any influence from microbes inhabiting other tissues. D) Defense mechanisms of the CNS include competition for nutrients and space between pathogens and the normal microbiota within the tissues.
55)
The Salk IPV and the Sabin OPV differ from one another in that
.
A) the IPV contains a version of the virus that is fully inactivated, whereas the virus within the OPV is attenuated, that is, reduced virulence but still viable B) the IPV must be administered by injection, whereas the OPV can be given orally C) the OPV, being viable, may revert back to the neurovirulent strain and be transmitted to other susceptible individuals, whereas the IPV, being inactivated, cannot revert to a virulent strain D) children in developing countries are administered the OPV, whereas in the USA, children are given the IPV by injection E) All of the choices are correct.
56) Which of the following statements is incorrect regarding the characteristics of Zika virus disease?
A) Individuals who live in mosquito-infested areas can be administered a vaccine as prevention. B) The Zika virus can be transmitted via a mosquito vector, sexually from an infected partner, and vertically from mother to fetus during gestation. C) Females who are infected with Zika during pregnancy are at risk of having a child with microcephaly. D) Adults who have been infected with the Zika virus occasionally go on to develop Guillain-Barre syndrome. E) Zika virus is an RNA virus in the family Flaviviridae.
Version 1
17
57) In an effort to limit transmission of the Zika virus, scientists have genetically modified the Aedes mosquito. Evaluate the statements below and determine which describes the most plausible method to minimize disease.
A) Male Aedes mosquitoes are genetically modified to have a "lethality" gene. They are raised in the lab being fed with the antidote tetracycline. Once released into the wild, they mate with females and the offspring, without access to tetracycline, die before reproducing. B) Male Aedes mosquitoes are genetically modified to contain a gene that codes for a bacterial toxin. Once released into the wild they die before they are able to mate with females. C) Male Aedes mosquitoes are genetically modified to be temperature-sensitive mutants. They are raised in the lab with air-conditioning, but once released into the wild, the heat and humidity in tropical climates causes their enzymes to denature, killing them before they can reproduce. D) Male Aedes mosquitoes are genetically modified to produce a protein that mimics the action of DEET in insect repellants. The males mate with females and during the mating they pass the protein to the female and both mosquitoes are killed in the process.
58) The nervous system can be divided into two parts: the nervous system, consisting of the brain and spinal cord, and the nervous system, which consists of the nerves that emerge from the brain and spinal cord to the sense organs and other parts of the body. A) central; peripheral B) peripheral; central C) motor; sensory D) integrative; sensory E) internal; external
59)
Choose the statement that best describes the normal microbiota of the nervous system.
Version 1
18
A) There are no normal biota present. B) There are limited types of bacteria present, and dormant herpes simplex virus present in the nervous system is considered normal microbiota. C) The Human Microbiome Project is sampling the nervous systemat the present time to study the microbes present in this area. D) The normal microbiota is very similar to that found in the gut.
60) Choose the microorganism that is the most common cause of bacterial meningitis in adults.
A) B) C) D)
Neisseriameningitidis Streptococcuspneumoniae Haemophilus influenzae Listeriamonocytogenes
61) Choose the microorganism that is the causative agent of the most serious acute form of bacterial meningitis to test your understanding of this disease.
A) B) C) D)
Version 1
Listeriamonocytogenes Streptococcuspneumoniae Neisseriameningitidis Haemophilusinfluenzae
19
62) NCLEX Prep—Test Bank Question: Please read the clinical scenario, and then answer the questions that follow to become familiar with the traditional NCLEX question format. A 20-year-old female college student is admitted to the intensive care unit from the emergency department. The patient lives in a dormitory at her university and her roommate brought her to the hospital because “she is not acting like herself.” The patient was previously healthy to the roommate’s knowledge, involved in campus activities, and an excellent student. The RN collects the patient’s history of present illness from her roommate. About two weeks prior, the patient began feeling ill. She had nasal congestion and was diagnosed with a sinus infection at the student health clinic. However, the patient reportedly decided against taking the prescribed antibiotics because she did not believe they were necessary. She developed high fevers and one day ago began complaining of a stiff neck. That morning, she was more lethargic, making nonsensical statements, and had a fever of 105°F. Her roommate was frightened by her change in behavior and drove her to the hospital.
62.1) Based upon the patient’s clinical presentation and course of illness, what is the most likely diagnosis?
A) B) C) D)
Septicemia, secondary to sinus infection Meningitis Tumor of the cervical spine Necrotizing encephalitis
62.2) Based upon the patient’s symptoms, which of the following tests is most important to obtain a diagnosis?
A) B) C) D)
Version 1
CT scan Chest X-ray Complete blood count Lumbar puncture
20
62.3) A lumbar puncture is performed, which confirms a diagnosis of meningitis. The pathology report indicates that the cerebral spinal fluid sample was found to have grampositive cocci in pairs and the patient is diagnosed with pneumococcus. What is the causative organism? A) B) C) D)
Streptococcuspneumoniae Neisseriameningitides Haemophilusinfluenzae Listeriamonocytogenes
62.4)
What is the best measure of prevention for pneumococcus?
A) B) C) D)
Avoidance of crowded living spaces, such as college dormitories Taking antibiotics for any cold symptoms Vaccination with Pneumovax 23 Drink plenty of fluids
62.5) Which antibiotic(s) would the nurse expect the medical team to prescribe to treat pneumococcus?
A) B) C) D)
Version 1
Vancomycin and ceftriaxone Amoxicillin Fluconazole None of the choices are correct.
21
63) NCLEX Prep—Test Bank Question: Please read the clinical scenario, and then answer the questions that follow to become familiar with the traditional NCLEX question format. A 75-year-old female is brought to the emergency department by ambulance after having a seizure at her home. The woman’s son reports she has not been acting normally. She complained of a severe headache and he noted a hand tremor and an unsteady gait. The past few days, she developed a high fever. Upon assessment by the medical team, the woman is not responding verbally. The nurse collects the patient’s history from her son, which is notable for a recent camping trip in the woods.
63.1) What components of the patient’s history are most important for the nurse to collect during the initial triage assessment?
A) B) C) D)
Past medical history, mosquito exposure, allergies, medications Medications, food allergies, religious preference Birth history, childhood development, level of education Mosquito exposure, use of DEET, level of water filtration
63.2) Multiple tests are performed, and the patient is diagnosed with Eastern Equine Encephalitis. What is the best initial treatment regimen?
A) B) C) D)
Acyclovir, antiepileptic drugs, lumbar puncture Lumbar puncture, CT scan, MRI Acyclovir, medical coma, heart catheterization Hydration, antiepileptic drugs, antipyretics
63.3) The RN updates the patient’s son on the plan of treatment. What is the most appropriate antibiotic to treat this illness?
Version 1
22
A) B) C) D)
Vanomycin Cefotaxime Acyclovir No antibiotictherapy is indicated.
63.4) The patient’s son is anxious about his mother’s prognosis. He asks the RN if his mother will survive the infection. What is the best response by the nurse?
A) That’s a good question. You should ask the doctor next time you see him. B) Almost all patients survive this disease. We will do our best to make sure your mother is one of them. C) I’m sorry, but your mother will probably not survive this disease. Would you like to see a chaplain? D) Unfortunately, we see about a 70% mortality rate for this disease. We are doing everything we can do to provide excellent care for your mother.
64) NCLEX Prep—Test Bank Question: Please read the clinical scenario, and then answer the questions that follow to become familiar with the traditional NCLEX question format. An RN is part of a team that has traveled to sub-Saharan Africa to volunteer in a medical clinic. The clinic is in an underdeveloped rural area. A 5-year-old boy is brought into the clinic with a high fever, sore throat, body aches, and weakness. Through the translator, the nurse finds out that the boy has been sick for about a week. He is noted to be barefoot and his mother reports he plays in area ponds to stay cool.
64.1) Which of the following diseases should the nurse be most concerned about, considering the patient’s presentation?
Version 1
23
A) B) C) D)
64.2)
A) B) C) D)
Meningitis Tetanus Polio Typhoid
How is the poliovirus transmitted?
Fecal-oral Arthropod Respiratory droplets Undercooked food
64.3) Serological testing on the patient’s cerebral spinal fluid confirms a diagnosis of polio. What is the best treatment modality?
A) B) C) D)
Antibiotics and fever control Acyclovir and pain control Immunization and respiratory support Pain control and respiratory support
64.4) The RN assists in educating the patient’s mother about the diagnosis through a translator. What is the causative agent for polio?
A) B) C) D)
Version 1
Poliovirus Streptococcuspneumoniae Naegleria fowleri Acanthamoeba
24
65) Pneumovax is a vaccine for prevention of disease caused by strains of Neisseria meningitidis. ⊚ ⊚
66)
Cryptococcal meningitis is highly communicable among humans. ⊚ ⊚
67)
true false
Some cases of CJD may be caused by a mutation of the PrP gene. ⊚ ⊚
70)
true false
Subacute sclerosing panencephalitis is caused by a prion. ⊚ ⊚
69)
true false
Viral meningitis normally requires aggressive antiviral treatment. ⊚ ⊚
68)
true false
true false
Prions can be destroyed by autoclaving. ⊚ ⊚
Version 1
true false
25
71)
In rabies, spasms of muscles for swallowing leads to a fear of water. ⊚ ⊚
true false
72) Up to 50% of persons who survived polio as children will suffer from post-polio syndrome later in life. ⊚ ⊚
true false
73) Tetanus can be prevented with the DTaP childhood immunization and boosters of the tetanus toxoid. ⊚ ⊚
74)
true false
Botulism is often referred to as lockjaw. ⊚ ⊚
true false
75) In both botulism and tetanus, respiratory muscles cannot facilitate breathing and, if untreated, respiratory collapse leads to death. ⊚ ⊚
76)
true false
The oral and inactivated polio vaccines are made from plant cell cultures.
Version 1
26
⊚ ⊚
true false
77) Following vaccination, no cases of attenuated polio virus reverting to a neurovirulent strain have been documented. ⊚ ⊚
true false
78) While the nervous system is believed to be free from microbiota in the healthy state, research has shown that the gut microbiome plays an important role in the development of the nervous system. ⊚ ⊚
true false
79) The CNS is an “immunologically privileged” site, which means that the immune response in these sites is enhanced. ⊚ ⊚
true false
80) The nervous system lacks normal biota, and the presence of any microorganism indicates a deviation from the healthy state. ⊚ ⊚
true false
81) Neonatal meningitis is almost always a result of an infection transmitted by the mother, either in utero or during the birthing process.
Version 1
27
⊚ ⊚
true false
82) The neurotoxin produced by Clostridium tetani blocks the inhibition of muscle contraction resulting in spastic paralysis. ⊚ ⊚
Version 1
true false
28
Answer Key Test name: Ch 17 4e 1) [B, C, E] 2) [A, B] 3) [B, C] 4) [A, C, D] 5) [A, C, E] 6) C 7) B 8) B 9) D 10) B 11) E 12) E 13) E 14) E 15) C 16) B 17) C 18) E 19) D 20) A 21) B 22) D 23) D 24) B 25) C 26) D Version 1
29
27) D 28) A 29) C 30) D 31) B 32) C 33) B 34) D 35) D 36) A 37) C 38) E 39) D 40) A 41) A 42) D 43) E 44) A 45) C 46) C 47) A 48) D 49) C 50) A 51) E 52) D 53) E 54) A 55) E 56) A Version 1
30
57) A 58) A 59) A 60) B 61) C 62) Section Break 62.1) B 62.2) D 62.3) A 62.4) C 62.5) A 63) Section Break 63.1) A 63.2) D 63.3) D 63.4) D 64) Section Break 64.1) C 64.2) A 64.3) D 64.4) A 65) FALSE 66) FALSE 67) FALSE 68) FALSE 69) TRUE 70) FALSE 71) TRUE 72) TRUE 73) TRUE Version 1
31
74) FALSE 75) TRUE 76) FALSE 77) FALSE 78) TRUE 79) FALSE 80) TRUE 81) TRUE 82) TRUE
Version 1
32
CHAPTER 18 1) Select all of the statements that apply to the pathogenesis of malaria to test your understanding of this disease.
A) Fever is constant without the presence of chills. B) Hypoglycemia is a symptom due to parasite metabolism of glucose. C) Damage to white blood cells results in anemia. D) Theliver and spleen become enlarged. E) RBCs adhere to blood vessels in the brain.
2) Select all of the statements that apply to malaria to test your understanding of this disease.
A) An effective vaccine is routinely used throughout the world. B) It is caused by five species of the protozoan Plasmodium. C) The vector is the mosquito. D) The parasite goes through a sexual and asexual phase during development. E) Bed nets soaked in insecticide help reduce risk of infection.
3)
Select all of the statements that apply to HIV and AIDS.
A) An effective vaccine is available to prevent HIV infection. B) HIV can infect and/or destroy many of the very cells needed to combat it, including the helper (T4 or CD4) class of lymphocytes, monocytes, macrophages, and even B lymphocytes. C) Neonates who have escaped HIV infection prior to and during birth can still become infected through nursing. D) In the United States, people are diagnosed with AIDS once they are positive for HIV. E) ART, the use of multiple simultaneous antiretrovirals, has shown to be an effective treatment and has reduced the rate of HIV drug resistance.
Version 1
1
4)
Select all of the phrasesthat apply to subacute endocarditis.
A) Most often causedby Staphylococcus aureus B) Most often causedby low pathogenicity oral streptococci C) Almost alwayspreceded by some form of damage to heart valves D) Involvescolonization of normal heart valves E) Preventioninvolves prophylactic antibiotic therapy before invasive procedures
5) Select the phrases that apply to all of the following nonhemorrhagic fever diseases: brucellosis, Q fever, cat-scratch disease, and Rocky Mountain spotted fever.
A) Caused by agram-negative bacterium B) Most can be treated with doxycycline C) Bacterium causingthe disease is transmitted by a vector D) Vaccines areavailable E) Cause capillary fragility
6)
The fibrous sac that encloses the heart is the
.
A) epicardium B) myocardium C) endocardium D) pericardium E) ectocardium
7)
The presence of viruses in the blood is called
Version 1
.
2
A) viremia B) fungemia C) hemovirus D) bacteremia E) septicemia
8)
occurs when bacteria flourish and grow in the bloodstream. A) Viremia B) Fungemia C) Hemovirus D) Bacteremia E) Septicemia
9)
All of the following are associated with subacute endocarditis except
.
A) occurs in patients that have prior heart damage B) caused by immune system autoantibodies that attack heart and valve tissue C) oral bacteria get introduced by dental procedures to the blood D) bacteria colonize previously damaged heart tissue resulting in a vegetation E) signs and symptoms include fever, heart murmur, and possible emboli
10)
The common causative agent of acute endocarditis is
.
A) Staphylococcus aureus B) Streptococcus pneumoniae C) Streptococcus pyogenes D) Neisseria gonorrhoeae E) All of the choicesare correct.
Version 1
3
11)
Acute endocarditis is most commonly contracted through
.
A) ingestion B) parenteral entry C) casual contact D) droplets E) fomites
12)
Most cases of septicemia are caused by
.
A) fungi B) viruses C) prions D) bacteria E) protozoans
13)
Which of the following is not true of septicemia?
A) Fever and shaking chills B) Respiratory acidosis C) Endotoxic shock D) Parenteral or endogenous transfer E) Drop in blood pressure
14)
Yersinia pestis
Version 1
.
4
A) was virulent in the Middle Ages but is no longer virulent B) has humans as an endemic reservoir C) does not respond to antimicrobial drugs D) is usually transmitted by a flea vector E) All of the choicesare correct.
15)
All of the following are associated with bubonic plague except
.
A) transmitted by human feces B) caused by Yersinia pestis C) patient often has enlarged inguinal lymph nodes D) patient has fever, headache, nausea, and weakness E) can progress to a septicemia
16)
Which is incorrect about Yersinia pestis?
A) Exhibits bipolar staining B) Gram-negative rod C) Transmitted by fleas D) Has a capsule E) Produces enterotoxin
17)
Plague includes
.
A) septicemic form; called Black Death B) bubonic form; buboes develop C) pneumonic form; sputum highly contagious D) disease control; control of rodent population E) All of thechoices are correct.
Version 1
5
18)
Bubonic plague is transmitted by
.
A) mosquitos B) flies C) animal bites D) sexual contact E) fleas
19)
Control of rodent populations is important for preventing
.
A) brucellosis B) plague C) malaria D) Q fever E) All of thechoices are correct.
20)
Which is not associated with tularemia?
A) A zoonosis B) Mammals are the chief reservoir C) A gram-positive bacterium D) Symptoms include fever, swollen lymph nodes, ulcerative lesions, conjunctivitis, and pneumonia E) Sometimes called rabbit fever
21)
Rabbits and other rodents are the reservoirs of the causative agent of
Version 1
.
6
A) tularemia B) anthrax C) malaria D) brucellosis E) mononucleosis
22)
The causative agent of Lyme disease is
.
A) Ixodes scapularis B) Borrelia hermsii C) Borrelia burgdorferi D) Ixodes pacificus E) Leptospira interrogans
23)
Erythema migrans, a bull's-eye rash, at the portal of entry is associated with
.
A) plague B) Rocky Mountain spotted fever C) Q fever D) Lyme disease E) yellow fever
24) The white-footed mouse, deer, and deer ticks are important to maintaining the transmission cycle associated with .
Version 1
7
A) Lyme disease B) yellow fever C) Q fever D) Rocky Mountain spotted fever E) plague
25)
Lyme disease involves
.
A) early symptoms of fever, headache, and stiff neck B) crippling polyarthritis, and cardiovascular and neurological problems C) people having contact with ticks D) treatment with antimicrobials E) All of thechoices are correct.
26)
Lyme disease is transmitted by
.
A) flies B) droplets C) lice D) fleas E) ticks
27)
Epstein-Barr virus has the following characteristics except it
.
A) is more commonly found in adults B) is transmitted by direct oral contact and saliva C) produces sudden leukocytosis D) has a 30- to 50-day incubation E) can be transmitted by contaminated blood transfusions and organ transplants
Version 1
8
28)
Symptoms of infectious mononucleosis include
.
A) vesicular lesions in oral mucosa B) fever and pocks on skin C) sore throat, fever, cervical lymphadenopathy, and splenomegaly D) fever, severe diarrhea, pneumonitis, hepatitis, and retinitis E) None of the choices are correct.
29)
Which of the following requires direct contact with infected body fluids?
A) Yellow fever B) Dengue fever C) Ehrlichiosis D) Lassa fever E) Chikungunya
30)
Which of the following is a hemorrhagic fever?
A) Yellow fever B) Ehrlichiosis C) Q fever D) Rocky Mountainspotted fever E) None of the choices are correct.
31)
Yellow fever and dengue fever are
Version 1
.
9
A) caused by arboviruses B) caused by viruses that disrupt capillaries and blood clotting C) zoonoses D) transmitted by a mosquito vector E) All of the choices are correct.
32)
Which of the following is not true of Ebola and Marburg?
A) Caused by filoviruses B) Disruption of clotting factors C) Transmitted by direct contact with body fluids D) Transmitted by mosquitoes E) There is no treatment for these viruses
33)
Which type of hemorrhagic fever can be treated with ribavirin?
A) Lassa fever B) Ebola C) Marburg D) Dengue fever E) Yellow fever
34) Which type of hemorrhagic fever is also known as "breakbone fever" because of the severe pain in bones?
Version 1
10
A) Lassa fever B) Ebola C) Marburg D) Dengue fever E) Yellow fever
35)
The reservoir for Lassa fever is the
.
A) monkey B) rat C) gorilla D) cat E) cattle
36)
Brucellosis is
.
A) a zoonosis B) seen in the patient as a fluctuating fever, with headache, muscle pain, and weakness C) also known as undulant fever D) an occupational illness of people who work with animals E) All of thechoices are correct.
37)
Pasteurization of milk helps to prevent
.
A) tularemia B) plague C) endocarditis D) brucellosis E) mononucleosis
Version 1
11
38)
The gram-negative bacillus associated with abscesses from cat bites or scratches is .
A) Salmonella typhimurium B) Yersinia enterocolitica C) Bartonella henselae D) Brucella suis E) Francisella tularensis
39)
Rocky Mountain spotted fever
.
A) is seen in highest numbers along the west coast B) is transmitted by Ixodes ticks C) symptoms include fever, headache, and rash D) never has severe complications E) All of thechoices are correct.
40)
Which is incorrect regarding Q fever?
A) Transmitted by lice B) Pathogen produces resistant spores C) Humans infected from unpasteurized milk and airborne spread D) Causes fever, muscle aches, rash, and sometimes pneumonia E) Is a zoonosis
41)
Which of the following is mismatched?
Version 1
12
A) Yersinia pestis—plague B) Coxiella burnetii—Q fever C) Brucellosis melitensis—undulating fever D) Bartonella henselae—cat-scratch disease E) Rickettsia typhi—Rocky Mountain spotted fever
42)
Cat-scratch fever can be prevented by
.
A) avoiding ticks B) pasteurizing milk C) vaccine D) animal control E) cleaning the scratch wound
43)
The symptoms that occur in cyclic 48- to 72- hour episodes in a malaria patient are . A) bloody, mucus-filled stools, fever, diarrhea, and weight loss B) fever, swollen lymph nodes, and joint pain C) urinary frequency and pain, and vaginal discharge D) chills, fever, and sweating E) sore throat, low-grade fever, and swollen lymph nodes
44)
Which is incorrect about malaria?
Version 1
13
A) Merozoites enter and multiply in liver cells. B) Trophozoites develop in RBCs. C) The mosquito inoculates human blood with sporozoites. D) Gametocytes enter the mosquito as she draws a blood meal from a human. E) Cerebral malaria is a potential serious complication.
45)
The cyclic bouts of fever and chills in malaria are caused by
.
A) liver cell lysis B) white blood cell lysis C) red blood cell lysis D) neurological involvement E) None of the choices are correct.
46)
Malaria may be prevented by
.
A) using bed nets sprayed with insecticide B) eliminating standing water C) taking prophylactic drugs D) decreasing the mosquito population E) All of thechoices are correct.
47)
The causative organism of malaria is a
.
A) bacterium B) virus C) fungus D) protozoan E) prion
Version 1
14
48)
Characteristics of Bacillus anthracis include all the following except
.
A) capsule and exotoxins are virulence factors B) a one-time vaccine provides lifelong immunity C) reservoir includes infected grazing animals and contaminated soil D) gram-positive bacillus E) spore former
49)
Anthrax is
.
A) a zoonosis B) transmitted by contact, inhalation, and ingestion C) a disease that, in humans, can cause a rapidly fatal toxemia and septicemia D) only seen sporadically in the United States E) All of thechoices are correct.
50)
A common cardiovascular/lymph system disease in AIDS patients is
.
A) acute endocarditis B) Burkitt's lymphoma C) herpes simplex virus D) ehrlichioses E) brucellosis
51)
Retroviruses have the following characteristics except
Version 1
.
15
A) glycoprotein spikes B) DNA genome C) enveloped D) reverse transcriptase E) viral genes integrate into the host genome
52)
All of the following pertain to patients with AIDS except they
.
A) have an immunodeficiency B) have CD4 T-cell titers below 200 cells/mm 3 of blood C) get repeated, life-threatening opportunistic infections D) can get unusual cancers and neurological disorders E) have the highest number of cases worldwide in the United States
53)
Documented transmission of HIV involves
.
A) mosquitoes B) unprotected sexual intercourse and contact with blood/blood products C) respiratory droplets D) contaminated food E) All of thechoices are correct.
54)
All of the following pertain to HIV except
.
A) attaches primarily to host cells with CD4 receptors B) initial infection often associated with vague symptoms C) becomes latent in host cells D) ELISA and Western blot tests detect HIV antigens E) can enter into nervous tissues and cause abnormalities
Version 1
16
55) Which drugs interfere with the action of an HIV enzyme needed for final assembly and maturation of the virus?
A) Reverse transcriptase inhibitors B) Protease inhibitors C) Fusion inhibitors D) Integrase inhibitors E) All of thechoices are correct.
56)
Which new class of drugs will interfere with docking onto host cells?
A) Reverse transcriptase inhibitors B) Protease inhibitors C) Fusion inhibitors D) Integrase inhibitors E) All of thechoices are correct.
57)
A frequent cancer that is seen in AIDS patients is
.
A) leukemia B) Hodgkin's lymphoma C) Kaposi's sarcoma D) melanoma E) myeloma
58) Which of the statements is incorrect regarding the similarities and differences between the cardiovascular and lymphatic systems?
Version 1
17
A) Although the cardiovascular and lymphatic systems run virtually parallel to each other, blood and lymphatic fluid never mixes. B) Both the cardiovascular system and the lymphatic system comprise a series of vessels that run roughly parallel to each other. C) The cardiovascular system is a closed circuit, whereas the lymphatic system directs fluid in one direction only. D) The cardiovascular system has a pump (the heart) to circulate blood, whereas the lymphatic system has no pumping mechanism.
59) While the cardiovascular system can be breached by microorganisms, mechanisms are in place to prevent infection. These include .
A) a close association with the lymphatic system which screens and filters the blood B) blood vessels that are impermeable to microorganisms C) toxic chemicals that will kill any microbes that enter the bloodstream D) a connective tissue fibrous mesh that filters the blood within the vessels, preventing pathogens from circulating
60)
Since the cardiovascular and lymphatic systems are not directly accessible to microbes .
A) it was initially believed that the systems lacked normal biota, however the Human Microbiome Project suggests that low levels of microbes may exist in the healthy state B) both systems are sterile in the healthy state C) the abundant microbiota enter via the gastrointestinal tract then cross the blood vessels D) the Human Microbiome Project confirmed that only pathogens exist in both systems
61)
Which of the following statements is true concerning the epidemiology of malaria?
Version 1
18
A) Malaria transmission is restricted to an area encompassing the equator, largely due to control of mosquito populations. B) Malaria is an endemic disease throughout the world. C) The majority of new malaria cases each year are in South America. D) The elderly are the most at risk population for death from malaria.
62)
Which of the following statements is false concerning the epidemiology of malaria?
A) B) C) D)
63)
Malaria has been eradicated from the United States. The majority of malaria cases are restricted to a band encompassing the equator. Africa sees the highest incidence of malaria in the world. Children and young adults are the most at-risk population for malaria.
Which of the following is true regarding HIV cases in the United States?
A) B) C) D)
HIV in the United States represents approximately 3.2% of cases worldwide. The % of new HIV cases is increasing each year. The majority of newly-diagnosed cases in 2015 were in the Latino population. Heterosexual transmission of HIV is decreasing in the United States.
64) Nowadays, HIV is more of a chronic disease than the death sentence it was when it first emerged in the 1980s. In addition to lifesaving medications, education and testing has also lowered the number of new cases annually. This means that in general .
A) B) C) D)
Version 1
incidence is decreasing and prevalence is increasing prevalence is decreasing and incidence is increasing incidence and prevalence are both increasing incidence and prevalence are both decreasing
19
65) Bacillus anthracis exhibits characteristics that make it a model organism to be exploited as an agent of bioterrorism. Those characteristics include .
A) the ability to form endospores, allowing it to be stored for long periods of time without nutrients or water B) the ability to infect and cause disease in several body systems including the lungs, cardiovascular system and gastrointestinal tract C) the ability to produce a tripartite toxin and polypeptide capsule as virulence factors D) difficulty diagnosing and therefore treating because the manifestations mimic other, more common diseases E) All of the choices reflect characteristics that make B. anthracis a candidate for bioterrorism.
66) The blood is full of leukocytes, such as , which are responsible for adaptive immunity, and various phagocytes, which are critical to both innate and adaptiveimmune responses. A) B) C) D)
lymphocytes mast cells neutrophils natural killer cells
67) Choose the statement that describes the normal biota of the cardiovascular system in a healthy state to test your understanding of the cardiovascular system.
A) B) C) D)
Version 1
Many types of microorganisms colonize the cardiovascular system. Only a few types of bacteria colonize the cardiovascular system. No microorganisms colonize the cardiovascular system. No microbes have ever been found to be present in the cardiovascular system.
20
68)
Choose the phrase that correctly describesacute endocarditis.
A) B) C) D)
Most often causedby Staphylococcus aureus Most often causedby oral cavity Streptococci Most oftenpreceded by some form of damage to the heart valve Even untreatedthis condition is not fatal
69) Choose the phrase that best describes the virulence factor shared by the yellow fever, Dengue fever, Ebola, Marburg, and Lassa fever viruses to test your understanding of hemorrhagic fever diseases.
A) Associated withlysis of red blood cells B) Associated withagglutination of red blood cells C) Associated withdisruption of clotting factors D) Associated withactivation of the complement system E) Associated with production of antihemoglobin antibodies
70)
Hemorrhagic fever diseases such as yellow fever, Dengue fever, and Ebola are caused by , whereas nonhemorrhagic fever diseases such as brucellosis, Q fever, and cat-scratch fever have as the causative agents of disease. A) B) C) D)
71)
viruses; bacteria bacteria; viruses bacteria; protozoans viruses; protozoans
Choose the phrase that best describes septicemia.
Version 1
21
A) B) C) D)
Occurs whenmicroorganisms are actively multiplying in the blood Occurs whenmicroorganisms are present in the blood Occurs whenbacterial toxins are found in the blood Occurs whenphagocytic cells release microbe debris into the blood
72) In anthrax, Bacillus anthracis gains access to the bloodstreamwhere it multiplies in large numbers resulting in death from an overwhelming . A) B) C) D)
skin infection septicemia state of dehydration viremia
73) NCLEX Prep—Test Bank Question: Please read the clinical scenario, and then answer the questions that follow to become familiar with the traditional NCLEX question format. A nurse is volunteering with a medical team in Southeast Asia. A 35-year-old female presents to the clinic reporting that the village doctors have diagnosed her with malaria. The nurse notes that the woman is febrile, tachypneic, and her eyes have a yellow tint. She reports generalized body aches and weakness. Through a translator, the nurse collects the woman’s medical history. She reports feeling ill for about ten days, experiencing several cycles of fever, sweating, and chills that are temporarily relieved before occurring again. The village doctors treated her with a combination of a mosquito paste and an indigenous root, but her symptoms have not improved. The medical team performs serological testing at the clinic and confirms a diagnosis of malaria.
73.1)
Version 1
What is the cause of the cyclical nature of the woman’s symptoms?
22
A) B) C) D)
She is being reinfected by a vector as she begins to recover. The village doctor’s treatments are working intermittently. Her symptoms reflect the synchronous rupture of RBCs. Her liver is failing in stages, and the symptoms coincide.
73.2) If the medical team had the ability to perform more thorough blood tests, what would the nurse expect to find?
A) B) C) D)
Hyperglycemia, increased RBC count Hyperglycemia, decreased RBC count Hypoglycemia, increased RBC count Hypoglycemia, decreased RBC count
73.3) Which of the following medications would the nurse expect to be ordered for this patient?
A) B) C) D)
Chloroquine Vancomycin Artemisinin Acyclovir
73.4) If pharmacological treatment is not successful, what may be the ultimate cause of death?
A) B) C) D)
Version 1
Malaremia Multiorgan failure Seizures Respiratory arrest
23
73.5) The nurse provides education to the woman’s family regarding malaria prevention. All of the following are recommended, except .
A) B) C) D)
washing all bedding and clothing in hot water eliminating areas of stagnant water use of bed nets prophylactic antimalarials
74) NCLEX Prep—Test Bank Question: Please read the clinical scenario, and then answer the questions that follow to become familiar with the traditional NCLEX question format. A pregnant 25-year-old woman with known HIV infection presents to an obstetric office for a 20-week prenatal visit. This is her first pregnancy and thus far it has been uncomplicated. The patient was first diagnosed as HIV-positive seven years prior and she has remained asymptomatic. She has been compliant with her antiretroviral therapies. The patient reports that the father of her child is not HIV-positive.
74.1) The patient asks the nurse what the risk is of her child being HIV-positive. What is the best response by the nurse?
A) The child will be HIV-positive and there is no way to prevent transmission. B) Since HIV is passed through the father, the child will be HIV negative. C) It is possible that HIV will be passed on to your child, but you can decrease the likelihood of transmission by compliance with antiretroviral therapy. D) Fetal immunity will ward off HIV during pregnancy, but the child should not breastfeed.
74.2) The patient states that she acquired HIV from a previous partner. What are the primary modes of HIV transmission?
Version 1
24
A) B) C) D)
Saliva and blood Sexual contact and blood Sexual contact and body fluids Blood and other body fluids
74.3) What is the most successful treatment during pregnancy to reduce transmission of HIV from mother to fetus?
A) B) C) D)
Pre-exposure prophylaxis Protease inhibitors Reverse transcriptase inhibitors Fusion inhibitors
74.4) When should the patient's newborn baby be tested for HIV, assuming she does not breastfeed? A) B) C) D)
At birth 1 month of age At birth and 1 month of age Every 6 months for 3 years
74.5) The patient asks the nurse for information about preventing HIV transmission to her sexual partner. What is the best form of prevention for HIV transmission, assuming sexual abstinence is not an option?
A) B) C) D)
Version 1
Use of barrier-method contraceptive Contraceptive drugs Prophylactic treatment of all sexual partners with antiviral chemotherapy Compliance with antiretroviral regimen
25
75) NCLEX Prep—Test Bank Question: Please read the clinical scenario, and then answer the questions that follow to become familiar with the traditional NCLEX question format. A 13-year-old girl with cerebral palsy is admitted to the pediatric surgical unit following a posterior spinal fusion. During the procedure, rods were implanted to straighten her spine. Per the orthopedic surgical team’s report, the patient tolerated the procedure well. Her post-operative vital signs were stable. 24 hours after she arrives to the inpatient unit, her blood pressure is 70/35, heart rate is 150 beats per minute, temperature is 104°F, and respiratory rate is 30 breaths per minute. She is difficult to arouse and is not responding to questions. The nurse calls for emergent assistance. Upon removing the dressing, it is noted that her incision is swollen, red, and oozing a cloudy white substance.
75.1) Which of the following is the most likely cause of the acute change in the patient’s status?
A) B) C) D)
75.2)
A) B) C) D)
Version 1
Localized infection Bacterial septicemia Postoperative hemorrhagic fever Fungemia
What is the most likely cause of septicemia?
Gram-positive bacteria Gram-negative bacteria Gram-positive and gram-negative bacteria Fungus
26
75.3) The medical team suspects that the patient has septicemia. Boluses of isotonic fluid are given and vasoactive medications are ordered to increase her blood pressure. Which of the following are the most important next interventions?
A) B) C) D)
Respiratory support, blood cultures, and antibiotics Wound cleansing, antipyretics, and antibiotics Maintenance intravenous fluids, blood cultures, and antibiotics Respiratory support, wound cleansing, and antipyretics
75.4) Preliminary testing on the blood cultures reveals the presence of gram-positive cocci. Although the final organism has not been identified, which of the following organisms is the most likely cause of the patient’s septicemia? A) B) C) D)
Staphylococcusaureus Streptococcuspneumoniae Haemophilus influenzae Neisseria gonorrhoeae
75.5) When the final results of the blood culture return three days later, the patient is diagnosed with S. aureus septicemia. Empiric antibiotic therapy was initiated with her initial clinical decompensation. What is the next step in the patient’s treatment regimen?
A) B) C) D)
76)
Continue initial empiric therapy as cessation could encourage resistance Change antibiotic coverage as indicated by susceptibility testing Discontinue antibiotics as three days of treatment is standard Continue empiric coverage until clinical symptoms improve
The cardiovascular and lymphatic systems have limited or no normal biota. ⊚ ⊚
Version 1
true false
27
77)
Plague is a zoonotic disease. ⊚ true ⊚ false
78)
Lyme disease is only seen in people living in Lyme, Connecticut. ⊚ ⊚
79)
A vaccine for Lyme disease is available. ⊚ ⊚
80)
true false
true false
Aedes mosquitoes are the vectors involved in yellow fever and dengue fever. ⊚ ⊚
true false
81) In severe cases of Rocky Mountain spotted fever, the enlarged lesions of the rash can become necrotic and predispose the patient to gangrene of toes and fingertips. ⊚ ⊚
true false
82) The highest numbers of cases of Rocky Mountain spotted fever occur along the eastern seaboard.
Version 1
28
⊚ ⊚
83)
true false
Rifampin is the drug of choice for malaria. ⊚ ⊚
true false
84) Under normal healthy circumstances the lymphatic system filters any microorganisms present. ⊚ ⊚
85)
true false
The cardiovascular and lymphatic systems are both circulatory in nature. ⊚ ⊚
true false
86) The lymphatic system is a major defense system in the body; it renders surveillance, recognition, and protection against foreign materials through a system of lymphocytes, phagocytes, and antibodies. ⊚ ⊚
true false
87) Due to the efficiency of the lymphatic system and the inaccessibility to microbes, data from the Human Microbiome Project confirmed that the cardiovascular system lacks microbiota in the healthy state. ⊚ ⊚
Version 1
true false
29
88) Of the 200 million new cases of malaria each year, 10% are in areas that roughly span the equator outside of the African continent. ⊚ ⊚
true false
89) The greatest rise in new HIV infections throughout the world is in the homosexual male population. ⊚ ⊚
true false
90) Anthrax is easy to diagnose and treat because the manifestations are localized to a single tissue and specific in nature. ⊚ ⊚
91)
true false
Both innate and adaptive immune mechanisms protect the cardiovascular system. ⊚ ⊚
true false
92) Like the nervous system, the cardiovascular and lymphatic systems do not havenormal access to microbes from the external environment. ⊚ ⊚
Version 1
true false
30
93) Spread of HIV through heterosexual contact has significantly increased as compared to data from the start of the epidemic. ⊚ ⊚
true false
94)
Once T-cell levelsdrop below 500 cells/μL, AIDS results. ⊚ true ⊚ false
95)
Hemorrhagic diseases have fever as a symptom whereas nonhemorrhagic diseases do not. ⊚ ⊚
true false
96) The vast majority of septicemias are caused by bacteria, therefore treatment should begin with broad-spectrum antibiotics. ⊚ true ⊚ false
97) Anthrax is a good potential bioterrorism weapon because Bacillus anthracis is a sporeforming bacterium that initiates disease symptoms that mimic many other infections. ⊚ ⊚
Version 1
true false
31
Answer Key Test name: Ch 18 4e 1) [B, D, E] 2) [B, C, D, E] 3) [B, C, E] 4) [B, C, E] 5) [A, B] 6) D 7) A 8) E 9) B 10) E 11) B 12) D 13) B 14) D 15) A 16) E 17) E 18) E 19) B 20) C 21) A 22) C 23) D 24) A 25) E 26) E Version 1
32
27) A 28) C 29) D 30) A 31) E 32) D 33) A 34) D 35) B 36) E 37) D 38) C 39) C 40) A 41) E 42) E 43) D 44) A 45) C 46) E 47) D 48) B 49) E 50) B 51) B 52) E 53) B 54) D 55) B 56) C Version 1
33
57) C 58) A 59) A 60) A 61) A 62) A 63) A 64) A 65) E 66) A 67) C 68) A 69) C 70) A 71) A 72) B 73) Section Break 73.1) C 73.2) D 73.3) A 73.4) B 73.5) A 74) Section Break 74.1) C 74.2) B 74.3) B 74.4) C 74.5) A 75) Section Break 75.1) B Version 1
34
75.2) C 75.3) A 75.4) A 75.5) B 76) TRUE 77) TRUE 78) FALSE 79) FALSE 80) TRUE 81) TRUE 82) TRUE 83) FALSE 84) TRUE 85) FALSE 86) TRUE 87) FALSE 88) TRUE 89) FALSE 90) FALSE 91) TRUE 92) TRUE 93) TRUE 94) FALSE 95) FALSE 96) TRUE 97) TRUE
Version 1
35
CHAPTER 19 1) Select all of the phrases that apply to healthcare-associatedpneumonia to test your understanding of the differences between healthcare-associated and community-acquired pneumonia.
A) Often arepolymicrobial in origin B) Bacteria enterlower respiratory tract due to abnormal breathing C) Bacteria enterlower respiratory tract due to mechanical ventilation D) Low mortalityrate E) Can be caused by coliform bacteria
2) Choose the fungi that cause pneumonia to test your understanding of microorganisms that cause lower respiratory tract diseases.
A) Streptococcuspneumoniae B) Legionella sp. C) Mycoplasmapneumoniae D) Histoplasmacapsulatum E) Pneumocystis jiroveci
3) Select all of the factors that can affecta person’s susceptibility to TB to test your understanding of the epidemiology of tuberculosis infection.
A) Presence ofmosquito vector B) Inadequatenutrition C) Poor access tomedical care D) Crowded livingconditions E) Compromised immune system
4)
Which of the following is not part of the upper respiratory tract?
Version 1
1
A) Mouth B) Nasal cavity C) Trachea D) Pharynx E) Larynx
5)
Which of the following is not part of the lower respiratory system?
A) Alveoli B) Bronchi C) Bronchioles D) Trachea E) Sinuses
6)
What features of the respiratory system protect us from infection?
A) Nasal hairs B) Cilia C) Mucus D) Macrophages E) All of thechoices are correct.
7)
Which antibody is concentrated in the respiratory tract?
A) IgA B) IgE C) IgG D) IgM E) All of thechoices are correct.
Version 1
2
8)
Normal biota of the upper respiratory tract include
.
A) Aspergillis B) Candida albicans C) Haemophilus influenzae D) Staphylococcus aureus E) All of thechoices are correct.
9)
Normal biota of the upper respiratory tract include
.
A) Streptococcus pyogenes B) Streptococcus pneumoniae C) Neisseria meningitidis D) alpha-hemolytic streptococci E) All of thechoices are correct.
10)
Normal biota of the lower respiratory tract include
.
A) Corynebacterium B) Candida albicans C) Haemophilus influenzae D) Staphylococcus aureus E) None of the choices are correct.
11)
What is the function of normal biota of the respiratory tract?
Version 1
3
A) Compete with pathogens for resources and space B) Microbial antagonism C) Provide antibodies D) Both compete withpathogens for resources and space and microbial antagonism are correct. E) Compete with pathogens for resources and space, microbial antagonism, andprovide antibodies are all correct.
12)
The most common type of virus leading to rhinitis is
.
A) coronavirus B) adenovirus C) rhinovirus D) herpes simplex virus E) retrovirus
13)
Cold viruses are transmitted by
.
A) droplet contact and indirect contact B) endogenous C) direct contact D) None of the choices are correct. E) All of the choices are correct.
14)
The most common mode of transmission for sinusitis is
Version 1
.
4
A) exotoxin B) direct contact C) indirect contact D) endogenous E) droplet contact
15)
Infection travels to the middle ear by way of the
.
A) auditory canal B) sinuses C) nasal cavity D) eustachian (auditory) tubes E) subarachnoid space
16)
The most common causative agent of otitis media is
.
A) Aspergillus fumigatus B) Haemophilus influenzae C) Candida albicans D) Corynebacterium E) Streptococcus pneumoniae
17)
Streptococcal pharyngitis involves all of the following except
.
A) a purulent exudate over the tonsils B) it can lead to scarlet fever if it is an erythrogenic toxin-producing strain C) it can lead to serious sequelae D) it is viral in origin E) it causes difficulty in swallowing and fever
Version 1
5
18)
The bright red rash and fever of scarlet fever are due to
.
A) enterotoxins B) hemolysins C) toxic shock syndrome toxin D) exfoliative toxin E) erythrogenic toxin
19)
Rheumatic fever is caused by
.
A) a primary streptococcal infection with Streptococcus pyogenes B) a combination of infection with Streptococcus pyogenes and the rheumatic fever virus C) a secondary streptococcal infection with a different type of streptococcus D) an immunological cross-reaction in which anti-streptococcal antibodies target the heart E) both a primary viral infection and a secondary streptococcal infection
20)
Superantigens
.
A) include streptolysin O B) include erythrogenic toxin C) induce production of tumor necrosis factor D) can initiate a cascade of immune responses E) All of the choicesare correct.
21)
Untreated streptococcal pharyngitis can lead to all of the following except
Version 1
.
6
A) scarlet fever B) pertussis C) rheumatic fever D) glomerulonephritis E) erythrogenic rash
22)
Virulence factors of Streptococcus pyogenes include
.
A) polysaccharides on the cell wall B) lipoteichoic acid C) M proteins D) hyaluronic acid capsule E) All of the choicesare correct.
23)
Which of the following is not true of Streptococcus pyogenes?
A) Group A streptococcus B) Gram positive C) Forms spores D) Sensitive to bacitracin E) Beta hemolytic
24)
Pertussis has the following characteristics except
.
A) the disease progresses through several distinct stages B) the catarrhal stage has persistent, hacking coughs with "whoops" C) the early stage resembles a cold with nasal discharge and sneezing D) DTaP immunization will prevent it E) transmission is by droplet contact
Version 1
7
25)
The causative organism of whooping cough is
.
A) Bordetella pertussis B) Streptococcus pneumoniae C) Haemophilus influenzae D) Streptococcus pyogenes E) Corynebacterium diphtheriae
26)
Which of the following is not true of whooping cough? A) Secondary infections can occur during the convalescent phase B) Caused by a small gram-negative coccobacillus C) Incubation period is 3–21 days D) Coldlike symptoms are common for the paroxysmal stage E) An uncontrollable cough occurs during the paroxysmal stage
27)
Which of the following is not a virulence factor of Bordetella pertussis?
A) Endotoxin B) Tracheal cytotoxin C) M protein D) Pertussis toxin E) Filamentous hemagglutinin
28)
Respiratory syncytial virus
Version 1
.
8
A) causes serious disease in infants 6 months old or younger B) is highly contagious C) is transmitted through droplet contact and fomite contamination D) cannot be prevented by vaccination at this time E) All of thechoices are correct.
29) The enzyme associated with the influenza virus that hydrolyzes the protective mucous coating of the respiratory tract is .
A) catalase B) reverse transcriptase C) hyaluronidase D) neuraminidase E) kinase
30)
Symptoms of influenza include
.
A) nasal discharge, mild fever, and absence of cough B) fever, diarrhea, and vomiting C) fever, myalgia, sore throat, cough, and nasal discharge D) fever, sore throat, rash, and cough E) fever and pneumonia
31)
Influenza vaccines include all the following except
Version 1
.
9
A) an inactivated dead virus B) the incorporation of usually three different strains C) an attenuated nasal drops vaccine D) the ability to provide lifelong immunity E) an overall effectiveness of 70%−90%
32)
Which of the following is not true of influenza?
A) It has glycoprotein spikes of hemagglutinin (H) and neuraminidase (N). B) Antigenic drift which changes the antigens to reduce immune recognition. C) Antigenic shift causes changes in the RNA strands. D) Symptoms are very similar to the common cold. E) The virus can infect both humans and swine.
33)
Which is not correct about Mycobacterium species?
A) All species are human pathogens. B) Cell walls have waxy lipids. C) They are acid fast. D) Usually have a slow growth rate. E) They are long, slender rods.
34)
All of the following pertain to tuberculosis except
Version 1
.
10
A) live bacilli can remain dormant in the lungs and reactivate later in life B) symptoms of active TB include low-grade fever, coughing, fatigue, weight loss, and night sweats C) lung infection can disseminate to many other organs in extrapulmonary TB D) the BCG vaccine is used in other countries E) antimicrobials cannot treat and cure tuberculosis
35)
Tuberculin skin testing
.
A) injects PPD intradermally B) uses a purified protein filtrate of Mycobacterium tuberculosis C) will be positive if the person has had previous exposure to TB D) will be positive if the person has active TB E) All of thechoices are correct.
36)
A diagnosis of tuberculosis involves a(n)
.
A) chest X-ray B) acid-fast stain of sputum C) sputum culture D) tuberculin skin test E) All of thechoices are correct.
37) Which of the following causes a noncommunicable pulmonary infection that resembles tuberculosis?
Version 1
11
A) Mycobacteriumavium complex (MAC) B) Mycobacteriumkansasii C) Mycobacteriumscrofulaceum D) Mycobacteriumfortuitum complex E) Mycobacteriummarinum
38)
The tubercles formed in primary tuberculosis are caused by an influx of
.
A) neutrophils B) basophils C) mononuclear cells D) polymorphonuclear leukocytes E) antibodies
39)
Extrapulmonary TB can involve the
.
A) kidneys B) long bones C) spine D) brain E) All of thechoices are correct.
40) When the centers of tubercles break down into calcification that replaces normal lung tissue.
Version 1
lesions, they gradually heal by
12
A) primary B) secondary C) necroticcaseous D) granuloma E) tertiary
41)
Streptococcus pneumoniae is
.
A) alpha-hemolytic on blood agar B) often referred to as pneumococcus C) a gram-positive diplococcus with a capsule D) most likely to cause infection in older adults E) All of thechoices are correct.
42)
Legionella pneumophila
.
A) is easily killed by chlorine B) often lives intracellularly in amoebas C) is transmitted through the soil D) always causes a severe pneumonia E) is a gram-positive bacterium
43) Which disease involves transmission by aerosolized water from whirlpool spas, air conditioners, cooling towers, and supermarket vegetable misters?
Version 1
13
A) Legionellosis B) Pertussis C) Brucellosis D) Plague E) Traveler's diarrhea
44)
Characteristics of Histoplasma capsulatum include
.
A) it grows in moist soil; rich in nitrogen from bird and bat droppings B) it is a yeast with a capsule C) it is commonly found in the southwestern United States D) it always results in severe infections E) its transmission occurs through water droplets
45)
The highest incidence of histoplasmosis in the United States occurs in the
.
A) Southeast B) Southwest C) East and Midwest D) Northeast E) Rocky Mountains
46)
Inhalation of respiratory secretions is involved in the transmission of the following except .
Version 1
14
A) influenza B) rhinitis C) legionella D) COVID-19 E) All of the choices involve respiratory secretions.
47)
Hantavirus has all of the following characteristics except
.
A) a reservoir is human carriers B) it is a zoonotic infection C) the symptoms are abrupt fever, lung edema, respiratory distress, and hypotension D) it is transmitted by aerosol transmission from rodent excreta E) the first U.S. outbreak occurred in 1993 in the Southwest
48) This opportunist is the most frequent cause of life-threatening pneumonia in AIDS patients. A) Cryptococcusneoformans B) Candidaalbicans C) Malasseziafurfur D) Pneumocystis(carinii) jiroveci E) None of the choices are correct.
49)
Influenza virus can exhibit constant mutation of viral glycoproteins called , or , a more serious phenomenon caused by the exchange of a viral gene with that of another influenza virus strain.
Version 1
15
A) antigenic shift; antigenic drift B) antigenic drift; antigenic shift C) None of these are correct.
50)
Which of the following pairs of defense mechanisms is mismatched?
A) Complement activation - third line of defense B) Ciliated epithelium - first line of defense C) Secretory IgA - third line of defense D) Antimicrobial peptides - second line of defense E) Coughing - first line of defense
51) Assess the following statements and determine which one is incorrect regarding antigenic shift.
A) The influenza virus genome mutates over time changing the amino acid sequence of the envelope glycoproteins. Each encounter with a new version of the virus requires activation of naive lymphocytes rather than memory cells, which is the reason we receive a new vaccination each year. B) The influenza genome contains 10 genes distributed on 8 separate RNA strands. These strands can be exchanged between different viruses. C) Some virus strains infect birds and swine, some infect swine and humans. If the strain that infects swine and humans exchanges strands with a bird flu strain while in a swine host cell, that virus can go on to infect a human who will have no recognition for the bird protein. D) If a new strain of virus emerged from an antigenic shift event, a vaccine different to the annual flu vaccine would have to be administered because the proteins coded for by the new strain would be different to those made as a result of antigenic drift.
52)
The main contributing factor to the evolution of MDR-TB is
Version 1
.
16
A) the slow growth of Mycobacterium tuberculosis requires antibiotic therapy for months rather than days, which is hard to sustain in some populations; cutting the regimen short allows the resistant bacteria to flourish B) the organisms that develop multidrug resistance have a different cell wall composition to the drug-susceptible organisms C) certain species of Mycobacterium tuberculosis are able to undergo bacterial transformation at a faster rate than other species, resulting in the transfer of resistance genes D) the abundance of antibiotics available in the United States and other developed countries makes it a particular problem for resistant strains to emerge in those parts of the world
53)
Which of the following is the first (uppermost) structure of the lower respiratory tract?
A) Larynx B) Trachea C) Bronchus D) Bronchioles E) Alveoli
54) Often followinga bout of the common cold, is an inflammatory condition of the sinuses that can result from a mixed infection due to the excellent growth media provided by the trapped inflammatory fluids. A) B) C) D)
sinusitis otitis media pharyngitis diphtheria
55)
The influenza virus is able to evade the immune response by two mechanisms: antigenic , which is the swapping out of one of the gene strands with a gene strand from a different influenza virus, and antigenic , in which the antigens gradually change their amino acid composition.
Version 1
17
A) shift; drift B) drift; shift C) switch; drift
56)
Choose the phrase that best describes the normal biota of the upper respiratory tract.
A) Limited to a fewtypes of gram-positive bacteria B) Is considered to be an area void of normal biota C) Contains avariety of microbes including some that can cause serious disease D) Contains avariety of microbes but not any that can cause serious disease E) Contains mainly harmless viruses andfungi
57) NCLEX Prep—Test Bank Question: Please read the clinical scenario, and then answer the questions that follow to become familiar with the traditional NCLEX question format. An RN in a primary care office is caring for a 10-year-old patient experiencing fever, chills, nasal congestion, and a sore throat.The patient’s mother indicates that the child had been ill for the past two days, experiencing a poor appetite and sleepiness.The nurse swabs the patient’s throat for rapid microbial analysis, suspecting the patient’s symptoms are due to a common pathogen.
57.1) The rapid microbial analysis is negative for group A Streptococcus and reveals the presence of only normal biota in the patient specimen.Based upon this information, all of the following microbes can be ruled out in the diagnosis except .
A) B) C) D)
Version 1
Streptococcus pyogenes Staphylococcus aureus Haemophilus influenzae Influenza A
18
57.2) Suspecting the patient is infected with influenza, the nurse prepares to obtain which of the following specimens for rapid testing at an external laboratory?
A) B) C) D)
Blood Cerebrospinal fluid Nasal swab Skin scraping
57.3) The laboratory findings report that the child is infected with influenza B virus.Which of the following medications would the nurse expect to be ordered for this child?
A) B) C) D)
Penicillin No medication Acyclovir Oseltamivir
58) The primary transmission route for acquiring agents of the common cold is by contact with hands and fomites contaminated with the nasal discharges of an infected person. ⊚ ⊚
59)
Secondary bacterial infections may occur with the common cold. ⊚ ⊚
60)
true false
true false
Antibiotic treatment for otitis media is standard procedure today.
Version 1
19
⊚ ⊚
61)
true false
Pertussis outbreaks still occur in the United States. ⊚ ⊚
true false
62) Respiratory syncytial virus is a major cause of global respiratory infection outbreaks in infants 6 months of age or younger. ⊚ ⊚
63)
true false
Influenza is predominantly transmitted by the fecal-oral route. ⊚ ⊚
true false
64) People who have received the BCG immunization will generally have a negative tuberculin skin test. ⊚ ⊚
true false
65)
An induration of less than 5 mm in the Mantoux test is negative for TB. ⊚ true ⊚ false
66)
The major reservoir for the hantavirus is the deer mouse.
Version 1
20
⊚ ⊚
67)
Legionellosis is a zoonosis. ⊚ ⊚
68)
true false
true false
A paroxysmal cough is associated with Legionnaires' disease. ⊚ ⊚
true false
69) Medical professionals actually observe MDR-TB patients take their medicine through a program called DOT. ⊚ ⊚
70)
true false
Anatomical features situated below the tonsils are considered the lower respiratory tract. ⊚ ⊚
true false
71) Antigenic drift differs from antigenic shift in that antigenic drift reflects the exchange of RNA strands between different virus strains within a common host cell, whereas antigenic shift is the gradual mutation over time of the glycoproteins on the viral envelope that bind to immune cells. ⊚ ⊚
Version 1
true false
21
72) Extensively drug-resistant tuberculosis, while harder to treat because some antibiotics are ineffective, has a similar prognosis and recovery rate compared to nonresistant tuberculosis. ⊚ ⊚
true false
73) To reduce the threat to public health, patients with MDR-TB are treated with directly observed therapy. ⊚ ⊚
true false
74) The respiratory system is protected from invading microorganisms by only first-line (innate) immune defenses. ⊚ ⊚
true false
75) The upper and lower respiratory tract, due to constant contact with the external environment, possesses a large amount of normal microbiota. ⊚ ⊚
true false
76) The influenza virus is able to decrease the effectiveness of the host immune response by a mechanism called antigenic drift, which involves the swapping of gene strands with other influenza viruses. ⊚ ⊚
Version 1
true false
22
77) Tuberculosis is a disease of poverty since susceptibility is increased in the face of inadequate nutrition and poor access to medical care. ⊚ ⊚
Version 1
true false
23
Answer Key Test name: Ch 19 4e 1) [A, B, C, E] 2) [D, E] 3) [B, C, D, E] 4) C 5) E 6) E 7) A 8) E 9) E 10) E 11) D 12) C 13) A 14) D 15) D 16) E 17) D 18) E 19) D 20) E 21) B 22) E 23) C 24) B 25) A 26) D Version 1
24
27) C 28) E 29) D 30) C 31) D 32) D 33) A 34) E 35) E 36) E 37) A 38) C 39) E 40) C 41) E 42) B 43) A 44) A 45) C 46) C 47) A 48) D 49) B 50) A 51) A 52) A 53) B 54) A 55) A 56) C Version 1
25
57) Section Break 57.1) D 57.2) C 57.3) D 58) TRUE 59) TRUE 60) FALSE 61) TRUE 62) TRUE 63) FALSE 64) FALSE 65) TRUE 66) TRUE 67) FALSE 68) FALSE 69) TRUE 70) FALSE 71) FALSE 72) FALSE 73) TRUE 74) FALSE 75) FALSE 76) FALSE 77) TRUE
Version 1
26
CHAPTER 20 1)
Select all of the accessory organs of the gastrointestinal system that add digestive fluids.
A) Stomach B) Salivary glands C) Liver D) Gallbladder E) Pancreas
2) Select all of the correct tissues that make up GALT to test your understanding of the defenses present in the gastrointestinal tract.
A) Tonsils and adenoids in the oral cavity B) Lymphoid tissues in the esophagus C) Peyer’s patches in the small intestine D) Appendix E) Gallbladder
3) Select all of the correct descriptions concerning the normal biota of the gastrointestinal tract.
A) B) C) D)
4)
Contains billions of microorganisms Provides a protective function Often disrupts the digestive process Helps educate the immune system
Select all of the correct statements about dental caries (tooth decay).
Version 1
1
A) Tooth decay is the most common infectious disease of humans. B) Decay is due to metabolic action of the bacteria. C) Dietary proteins must be present for the decay process to take place. D) Staphylococcus species are the main cause of dental caries. E) Symptoms of tooth decay are always noticeable.
5)
Select the correct descriptions of mumps, a gastrointestinal tract disease.
A) Inflammation of the salivary glands B) Caused by a DNA herpesvirus C) No vaccine available D) Humans are the only host for the virus E) No complications associated with the disease
6) Select the correct statement(s) concerning gastric ulcers to test your understanding of gastrointestinal tract diseases caused by microorganisms.
A) The cause is Helocobacter pylori, a gram-positive coccus. B) They are transmitted through the oral-oral route and the fecal-oral route. C) Lysozyme secretion by the bacteria is a protective adaptation. D) Diagnosis is through endoscopy. E) Antibiotic treatment canhelp treat the underlying bacterial infection.
7) Select all of the correct answers concerning the disease schistosomiasis to test your understanding of GI tract diseases caused by Schistosoma sp.
Version 1
2
A) Chronic infection results in liver disease, splenomegaly, and hepatomegaly. B) Schistosomes are trematodes often referred to as blood flukes. C) Schistosomiasis follows a Cycle A mode of transmission. D) Fluke cloaks itself with host proteins to evade the immune response. E) Fish are an intermediate host in the life cycle.
8)
Defenses of the GI tract against pathogens include
.
A) mucus, acid, and saliva B) secretory IgA and lysozyme C) peristalsis D) GALT E) All of thechoices are correct.
9)
The mucinous glycoprotein covering teeth that streptococci attach to is called the .
A) enamel B) pellicle C) gingiva D) lactoferrin E) oral membrane
10)
Which organ of the GI tract has a large commensal population of microorganisms?
Version 1
3
A) Liver B) Salivary glands C) Pancreas D) Large intestine E) Small intestine
11)
The most common infectious disease in humans is
.
A) the common cold B) dental caries C) pharyngitis D) diarrhea E) gastritis
12)
The predominant causative organism of dental caries seem to be
.
A) Streptococcus pyogenes B) Streptococcus mutans C) Streptococcus pneumoniae D) Staphylococcus epidermidis E) Staphylococcus aureus
13)
Oral biota on the tooth surface
Version 1
.
4
A) develop a biofilm B) use fimbriae and slime layers to adhere C) include Streptococci that metabolize sucrose, produce sticky glucans, and form plaque D) include Lactobacilli and Streptococci,which ferment carbohydrates and produce acids that cause caries E) All of the choices are correct.
14)
The causative organism for mumps is
.
A) Paramyxovirus B) Morbillivirus C) Streptococcus pyogenes D) Corynebacterium E) Vibrio
15)
The virulence factor for mumps is
.
A) inflammation B) acid production C) endotoxin D) enterotoxin E) spike-induced syncytium
16)
Orchitis and epididymitis are seen in young adult males as a complication of
Version 1
.
5
A) croup B) mumps C) influenza D) measles (rubeola) E) rubella
17)
Helicobacter pylori causes
.
A) gastritis B) duodenal ulcers C) stomach ulcers D) increased risk for stomach cancer E) All of thechoices are correct.
18)
Which is not a characteristic of Helicobacter pylori?
A) Gram negative B) Produces enteroxin that causes diarrhea C) Curved rods D) Lives in the stomach E) Produces urease that buffers stomach acidity
19)
Which of the following is mismatched?
A) H antigen -flagellar B) K antigen - capsule C) O antigen - cellwall D) All of thechoices are correct. E) None of the choices are correct.
Version 1
6
20)
Salmonella are
.
A) methane producers B) motile C) gram-positive rods D) lactose fermenters E) All of thechoices are correct.
21)
Salmonelloses are
.
A) not caused by Salmonella typhi B) associated with undercooked poultry and eggs, and reptile and rodent feces C) types of gastroenteritis with vomiting and diarrhea D) treated with fluid and electrolyte replacement E) All of thechoices are correct.
22)
Which is incorrect about shigellosis?
A) Outbreaks have occurred in day care centers B) Watery diarrhea with bloody, mucoid stools and abdominal cramps C) It frequently involves septicemia D) Human carriers cause fecal transmission E) It is also called dysentery
23)
Shigella species have the following characteristics except
Version 1
.
7
A) they are nonmotile, gram-negative rods B) the reservoir is human carriers C) produce urease and hydrogen sulfide D) they infect the large intestine E) they produce enterotoxin called shiga toxin
24)
E. coli O157:H7 characteristics include all the following except it
.
A) only causes occupational illness in people who work with animals B) is transmitted by ingestion of contaminated, undercooked food, especially hamburger C) causes a bloody diarrhea D) has a reservoir of cattle intestines E) in some cases goes on to hemolytic uremic syndrome (HUS) with possible kidney failure
25) Which of the following statements describes the main virulence factor of E. coli O157:H7?
A) A pathogen-encoded exotoxin B) A bacteriophage-encoded exotoxin C) An endotoxin used for host attachment D) A coagulase for host clotting E) A hemolysin for red blood cell destruction
26)
Escherichia coli infections
Version 1
.
8
A) are often transmitted by fecal-contaminated water and food B) have been due to undercooked meat C) involve enterotoxin in traveler's diarrhea D) can beself-limiting with the only treatment being rehydration E) All of thechoices are correct.
27)
All of the following are characteristics of Campylobacter jejuni except it
.
A) is transmitted through ingestion of contaminated chicken, meat, milk, and water B) causes fever and a watery stool C) infects the stomach D) produces an enterotoxin that stimulates diarrhea E) has gram-negative curved rods with darting motility
28)
The most common bacterial cause of diarrhea in the United States is
.
A) E. coli B) Salmonella C) Shigella D) Campylobacter E) Yersinia
29)
Campylobacter infection has all of the following characteristics except it
.
A) can last over 2 weeks B) can burrow into the mucosa of the ileum and multiply C) can lead to Guillain-Barre syndrome D) can cause acute temporary paralysis E) is caused by a shiga toxin
Version 1
9
30)
Production of enterotoxin is a characteristic of
.
A) Clostridium botulinum B) Clostridium perfringens C) Clostridium difficile D) Clostridium tetani E) All of the choicesare correct.
31)
All of the following pertain to Clostridium difficile infection except it
.
A) is due to ingestion of contaminated, improperly stored, cooked meats and gravies B) is a colitis that is a superinfection C) often has an endogenous source D) is associated with disruption of normal flora due to broad-spectrum antimicrobials E) is the major cause of diarrhea in hospitals
32)
Which is not true of Clostridium difficile?
A) A gram-positive, endospore forming rod B) Part of normal intestinal biota C) Infection precipitated by broad-spectrum antibiotic therapy D) Also called pseudomembranous colitis E) Produces "rice-water" stools
33)
The virulence of Vibrio cholerae is due to its
Version 1
.
10
A) capsule B) neurotoxin C) invasive enzymes D) enterotoxin E) All of the choicesare correct.
34)
Cholera symptoms include
.
A) copious watery diarrhea B) loss of blood volume C) acidosis, sunken eyes, and thirst D) hypotension, tachycardia, and shock E) All of the choicesare correct.
35) is
The most immediate and important treatment needed to prevent death in cholera victims .
A) water and electrolyte replacement B) antimicrobials C) antitoxin D) surgery E) None of the choices are correct.
36)
"Rice-water stools" are associated with disease caused by which organism?
Version 1
11
A) Vibriovulnificus B) Vibrioparahaemolyticus C) Vibriocholerae D) Campylobacterjejuni E) Helicobacterpylori
37) Outbreaks of which disease have been associated with fecal-contaminated drinking water?
A) Cryptosporidiosis B) Cyclosporiasis C) Trichinosis D) Toxoplasmosis E) None of the choices are correct.
38)
Signs and symptoms of cryptosporidiosis include
.
A) headache, sweats, vomiting, severe abdominal cramps, and diarrhea B) chills, fever, and sweats C) bloody, mucus-filled stools and fever D) a red-skin papule that spreads to a large ulcer E) None of the choices are correct.
39)
The primary viral cause of chronic diarrhea, globally, is
Version 1
.
12
A) cryptosporidium B) rotavirus C) adenovirus D) norovirus E) astrovirus
40)
Which of the following is not a normal causative agent of food poisoning?
A) Clostridiumperfringes B) Staphylococcusaureus exotoxin C) Clostridiumdifficile D) Bacilluscereus E) All of the choicesare correct.
41)
A common food intoxication is caused by enterotoxin-producing strains of
.
A) Staphylococcus aureus B) Staphylococcus epidermidis C) Staphylococcus saprophyticus D) Streptococcus pyogenes E) Streptococcus agalactiae
42)
Which is incorrect about Staphylococcus aureus food intoxication?
Version 1
13
A) Food gets contaminated by a human carrier B) Common associated foods include custards, ham, cream pastries, and processed meats C) After contamination, food must be left unrefrigerated for a few hours D) Ingestion of the pathogen allows it to multiply and damage the GI tract lining E) Symptoms come on quickly and include nausea, vomiting, and diarrhea
43) Outbreaks of which protozoan disease have been associated with fecal-contaminated imported raspberries and also fresh greens and drinking water?
A) Cryptosporidiosis B) Cyclosporiasis C) Trichinosis D) Toxoplasmosis E) None of the choices are correct.
44)
Entamoeba histolytica is acquired by A) B) C) D)
45)
.
ingestion of cysts of the pathogen contaminated food contaminated water All of thechoices are correct.
All of the following can be transmitted by drinking contaminated water except
Version 1
.
14
A) cryptosporidiosis B) cyclosporiasis C) malaria D) campylobacter E) giardiasis
46)
Giardiasis involves
.
A) a protozoan that does not form cysts B) infection by a bacteria C) symptoms of abdominal pain, flatulence, and diarrhea D) fimbriae for adherence E) All of thechoices are correct.
47) Inflammation of the liver with necrosis of the hepatocytes and swelling due to a mononuclear response is a disease called .
A) hepatitis B) jaundice C) liver cancer D) mononucleosis E) None of the choices are correct.
48)
Hepatitis B virus
Version 1
.
15
A) is principally transmitted by blood B) transmission risks include shared needles, anal intercourse, and heterosexual intercourse C) is transmitted to the newborn from chronic carrier mothers D) has many chronic carriers E) All of thechoices are correct.
49)
Hepatitis B infection
.
A) has an incubation of 2 to 7 weeks B) can be transmitted by the fecal-oral route C) increases risk for hepatocellular cancer D) is responsible for most cases of posttransfusion hepatitis E) All of thechoices are correct.
50)
Which is incorrect about hepatitis A infection?
A) Transmitted by the oral-fecal route B) Flulike symptoms with discomfort near the liver and darkened urine C) Predisposes a person for liver cancer D) Immunoglobulin therapy helps decrease the severity E) HAVRAX is an inactivated vaccine for prevention
51)
Which is mismatched?
Version 1
16
A) Hepatitis A virus – nonenveloped, single-stranded RNA enterovirus B) Hepatitis B virus – enveloped DNA virus C) Hepatitis C virus – RNA virus D) Hepatitis D virus – defective RNA virus that coinfects with HAV E) Hepatitis E virus – self-limiting RNA virus
52)
The most common cause of liver cancer in the United States is
.
A) HAV B) HBV C) HCV D) HDV E) HEV
53)
Parasitic helminths have the following characteristics except
.
A) are multicellular animals B) have a definitive host where the adult form lives C) do not exhibit drug resistance D) include roundworms E) include tapeworms
54)
Ascaris lumbricoides
.
A) is an intestinal roundworm B) spreads to humans by ingestion of Ascaris eggs in food C) larvae penetrate into lymphatics and capillaries around intestines D) larvae migrate to the pharynx, get swallowed, and return to intestines to mature E) All of thechoices are correct.
Version 1
17
55)
Enterobius vermicularis is
.
A) a whipworm B) common only to the tropics and subtropics C) an intestinal worm that easily contaminates fingers and fomites D) often fatal in heavy infestations E) All of thechoices are correct.
56)
All of the following pertain to tapeworms except
.
A) scolex is the head B) strobila is made up of proglottids C) proglottids are reproductive segments D) cysticerci are young infective larvae ingested in undercooked meat E) eosinophilia does not occur
57)
General symptoms of helminth infection include all of the following except
.
A) eggs, larvae, or adult worms found in feces B) increased sensitivity to helminth antigens C) intense abdominal pain D) increased eosinophil count E) vague nausea
58)
Trichinosis includes all the following except
Version 1
.
18
A) encysted larvae get ingested B) associated with undercooked pork or bear meat C) migrate from intestines to blood and various body tissues D) coiled larvae encyst in skeletal muscle E) All of thechoices are correct.
59)
Liver flukes live in all of the following areas except the
.
A) small intestine B) liver C) gallbladder D) bile duct E) All of the choicesare correct.
60)
Which of the following is not true of schistosomiasis?
A) It is caused by blood flukes B) Larvae called cercariae can invade intact skin C) It may cause liver swelling or malfunction D) Worms can invade the brain E) It can "cloak" itself with proteins in the blood
61) Which of the following helminths is paired with correct characteristics of its pathogenesis?
Version 1
19
A) Ascaris lumbricoides - life and transmission cycleB B) Necator americanus - life and transmission cycleA C) Trichuris trichuria - life and transmission cycleB D) Strongyloides stercoralis - life and transmissioncycle B E) Taenia solium - life and transmission cycle A
62)
All of the following are components of the digestive tract except the
.
A) kidneys B) large intestine C) mouth D) esophagus E) rectum
63)
Which of the following is not considered an accessory organ to the gastrointestinal tract?
A) Stomach B) Liver C) Salivary glands D) Pancreas E) Gallbladder
64) Certain molecules contribute to the first line of defense in the gastrointestinal tract, including .
A) B) C) D)
Version 1
lysozyme and lactoferrin complement and interleukin-1 interferon and complement histamine and leukotriene
20
65) Which of the following statements is incorrect regarding the microbiome of the gastrointestinal tract?
A) The gut microbiome, while playing an important role in gastrointestinal function, does not influence other systems in the body. B) Many organisms of the microbiome are commensals but others are mutualistic; they synthesize important molecules, such as vitamins, that humans cannot make themselves. C) The GI microbiome plays a role in "teaching" the immune system how to recognize and react to foreign cells. D) The gut microbiome plays a protective role by preventing the growth of pathogens through microbial antagonism.
66)
Which of the following is incorrect regarding the microbiome of the stomach?
A) The high acid content of the stomach prevents any microbes from colonizing the organ. B) Researchers for the Human Microbiome Project revealed the molecular signatures of over 100 microbial species in the stomach. C) Microbes including Staphylococcus, Clostridium and Bacillus are known to be part of the stomach microbiome. D) Helicobacter pylori, the organism known to cause stomach ulcers and cancer, is a known member of our stomach microbiome.
67) Evaluate the following statements and choose the one that best explains why infection with Taenia solium is one of the five neglected parasitic infections in the United States.
Version 1
21
A) Thousands of Latinos are thought to be infected with T. solium but is is underreported because American physicians are unaware of the signs and symptoms. B) There is no treatment for T. solium so the infected individuals are neglected. C) There are so few cases of T. solium in the United States, so those individuals that are infected are ignored. D) Because infection with T. solium is not a deadly disease, physicians rarely treat those that are infected.
68)
Choose the best description ofthe normal microbiota of the oral cavity.
A) B) C) D)
69)
Very large, diversepopulation that includes only bacteria Very large butnondiverse population of just a few species Very large population composed of only streptococci Very large, diversepopulation that includes bacteria, fungi, and protozoans
Choose the statement that describes the normal biota of the stomach. A) B) C) D)
The stomach isconsidered to be sterile. The stomach issparsely populated by just a few bacteria species. There are over 100different species of microorganisms living in the stomach. The normal biota of the stomach is similar to that of the large intestine.
70) Choose the statement that describes rotavirus infections to test your understanding of GI tract diseases.
Version 1
22
A) The virus is transmitted by the oral-oral route. B) The virus has an unusual double-stranded RNA genome. C) No vaccine is available for rotavirus. D) Infection leads to mild disease without hospitalization. E) Newborns are the most susceptible to infection.
71)
Choose the statement that describes the best method of prevention of dental caries.
A) Dietary restriction of carbohydrates B) Regular brushing and flossing C) Adding fluoride to the water system D) Chewing sugarless gum E) Use of probiotics
72) infections. A) B) C) D)
73)
, an increase in certain granular leukocytes, is a hallmark of helminthic
Eosinophilia Leukopenia Leukocytosis Neutrophila
Choose the phrase that does not describe helminths.
A) Possess a complex neurological system B) Specialized mouthparts for attaching to host tissues C) Possess a protective cuticle D) Release enzymes to penetrate tissues E) May be acquired by the fecal-oral route
Version 1
23
74) NCLEX Prep - Test Bank Question: Please read the clinical scenario, and then answer the questions that follow to become familiar with the traditional NCLEX question format. A nurse is caring for an 18-month-old girl in the pediatric intensive care unit. She was admitted following three days of bloody diarrhea and fever. Her parents report she had fewer wet diapers than normal. She was admitted with hyperkalemia, as well as increased blood urea nitrogen (BUN) and creatinine levels, indicating acute renal failure. After further lab work, the toddler is diagnosed with hemolytic uremic syndrome (HUS), possibly caused by drinking unpasteurized milk.
74.1)
A) B) C) D)
What organism is the leading cause of HUS in the United States?
Shigella dysenteriae Enteroinvasive Escherichia coli Shiga-toxin-producing Escherichia coli Clostridium difficile
74.2) The child’s parents are noticeably distressed. They ask the nurse what could have been done to prevent their child from contracting this illness. Which of the following responses by the nurse is most appropriate? A) While there are some individuals who advocate for health benefits of consuming unpasteurized milk, the risks of acquiring organisms such as pathogenic E. coli, Salmonella, Listeria, and Campylobacter, among others, outweighs any benefit. B) You should consider having your daughter vaccinated with the E. coli vaccine. C) Next time your daughter exhibits these symptoms, bring her in to the emergency department sooner to prevent this severe illness. D) Since nobody knows the true cause of HUS, there’s nothing you could have done to prevent this from happening. E) You should seek out a safer supplier of unpasteurized milk.
Version 1
24
74.3)
A) B) C) D)
Which of the following clinical manifestations are diagnostic of HUS?
Hemolytic anemia Thrombocytopenia Acute kidney failure All of the choices are correct.
74.4) The child’s parents ask the nurse to explain why their child has not received antibiotics. Which of the following statements are true about the use of antibiotic therapy in HUS?
A) Antibiotics are of no help and may increase the pathology B) There are no antibiotics that can effectively treat E. coli C) Antibiotic therapy may stunt the immune system response D) Antibiotics are reserved for the most virulent cases of HUS, specifically patients receiving dialysis
74.5)
Why does a patient with HUS present with bloody diarrhea?
A) The blood is actually from the kidneys, which are in failure B) The diarrhea is red in color because the body is shedding shiga toxin C) Shiga toxin damages the gut lining, causing it to bleed D) E. coli attaches to small bowel outpouchings and because of underlying thrombocytopenia, causes bleeding
Version 1
25
75) NCLEX Prep - Test Bank Question: Please read the clinical scenario, and then answer the questions that follow to become familiar with the traditional NCLEX question format. An inner city medical clinic provides medical screening and basic care for low-income individuals and families. A nurse completes an intake assessment for a 37-year-old homeless male. He denies current intravenous drug use and presents for hepatitis, HIV, and STD screening. He is found to be positive for hepatitis B.
75.1) The patient asks how he could have contracted this infection. Which of the following are possible modes of hepatitis B transmission?
A) B) C) D)
Needle stick Sexual contact Sharing a toothbrush All of thechoices are correct.
75.2) The patient asks what treatment is available for hepatitis infections. What is the best response by the nurse?
A) Curing hepatitis is challenging and expensive. What kind of support structure do you have in place to help you through this? B) There are many treatment modalities available. You should talk to our doctors right away about starting treatment. C) Right now, the only treatable hepatitis is hepatitis C, but new modes of treatment are being developed and studied every day. D) Hepatitis can be well controlled with lifestyle change. If we can figure out how you contracted the disease, we can figure out how to cure it.
75.3)
Version 1
Which test(s) is (are) used to detect hepatitis B?
26
A) B) C) D)
Radioimmunoassay ELISA Antibody test All of thechoices are correct.
75.4) After drawing the patient’s blood for hepatitis B testing, the healthcare worker set the needle on a counter instead of disposing of it in a sharps container. Several hours later, another healthcare worker accidentally stuck herself with the needle. How concerned should she be that she could contract hepatitis B?
A) B) C) D)
76)
In the absence of dietary carbohydrates, bacteria do not cause tooth decay. ⊚ ⊚
77)
true false
Mumps causes permanent sterility in young male adults. ⊚ ⊚
78)
She should not beconcerned at all. The virus dies once the blood dries. She should not be concerned at all. The virus is not transmitted via blood. She should be very concerned. The virus can survive several days in dried blood. She should be very concerned. The virus multiplies indefinitely on hard surfaces.
true false
E. coli O157:H7 secretes shiga exotoxin. ⊚ ⊚
Version 1
true false
27
79)
Diarrhea caused by ⊚ ⊚
80)
E. coli O157:H7 is treated with antibiotics.
true false
The best treatment for acute diarrhea is oral replacement of electrolytes and water. ⊚ ⊚
true false
81) Only feces from humans that contaminate food and water can be involved in transmission of amoebiasis. ⊚ true ⊚ false
82)
Boiling will not kill Giardia lamblia cysts in contaminated water. ⊚ ⊚
83)
Carriers of hepatitis B virus are not common. ⊚ ⊚
84)
true false
true false
There are vaccines for immunity to the hepatitis B virus. ⊚ ⊚
Version 1
true false
28
85)
The only body fluid that can transmit hepatitis B virus is blood. ⊚ ⊚
86)
Most antihelminth medications act only against the worms, not the human host. ⊚ ⊚
87)
true false
true false
There are no infectious agents that can invade intact skin. ⊚ ⊚
true false
88) Research does not indicate a link between the numbers and types of bacteria causing periodontitis and thicker carotid arteries in an individual. ⊚ ⊚
true false
89) Despite acidic conditions, some microorganisms have been found residing within the human stomach. ⊚ ⊚
true false
90) The gallbladder, while not strictly a component of the GI tract, is considered an accessory organ since it aids digestion by secreting necessary fluid and enzymes into the system.
Version 1
29
⊚ ⊚
true false
91) Natural defenses of the GI tract include mechanical factors, such as peristalsis, and chemical factors, such as secretory IgA on the surface of the intestines. ⊚ ⊚
true false
92) Helicobacter pylori is an acidophile that causes stomach ulcers and cancer; because it is a pathogen it is not part of our stomach microbiome. ⊚ ⊚
true false
93) Cysticercosis is common in Latin America. The increasing population of Latinos in the United States has brought with it an increase in T. solium infection, so American physicians can now rapidly diagnose and treat all cases. ⊚ ⊚
true false
94) The gastrointestinal tract is thought of as a long tube starting from the mouth and ending with the anus. ⊚ ⊚
true false
95) The gastrointestinal tract is protected by gut-associated lymphoid tissue (GALT), which confers both innate and adaptive immune defenses against invaders.
Version 1
30
⊚ ⊚
true false
96) Bacteria appear transiently in the stomach as they travel through the GI tract, but there are no normal biota members associated with the stomach. ⊚ ⊚
true false
97) Evidence suggests that people with high numbers of the bacteria associated with periodontitis may also have increased rates of cardiovascular disease. ⊚ ⊚
true false
98) Dental diseases, in particular periodontal disease, provide excellent models of diseases mediated by single organisms. ⊚ ⊚
true false
99) Hepatitis C is known as the "silent epidemic" for more than 3.5 million Americans are infected with this virus; however, infection rates in the United States and abroad are decreasing due to the implementation of a successful vaccine. ⊚ ⊚
true false
100) Chronic infection by Schistosoma mansoni and Schistosoma japonicum results in liver disease and splenomegaly but other schistosomes can cause bladder obstruction and blood in the urine.
Version 1
31
⊚ ⊚
Version 1
true false
32
Answer Key Test name: Ch 20 4e 1) [B, C, D, E] 2) [A, B, C, D] 3) [A, B, D] 4) [A, B] 5) [A, D] 6) [B, D, E] 7) [A, B, D] 8) E 9) B 10) D 11) B 12) B 13) E 14) A 15) E 16) B 17) E 18) B 19) D 20) B 21) E 22) C 23) C 24) A 25) B 26) E Version 1
33
27) C 28) D 29) E 30) C 31) A 32) E 33) D 34) E 35) A 36) C 37) A 38) A 39) B 40) C 41) A 42) D 43) B 44) D 45) C 46) C 47) A 48) E 49) C 50) C 51) D 52) B 53) C 54) E 55) C 56) E Version 1
34
57) C 58) E 59) A 60) D 61) D 62) A 63) A 64) A 65) A 66) A 67) A 68) D 69) C 70) B 71) A 72) A 73) A 74) Section Break 74.1) B 74.2) A 74.3) D 74.4) A 74.5) C 75) Section Break 75.1) D 75.2) B 75.3) D 75.4) C 76) TRUE 77) FALSE Version 1
35
78) TRUE 79) FALSE 80) TRUE 81) FALSE 82) FALSE 83) FALSE 84) TRUE 85) FALSE 86) FALSE 87) FALSE 88) FALSE 89) TRUE 90) TRUE 91) TRUE 92) FALSE 93) FALSE 94) TRUE 95) TRUE 96) FALSE 97) TRUE 98) FALSE 99) FALSE 100) TRUE
Version 1
36
CHAPTER 21 1) Select all of the correct descriptions of the protective characteristics of urine to test your understanding of the natural defenses of the genitourinary tract.
A) Has acidic pH B) Contains lysozyme, which breaks down peptidoglycan C) Contains lactoferrin, an iron-binding protein D) Contains secretoryIgA E) Contains mucus
2) Select all of the correct statements concerning the female genital tract to test your understanding of the normal biota present in the genitourinary tract.
A) Throughout life, the pH of the vagina is approximately pH 7. B) The vagina, cervix, and ovaries are sterile areas. C) After puberty, estrogen production leads to glycogen release in the vagina lowering the pH. D) After puberty, lactobacilli dominate discouraging growth of other microbes. E) After puberty, lactobacilli dominate encouraging growth of other microbes.
3) Select all of the statements that apply to gonorrhea to test your understanding of genitourinary tract diseases characterized by discharge.
A) It is caused by Neisseria gonorrhoeae, a gram-negative diplococcus. B) Infection can render both males and females sterile. C) It can result in eye and respiratory infections of neonates during birthing process. D) Penicillin is used for treatment. E) Condoms are ineffective at reducing transmission.
Version 1
1
4) Select all of the statements that apply to Chlamydia infection to test your understanding of genitourinary tract diseases characterized by discharge.
A) It is caused by Chlamydia trachomatis, a large bacterium. B) The causative agent is an obligate intracellular parasite. C) The causative agent has two stages: elemental body and reticulate body. D) The causative agent has two stages: cercaria and miracidium. E) Neonate eye infections with Chlamydia are more prevalent than those associated with gonorrhea.
5) Select all of the correct descriptions of vaginitis to test your understanding of vaginal diseases caused by microorganisms.
A) Most commonlycaused by Candida albicans B) Characterized byvaginal itching and burning but no discharge C) Most commonlycaused by Gardnerella D) Discharge withfishy odor E) Most infections are opportunistic
6)
Select all of the sequelae associated with group B Streptococcus infections in infants.
A) Meningitis B) Pneumonia C) Impaired senses D) Club feet or missing digits E) Developmentaldisabilities
7)
Which of the following is not a defense of the genitourinary tract?
Version 1
2
A) Flushing action of urine B) Lysozyme C) IgA D) IgG E) Mucus secretions
8)
Normal biota of the urethra include all of the following except
.
A) nonhemolytic Streptococcus B) Staphylococcus C) Corynebacterium D) Escherichia coli E) Lactobacillus
9)
Lactobacillus in the female reproductive tract
.
A) is protective B) is indicative of underlying infection C) is the causative agent in common yeast infections D) can lead to pelvic inflammatory disease E) can contribute to STDs
10) The predominant microorganism in the female reproductive tract during childbearing years is .
Version 1
3
A) Corynebacterium B) Staphylococcus C) Escherichia coli D) Lactobacillus E) Candida albicans
11) Which of the following is not a characteristic of the normal biota of the female reproductive tract during childbearing years?
A) Vaginal pH is neutral B) Estrogen causes glycogen release C) Lactobacilli convert sugars to acid D) Candida albicans is present in small amounts E) Secretory IgA provides protection
12)
The most common causative agent of urinary tract infections is
.
A) Escherichia coli O157:H7 B) Escherichia coli C) Staphylococcus aureus D) Streptococcus pyogenes E) Pseudomonas aeruginosa
13)
All of the following are signs and symptoms of urinary tract infections except
Version 1
.
4
A) red blood cells in urine B) painful urination C) white blood cells in urine D) nausea E) diarrhea
14)
Which of the following organisms causes urinary tract infections?
A) Proteusmirabilis B) Schistosomahaematobium C) Treponemapallidum D) Group B Streptococcus E) Gardnerella
15)
Infection of the urinary bladder is called
.
A) urethritis B) pyelonephritis C) cystitis D) vaginitis E) PI
16)
The most common mode of disease transmission in UTIs is
.
A) fomites B) indirect contact C) opportunism D) aerosol E) endogenous transfer
Version 1
5
17)
The best prevention for UTIs is
A) B) C) D)
18)
.
to avoid contact hygiene to flush tract with alcohol to consume a high-carbohydrate diet
Infected animals shed Leptospira
interrogans in their
.
A) feces B) blood C) urine D) respiratory secretions E) saliva
19)
Leptospirosis has all of the following characteristics except
.
A) most common in cattle, horses, pigs, and dog B) pathogen is a spirochete C) infects kidneys, liver, brain, and eyes D) humans acquire it by contact with abraded skin or mucous membranes E) can be transmitted by animal bites
20)
Leptospirosis
Version 1
.
6
A) has only humans as a reservoir B) is communicable C) can be contracted from the environment D) is strictly transmitted by sexual contact E) is contracted by the fecal-oral route
21)
Trichomonas vaginalis
.
A) does not produce cysts B) in males causes urethritis C) in females causes vaginitis with a foul-smelling discharge D) has four flagella and an undulating membrane E) All of thechoices are correct.
22)
The most common cause of vaginitis is
.
A) Candida albicans B) Escherichia coli C) Staphylococcus aureus D) Streptococcus pyogenes E) Pseudomonas aeruginosa
23)
Candida albicans is a
.
A) fungus B) bacteria C) helminth D) protozoa E) None of the choices are correct.
Version 1
7
24)
Trichomonas vaginalis is a
.
A) fungus B) bacteria C) helminth D) protozoan E) None of the choices are correct.
25)
Which of the following is not true of prostatitis?
A) Caused by GI tract biota B) Can be chronic or acute C) The specific agent is easy to determine D) Patients may be very ill with the acute form E) Accompanied by pain and frequent, difficult urination
26)
Neisseria gonorrhoeae is
.
A) the cause of ophthalmia neonatorum B) the cause of gonorrhea C) called the gonococcus D) virulent due to fimbriae and a protease that inactivates IgA E) All of thechoices are correct.
27)
All of the following pertain to gonorrhea except
Version 1
.
8
A) a chancre-type lesion develops at the portal of entry B) is a sexually transmitted disease C) pelvic inflammatory disease (PID), epididymitis, and infertility are complications D) females can have asymptomatic infection E) symptoms include painful urination and discharge
28)
The primary virulence factor of Neisseria gonorrhoeae is
.
A) endotoxin B) exotoxin C) kinase D) fimbriae E) All of thechoices are correct.
29)
Pelvic inflammatory disease results from infection of the
.
A) vagina B) fallopian tubes C) ovaries D) Both vagina andfallopian tubesare correct. E) Both fallopiantubesand ovaries are correct.
30)
The leading cause of pelvic inflammatory disease is
.
A) gonorrhea B) chlamydia C) genital herpes D) syphilis E) HIV
Version 1
9
31)
Pelvic inflammatory disease often leads to
.
A) ovarian cancer B) uterine cancer C) cervical cancer D) infertility E) kidney cancer
32)
The following are characteristics of chlamydias except
.
A) they are gram-negative B) they are obligate parasites that need host cells for growth C) their elementary bodies are the infectious form D) the formation of elementary bodies is not part of the life cycle E) their reticulate bodies differentiate into elementary bodies
33)
Chlamydia trachomatis causes
.
A) nongonococcal urethritis in males B) cervicitis in females C) congenital conjunctivitis D) pelvic inflammatory disease in females E) All of thechoices are correct.
34)
The most common reported STD in the United States is
Version 1
.
10
A) gonorrhea B) chlamydia C) genital herpes D) syphilis E) HIV
35)
Lymphogranuloma venereum is a complication of
.
A) gonorrhea B) chlamydia C) genital herpes D) syphilis E) HIV
36)
The best way to directly observe spirochetes is
microscopy.
A) bright-field B) dark-field C) fluorescent D) phase-contrast E) electron scanning
37)
Treponema pallidum
.
A) has humans as the reservoir B) can cross the placenta C) has a hooked tip to attach to epithelium D) is transmitted by direct sexual contact E) All of thechoices are correct.
Version 1
11
38)
The chancre of syphilis
.
A) occurs due to small hemorrhaging of capillaries B) is very painful C) occurs during the tertiary stage D) develops into a lesion with firm margins and an ulcerated, central crater E) All of thechoices are correct.
39)
The secondary stage of syphilis
.
A) is when the patient is no longer infectious to others B) occurs within 10 days of the primary stage C) is a time when the pathogen enters and multiplies in the blood D) has no symptoms E) is when gummas develop in tissues
40) During which stage of syphilis does fever, lymphadenopathy, and a red to brown rash occur?
A) Primary B) Secondary C) Tertiary D) Latent E) All of thechoices are correct.
41)
Permanent cardiovascular and neurological damage is seen in which stage of syphilis?
Version 1
12
A) Primary B) Secondary C) Tertiary D) Latent E) All of the choices are correct.
42) Syphilitic lesions called tertiary stage of syphilis.
develop in the liver, skin, bone, and cartilage during the
A) chancres B) gummas C) ulcers D) nodules E) None of the choices are correct.
43)
The rash of secondary syphilis
.
A) causes severe itching B) is intensely painful C) only lasts a few days D) usually disappears spontaneously after a few weeks E) appears within 1 to 2 days of infection
44)
The latency period of syphilis can last up to
Version 1
.
13
A) 3 to 6 months B) 1 year C) 10 years D) 20 years or more E) Syphilis is never a latent infection.
45)
Which of the following antivirals is not used for treatment of HSV-1 and HSV-2?
A) Interferon B) Valacyclovir C) Acyclovir D) Famciclovir E) All of thechoices are correct.
46)
Chancroid
.
A) is caused by Treponema pallidum B) is caused by a spirochete C) presents with a hard chancre D) is caused by Haemophilus ducreyi E) is very painful in both sexes
47)
Herpes simplex 1
.
A) is exclusive to oral mucosa B) is exclusive to genitourinary tract C) confers immunity to herpes simplex 2 D) is cleared by acyclovir E) None of the choices are correct.
Version 1
14
48)
Herpes simplex 2 (HSV-2) causes
.
A) genital lesions B) intensely sensitive vesicles on or near the genitals C) symptoms that include urethritis, cervicitis, and itching D) infection in neonates that have contact with lesions in the birth canal E) All of thechoices are correct.
49)
An occasional serious complication of herpes simplex 1 is
.
A) shingles B) paralysis C) encephalitis D) myocarditis E) kidney failure
50)
All of the following are true for HSV-2 except
.
A) becomes latent in sacral ganglion B) virus is only shed from active lesions C) is reactivated by bacterial infections D) can infect oral mucosa E) All of the choicesare correct.
51)
All of the following pertain to genital warts except
Version 1
.
15
A) are not common in the United States B) are sexually transmitted C) often occur on the penis, vagina, and cervix D) includes large cauliflower-like masses called condylomata acuminata E) certain strains strongly predispose a person to cancer of the cervix or penis
52)
Warts are caused by
.
A) bacteria B) enveloped DNA viruses C) nonenveloped DNA viruses D) enveloped RNA viruses E) nonenveloped RNA viruses
53)
Which group is at greatest risk for group B Streptococcus infection?
A) Pregnant women B) Neonates C) 21-year-old females D) 21-year-old males E) Older adults (age 65 years and older)
54) Even after a positive test for a microbial cause, prostatitis is unresponsive to antibiotic therapy. Why?
Version 1
16
A) It is a viral infection B) Drug-induced side effects are too great C) Mixed biofilms are hard to kill D) The infection is sporadic E) Prostatitis is never caused by bacteria
55) Which of the following are characteristics that can be utilized to distinguish between vaginitis and vaginosis?
A) B) C) D)
56)
Causative agent Presence of vaginal inflammation Presence of discharge All of thechoices are correct.
The male urinary tract differs from the female tract in that
.
A) in males the urethra acts as the terminal organ for both the urinary and reproductive systems, whereas in females, the urethra is separate from the vagina B) in males the kidneys attach to the bladder via the ureter, whereas in females it attaches via the urethra C) in females the bladder is significantly smaller than the male bladder to provide room for the uterus D) in males the kidneys are significantly larger than in females due to testosterone production
57)
Which of the following isnot a component of the female reproductive tract?
Version 1
17
A) Vas deferens B) Fallopian tubes C) Ovary D) Cervix E) Uterus
58)
The male reproductive system comprises all of the following except
.
A) the fallopian tubes B) vas deferens C) epididymides D) the scrotum E) seminal vesicles
59) as
The urinary tract defenses in both males and females consists of mechanical factors such , and chemicals like , to prevent the growth of pathogens in the system.
A) B) C) D)
the flushing action of urine; lysozyme phagocytosis; interleukins lysis of foreign cells; interferons the inflammatory response; leukotrienes
60) Which of the following statements is incorrect regarding the defense mechanisms of the female reproductive tract?
Version 1
18
A) Vaginal pH is continually around 5 throughout the lifespan of the female to prevent the growth of pathogenic organisms. B) After puberty, glycogen is secreted into the vaginal mucus, which is fermented into acid by the normal biota thus lowering the pH. C) Before puberty and after menopause, vaginal mucus remains around pH 7. D) The microbiota of the female vaginal tract varies throughout her lifetime in response to vaginal pH; lower pH encourages the growth of organisms that prevent invasion by pathogens that could harm a developing fetus.
61)
Prostatitis is usually caused by bacteria that are transmitted by
A) B) C) D)
.
endogenous transfer from the gastrointestinal tract direct contact from a sexual partner poor hygiene during urination opportunism by the microbiota following an antibiotic regime
62) Evaluate the following statements and determine which is incorrect regarding the development of genital warts.
A) Warts caused by human papillomavirus can be inconspicuous, whereas warts caused by the poxvirus are large and unsightly, a condition called condyloma acuminata. B) Two types of virus, HPV and a poxvirus can cause genital warts although HPV infection has more serious consequences. C) Genital warts caused by the human papillomavirus range from being inconspicuous or extensive, whereas those from the poxvirus do not vary. D) Regardless of the viral cause of genital warts, the condition is not treatable, although the warts can be removed.
63) Evaluate the statements below and determine which one is incorrect regarding the HPV vaccine.
Version 1
19
A) Only females have a cervix so it is not necessary to vaccinate males against HPV since they cannot develop cervical cancer. B) Even if a female retains her virginity until she marries, it is important she is vaccinated before having sex because the virus is transmitted from male to female regardless of marital status. C) Males can develop cancer from HPV, most often in the penis, anus, mouth and throat, so it important that males as well as females are vaccinated during adolescence. D) HPV is so pervasive in our society that the CDC states that nearly all sexually active adults will be exposed to HPV at some stage of their lives. E) Since HPV is incurable and some strains house oncogenes, vaccination before first exposure is the best cancer prevention.
64)
The HPV vaccine is recommended for all adolescents because
.
A) it causes a variety of cancers in both males and females B) the vaccine prevents infection by up to nine strains of the virus that cause both cancers and genital warts C) the virus can be transmitted from male to female, female to male, male to male, and female to female, so everyone, regardless of sexual preference, should receive the vaccine D) the virus is so common that most sexually active individuals will be exposed to the virus at some point in their lives E) All of the choices are correct.
65) Colonization of pregnant females by Group B Streptococcus puts the neonate at risk because .
Version 1
20
A) the organism produces hemolysins that can break down red blood cells B) colonization during pregnancy can result in preterm labor C) the neonate can develop meningitis or pneumonia through exposure to the organism in the birth canal D) permanent damage such as hearing or vision loss can result following recovery from an acute condition after delivery E) All of the choices are correct.
66)
Choose the statement that best describes characteristics of leptosporosis.
A) Leptosporosis has four phases. B) Leptosporosis is not considered a zoonotic infection. C) Leptosporosis in humans is caused by Leptospira interrogans, a gram-negative bacillus. D) Leptosporosis can be marked by the development of Weil’s syndrome, a cluster of symptoms characterized by kidney invasion, hepatic disease, jaundice, anemia, and neurological disturbances.
67)
An inflammation of the vagina most commonly caused by Candida, referred to as ,is characterized by some degree of vaginal itching; , however, does not appear to induce vaginal inflammation and is most likely due to a reduction in Lactobacillus species. A) B) C) D)
vaginitis; vaginosis trichomoniasis; vaginosis vaginosis; vaginitis vaginitis; trichomoniasis
68) Inflammation of a male reproductive gland known as is accompanied by pain in the pelvic area and lower back, difficulty urinating, and painful ejaculation.
Version 1
21
A) B) C) D)
prostatitis chancroid trichomoniasis molluscum contagiosum
69) Warts produced by human papillomavirus infection range from tiny, flat bumps to extensive, branching, cauliflower-like masses called . A) B) C) D)
condyloma acuminata molluscum contagiosum pediculosis pubis chancroids
70) NCLEX Prep - Test Bank Question: Please read the clinical scenario, and then answer the questions that follow to become familiar with the traditional NCLEX question format. A nurse in the emergency department is caring for a 23-year-old female who is on vacation for her honeymoon. Two days ago, she began experiencing burning while voiding and a tactile fever. The patient reports she has been treating the symptoms with cranberry tablets and increased fluid intake. Her symptoms have not improved and now she is experiencing a frequent urge to void and lower abdominal pain. Upon assessment, the nurse finds the patient has a lowgrade fever.
70.1) Based on the patient’s clinical presentation, which of the following diagnoses is most likely?
A) B) C) D)
Version 1
Urethritis Cystitis Pyelonephritis General urinary tract infection
22
70.2)
A) B) C) D)
What findings on urinalysis would be consistent with cystitis?
Increased glucose Depleted protein Presence of WBCs Decreased specific gravity
70.3) The nurse provides education for the patient prior to discharge. She is instructed to seek care for which of the following symptoms?
A) B) C) D)
High fever and low back pain Lack of fever and RLQ tenderness Nausea and vomiting Lack of pain and pink urine
70.4) The medical provider prescribes an antibiotic regimen. The patient asks the nurse if antibiotics are necessary if she increases her intake of cranberry tablets. What is the most appropriate response by the nurse?
A) Cranberry tablets should acidify your bladder and urinary tract, effectively killing bacteria. Antibiotics shouldn’t be necessary if you increase your dose. B) Your genitourinary tract defenses should be able to fight this infection without the use of antibiotics or additional therapies. If your symptoms worsen, you should take the antibiotics. C) It is important to complete a course of antibiotics to prevent this infection from becoming more serious. D) You must take the antibiotics to prevent this infection from spreading to others.
Version 1
23
71) NCLEX Prep - Test Bank Question: Please read the clinical scenario, and then answer the questions that follow to become familiar with the traditional NCLEX question format. A 20-year-old male college student presents to the campus health clinic. The patient reports no significant past medical history. He has been experiencing painful urination for several days and yellow discharge from his urethra. The nurse asks the student about his sexual activity. He reports engaging in unprotected sex, and is concerned that he may have contracted a sexually transmitted infection. The nurse sends a sample of the urethral discharge to the microbiology laboratory. The laboratory reports gram-negative diplococci and many neutrophils on the smear, suggestive of gonorrhea.
71.1) Based on the patient’s clinical presentation and history alone, how could the nurse have distinguished gonorrhea from chlamydia?
A) B) C) D)
71.2)
A) B) C) D)
Chlamydia rarelypresents with a yellow discharge. Painful urinationis the hallmark of gonorrhea. Chlamydiaadditionally presents with lower abdominal pain. Gonorrhea andchlamydia mimic each other in presentation.
Which antibiotic(s) does the nurse anticipate will be ordered to treat the patient?
Amoxicillin Bactrim Gonorrhea is not treated with antibiotics Ceftriaxone and azithromycin
71.3) The nurse provides education about prevention of future gonorrheal infection. Which of the following topics would be included in the teaching?
Version 1
24
A) B) C) D)
Always use a condom for sexual contact Vaccination with Gardisil Strict abstinence Gonorrhea will always be latent in the patient’s genitourinary system.
71.4) The patient asks the nurse to keep his diagnosis confidential. What is the nurse’s best response?
A) I’m bound by HIPAA laws to maintain patient confidentiality, including your diagnosis. B) I am required by law to contact the health department, and you should notify any recent sexual partners. C) Once a sexually transmitted infection is detected, HIPAA laws no longer apply. I am obligated to share this with the school to track occurrences of gonorrhea on campus. D) I am required, by law, to contact the health department, unless you sign a ‘Waiver of Liability’ form.
72) NCLEX Prep - Test Bank Question: Please read the clinical scenario, and then answer the questions that follow to become familiar with the traditional NCLEX question format. A 21-year-old college student presents to the university campus clinic. The nurse collects the patient’s medical history and performs an initial assessment. The patient states, “I’m having a herpes flair, and I need a prescription for acyclovir.” The nurse examines the lesion on the shaft of his penis. The ulcer is 2 cm in size and seems painless and hard upon palpation.
72.1)
Version 1
Based upon the assessment of the lesion, what is the most likely diagnosis?
25
A) B) C) D)
72.2)
A) B) C) D)
Herpes Primary syphilis Chancroid Human papillomavirus infection
How would the diagnosis of syphilis be confirmed?
Dark-field microscopy Blood antibody test PCR All of these tests can be used in diagnosis.
72.3) The physician in the clinic wrote a prescription for acyclovir when the student called him on the phone that morning. What is the most appropriate action by the nurse?
A) Give the student the prescription for acyclovir B) Give the student the prescription for acyclovir and instruct him to use it in the future C) Call the physician, report the assessment findings, and suggest a prescription for penicillin G D) Ask the physician to write a prescription for a broad-spectrum antibiotic
72.4) The student states, “My friend had syphilis, but the lesion healed without antibiotics. Do I really need to take medications?” What is the best response by the nurse?
Version 1
26
A) The lesion will probably heal independently, but antibiotics will make it heal faster. B) The lesion will heal on its own, but latent syphilis will cause serious disease later on and must be treated. C) Syphilis is primarily viral in nature so an antibiotic is used to prevent bacterial infection of the healing lesion. D) Antibiotics will protect you from contracting other sexually transmitted infections while the lesion is healing.
73)
Chlamydia trachomatis is commonly transmitted by direct sexual activity. ⊚ ⊚
74)
Chlamydias can cross the placenta and cause intrauterine infection. ⊚ ⊚
75)
true false
true false
Chlamydiosis can lead to cervicitis, salpingitis, and pelvic inflammatory disease. ⊚ ⊚
true false
76) The antivirals used for herpes simplex infections have the ability to completely destroy the virus and permanently cure the latent infection. ⊚ ⊚
Version 1
true false
27
77) Group B streptococcal infections can cause serious infections in infants through vertical transmission. ⊚ true ⊚ false
78)
The HPV vaccine prevents infection by a single pathogenic strain of HPV. ⊚ ⊚
true false
79) The flushing action of urine is a defense mechanism that prevents microbial invasion of the urinary system. ⊚ ⊚
true false
80) In both genders, the kidneys, ureters, bladder, and the entire urethra are kept sterile by urine flow. ⊚ ⊚
true false
81) Chancroid is an ulcerative disease caused by a spirochete and produces systemic complications similar to that seen with syphilis. ⊚ ⊚
true false
82) Molluscum contagiosum is an STI that produces warts, and it can also be associated with a number of serious complications.
Version 1
28
⊚ ⊚
83)
true false
The HPV vaccine prevents infection by all types of human papillomaviruses. ⊚ ⊚
true false
84) Pregnant women are an important risk group for group B Streptococcus infections due to their ability to pass this dangerous organism to the infant during the birthing process. ⊚ ⊚
Version 1
true false
29
Answer Key Test name: Ch 21 4e 1) [A, B, C, D] 2) [C, D] 3) [A, B, C] 4) [B, C, E] 5) [A, E] 6) [A, B, C, E] 7) D 8) D 9) A 10) D 11) A 12) B 13) E 14) A 15) C 16) E 17) B 18) C 19) E 20) C 21) E 22) A 23) A 24) D 25) C 26) E Version 1
30
27) A 28) D 29) E 30) B 31) D 32) D 33) E 34) B 35) B 36) B 37) E 38) D 39) C 40) B 41) C 42) B 43) D 44) D 45) A 46) D 47) E 48) E 49) C 50) B 51) A 52) C 53) B 54) C 55) D 56) A Version 1
31
57) A 58) A 59) A 60) A 61) A 62) A 63) A 64) E 65) E 66) D 67) A 68) A 69) A 70) Section Break 70.1) B 70.2) C 70.3) A 70.4) C 71) Section Break 71.1) D 71.2) D 71.3) A 71.4) B 72) Section Break 72.1) B 72.2) D 72.3) C 72.4) B 73) TRUE 74) FALSE Version 1
32
75) TRUE 76) FALSE 77) TRUE 78) FALSE 79) TRUE 80) FALSE 81) FALSE 82) FALSE 83) FALSE 84) TRUE
Version 1
33
CHAPTER 22 1) Increased rainfall in some areas of the world could lead to an increased incidence of which of the following diseases? A) B) C) D)
2)
Malaria Yellow fever Hepatitis A Rabies
Using microbes to break down or remove toxic wastes in water and soil is called .
A) decomposition B) synergism C) mineralization D) bioremediation E) recycling
3)
The breakdown of man-made compounds by decomposers is called
.
A) mineralization B) biodegradation C) decomposition D) parasitism E) saprobism
4)
Potable water is
Version 1
.
1
A) contaminated B) polluted C) safe to drink D) foul tasting E) None of the choices are correct.
5)
Indicator bacteria are
.
A) coliforms B) enterococci C) evidence of fecal contamination D) used in water quality tests E) All of thechoices are correct.
6)
Which of the following is not true of coliforms?
A) They are gram-negative, lactose-fermenting, and gas-producing bacteria B) They include E. coli, Enterobacter, and Citrobacter C) Coliform levels are not always directly correlated to E. coli levels D) Fecal coliform assays are very specific E) Coliform bacteriophages are currently used as good fecal indicators
7)
The most prominent waterborne pathogens of recent times include
.
A) Giardia B) Cryptosporidium C) Campylobacter D) Salmonella E) All of thechoices are correct.
Version 1
2
8)
Which is the first step in water purification?
A) Chlorination B) Aeration and settling C) Sedimentation D) Storage E) Filtration
9) In which step of water purification does water move through sand beds and activated charcoal?
A) Chlorination B) Aeration and settling C) Sedimentation D) Storage E) Filtration
10)
Primary sewage treatment includes
.
A) sludge digesting B) skimming C) filtration D) chlorination E) aeration
11)
The final step of sewage treatment before release into the environment is
Version 1
.
3
A) sludge digesting B) skimming C) filtration D) chlorination E) aeration
12) What chemical is used in both water purification and sewage treatment to give long-term disinfection?
A) Fluorine B) Activated charcoal C) Copper sulfate D) Chlorine E) All of thechoices are correct.
13) An infectious disease typically seen among animals that may be acquired by humans is referred to as . A) B) C) D)
zoophilic zoonotic nosocomial agronotic
14) Which of the following factors contributed to the emergence and spread of Lyme disease?
Version 1
4
A) Increase in deer and mouse populations B) Increase in deer tick populations C) Increase in new home developments in wooded areas D) Climate change E) All of these factors are correct.
15)
Which of the statements about the emergence of HIV/AIDS is incorrect?
A) It is likely that HIV/AIDS began as a zoonotic disease, evolving from SIV, the simian immunodeficiency virus. B) HIV spread throughout sub-Saharan Africa via colonization and urbanization. C) It is believed that HIV was spread to the United States from Southeast Asia during the Vietnam War. D) The first cases of AIDS in the United States were seen in 1980.
16)
What two strategies are used in the United States to ensure potable drinking water? A) Filtration and chlorination B) Chlorination and gamma radiation C) Ultraviolet disinfection and filtration D) Pasteurization and chlorination E) Filtration and pasteurization
17) Monitoring of drinking water and recreational water is done by testing for indicator bacteria. Indicator bacteria . A) B) C) D)
Version 1
are bacteria typically found in the mammalian mouth, nose, and throat are bacteria typically found in the mammalian gastrointestinal tract are bacteria known to proliferate in aquatic environments and fish are bacteria known to cause human disease
5
18)
Which of the following bacteria would be most indicative of fecal contamination? A) B) C) D)
Staphylococcus aureus Streptococcus pyogenes E. coli Enterobacter aerogenes
19) Which of the following viruses can be readily transmitted by contaminated food and water? A) Hepatitis B virus B) Hepatitis A virus C) Hepatitis C virus D) Herpes simplex virus E) Varicella zoster virus
20) Which of the following atmospheric gases have been increasing in concentration and are believed to be contributors to climate change? A) Oxygen and carbon dioxide B) Hydrogen and carbon dioxide C) Nitrogen and oxygen D) Carbon dioxide and methane E) Methane and oxygen
21)
Which diseases are predicted to increase as a result of global climate change?
Version 1
6
A) Malaria and cholera B) Malaria and bacterial meningitis C) Bacterial meningitis and cholera D) MRSA infections and bacterial meningitis E) Campylobacter infections and E. coli infections
22) The incidence of which disease is most likely to be impacted as a result of increased rainfall? A) B) C) D)
Malaria Lyme disease Salmonella gastroenteritis Hepatitis A
23) The transmission of which diseases are dependent on maintenance of the mosquito vector? A) Malaria B) Lyme disease C) Cholera D) West Nile virus – encephalitis E) Both malaria and West Nile virus encephalitis
24) One of the goals of wastewater treatment is to (BOD).
the biological oxygen demand
A) increase B) decrease C) neutralize
Version 1
7
25)
The primary phase of wastewater treatment A) B) C) D)
26)
removes harmful microorganisms separates solid from liquid wastes increases the biological oxygen demand often uses ultraviolet radiation
The secondary phase of wastewater treatment is largely A) B) C) D)
27)
.
physical biological chemical irradiation
The primary phase of wastewater treatment is largely A) B) C) D)
.
.
physical biological chemical irradiation
28) Use of a trickling filter that sprays wastewater over rocks coated with biofilms which aerobically degrade organic matter is an example of the . A) primary phase of wastewater treatment B) secondary phase of wastewater treatment C) tertiary phase of wastewater treatment
Version 1
8
29)
The activated sludge step in wastewater treatment is an example of the
.
A) primary phase of wastewater treatment B) secondary phase of wastewater treatment C) tertiary phase of wastewater treatment
30)
Which of the following may be removed by tertiary water treatment? A) Ammonia B) Nitrate C) Phosphate D) Pathogenic microorganisms E) All of the choices are correct.
31) Agricultural water run-off, rich in nitrates, ammonia, and phosphates, may be treated to remove these chemicals so that the treated water does not support algal blooms. Such treatment is an example of . A) primary treatment B) secondary treatment C) tertiary treatment
32)
Which wastewater treatment process can generate methane which to be used as a fuel? A) B) C) D)
33)
Settling Trickling filters Activated sludge process Anaerobic digestors
Backyard composting in either a pile or a bin is generally an
Version 1
process.
9
A) aerobic B) anaerobic
34)
The decomposition that occurs in a sanitary landfill is largely
.
A) aerobic B) anaerobic
35) Which term is used to describe a novel, chemically synthesized compound that does not occur naturally but can be made to be biodegradable? A) Antibiotic B) Probiotic C) Autobiotic D) Xenobiotic E) Pseudobiotic
36)
Which of the following has been chemically modified to make it biodegradable? A) B) C) D)
Glass Aluminum foil Styrofoam Laundry detergent
37) The greater the amount of organic matter in wastewater, the oxygen demand.
Version 1
its biological
10
A) higher B) lower
38)
Which of the following activities/processes depends upon the activities of microbes? A) Composting B) Wastewater treatment C) Bioremediation D) Biodegradation E) All of the choices are correct.
39)
Which term would be used to describe a compound totally resistant to microbial attack? A) B) C) D)
40)
Inert Recalcitrant Antibacterial Bioaccumulative
Disruption of the One Health balance can be caused by
.
A) burning of fossil fuels leading to climate change B) transporting organisms outside of their usual habitat C) excessive use of antibiotics D) increase in population density E) All of the choices are correct.
41)
The concept of One Health demonstrates the interrelationship between
Version 1
.
11
A) human health, animal health, and environmental health B) global economics, animal health, and climate change C) human health, global warming, and economics D) global travel, communication, and economics E) environmental health, microbial growth, and economics
42) Which of the following statements describes the cause of the hantavirus pulmonary syndrome outbreak in the Southwest during the 1990s, demonstrating the interconnected web of human health, animal health and environmental health?
A) Excess rain enhanced the production of pine nuts, which led to a large population of deer mice, the reservoir of hantavirus that are shed in urine and feces, and can aerosolize into human lungs. B) Global warming evaporated the lakes and drove deer mice, the reservoir of hantavirus, into the suburbs, where they passed the virus to humans in their feces. C) Polluted rivers from excess watershed run-off killed the native plants so the starving deer mice were forced to migrate to urban areas where they shed hantavirus that aerosolized into human lungs. D) Increased temperatures enhanced the mosquito population in the Southwest, leading to more humans being bitten and greater transmission of hantavirus.
43) Which of the following did not contribute to the outbreak of hantavirus pulmonary syndrome in the Southwest during the 1990s?
A) B) C) D)
Version 1
Excessive winter rainfall Enhanced production of the pine nut Increase in deer mice population Global warming
12
44) All of the following are factors that contribute to the increase in Lyme disease cases in the United States except .
A) increased pollution of lakes and rivers B) decrease in areas of farmland C) increase in areas of woodland D) warming trends E) suburban development expanding into natural areas
45)
Which of the following statements is incorrect regarding zoonotic infections?
A) Infections agents pass from animals to humans, not from humans to animals. B) The majority of emerging human diseases in the past 30 years have been zoonotic. C) Fewer than 1% of vertebrate viruses have been described. D) Some zoonotic diseases can originate from animal-to-human transmission, then human-to-human transmission can further the spread.
46) Choose the answer that explains the emergence of Lyme disease in the northeastern United States. A) Restoration of woodland areas increased the deer tick population, the vector for transmission of Lyme disease to humans. B) Larger amounts of rainfall increased the mosquito population, the vector for transmission of Lyme disease to humans. C) The Northeast area of the country was slower to develop adequate wastewater treatment plans, leading to the spread of Lyme disease through contaminated drinking water.
47) Which of the following most likely explains the transmission of HIV from animals to humans?
Version 1
13
A) HIV evolved from a similar virus in chimpanzees that was transmitted to hunters by bites, scratches, and blood products. B) Sexual contact between bush hunters and chimpanzees led to transmission of HIV from animals to humans. C) Urine and fecal matter transferred SIV from monkeys to humans, and then the virus mutated to its current HIV form.
48) Enteric bacteria most useful in the routine monitoring of microbial pollution are gramnegative rods called , because finding them in high numbers may indicate recent or high levels of fecal contamination. A) B) C) D)
49)
coliforms biosensors streptococci xenobiotics
Which of the following xenobiotic chemicals was redesigned to be biodegradable? A) Laundry detergents B) Inorganic nitrogen compounds C) Greenhouse gases
50)
What is meant by the term "biological oxygen demand"?
A) It is the amount of oxygen required for aerobic organisms to break down organic matter in a body of water B) It is the amount of oxygen needed for anaerobic degradation in wastewater C) It is the required amount of oxygen for microbes to break down inorganic phosphates and nitrogen compounds in oil spills
Version 1
14
51)
Which of the following accurately describes the concept of "social distancing?"
A) Limiting the number of people in a certain space to minimize the risk of transmission of an infectious disease B) Isolating sick individuals in hospital rooms away from other patients C) Keeping ventilated patients at least six feet away from one another D) Lowering the number of cases that enter the hospital at one time
52) NCLEX Prep—Test Bank Question: Please read the clinical scenario, and then answer the questions that follow to become familiar with the traditional NCLEX question format. A clinic nurse at the county health department provides health counseling for a 26-year-old male who will be traveling internationally. The nurse consults the Centers for Disease Control and Prevention vaccination recommendations for the specific areas to which he is traveling. The man has many questions about the vaccines as well as infectious disease prevention.
52.1) The traveler is concerned about the safety of drinking water. Which of the following is a common pathogen that can cause a waterborne diarrheal illness? A) B) C) D)
Staphylococcus Lactobacillus Shigella All of the choiceswill cause diarrheal illness.
52.2) The nurse educates the traveler about markers of fecal contamination, as he is bringing a water indicator kit that tests for coliforms. The nurse advises that the kit may not detect other fecal pathogens. Which of the following organisms are coliforms?
Version 1
15
A) B) C) D)
E. coli Enterobacter Klebsiella All of the choicesare coliforms.
52.3)
The presence of coliforms necessarily indicates the presence of fecal matter.
⊚ ⊚
true false
52.4) The presence of trace amounts of fecal coliforms in a water sample means that the entire body of water is considered unsafe to drink. ⊚ ⊚
true false
52.5)
What type of bacteria is water tested for when determining its safety?
A) B) C) D)
Version 1
Presence bacteria Indicator bacteria Confirming bacteria Yielding bacteria
16
53) NCLEX Prep—Test Bank Question: Please read the clinical scenario, and then answer the questions that follow to become familiar with the traditional NCLEX question format. A 47-year-old male and father of five works as a high school English teacher. He recently read in the community newspaper that local sewage is treated and repurposed to the community as drinking water. His senior English class is concerned about the quality of drinking water in the city and they want to petition the state senate to pass a bill barring the practice. He comes to the local health department asking for information.
53.1)
A) B) C) D)
53.2)
A) B) C) D)
53.3)
A) B) C) D)
Version 1
What is the role of nonpathogenic microbes in the wastewater treatment process?
Degradation of organic compounds Outcompeting pathogens Anaerobic decomposition of solid waste All of these are true.
What is the initial step in wastewater treatment?
Skimming Aeration Chlorination Digestion
Which gas is used as a mode of purification in the final phase of treatment?
Hydrogen Carbon dioxide Ammonia Chlorine
17
54)
There are very few viruses in aquatic environments. ⊚ ⊚
55)
true false
The use of "coliform" and " E. coli" when discussing potable water is interchangeable. ⊚ ⊚
true false
56) In the secondary phase of water treatment, organic matter undergoes biodegradation by a diverse mix of bacteria, algae, and protozoa. ⊚ ⊚
true false
57) Coliforms, used as indicator bacteria, are gram-negative bacteria that ferment lactose and produce gas. ⊚ true ⊚ false
58) There are cities in the United States that obtain drinking water from the same river in which sewage was disposed of upstream. ⊚ true ⊚ false
59) After secondary treatment, treated wastewater can usually be safely released into nearby bodies of waters.
Version 1
18
⊚ ⊚
true false
60) Most of the oil released into the Gulf of Mexico during the Deepwater Horizon explosion was biodegraded by microorganisms. ⊚ true ⊚ false
61)
Biodegradable plastics or plastic substitutes exist today that can be composted. ⊚ true ⊚ false
62) As wastewater moves through the wastewater treatment process, its biological oxygen demand increases. ⊚ true ⊚ false
63) The concept of One Health demonstrates the interconnected relationship between global travel, economics and communication. ⊚ ⊚
true false
64) Global warming increased the mosquito population in the Southwest during the 1990s, which led to an outbreak of hantavirus pulmonary syndrome. ⊚ ⊚
Version 1
true false
19
65) The increase in cases of Lyme disease in the northeast as well as other parts of the United States is due to several overlapping influences; general warming trends, an increase in natural areas as farmlands disappear, and increasing human populations encroach on natural areas as suburban development spreads. ⊚ ⊚
true false
66) Transmission of organisms that cause zoonotic diseases can only pass from animal to human, not from human to animal. ⊚ ⊚
true false
67) In wastewater treatment, the primary stage separates out large solid matter, the secondary stage reduces the remaining matter using microbial biodegradation, and an optional third stage removes additional substances like ammonia and nitrates. ⊚ true ⊚ false
68) Waterborne pathogens are always bacterial in nature and have been known to cause disease epidemics. ⊚ ⊚
true false
69) All coliform species are fecal bacteria; therefore, a high total coliform count indicates a substance is absolutely unsafe to drink. ⊚ true ⊚ false
Version 1
20
70) A xenobiotic is any compound that has been chemically altered to undergo biodegradation. ⊚ true ⊚ false
71)
COVID-19 is a zoonotic infection. ⊚ ⊚
Version 1
true false
21
Answer Key Test name: Ch 22 4e 1) [A, B] 2) D 3) B 4) C 5) E 6) D 7) E 8) C 9) E 10) B 11) D 12) D 13) B 14) E 15) C 16) A 17) B 18) C 19) B 20) D 21) A 22) A 23) E 24) B 25) B 26) B Version 1
22
27) A 28) B 29) C 30) E 31) C 32) D 33) A 34) B 35) D 36) D 37) A 38) E 39) B 40) E 41) A 42) A 43) D 44) A 45) A 46) A 47) A 48) A 49) A 50) A 51) A 52) Section Break 52.1) C 52.2) D 52.3) FALSE 52.4) FALSE Version 1
23
52.5) B 53) Section Break 53.1) D 53.2) A 53.3) D 54) FALSE 55) FALSE 56) TRUE 57) TRUE 58) TRUE 59) TRUE 60) TRUE 61) TRUE 62) FALSE 63) FALSE 64) FALSE 65) TRUE 66) FALSE 67) TRUE 68) FALSE 69) FALSE 70) FALSE 71) TRUE
Version 1
24